You are on page 1of 398

IMAS 3-4

(2012 – 2022)
STT NĂM VÒNG ĐỀ LỜI GIẢI
1 3 9
1 2012
2 19 26
1 31 39
2 2013
2 53 60
1 69 77
3 2014
2 88 96
1 106 114
4 2015
2 123 129
1 136 144
5 2016
2 157 164
1 174 181
6 2017
2 193 200
1 210 217
7 2018
2 230 237
1 248 255
8 2019
2 271 278
1 289 297
9 2020
2 312 320
1 332 340
10 2021
2 356 364
11 2022 1 374 381
注意:

允許學生個人、非營利性的圖書館或公立學校合理使用
本基金會網站所提供之各項試題及其解答。可直接下載
而不須申請。

重版、系統地複製或大量重製這些資料的任何部分,必
須獲得財團法人臺北市九章數學教育基金會的授權許
可。

申請此項授權請電郵 ccmp@seed.net.tw

Notice:

Individual students, nonprofit libraries, or schools are


permitted to make fair use of the papers and its
solutions. Republication, systematic copying, or
multiple reproduction of any part of this material is
permitted only under license from the Chiuchang
Mathematics Foundation.
Requests for such permission should be made by
e-mailing Mr. Wen-Hsien SUN ccmp@seed.net.tw

1
International Mathematics Assessments for Schools

2012 MIDDLE PRIMARY DIVISION FIRST ROUND PAPER


Time allowed:75 minutes

INSTRUCTION AND INFORMATION


GENERAL
1. Do not open the booklet until told to do so by your teacher.
2. No calculators, slide rules, log tables, math stencils, mobile phones or other
calculating aids are permitted. Scribbling paper, graph paper, ruler and compasses
are permitted, but are not essential.
3. Diagrams are NOT drawn to scale. They are intended only as aids.
4. There are 20 multiple-choice questions, each with 5 choices. Choose the most
reasonable answer. The last 5 questions require whole number answers between
000 and 999 inclusive. The questions generally get harder as you work through the
paper. There is no penalty for an incorrect response.
5. This is a mathematics assessment, not a test; do not expect to answer all questions.
6. Read the instructions on the answer sheet carefully. Ensure your name, school
name and school year are filled in. It is your responsibility that the Answer Sheet
is correctly coded.
7. When your teacher gives the signal, begin working on the problems.
THE ANSWER SHEET
1. Use only lead pencils.
2. Record your answers on the reverse side of the Answer Sheet (not on the question
paper) by FULLY filling in the circles which correspond to your choices.
3. Your Answer Sheet will be read by a machine. The machine will see all markings
even if they are in the wrong places. So please be careful not to doodle or write
anything extra on the Answer Sheet. If you want to change an answer or remove
any marks, use a plastic eraser and be sure to remove all marks and smudges.
INTEGRITY OF THE COMPETITION
The IMAS reserves the right to re-examine students before deciding whether to
grant official status to their scores.

2
─────────────────────────────────────────────────
2012 MIDDLE PRIMARY DIVISION FIRST ROUND PAPER
─────────────────────────────────────────────────

Questions 1-10, 3 marks each

1. We want to cut a 20 m stick into shorter sticks of length 4 m. We can only cut
one piece of stick at a time. How many cuts are required?
(A)3 (B)4 (C)5 (D)6 (E)7
─────────────────────────────────────────────────
2. What number must replace the □ in the mathematical sentence
□ + 5 = 13-6 to make it correct?
(A)1 (B)2 (C)3 (D)4 (E)5
─────────────────────────────────────────────────
3. Which of the following number is less than 2,010,000?
(A) two millions and ten thousands (B) 2,100,000
(C) one million and twenty thousands (D) 20,100,000
(E) two millions and one hundred thousands
─────────────────────────────────────────────────
4. What is the value of 999+99?
(A)1088 (B)1098 (C)1099 (D)1889 (E)1989
─────────────────────────────────────────────────
5. The following figures are formed by identical squares. Which figure has the
longest perimeter?
(A) (B) (C) (D) (E)

─────────────────────────────────────────────────
6. Starting from the forest, a rabbit hopped 500 m to the north, and then reached a
farm by hopping 500 m to the west. In which direction from the forest is the
farm?
(A)East (B)South (C)Northwest(D)North (E)West
─────────────────────────────────────────────────
7. Which of the following figures cannot be drawn without lifting the pencil off the
paper and without going over any line twice?
(A) (B) (C) (D) (E)

──────────────────────────────────────────────

3
───────────────────── MP 2 ──────────────────────
8. What operation signs must replace ○ and □ so that the mathematical
sentence 2 + 8 + 3 = 2 ○ 8 □ 3 is correct?
(A)+ for ○ and × for □ (B)× for ○ and - for □
(C)+ for ○ and ÷ for □ (D)× for ○ and ÷ for □
(E)× for ○ and+for □
─────────────────────────────────────────────────
9. A ray consists of a point on a line and all the points on that line on one side of
that point. How many rays can we draw through a point on a plane?
(A) 1 (B) 2 (C) 4
(D) 8 (E)Infinitely many
─────────────────────────────────────────────────
10. The birthdays of two sisters are on the same day. The elder sister is 4 years
older than the younger sister. When the sum of their ages reaches 50 years, how
old is the younger sister?
(A)12 (B)20 (C)23 (D)25 (E)27
─────────────────────────────────────────────────

Questions 11-20, 4 marks each

11. The diagram below shows two squares AMND and PQRS inside a rectangle
ABCD. The areas of the two squares are 16 cm2 and 4 cm2 respectively.
What is the sum of area of the shaded regions in cm2?
D N C
P S

Q R
A M B

(A)3 (B)4 (C)5 (D)6 (E)7


─────────────────────────────────────────────────
12. How many zeros are there in the end of the product of 20×30×40×50?
(A)1 (B)2 (C)4 (D)5 (E)6
─────────────────────────────────────────────────
13. Some students form a rectangle. Joseph is in the fourth row if we count from
the front and in the seventh row if we count from the back. He is in the third
column if we count from left and in the ninth column if we count from the right.
How many students are there?
(A)90 (B)100 (C)110 (D)120 (E)132
─────────────────────────────────────────────────

4
────────────────────── MP 3 ─────────────────────
14. Two pieces of straw can be joined together by overlapping one with the other for
a length of 2 cm. If three pieces of straw are joined together to form a magic
wand, what is the length, in cm, of the magic wand?
(A)54 (B)55 (C)56 (D)58 (E)60
─────────────────────────────────────────────────
15. In the mathematical sentence below, A, B, C, D, E and F represent six distinct
digits from 0 to 9. What is the numeral value of E?
6 A
× 3 5
3 3 B
1 C 8
D E F B
(A)0 (B)1 (C)2 (D)3 (E)4
─────────────────────────────────────────────────
16. A soccer match consists of two halves each lasting 45 minutes, with a 15-minute
break in between. If a soccer match begins at 13:00 and there is no extension of
the game, at what time will it end?
(A)13:45 (B)14:00 (C)14:30 (D)14:35 (E)14:45
────────────────────────────────────────────────
17. Study the mathematical sentences below:
74 × 6 = 444 ,
74 × 12 = 888 ,
74 × ( ) = 444888
What number must be filled in the parentheses so that the last sentence is
correct?
(A)1266 (B)1206 (C)6088 (D)6612 (E)6012
─────────────────────────────────────────────────
18. Let us fold a square piece of paper along a diagonal to make a triangle. Cut a
small round hole near each of the three corners of the triangle. Which of the
following figures is obtained when we unfold the piece of paper?
(A) (B) (C) (D) (E)

─────────────────────────────────────────────────
19. Susan wrote five numbers on the board, represented by A, B, C, D and E. A is
larger than B, C is larger than D, C is smaller than E, D is larger than B, and E is
smaller than A. Which number is the third largest?
(A)A (B)B (C)C (D)D (E)E

5
─────────────────────────────────────────────────
────────────────────── MP 4 ─────────────────────
20. The three figures in the diagram below have equal perimeters. What is a:b:c ?

2a c c

2b 2b
a a
c c
2a c
a
(A)1:1:1 (B)2:4:5 (C)4:6:5
(D)16:20:25 (E)20:25:24
────────────────────────────────────────────────

Questions 21-25, 6 marks each

21. The mirror image of a mathematical expression reads .


What is the correct difference?
─────────────────────────────────────────────────
22. The following figure is composed of eight line segments. At the intersection of
every two segments is a right angle, and each line segment is marked with a
number. The easiest method to find the perimeter of the figure is by measuring
three segments in the figure. What is the smallest three-digit number that is
formed by the corresponding numbers of the three segments?

3


4


5

6

2

7


8

○1

─────────────────────────────────────────────────
23. Let △, □ and ☆ represent three distinct digits. If 7△90901 is larger than
79□9001, which is in turn larger than 798900☆, what is the value of
△ + □ + ☆?
─────────────────────────────────────────────────
24. The number of bicycles in the school bicycles lot is a three-digit number, and the
number of bicycle wheels is also a three-digit number. These six digits are 2, 3,
4, 5, 6 and 7 in some order. At most how many bicycles are there?
────────────────────────────────────────────────

6
────────────────────── MP 5 ─────────────────────
25. Helen has some $1, $2 and $5 coins. The total value is $80. All the $1 coins
may be traded in for $10 coins, resulting in 36 fewer coins. All the $5 coins
may be traded in for $10 coins, and all the $2 coins may be traded in for $5 coins.
What is the largest possible numbers of coins Helen has?
────────────────────────────────────────────────

***

7
注意:

允許學生個人、非營利性的圖書館或公立學校合理使用
本基金會網站所提供之各項試題及其解答。可直接下載
而不須申請。

重版、系統地複製或大量重製這些資料的任何部分,必
須獲得財團法人臺北市九章數學教育基金會的授權許
可。

申請此項授權請電郵 ccmp@seed.net.tw

Notice:

Individual students, nonprofit libraries, or schools are


permitted to make fair use of the papers and its
solutions. Republication, systematic copying, or
multiple reproduction of any part of this material is
permitted only under license from the Chiuchang
Mathematics Foundation.
Requests for such permission should be made by
e-mailing Mr. Wen-Hsien SUN ccmp@seed.net.tw

8
─────────────────────────────────────────────────
2012 MIDDLE PRIMARY FIRST ROUND SOLUTION
─────────────────────────────────────────────────
1. We want to cut a 20 m stick into shorter sticks of length 4 m. We can only cut
one piece of stick at a time. How many cuts are required?
(A)3 (B)4 (C)5 (D)6 (E)7
【Suggested Solution】
The wooden stick was cut into 20 ÷ 4 = 5 pieces, therefore we needs to cut 5-1 = 4
times. Answer:(B)
2. What number must replace the □ in the mathematical sentence
□ + 5 = 13-6 to make it correct?
(A)1 (B)2 (C)3 (D)4 (E)5
【Suggested Solution】
2+5=13-6. (B)
Answer:
3. Which of the following number is less than 2,010,000?
(A) two millions and ten thousands (B) 2,100,000
(C) one million and twenty thousands (D) 20,100,000
(E) two millions and one hundred thousands
【Suggested Solution】
2 010 000 is equal to A, less than B, greater than C, less than D, less than E.
Answer:(C)
4. What is the value of 999+99?
(A)1088 (B)1098 (C)1099 (D)1889 (E)1989
【Suggested Solution】
999+99=(1000-1)+ (100-1)=1100-2=1098.
Answer:(B)
5. The following figures are formed by identical squares. Which figure has the
longest perimeter?
(A) (B) (C) (D) (E)

【Suggested solution】
Let the side lengths of squares are 1.The perimeter of figure A is 18; the perimeter of
figure B is 22; the perimeter of figure C is 20; the perimeter of figure D is 16; the
perimeter of figure E is 18. Thus the figure with largest perimeter is B.
Answer:(B)
6. Starting from the forest, a rabbit hopped 500 m to the north, and then reached a
farm by hopping 500 m to the west. In which direction from the forest is the
farm?
(A)East (B)South (C)Northwest(D)North (E)West

9
【Suggested Solution】
From the diagram below, we can determine the location of the Farmland and it is in
the Northwest of the Forest.
500 m
Farmland
N
500 m

Forest
Answer:(C)
7. Which of the following figures cannot be drawn without lifting the pencil off the
paper and without going over any line twice?
(A) (B) (C) (D) (E)

【Suggested Solution】
Figures A, B, D and E can be drawn without lifting the pencil off the paper and
without going over any line twice; it is only figure C that is not.
Answer:(C)
8. What operation signs must replace ○ and □ so that the mathematical
sentence 2 + 8 + 3 = 2 ○ 8 □ 3 is correct?
(A)+ for ○ and × for □ (B)× for ○ and - for □
(C)+ for ○ and ÷ for □ (D)× for ○ and ÷ for □
(E)× for ○ and+for □
【Suggested Solution】
The value of the mathematical sentence on the left is equal to 13, when we fill × and
-symbol in the mathematics sentence on the right, we have 2×8-3=13.
Answer:(B)
9. A ray consists of a point on a line and all the points on that line on one side of
that point. How many rays can we draw through a point on a plane?
(A) 1 (B) 2 (C) 4
(D) 8 (E)Infinitely many
【Suggested Solution】
Using the given point as the endpoint, we can draw as many rays in different
directions. Thus, we can draw many infinite rays. Answer:(E)
10. The birthdays of two sisters are on the same day. The elder sister is 4 years
older than the younger sister. When the sum of their ages reaches 50 years, how
old is the younger sister?
(A)12 (B)20 (C)23 (D)25 (E)27

10
【Suggested Solution】
The difference between the ages of two sisters is 4 years old, so when the sum of their
ages is 50 years old, the age of the younger sister is (50-4)÷2=23 years old.
Answer:(C)
11. The diagram below shows two squares AMND and PQRS inside a rectangle
ABCD. The areas of the two squares are 16 cm2 and 4 cm2 respectively. What
is the sum of area of the shaded regions in cm2?
D N C
P S

Q R
A M B

(A)3 (B)4 (C)5 (D)6 (E)7


【Suggested solution】
The side lengths of two squares AMND and PQRS are 4cm and 2cm respectively.
Hence MN = 4 cm,PS = PQ = RQ = 2 cm, the area of rectangle ABCD is 6×4=24cm2
and the sum of areas of the shaded regions in the figure is24-16-4 =4 cm2.
Answer:(B)
12. How many zeros are there in the end of the product of 20×30×40×50?
(A)1 (B)2 (C)4 (D)5 (E)6
【Suggested solution】
20×30×40×50=1 200 000. There are 5 zeroes in the end of the product.
【Suggested solution】
There are 1 zero each in the end of 20, 30, 40 and 50. When 2 times 5 have one more
zero in the end of the product, hence there are 5 zeroes in the end of the product.
Answer : (D)
13. Some students form a rectangle. Joseph is in the fourth row if we count from
the front and in the seventh row if we count from the back. He is in the third
column if we count from left and in the ninth column if we count from the right.
How many students are there?
(A)90 (B)100 (C)110 (D)120 (E)132
【Suggested Solution】
From the given information, each row of the rectangular queue has 3 + 9 -1 = 11
(student) while each column has 4 +7-1 = 10 (student), so the total number of
students is 11 × 10 = 110.
Answer:(C)
14. Two pieces of straw can be joined together by overlapping one with the other for
a length of 2 cm. If three pieces of straw are joined together to form a magic
wand, what is the length, in cm, of the magic wand?
(A)54 (B)55 (C)56 (D)58 (E)60

11
【Suggested Solution】
If there is no joint between any two straws, then the three straws will have a total
length 20 × 3 = 60 cm, a joint is 2 cm, two joints need to be 4 cm. Therefore, the
length of the magic wand is 60-4 = 56 cm.
Answer:(C)
15. In the mathematical sentence below, A, B, C, D, E and F represent six distinct
digits from 0 to 9. What is the numeral value of E?
(A)0 (B)1 (C)2 (D)3 (E)4
6 A
× 3 5
3 3 B
1 C 8
D E F B
【Suggested Solution】
From the mathematical sentence, since the product of A and 3 is 8, then the only
possible value of A is 6, so the value of C must be 9, it follows that B=0, G=0; thus
F=1, E=3 and D=2. Answer:(D)
16. A soccer match consists of two halves each lasting 45 minutes, with a 15-minute
break in between. If a soccer match begins at 13:00 and there is no extension of
the game, at what time will it end?
(A)13:45 (B)14:00 (C)14:30 (D)14:35 (E)14:45
【Suggested Solution】
A complete soccer match will take 45+45+15=105 minutes in all, therefore the soccer
match will end at 14:45. Answer: (E)
17. Study the mathematical sentences below:
74 × 6 = 444 ,
74 × 12 = 888 ,
74 × ( ) = 444888
What number must be filled in the parentheses so that the last sentence is correct?
(A)6 (B)12 (C)444 (D)888 (E)6012
【Suggested Solution】
444888=444000+888=74×6000+74×12=74×6012。 Answer:(E)
18. Let us fold a square piece of paper along a diagonal to make a triangle. Cut a
small round hole near each of the three corners of the triangle. Which of the
following figures is obtained when we unfold the piece of paper?
(A) (B) (C) (D) (E)

12
【Suggested Solution】
Refer to the illustrations below, the first diagram is the result of a square piece of
paper, folded diagonally and then hole on three corners; the second diagram is how
piece of paper looks like when unfolded.

Answer:(C)
19. Susan wrote five numbers on the board, represented by A, B, C, D and E. A is
larger than B, C is larger than D, C is smaller than E, D is larger than B, and E is
smaller than A. Which number is the third largest?
(A)A (B)B (C)C (D)D (E)E
【Suggested Solution】
Since A > E > C > D > B, therefore the third largest number corresponds to C.
Answer:(C)
20. The three figures in the diagram below have equal perimeters. What is a:b:c?

2a c c

2b 2b
a a
c c
2a c
a
(A)1:1:1 (B)2:4:5 (C)4:6:5
(D)16:20:25 (E)20:25:24
【Suggested solution】
We have 4b+a=6a =5c so that 4b = 5a and 6a =5c. Thus a:b =4:5 and a:c =5:6,
so that a:b:c = 20:25:24.
Answer:(E)
21. The mirror image of a mathematical expression reads .
What is the correct difference?
【Suggested Solution】
The mathematical expression that we saw in the mirror must be
Therefore, the correct answer is 84.
Answer:084
22. The following figure is composed of eight line segments. At the intersection of
every two segments is a right angle, and each line segment is marked with a
number. The easiest method to find the perimeter of the figure is by measuring
three segments in the figure. What is the smallest three-digit number that is
formed by the corresponding numbers of the three segments?

13

3


4


5

6

2

7


8


1

【Suggested Solution】
Since the intersection of every two line segments is a right angle, we can see the line
segments ○ 1 and ○ 3 are of equal length, line segments ○ 5 and ○ 7 are also of equal

length, ○ 4 +○6 +○
8 =○ 2 , so if simply measuring the segment ○ 1 (or segment ○ 3 ),

segment ○ 2 and segment ○ 5 (or segment ○ 7 ), then the perimeter of the given

diagram can be determined easily. Therefore, the smallest three-digit number


corresponding to three line segments is 125.
Answer:125
23. Let △, □ and ☆ represent three distinct digits. If 7△90901 is larger than
79□9001, which is in turn larger than 798900☆, what is the value of
△ + □ + ☆?
【Suggested Solution】
In order 7△90901>79□9001,then △must be filled in by the digit 9 only.
So that 79□9001>798900☆,then 8 must be filled in □ while ☆ must filled in by
the digit 0. Hence the sum of the three digits represented by △, □ and ☆ is
9+8+0=17. Answer:017
24. The number of bicycles in the school bicycles lot is a three-digit number, and the
number of bicycle wheels is also a three-digit number. These six digits are 2, 3,
4, 5, 6 and 7 in some order. At most how many bicycles are there?
【Suggested solution】
Let the total number of bicycles be abc and the total number of wheels be def .
From the question , we have def = 2abc ,so a can only be 2 or 3.
If a = 3,then d can only be 6 or 7。
When d = 6,b can only be 2, then c = 7, thus abc = 327 , def = 654 。
When d = 7,b can only be 6, there does not exist any suitable value for c。
If a = 2,the total number of bicycles is obviously less than 327, therefore there are at
most 327 bicycles. Answer:327
25. Helen has some $1, $2 and $5 coins. The total value is $80. All the $1 coins
may be traded in for $10 coins, resulting in 36 fewer coins. All the $5 coins
may be traded in for $10 coins, and all the $2 coins may be traded in for $5 coins.
What is the largest possible numbers of coins Helen has?

14
【Suggested Solution】
When Helen traded every ten $1-coin into one $10-coin, the number of coins reduced
by 9 pieces, there are 36 coins reduced from piggy bank, then a total of $ 40 was
changed, so there are 40 pieces of $1 coins, which gives an amount of $40. Let the
number of $2 coins as m and number of $5 coins as n, then 2m+5n=80-40. From the
given information, we know that m is a multiple of 5 and n is a multiple of 2. When
m=5, n=6; when m=10, n=4;when m=15, n=2. When we consider m as a larger
value, there is no n satisfies the given condition of the problem. The above three
cases can conclude that the number of coins are 51, 54 and 57. Hence, there are at
most 57 coins.
Answer:057

15
注意:

允許學生個人、非營利性的圖書館或公立學校合理使用
本基金會網站所提供之各項試題及其解答。可直接下載
而不須申請。

重版、系統地複製或大量重製這些資料的任何部分,必
須獲得財團法人臺北市九章數學教育基金會的授權許
可。

申請此項授權請電郵 ccmp@seed.net.tw

Notice:

Individual students, nonprofit libraries, or schools are


permitted to make fair use of the papers and its
solutions. Republication, systematic copying, or
multiple reproduction of any part of this material is
permitted only under license from the Chiuchang
Mathematics Foundation.
Requests for such permission should be made by
e-mailing Mr. Wen-Hsien SUN ccmp@seed.net.tw

16
2nd International Mathematics Assessments for Schools
(2012-2013 )

Middle Primary Division Round 2


Time: 120 minutes

Printed Name;! ! Code; ! Score;! !

Instructions:!
z Do not open the contest booklet until you are told to do so.
z Be sure that your name and code are written on the space provided above.
z Round 2 of IMAS is composed of three parts, total score is 100 marks.
z Questions 1 to 5 are given in multiple-choice test. Each question has five possible
options marked as A, B, C, D and E. Only one of these options is correct. After
making your choice, fill in the appropriate letter on the space provided. Each
correct answer is worth 5 marks. There is no penalty for an incorrect answer.
z Questions 6 to 13 are short answer test. Only Arabic numerals are accepted; using
other written text will not be honored or credited. Some questions have more than
one answer, as such all answers are required to be written down on the space
provided to obtain full marks. Each correct answer is worth 5 marks. There is no
penalty for incorrect answer.
z Questions 14 and 15 require detailed solution or process in which 20 marks are to
be awarded to completely written solution. Partial marks may be given to
incomplete presentation. There is no penalty for an incorrect answer.
z Using of electronic computing devices is not allowed.
z Only pencil, blue or black ball-pens may be used to write your solution or answer.
z Diagrams are not drawn to scale. They are intended as aids only.
z After the contest the invigilator will collect the contest paper.
The following area is to be filled up by the judges;
the contestants are not supposed to mark anything here.
Total
Question 1 2 3 4 5 6 7 8 9 10 11 12 13 14 15 Score Signature

Score

Score

17
18
Middle Primary Division Round 2
Questions 1 to 5, 4 marks each
1. When 86 is divided by a number, the remainder is 6. Which of following cannot
be the value of that number?
(A)10 (B)20 (C)30 (D)40 (E)80

Answer:

2. Edmond was one of 14 athletes in the track-and-field team. They are numbered
from 1 to 14. The sum of the numbers of the other 13 athletes is 100. What is
Edmond’s number?
(A)5 (B)7 (C)11 (D)14 (E)17

Answer:

3. A department store sells three different gift items costing 30 dollars, 60 dollars
and 90 dollars, and three kinds of gift boxes costing 20 dollars, 50 dollars and 80
dollars. Helen buys one gift item and a gift box. How many different values can
her total bill be?
(A)4 (B)5 (C)6 (D)7 (E)8

Answer:

4. The diagram below shows five squares of equal size. The shaded part of each is
removed. Of the resulting figures, whose perimeter is equal to the perimeter of
the uncut square?
(A) (B) (C) (D) (E)

Answer:

5. A man standing upside down saw in a mirror a woman wearing a shirt with a
number as shown below. What is the actual number on the woman’s shirt?

(A)2009 (B)2006 (C)5006 (D)5009 (E)6005

Answer:

19
MP 2

Questions 6 to 13, 5 marks each


6. The weight of box A is four times the weight of box B. Box A is heavier than box
B by 12 kg. What is the weight of box A in kg?

Answer: kg

7. The distance between Mark’s home and the park is 3000 m. Mark walked from
his home to the park at a speed of 100 m per minute and then walked back home
at a speed of 150m per minute. What is Mark’s average speed in meter per
minute?

Answer: m per minute

8. There are three 1 by 3 cards, each containing three digits in a vertical column. If
these three cards are placed side by side as shown in the diagram below, we can
read off three three-digit numbers: 264, 918 and 689. What is the smallest
three-digit number that can be obtained by rearranging the order of the cards?
2 6 4
9 1 8
6 8 9

Answer:

9. In the 6 × 8 square grid, each square is 1 cm2 , what is the shaded area, in cm2 ?

Answer: cm2

20
MP 3
10. The diagram shows two identical graduated cylinders of capacity 900 ml, each
containing some water. In order for both cylinders to contain the same amount of
water, how much water should be poured from the cylinder on the left into the
cylinder on the right?

Answer: ml

11. Observe the following pattern:


29
= 0.725
40
31
= 0.775
40
33
= 0.825
40
27
What is the decimal expansion of ?
40

Answer:

12. The diagram shows that if a rope is folded once and be cut in halves, it will
separate into 3 pieces; and if it is folded twice instead, it will separate into 5
pieces. If it is folded 6 times instead, into how many pieces will it separate?

Fold 1 time Fold 2 times


Answer:

13. Andy and Ben started at 7:00AM from town A and jogged along the same road in
the same direction. Andy jogged at a constant speed of 6 km/h while Ben jogged
at a constant speed of 4 km/h. At 9:00AM, Ben borrowed a bike along the road
and rode at a constant speed of 10 km/h. He caught up with Andy at town B.
What was the distance between town A and town B?

Answer: km

21
MP 4
Questions 14 to 15, 20 marks each
(Detailed solutions are needed for these two problems)
14. The total value of Jerome’s 4 coins is 3 dollars less than Roberta’s 3 coins. Each
is a 1-dollar, 2-dollar or 5-dollar coin, but there are only two different kinds
among these 7 coins. What is their minimum total value?

Answer:

22
MP 5
15. Unlimited numbers of two types of two square cardboard papers are available:
side lengths of 1 cm or 2 cm.

Now, we are going to use these two kinds of cardboard paper to assemble a
rectangular shape with length of 5 cm by 3 cm. How many different ways can we
make such kind of rectangular shape paper? (rotating and flipping of each other
to make a rectangular paper is considered the same way of making the rectangular
paper. For example, the following three rectangular pieces of paper is the same
way of making a new rectangular pieces of paper, because the first rectangular
piece of paper becomes the second piece through a clockwise rotation of 90
degrees and the third piece can be obtained from the second piece by flipping it
upside down)

Answer:

23
24
注意:

允許學生個人、非營利性的圖書館或公立學校合理使用
本基金會網站所提供之各項試題及其解答。可直接下載
而不須申請。

重版、系統地複製或大量重製這些資料的任何部分,必
須獲得財團法人臺北市九章數學教育基金會的授權許
可。

申請此項授權請電郵 ccmp@seed.net.tw

Notice:

Individual students, nonprofit libraries, or schools are


permitted to make fair use of the papers and its
solutions. Republication, systematic copying, or
multiple reproduction of any part of this material is
permitted only under license from the Chiuchang
Mathematics Foundation.
Requests for such permission should be made by
e-mailing Mr. Wen-Hsien SUN ccmp@seed.net.tw

25
2012 Middle Primary Division Second Round Solution
1. Because 86 = 10 × 8 + 6 = 20 × 4 + 6 = 30 × 2 + 26 = 40 × 2 + 6 = 80 × 1 + 6 , the
remainder is . So cannot be the value of the quotient.
Answer:(C)
2. The sum of numbers of 14 athletes is 1+2+3+…+13+14=105, and 105-100=5.
That is, this extra 5 is Edmond’s number.
Answer:(A)
3. Arrange a gift and a gift box in pairs, there are 9 condition: (30, 20), (30, 50), (30,
80), (60, 20), (60, 50), (60, 80), (90, 20), (90, 50), (90, 80). The sums of two
numbers in parentheses are 50, 80, 110, 80, 110, 140, 110, 140, 170. So there are
totally 5 different possible prices: 50, 80, 110, 140, 170.
Answer:(B)
4. The perimeter of (A) is longer than the original one; that of (B) is equal to the
original one; that of (C) is longer than the original one; that of (D) is longer than
the original one; that of (E) is shorter than the original one.
(A) (B) (C) (D) (E)

So we choose (B).
Answer:(B)
5. If the man does not stand upside down, he should see in the mirror. If he
doesn’t see the number in the mirror, he should see . Hence, the number on
the shirt is 5006.
Answer:(C)
6. Assume that the weight of box B is 1 unit, than the weight of box A is 4 units.
That is, the weight of box A is 3 units more than box B. Hence, 3 units are equal
to 12 kilograms, 1 unit is equal to 12÷3=4 kilograms. So the weight of box A is
4×4=16 kilograms.
Answer:16 kg
7. If we want to know Mark’s average speed, we should find out how long he
walked and how long he spent. The distance he walked is totally 3000 × 2 = 6000
meters. The time he spent from home to the park is 3000 ÷ 100 = 30 minutes; that
of he spent from the park to home is 3000 ÷ 150 = 20 minutes, totally 30 + 20 =
50 minutes. So Mark’s average speed is 6000 ÷ 50 = 120 meters per minute.
Answer:120 m per minutes
8. If we want to find out the smallest three-digit number, the hundreds digit should
be as small as possible, so the second card should be placed on the left side. The
numbers correspond to 1 are 8 and 9. To get the smallest three-digit number, the
tens digit should be smaller than the single digit. So the third card is in the
middle, and the first card is on the right side. Hence, the smallest number we get
is 189.
Answer:189

26
9. The shaded area consists of 16 triangles, which are equal to
8 squares, and 7 squares. So there are 8 + 7 = 15 squares
are colored. That is, the shaded area is 15 cm2.

Answer:15 cm2
10. Notice that the water in the left cylinder is 900-600 = 300 ml milliliters more
than the right that in the right one, so we should pour 300 ÷ 2 = 150 milliliters
into the right cylinder.

Answer:150 ml
11. Observe that the denominators are all 40. When we add 2 to the numerator, the
fraction becomes 0.05 larger than the original one. Because 27 is 2 smaller than
29, the result should 0.05 smaller than 0.725, that is, 0.675.
Answer:0.675
12. 【Solution 1】 】We can get 2 strands of rope when folding for one time; 4 strands
of rope when folding for two times; 8 strands of rope when folding for three
times; 16 strands of rope when folding for four times; 32 strands of rope when
folding for five times. If we cut the rope in halves after folding five times, the
rope will be cut at 32 different places. That is, it will separate into 33 pieces.
【Solution 2】 】the pieces we get when cutting in halves after folding for several
times have the rule as follow:
Cut the rope in halves after folding for 1 time, we will get 21 + 1= 3 pieces;
Cut the rope in halves after folding for 2 time, we will get 22 + 1 = 5 pieces;
Cut the rope in halves after folding for 3 time, we will get 23 + 1 = 9 pieces;
Cut the rope in halves after folding for 4 time, we will get 24 + 1 =17 pieces;
Cut the rope in halves after folding for 5 time, we will get 25 + 1 =33 pieces.
Answer:33 pieces
13. 【Solution 1】Ben was (6-4)×2=4 km behind Andy when he rode on the bike,
so it took him 4÷(10-6)= 1 to catch up with Andy. At the same time, they
arrived at town B together. Therefore, it took Andy 3 hours to arrive town B from
town A. Then, the distance between two towns is 6×3=18 km.
【Solution 2】Assume that it took Ben t hours to arrive town B after starting to
ride the bike, so
2 × 4 + 10t = 2 × 6 + 6t , t = 1.
Therefore, the distance between town A and town B is 2 × 4 + 10 × 1 = 18 km.
Answer:18 km

27
14. If we want to know the minimum total value, Jerome and Roberta should both
have 1-dollar coin. Hence, Jerome has 4 dollars, Roberta has 3 dollars. (5 marks)
However, from the problem we know that Jerome’s 4 coins are 3 dollars less than
Roberta’s 3 coins, and there are only two different kinds among these coins.
Assume that the coins Roberta has are all 2-dollar, then he has 6 dollars, while
his money is only 2 dollars more than Jerome’s. So Roberta must have a 5-dollar
coin and two 1-dollar coins. (10 marks)
Hence, their minimum total value is 5 + 1 + 1 + 1 × 4 = 11 dollars. (5 marks)
Answer:11 dollars
【Marking Scheme】
Only exact solution without the solving process, 5 marks.

15. Because we will use at most two pieces of square of 2 cm when making paper
which length is 5 cm and width is 3 cm, so we turn the problem into several cases:
(5 marks)
(Case 1) There is only 1 way if we use only squares of 1 cm: (5 marks)

(Case 2) We use one piece of square of 2 cm. According to the symmetry,


there are 2 ways to make the paper: (5 marks)

(Case 3)We use two pieces of squares of 2 cm. According to the symmetry,
there are 4 ways to make the paper: (5 marks)

To sum up, there are 7 different ways.


Answer:7 ways
【Marking Scheme】
5 marks for finding out that at most 2 pieces of squares of 2 cm, and turning the
problem into different cases by pieces of squares of 2cm.
5 marks for each exactly discussion of situation;
Only exact solution without the solving process, 5 marks.

28
注意:

允許學生個人、非營利性的圖書館或公立學校合理使用
本基金會網站所提供之各項試題及其解答。可直接下載
而不須申請。

重版、系統地複製或大量重製這些資料的任何部分,必
須獲得財團法人臺北市九章數學教育基金會的授權許
可。

申請此項授權請電郵 ccmp@seed.net.tw

Notice:

Individual students, nonprofit libraries, or schools are


permitted to make fair use of the papers and its
solutions. Republication, systematic copying, or
multiple reproduction of any part of this material is
permitted only under license from the Chiuchang
Mathematics Foundation.
Requests for such permission should be made by
e-mailing Mr. Wen-Hsien SUN ccmp@seed.net.tw

29
International Mathematics Assessments for Schools

2013 MIDDLE PRIMARY DIVISION FIRST ROUND PAPER


Time allowed:75 minutes

INSTRUCTION AND INFORMATION


GENERAL
1. Do not open the booklet until told to do so by your teacher.
2. No calculators, slide rules, log tables, math stencils, mobile phones or other
calculating aids are permitted. Scribbling paper, graph paper, ruler and compasses
are permitted, but are not essential.
3. Diagrams are NOT drawn to scale. They are intended only as aids.
4. There are 20 multiple-choice questions, each with 5 choices. Choose the most
reasonable answer. The last 5 questions require whole number answers between
000 and 999 inclusive. The questions generally get harder as you work through the
paper. There is no penalty for an incorrect response.
5. This is a mathematics assessment, not a test; do not expect to answer all questions.
6. Read the instructions on the answer sheet carefully. Ensure your name, school
name and school year are filled in. It is your responsibility that the Answer Sheet
is correctly coded.
7. When your teacher gives the signal, begin working on the problems.
THE ANSWER SHEET
1. Use only lead pencils.
2. Record your answers on the reverse side of the Answer Sheet (not on the question
paper) by FULLY filling in the circles which correspond to your choices.
3. Your Answer Sheet will be read by a machine. The machine will see all markings
even if they are in the wrong places. So please be careful not to doodle or write
anything extra on the Answer Sheet. If you want to change an answer or remove
any marks, use a plastic eraser and be sure to remove all marks and smudges.
INTEGRITY OF THE COMPETITION
The IMAS reserves the right to re-examine students before deciding whether to
grant official status to their scores.

30
2013 MIDDLE PRIMARY DIVISION FIRST ROUND PAPER

Questions 1-10, 3 marks each


1. In an arts class, the teacher displays the following figures. How many of them are
circular?
■ ▲ ● ★ ■ ▲ ● ★ ■ ●
(A)1 (B)2 (C)3 (D)4 (E)5

2. The clock in the diagram shows the time 7:30. The hour hand is between the
numbers 7 and 8 and the minute hand points to the number 6. To which number
will the minute hand be pointing 40 minutes later?

(A)2 (B)4 (C)6 (D)8 (E)10

3. Which of the following is the closest in length to one day?


(A)half day (B)2 days (C)23 hours
(D)26 hours (E)1410 minutes

4. In the amusement park, a roller-coaster ride requires 5 tokens. Each token costs 5
dollars. How many dollars does Mickey have to spend for one ride?

(A)5 (B)10 (C)15 (D)20 (E)25

5. How many different triangles can be found in the diagram?

(A)1 (B)3 (C)4 (D)5 (E)6

31
MP 2
6. Walter has two options when going to school. He can (a) walk 8 minutes to the
bus stop and then ride the bus for 15 minutes to the school, or (b) walk 10
minutes to the subway stop and then ride the subway for 10 minutes to school. If
he does not have to wait for the bus at the bus stop, nor the train on the subway
station, what is the minimum number of minutes required for him to get to
school?
(A)18 (B)20 (C)23 (D)25 (E)33

7. The diagram shows the water distribution system in the neighbourhood. There
are five valves indicated by capital letters in black circles. When water flows into
a house, it will not flow out of that house into another. Which valve must we
close in order to shut off the water to exactly four houses?

D
Source of
water supply A B C
E

(A)A (B)B (C)C (D)D (E)E

8. A giraffe invites 28 small animals to a Plain Peach Party. In the group photo, the
giraffe is in the middle. Counting from the left, which position does the giraffe
occupy?

(A)1 2 (B)13 (C)14 (D)15 (E)16

9. A kangaroo jumps 6 metres forward, 4 metres backward, 7 metres forward, 8


metres backward and then rests. How many metres apart are the current position
and the initial position of the kangaroo?

(A)1 (B)3 (C)4 (D)6 (E)8

32
MP 3
10. A round table can seat 5 guests and a square table can seat 4 guests. Which of the
following combinations of tables can seat 36 guests without any empty seat?
(A)1 round table and 2 square tables (B)2 round tables and 4 square tables
(C)3 round tables and 5 square tables (D)4 round tables and 4 square tables
(E)5 round tables and 3 square tables

Questions 11-20, 4 marks each


11. In the supermarket, apples sell at 150 dollars for 6, and pears sell at 30 dollars for
2. By how many dollars is the average price of an apple more than the average
price of a pear?

(A)1 (B)3 (C)5 (D)10 (E)12

12. Four children together own 240 books. Ace gives Bea 3 books, Bea gives Cec 4
books, Cec gives Dee 5 books and Dee gives Ace 6 books. Then each has the
same number of books. Initially, how many books belong to the child with the
least number of books?
(A)57 (B)58 (C)59 (D)60 (E)61

13. Zachary has a computer program which accepts an input and produces an output.
Some of the data are shown in the following table.
Input 1 2 3 4 5 6 7
Output 4 7 10 13 16 ? 22
What is the output when the input is 6?
(A)17 (B)18 (C)19 (D)20 (E)21

14. Hana divides a circular piece of paper into 5 regions as shown in the diagram.
She wants to paint each region in a colour so that two regions sharing a common
side have different colours. What is the smallest number of colours she needs?

(A)1 (B)2 (C)3 (D)4 (E)5

33
MP 4
15. Three rabbits are digging for parsnips in a field. The White Rabbit and the
Spotted Rabbit dig up 13 parsnips between them. The Spotted Rabbit and the
Black Rabbit dig up 11 parsnips between them. The Black Rabbit and the White
Rabbit dig up 16 parsnips between them. What is the total number of parsnips
dug up by the three of them?
(A)10 (B)11 (C)15 (D)16 (E)20

16. Three travellers are crossing a desert together. When Mickey has finished his
water supply, Don still has 5 bottles of mineral water and Jan has 4 bottles. They
share the water equally among them. Mickey pays the others 36 dollars for the
water he has received. How many dollars should go to Don?
(A)8 (B)12 (C)16 (D)20 (E)24

17. From a box of chocolate, Mickey takes out half the number of pieces and puts
one piece back. Then he takes out half of the remaining number of pieces and
puts one piece back. After he has done this a total of 5 times, only three pieces
are left in the box. How many pieces of chocolate are there in the box initially?

(A)158 (B)78 (C)38 (D)34 (E)18

18. A necklace consists of 27 beads. When the necklace is opened, the first two beads
are black, the next two are white, the next two are black, the next two are white,
as shown in the diagram. If this pattern continues, what is the total number of
black beads in the necklace?

(A)13 (B)14 (C)15 (D)16 (E)17

19. The digits 1 to 9 are placed inside the squares in the diagram, with a different
digit in each of the boxes. Only the digit 2 is shown. If the equations are correct,
what is the two-digit number formed by the digits in the first two boxes from the
left?
÷ = - = 2
(A)98 (B)86 (C)78 (D)76 (E)68

34
MP 5
20. There are 54 grid points on the 5 by 8 grid, shown in the diagram, where the side
of each small square is 1 cm. Starting from point P, an ant crawls from grid point
to grid point along the grid lines, visiting each grid point exactly once before
returning to P. What is the maximum length of its path, in cm?
P

(A)26 (B)30 (C)36 (D)54 (E)93

Questions 21-25, 6 marks each


21. Some cards are missing from a deck of 52 cards. If the incomplete deck is dealt
to four players so that each receives the same number of cards, then 3 cards are
left. If it is dealt to three players instead, with each still receiving the same
number of cards, then 1 card is left. What is the maximum number of cards
possible in the incomplete deck?

22. The diagram shows a 4 cm by 4 cm piece of paper overlapping a 3 cm by 5 cm


piece of paper. By how many cm2 does the area of the non-overlapped part of the
square piece of paper exceed the area of the non-overlapped part of the
rectangular piece of paper?

23. For the class photo of 42 students, the photo shop charges 10 dollars for the first
copy and 3 dollars for each additional copy. Moreover, 2 bonus copies are given
for any order over 30 copies. If each student gets one copy, how many dollars
must they pay the photo shop altogether?

35
MP 6
24. The diagram shows 7 flowers printed on a piece of paper. What is the smallest
number of straight lines we must draw to divide the piece of paper into a number
of regions, so that each flower is in a different region?

25. The six faces of a cubical die are labelled with six different positive integers. If
the numbers on any two adjacent faces differ by at least 2, what is the minimum
value of the sum of these six numbers?

***

36
37
注意:

允許學生個人、非營利性的圖書館或公立學校合理使用
本基金會網站所提供之各項試題及其解答。可直接下載
而不須申請。

重版、系統地複製或大量重製這些資料的任何部分,必
須獲得財團法人臺北市九章數學教育基金會的授權許
可。

申請此項授權請電郵 ccmp@seed.net.tw

Notice:

Individual students, nonprofit libraries, or schools are


permitted to make fair use of the papers and its
solutions. Republication, systematic copying, or
multiple reproduction of any part of this material is
permitted only under license from the Chiuchang
Mathematics Foundation.
Requests for such permission should be made by
e-mailing Mr. Wen-Hsien SUN ccmp@seed.net.tw

38
Solution to First Round of 2013 IMAS Middle Primary Division

1. In an art class, the teacher presented shaped patterns. How many circular figures
are there?
■▲●★■▲●★■●
(A)1 (B)2 (C)3 (D)4 (E)5
【Suggested Solution】
There are three (3) circles shown in the above figures. Hence, we must select C.
Answer: C
2. The clock in the diagram shows the time 7:30. The hour hand is between the
numbers 7 and 8 and the minute hand points at number 6. What number will the
minute hand be pointing 40 minutes later?

(A)2 (B)4 (C)6 (D)8 (E)10


【Suggested Solution #1】
After 40 minutes, the time indicated in the clock is 8:10, the minute hand must be
pointing at 2. Hence, we select A.
【Suggested Solution #2】
The minute hand will move from one digit to the next digit every 5 minutes and 40
minutes later it will move 8 digits, that is 6 + 8-12 = 2, the minute hand will
pointing at digit 2, so we select option A.
Answer: A
3. Which of the following is the closest length of time for one day?
(A)half day (B)2 days (C)23 hours
(D)26 hours (E)1410 minutes
【Suggested Solution #1】
Let us first convert all the time into minutes’ measurement. Since 1 day = 1440
minutes, half day = 720 minutes, two days = 2880 minutes, 23 hours = 1380 minutes,
26hours = 560 minutes. Then it follows
1440 − 720 = 720
2880 − 1440 = 1440
1440 − 1380 = 60
1560 − 1440 = 120
1440 − 1410 = 30,
Hence, 1410 minutes is closest to one day. Therefore, we choose E.

39
【Suggested Solution #2】
Since 1 day = 24 hours= 1440 minutes and half day must be one-half of 1440 minutes,
so there is a difference of 720 minutes in half day while two days is more than a
whole day by 1440 minutes. 23 hours is one hour less than a whole day, there is a
difference of 60 minutes. The difference of 26 hours and a whole day is 2 hours,
which means they differ by 120 minutes. Lastly, the difference between 1410
minutes and a whole day is 30 minutes only. Hence, we conclude that 1410 minutes
is nearer to one whole day. So, we select E.
Answer: E
4. In the amusement park, a roller-coaster ride requires 5 tokens. Each token costs 5
dollars. How many dollars does Mickey have to spend for one ride?
(A)5 (B)10 (C)15 (D)20 (E)25
【Suggested Solution】
To be able ride a roller-coaster, he requires 5 tokens and each token costs $ 5.
Hence, Mickey must spend $5 × 5 = $25. So, we select E.
Answer: E
5. How many different triangles can be found in the diagram?
(A)1 (B)3 (C)4
(D)5 (E)6
【Suggested Solution #1】
A

D E

C B
Based on the given diagram, we can name five different triangles as △ADE, △DEB,
△DBC, △ADB, △ABC. Therefore, we choose D for our answer.
【Suggested Solution #2】
From the given diagram, we easily recognize each of the
regions A, B and C as triangles of different sizes. When we
A
combine regions A and B, they produce another triangle of
different size. When regions A, B and C are combined, the B
C
biggest triangle is produced. Therefore, there are 5 different
triangles in the diagram.
Answer: D
6. Walter has two options in going to school. He can (a) walk 8 minutes to the bus
stop, and rides the bus for 15 minutes to the school, or (b) walks 10 minutes to
the LRT station, rides the train for 10 minutes to school. If he does not have to
wait for the bus at the bus stop, nor the train on the subway station, what is the
minimum number of minutes required for him to get to school?
(A)18 (B)20 (C)23 (D)25 (E)33

40
【Suggested Solution 】
When Walter selects option (a) for going to school from his house, he needs to travel
8 + 15 =23 minutes; when he selects option (b), then he needs to travel 10 + 10 =20
minutes. Therefore, Walter needs at least 20minutes for him to get to school from his
house.
Answer: B
7. The diagram shows the water distribution system in the neighbourhood. There
are five valves indicated by capital letters. When water flows into a house, it will
not flow out to another houses. Which valve must be closed in order to shut off
the water to exactly four houses?
l
g h i m

D n
Source of Water
Supply A B C
E
o
k
f j

(A)A (B)B (C)C (D)D (E)E


【Suggested Solution】
Let us name the 10 houses as f, g, h, i, j, k, l, m, n and o and analyze the switch
(on/off) positions of the valve to determine our target :
When only valve A is shut off, then all the users will have no water, that is; a total of
10 houses will be waterless.
When only valve B is shut off, since the water of house j can be supplied by valve A,
so there are two houses f and j will still have water, that is; 8 houses will have no
water.
When only valve C is shut off, since the water of house l can be supplied by valve B,
so there are four houses k, m, n and o will have no water.
When only valve D is shut off, since the water of house l can be supplied by valve B
while n can be supplied by valve E, so only house m is affected with water
interruption. When only valve E is shut off, since the water of house n can be
supplied by valve E while k can be supplied by valve C, so there will be only one
house o be affected with no water.
Thus, we choose C.
Answer: C
8. A giraffe invites 28 small animals to a Plain Peach Party. In a group photo, the
giraffe is in the middle. Counting from the left, which position does the giraffe
occupy?
(A)1 2 (B)13 (C)14 (D)15 (E)16

41
【Suggested Solution】
If the giraffe’s position is at the middle, then animals at the left and right sides are
equal in number. Hence, there are 28 ÷ 2 =14 animals on either side; therefore the
giraffe is on the 15th position.
Answer: D
9. A kangaroo jumps 6 meters forward, 4 meters backward, 7 meters forward,
8 meters backward, and then it rests. How many meters apart are the current
position and the initial position of the kangaroo?
(A)1 (B)3 (C)4 (D)6 (E)8
【Suggested Solution #1】
Since the kangaroo jumps 6 m forward, then the kangaroo is 6 m from the initial
position, then he turns back and jumps 4 m, so he is now 6 − 4 = 2 m from the initial
position, and this time he turns forward and jumps 7 m again, which is 2 + 7 = 9 m
from the initial position, and lastly turns back to jump 8 m, which is 9 − 8 = 1 m from
the initial position.
【Suggested Solution #2】
From the given information, we know the kangaroo jumps to a forward direction in a
total distance of 6 +7 = 13 m while he jumps to a backward direction in a total
distance of 4 +8 = 12 m. Therefore, the distance that the kangaroo jumps from the
initial position to the current position is 13 − 12 = 1 m.
Answer: A
10. A round table can seat 5 guests and a square table can seat 4 guests. Which of the
following combinations of tables can seat 36 guests without leaving any empty
seat?
(A)1 round table and 2 square tables (B)2 round tables and 4 square tables
(C) 3 round tables and 5 square tables (D)4 round tables and 4 square tables
(E)5 round tables and 3 square tables
【Suggested Solution】
If there is 1 round table and 2 square tables, then only 1×5+2×4=13 guests can be
seated. The number of tables available is insufficient for the guests. Hence, it can’t be
the solution!
If there are 2 round tables and 4 square tables, then only 2×5+4×4=26 guests can be
seated. The number of tables available is insufficient for the guests. Hence, it can’t be
the solution!
If there are 3 round tables and 5 square tables, it can accommodate 3×5+6×4=39
guests. There are several empty seats. Hence, it can’t be the solution!
If there are 4 round tables and 4 square tables, it can accommodate 4×5+4×4=36
guests, number of seats is exactly the same as the number guests. This kind of
arrangement can be the solution.
If there are 5 round tables and 3 square tables, it can accommodate 6×5+2×4=38
guests, there are several empty seats. Hence, it can’t be the solution!
Answer: D

42
11. In a supermarket, apples are sold at 150 dollars for 6 pieces, and pears are sold at
30 dollars for 2 pieces. How much more expensive is the cost of each apple than
a cost of each pear?
(A)1 (B)3 (C)5 (D)10 (E)12
【Suggested Solution】
We can easily determine that each apple costs 150 ÷ 6 = $25, each pear cost 30 ÷ 2 =
$15, then the price of an apple is expensive than each pear by 25 − 15 = $10.
Answer: D
12. Altogether, there are 240 books owned by 4 children. If Ace gives Bea 3 books,
Bea gives Cec 4 books, Cec gives Dee 5 books and Dee gives Ace 6 books. Then
each has the same number of books. Initially, how many books belong to the
child with the least number of books?
(A)57 (B)58 (C)59 (D)60 (E)61
【Suggested Solution】
From the given information, we know that 4 children own 240 books, so when those
books are equally divided, each of them are supposed to have 240 ÷ 4 = 60 books.
But from the problem, we know that Ace has 6 − 3 = 3 more books, Bea has
4 − 3 = 1 book less, Cec has 5 − 4 = 1 book less and Dee has 6 − 5 = 1 book less.
Thus, we know that originally Ace must have 57 books while each of Bea, Cec and
Dee has 61 books.
Answer: A
13. Zachary has a computer program which accepts an input and produces an output.
Some of the data are shown in the table below.
Input 1 2 3 4 5 6 7
Output 4 7 10 13 16 ? 22
What is the output when the input is 6?
(A)17 (B)18 (C)19 (D)20 (E)21
【Suggested Solution #1】
From the given information, it shows that the output data is 1 more than three times
than the input data. Thus, when we enter, the output becomes 3 × 6 +1 = 19.
【Suggested Solution #2】
From the given information, we discover that the output data is an arithmetic
sequence composed of 4, 7, 10, 13, 16, ..., with a common difference of 3. Hence,
when the input data is 6, the output data is 16 +3 = 19.
Answer: C
14. Hanna divides a circular piece of paper into 5 regions as shown in
the diagram. She wants to paint each region using a color so that
two regions sharing a common side with different colors. What is
the smallest number of colors she needs?
(A)1 (B)2 (C)3
(D)4 (E)5

43
【Suggested Solution #1】
Let us assign the five different regions as regions 1 to 5 as in the
diagram. If region 1 is painted with color A, since we are required to 4
use the least number of colors, so we may paint color B in region 2
3 5
and 5, now region 4 must be painted with a color different from
region 5, that is; region 4 may use color A which is the same as region 2 1
1 and finally region 3 will be painted with a color different from that
of region 2 (color B) and region 4 (color A). So, we know region 3 must be painted with
a third color. Thus, Hanna must use at least 3 colors to paint the circular piece of paper.
【Suggested Solution # 2】
If there are only two color pens available, then by Pigeonhole
Principle we know that at least one color must be painted in three
regions, but the diagram proves it not possible because every three
regions there are at least two regions which are adjacent to one
another, so there must be at least 3 color pens in order to paint the
regions as shown in the diagram at the right.
Answer: C
15. Three rabbits are digging for radishes in a field. The White Rabbit and the
Spotted Rabbit dig up 13 radishes between them. The Spotted Rabbit and the
Black Rabbit dig up 11 radishes between them. The Black Rabbit and the White
Rabbit dig up 16 radishes between them. What is the total number of radishes
dug up by the three rabbits?
(A)10 (B)11 (C)15 (D)16 (E)20
【Suggested Solution】
Since 13 + 11 + 16 = 40, is exactly two times the number of radishes pulled out by
the three rabbits, so the three rabbits pulled out a total of 40 ÷ 2 = 20 radishes.
Answer: E
16. Three travellers are crossing a desert together. When Mickey has finished his
water supply, Don still has 5 bottles of mineral water and Jan has 4 bottles. They
share the water equally among them. Mickey pays the others 36 dollars for the
water he has received. How many dollars should go to Don?
(A)8 (B)12 (C)16 (D)20 (E)24
【Suggested Solution】
From the information, each person shares (5 + 4) ÷ 3 = 3 bottles of mineral water, so
that Mickey must pay 36 ÷ 3 = $12 for each bottle of mineral water, that is; Don
gives Mickey 5 − 3 = 2 bottles of mineral water while Jan gives 4 − 3 = 1 bottle of
mineral water to Mickey. Therefore, Mickey should pay 2 × 12 = $24 to Don as the
cost of mineral water. Answer: E
17. From a box of chocolate, Mickey takes out half the number of pieces and put one
piece back. Then he takes out half of the remaining number of pieces of
chocolates and puts one piece back. After he does this for a total of 5 times, there
are only three pieces of chocolates are left in the box. How many pieces of
chocolates are in the box initially?
(A)158 (B)78 (C)38 (D)34 (E)18

44
【Suggested Solution】
Working backward, we know that there are (3 − 1) × 2 = 4 pieces of chocolates in the
box before the 5th operation, it follows that there are (4 − 1) × 2 = 6 pieces of
chocolates in the box before the 4th operation , (6 − 1) × 2 = 10 pieces of chocolates in
the box before the 3rd operation, there are (10 − 1) × 2 = 18 pieces of chocolates in the
box before the 2nd operation and there are (18 − 1) × 2 = 34 pieces of chocolates in
the box before the 1st operation starts. Hence, there are 34 pieces of chocolate in the
box originally.
Answer: D
18. A necklace has 27 beads. When part of the necklace
becomes visible, it appears that the first two beads
are black, the next two are white, the next two are
black, the next two are white, as shown in the diagram. If this pattern continues,
what is the total number of black beads in the necklace?
(A)13 (B)14 (C)15 (D)16 (E)17
【Suggested Solution】
Let us consider two black beads and two white beads as a group, then 27 beads have
six groups and there remain 3 beads, and by rules of the pattern in the string of beads,
there must be 2 black beads in the remaining 3 beads. Since in each group, there are 2
black beads, so a total of 2 × 6 +2 = 14 black beads must be in the necklace.
Answer: B
19. The digits 1 to 9 are placed inside the squares in the diagram, with a different
digit in each of the boxes. Only the digit 2 is shown. If the equations are correct,
what is the two-digit number formed by the digits in the first two boxes from the
left?
÷ = - = 2
(A)98 (B)86 (C)78 (D)76 (E)68
【Suggested Solution #1】
Let us assume the above mathematics expression as AB ÷ CD = EF − GH = 2 .
Then from the expression AB ÷ CD = 2 , let us consider the following cases:
When AB = 98 , then CD =49 , the digit 9 repeated twice. Hence option E cannot be
the answer.
When AB = 86 , we have CD =43 , at this instance the possible value of E, F, G and
H are 1, 5, 7, and 9. Since it was given EF − GH = 2 , then F = 7, H = 5 or F = 9,
H = 7 or F = 1, H = 9. No matter what values will be assign to E and G, no value
will satisfy the mathematics sentence EF − GH = 2 . Hence option B is not the
answer.
When AB = 78 , it follow CD =39 , so the values of E, F, G and H may be the digits
1, 4, 5 and 6. But EF − GH = 2 , this implies F = 6, H = 4 and we know that there are
no possible values can be assign to E, G to meet the condition EF − GH = 2 . Hence
option C cannot be the answer.

45
When AB = 76 , then CD =38 , it follow the values of E, F, G and H can assign as 1,
4, 5 and 9. Since EF − GH = 2 , so we have F = 1, H = 9, so that E = 5, G = 4 in
order EF − GH = 2 ; that is; 76 ÷ 38 = 51 − 49 = 2 . This met the condition of the
problem.
When AB = 68 , it follow CD =34 , so the value of digits E, F, G and H could be 5, 7,
8 and 9. But EF − GH = 2 , so we have F = 7, H = 5 or F = 9, H = 7. Then for any
value of E and G, we can’t establish EF − GH = 2 . Hence option E is not the answer.
【Suggested Solution #2】
We may assume the given mathematical expression as AB ÷ CD = EF − GH = 2 .
Since AB ÷ CD = 2 , we can rewrite it as AB = 2 × CD while EF − GH = 2 where
E, G must be distinct digits and there is a regrouping in the subtraction, so we predict
F = 1 and H = 9:
(i) If EF = 81 , GH = 79 , at this instance the values of A, B, C and D are 3, 4, 5 and
6. But AB = 2 × CD , it follow that B = 6, D = 3 and for whatever the values of
A and C, it cannot satisfy AB = 2 × CD . Not Possible!
(ii) If EF = 71 , GH = 69 , the values of A, B, C and D can be assign by the digits 3,
4, 5 and 8. Since AB = 2 × CD , it follow that B = 8, D = 4 and whatever the
values of A and C, it cannot satisfy AB = 2 × CD . Not Possible!
(iii)If EF = 61 , GH = 59 , then the possible values of A, B, C and D are 3, 5, 7 and 8.
But AB = 2 × CD , we conclude B = 8, D = 4 or B = 4, D = 7; and whatever the
values of A and C, it cannot satisfy AB = 2 × CD . Not Possible!
(iv) If EF = 51 , GH = 49 , if follow that the possible values of A, B, C and D are 3,
6, 7 and 8; so that for AB = 2 × CD we have B = 6, D = 3 or B = 6, D = 8.
When B = 6, D = 3, for whatever values of A and C to be assign, there is no
possible way to establish AB = 2 × CD . Not Possible!
When B = 6, D = 8, we know that A = 7, C = 3 so that AB = 2 × CD , we have the
correct mathematical sentence; that is; 76 ÷ 38 = 51 − 49 = 2 .
(v) If EF = 41 , GH = 39 , the only possible values of A, B, C and D are 5, 6, 7 and
8. But AB = 2 × CD , it follow that B = 6, D = 8, and whatever the values of A
and C, it cannot satisfy AB = 2 × CD . Not Possible!
Answer: D
20. There are 54 grid points on a 5 by 8 grid as P
shown in the diagram where the side of each
small square is 1 cm. Starting from point P, an
ant crawls from point to point along the grid lines,
visiting each grid point exactly once before
returning to P. What is the maximum length of its
path, in cm?
(A)26 (B)30 (C)36 (D)54 (E)93

46
【Suggested Solution】
An ant starts from point P crawling along the path as P
shown by passing thru all the points and finally back
to point A. Because the ant must pass thru each grid
point once and only once, we also know the distance
between two adjacent points is 1 cm, and the path has
a total 54 points. So the ant must crawl through with
a total distance of 54 cm, which is also the longest
path.
Answer: D
21. Some cards are missing from a deck of 52 cards. If the incomplete deck is dealt
to four players so that each receives the same number of cards, then 3 cards are
left. If it is dealt to three players instead, with each still receiving the same
number of cards, then 1 card is left. What is the maximum number of cards are
there in the incomplete deck?
【Suggested Solution #1】
When the four players together play the cards, each of them gets the same number of
cards, and we discover that 3 cards are left, hence the possible number of cards is 3, 7,
11, 15, 19, 23, 27, 31, 35, 39, 43, 47, 51.
When three individuals play together, each of them gets the same number of cards.
Again, 3 cards are left; hence the possible number of cards is 1, 4, 7, 10, 13, 16, 19,
22, 25, 28, 31, 34, 37, 40, 43, 46, 49.
From above two cases, we can infer the possible the number of cards as 7, 19, 31 or
43. Thus, the maximum number of cards is 43.
【Suggested Solution #2】
Assume there are n cards in the poker, where n < 52, since we know that n − 3 is
divisible by 4 and n − 1 is divisible by 3. So the following cases are established:
When n − 3 = 48 , then n − 1 = 50 , which is not divisible by 3!
When n − 3 = 44 ,then n − 1 = 46 , which is not divisible by 3 also!
When n − 3 = 40 , then n − 1 = 42 , divisible by 3
Thus, the largest possible number of cards must be n = 43.
【Suggested Solution #3】
Assume there are n cards in the poker, where n < 52, then n must be a multiple of 3
when it is increased by 3, likewise n is also divisible by 3 when it is increased by 1.
From the statement that n must be a multiple of 3 when it is increased by 3, we can
also conclude that n is a multiple of 12 when increased by 3, 7 or 11.
From the statement that n is also divisible by 3 when it is increased by 1, we also
conclude that n is a multiple of 12when increased by 1, 4, 7 or 10.
Hence, we know that n must be a multiple of 12when increased by 7, that is the
possible value of n are 7, 12+7=19, 24+7=31 or 36+7=43.。Therefore, the maximum
number is n = 43.
Answer: 043

47
22. The diagram shows a 4 cm by 4 cm piece of paper overlapping a 3 cm by 5 cm
piece of paper. By how many cm2 does the area of the non-overlapped part of the
square piece of paper exceeds the area of the non-overlapped part of the
rectangular piece of paper?

C
B
A

【Suggested Solution】
The given figure shows that the square is composed of regions A and B, while the
rectangle is composed of regions B and C. In order to find the difference of the two
non-overlapping regions is the same as determining the difference of area of square
and area of a rectangle. Hence, the area of the difference of the two non-overlapping
regions is 4 × 4-5 × 3 = 1 cm2.
Answer: 001
23. For the class photo of 42 students, the photo shop charges 10 dollars for the first
copy and 3 dollars for each additional copy. Moreover, a bonus of 2 copies is
given for an order exceeding 30 copies. If each student gets one copy, how much
must they pay the photo shop altogether?
【Suggested Solution】
From the given information, we know the printing of first copy of photo costs $10
and 2 copies are given free if printing is more than 30 copies. Hence, the class shall
ask the photo shop to develop one copy plus 42-1-2 = 39 copies, so the total
payment is 10 + 39 × 3 or $127.
Answer: 127
24. The diagram shows 7 flowers printed on a piece of paper. What is the smallest
number of straight lines we must draw to divide the piece of paper into several
regions, so that each flower is in a different region?

【Suggested Solution】
We may think that 2 straight lines will divide the piece of paper into the four regions,
but each flower is not necessary on a separate region! By drawing 3 straight lines as
in the diagram will split each flower in a separate regions.
Answer: 003

48
25. The six faces of a cubical die are labeled with six different positive integers. If
the numbers on any two adjacent faces differ by at least 2, what is the minimum
value of the sum of these six numbers?

【Suggested Solution】
In order the sum of all the numbers in the six surfaces to be a minimum, then each
of the six sides must be 1, otherwise the number appear in each face must reduced by
1, then the sum will also decreasing. Similarly, in a standard cube, the opposite side
of a number 1 must be the number 2, or each number on each face (except the face
with number 1) must decrease by 1 also, then the sum will become smaller. We
know the remaining four faces which are adjacent to number 2, we can predict the
minimum sum will be 2 + 2 = 4, if the sum is more than 4, then each number on the
four sides must each decreased by 1, so that the total will also reduced. We can now
entered in the opposite face of 4 by the number 5, then the remaining two faces which
are adjacent with 5, so that the minimum sum of numbers in those two faces must be
5 +2 = 7, if more than 7, then each number on the two sides must each decreased by 1,
so that the total will also reduced. Now we can entered in 8 in the opposite face of
the number 7, at this time we have the minimum sum of those six faces, where these
six numbers 1, 2, 4, 5, 7 and 8. Therefore, the sum of the six faces of the minimum
number is 1 +2 +4 +5 +7 +8 = 27.
Answer: 027

49
注意:

允許學生個人、非營利性的圖書館或公立學校合理使用
本基金會網站所提供之各項試題及其解答。可直接下載
而不須申請。

重版、系統地複製或大量重製這些資料的任何部分,必
須獲得財團法人臺北市九章數學教育基金會的授權許
可。

申請此項授權請電郵 ccmp@seed.net.tw

Notice:

Individual students, nonprofit libraries, or schools are


permitted to make fair use of the papers and its
solutions. Republication, systematic copying, or
multiple reproduction of any part of this material is
permitted only under license from the Chiuchang
Mathematics Foundation.
Requests for such permission should be made by
e-mailing Mr. Wen-Hsien SUN ccmp@seed.net.tw

50
3rd International Mathematics Assessments for Schools
(2013-2014 )

Middle Primary Division Round 2


Time: 120 minutes

Printed Name;! ! Code; ! Score;! !

Instructions:!
z Do not open the contest booklet until you are told to do so.
z Be sure that your name and code are written on the space provided above.
z Round 2 of IMAS is composed of three parts; the total score is 100 marks.
z Questions 1 to 5 are given as a multiple-choice test. Each question has five
possible options marked as A, B, C, D and E. Only one of these options is correct.
After making your choice, fill in the appropriate letter in the space provided. Each
correct answer is worth 4 marks. There is no penalty for an incorrect answer.
z Questions 6 to 13 are a short answer test. Only Arabic numerals are accepted;
using other written text will not be honored or credited. Some questions have
more than one answer, as such all answers are required to be written down in the
space provided to obtain full marks. Each correct answer is worth 5 marks. There
is no penalty for incorrect answers.
z Questions 14 and 15 require a detailed solution or process in which 20 marks are
to be awarded to a completely written solution. Partial marks may be given to an
incomplete presentation. There is no penalty for an incorrect answer.
z Use of electronic computing devices is not allowed.
z Only pencil, blue or black ball-pens may be used to write your solution or answer.
z Diagrams are not drawn to scale. They are intended as aids only.
z After the contest the invigilator will collect the contest paper.
The following area is to be filled in by the judges;
the contestants are not supposed to mark anything here.
Total
Question 1 2 3 4 5 6 7 8 9 10 11 12 13 14 15 Score Signature

Score

Score

51
52
Middle Primary Division Round 2
Questions 1 to 5, 4 marks each
1. The diagram shows an aquarium containing five starfish, each occupying a
labelled compartment. Water is pumped into the aquarium through the pipe on
the left side. Which compartment is the first to be flooded with water?
Top

A B

C D
E

Bottom
(A)A (B)B (C)C (D)D (E)E

Answer:

2. When 10101 is subtracted from 10000000, how many times does the digit 9
appear in the difference?
(A)3 (B)4 (C)5 (D)6 (E)7

Answer:

3. What is the sum of 32 copies of 1000, 19 copies of 100 and 29 copies of 10?
(A)3219290 (B)321929 (C)342190 (D)34190 (E)32129

Answer:

4. The diagram shows a 6 × 8 chessboard with


→ →
squares painted in black and white in an unusual
pattern. Starting from the top left corner, a
marker must move between squares which have
opposite colours and share a common border.
What is the minimum number of black squares
it must visit in order to arrive at the top right
corner, counting it as one of the black squares visited?
(A)3 (B)8 (C)9 (D)10 (E)11
Answer:

53
MP 2
5. Max gives 27 apples to a group of friends. The numbers of apples they receive
are consecutive positive integers. What is the maximum size of this group?
(A)2 (B)3 (C)4 (D)5 (E)6

Answer:

Questions 6 to 13, 5 marks each


6. When the digits 0, 1, 2, 5, 6, 8 and 9 are rotated 180°, they become 0, 1, 2, 5, 9, 8
and 6 respectively. What does 9105 become when the four-digit number is
rotated 180°?

Answer:

7. An ant by itself is unable to drag a slice of bread back to the anthill. So summons
9 other ants to help, but the slice is still too heavy. So each of these 10 ants
summons 9 other ants to help, and they manage to drag the slice back to the
anthill. How many ants are involved?

Answer: ants

8. Lily has 100 chocolates. She eats one on the first day. Each day after, she eats
twice as many as the day before, until all the chocolates have been eaten. How
many chocolates did she eat on the last day?

Answer:

9. A class is putting up 10 rectangular posters of the same shape and size on a wall.
Each poster must be held in place by one nail near each corner. Adjacent posters
may overlap slightly so that the same nail can serve to hold both of them. The
diagram shows how 9 nails can hold four posters adjacent diagonally. What is the
minimum number of nails required to hold all 10 posters?

Answer: nails

54
MP 3
10. The diagram shows seven marked points, six on a semicircular arc, including
both endpoints of the diameter, along with the centre of the arc. How many
triangles are there whose vertices are all chosen from these points?

Answer: triangles

11. The diagram shows an addition of a three-digit number, a two-digit number and a
one-digit number, with a three-digit sum. The same letter stands for the same
digit and different letters stand for different digits. A question mark can stand for
any digit, including those represented by a letter. What is the maximum value of
the sum?
X Y Z
Y Z
+ Z
? ? ?

Answer:

12. Leon uses a code to convert a letter string consisting only of As, Bs and Cs, into a
number string consisting only of 0s and 1s, by replacing A with 101, B with 11
and C with 0. If the number string obtained is 110101101110101, what is the
number of letters in the original letter string?

Answer: letters

13. The total number of players on three badminton teams is 29. No two players on
the same team play against each other, while every two players on different
teams play each other exactly once. What is the maximum number of games
played?

Answer: games

55
MP 4
Questions 14 to 15, 20 marks each
(Detailed solutions are needed for these two problems)
14. Some of the squares in the 6 × 6 table are shaded. The numbers of shaded squares
in the respective rows and columns are indicated on the edge of the table, and
there are no gaps between the shaded squares in any row or column. Show where
the shaded squares are.

1
5
5
4
2
2
2 3 6 5 2 1

1
5
5
4
2
2
2 3 6 5 2 1
Answer:

56
MP 5
15. A three-digit number is 13 times the product of its digits. The hundreds digit is
larger than either of the other two digits. What is this number?

Answer:

57
58
注意:

允許學生個人、非營利性的圖書館或公立學校合理使用
本基金會網站所提供之各項試題及其解答。可直接下載
而不須申請。

重版、系統地複製或大量重製這些資料的任何部分,必
須獲得財團法人臺北市九章數學教育基金會的授權許
可。

申請此項授權請電郵 ccmp@seed.net.tw

Notice:

Individual students, nonprofit libraries, or schools are


permitted to make fair use of the papers and its
solutions. Republication, systematic copying, or
multiple reproduction of any part of this material is
permitted only under license from the Chiuchang
Mathematics Foundation.
Requests for such permission should be made by
e-mailing Mr. Wen-Hsien SUN ccmp@seed.net.tw

59
2013 Middle Primary Division Second Round Solution
1. The diagram shows an aquarium containing five starfish, each occupying a
labelled compartment. Water is pumped into the aquarium through the pipe on
the left side. Which compartment is the first to be flooded with water?
Top

A B

C D
E

Bottom
(A)A (B)B (C)C (D)D (E)E
【Solution】
The diagram shows the moment when the first compartment is about to be flooded
with water. The answer is (C).

A B

C D
E

Answer:(C)
2. When 10101 is subtracted from 10000000, how many times does the digit 9
appear in the difference?
(A)3 (B)4 (C)5 (D)6 (E)7
【Solution】
In the difference 10000000 − 10101 = 9989899 , the digit 9 appears 5 times. The
answer is (C).
Answer:(C)
3. What is the sum of 32 copies of 1000, 19 copies of 100 and 29 copies of 10?
(A)3219290 (B)321929 (C)342190 (D)34190 (E)32129
【Solution】
We have 32 × 1000 + 19 × 100 + 29 × 10 = 32000 + 1900 + 290 = 34190 . The answer is
(D).
Answer:(D)

60
4. The diagram shows a 6 × 8 chessboard with → →
squares painted in black and white in an unusual
pattern. Starting from the top left corner, a
marker must move between squares which have
opposite colours and share a common border.
What is the minimum number of black squares
it must visit in order to arrive at the top right
corner, counting it as one of the black squares visited?
(A)3 (B)8 (C)9 (D)10 (E)11
【Solution】
The diagram shows a main path which takes the marker over 9 black squares. There
are two detours near the bottom left corner, but each involves increasing the number
of black squares visited, and are ignored. There are also two places with alternative
routes near the bottom right corner, and in each case, either can be taken without
affecting the number of black squares visited. The answer is (C).
→ →

Answer:(C)
5. Max gives 27 apples to a group of friends. The numbers of apples they receive
are consecutive positive integers. What is the maximum size of this group?
(A)2 (B)3 (C)4 (D)5 (E)6
【Solution 1】
A sum of consecutive positive integers may be represented geometrically as a
staircase. Two identical staircases can be put together to form a rectangle. If the sum
of the consecutive integers is 27, the area of the rectangle is 54. Not counting the 1 ×
54 rectangle, there are three others with integral dimensions, namely, 2 × 27, 3 × 18
and 6 × 9. They are shown in the diagram, partitioned into two identical staircases.
The sums they generated are 13+14, 8+9+10 and 2+3+4+5+6+7. The answer is (E).
13 14

14 13
2 7
8 10

10 8
7 2
61
【Solution 2】
There are three ways of expressing 27 as a sum of consecutive positive integers,
namely, 27 = 13+14, 27 = 9+9+9 = 8+9+10 and 27 = 9+9+9 = (4+5)+(3+6)+(2+7) =
2+3+4+5+6+7. It follows that 27 can be expressed as a sum of at most 6 consecutive
positive integers. The answer is (E).
Answer:(E)
6. When the digits 0, 1, 2, 5, 6, 8 and 9 are rotated 180°, they become 0, 1, 2, 5, 9, 8
and 6 respectively. What does 9105 become when the four-digit number is
rotated 180°?
【Solution】
The last digit 5 of the given number becomes the first digit 5 of the number we seek.
The other three digits, namely, 0, 1 and 9, become 0, 1 and 6 respectively.
Hence the number we seek is 5016.
Answer:5016
7. An ant by itself is unable to drag a slice of bread back to the anthill. So summons
9 other ants to help, but the slice is still too heavy. So each of these 10 ants
summons 9 other ants to help, and they manage to drag the slice back to the
anthill. How many ants are involved?
【Solution】
The total number of ants involved is (1 + 9) + 10 × 9 = 100 .
Answer:100 ants
8. Lily has 100 chocolates. She eats one on the first day. Each day after, she eats
twice as many as the day before, until all the chocolates have been eaten. How
many chocolates did she eat on the last day?
【Solution】
During the first six days, Lily eats respectively 1, 2, 4, 8, 16 and 32 chocolates, Since
1+2+4+8+16+32=64-1=63, 100-63=37 chocolates are left, which is less than 64.
Hence the number of chocolates Lily eats on the last day is 37.
Answer:37
9. A class is putting up 10 rectangular posters of the same
shape and size on a wall. Each poster must be held in place
by one nail near each corner. Adjacent posters may overlap
slightly so that the same nail can serve to hold both of
them. The diagram shows how 9 nails can hold four
posters adjacent diagonally. What is the minimum number
of nails required to hold all 10 posters?
【Solution 1】
Clearly, the 10 posters should be arranged in a rectangular array, and there are four
ways to do so.
For a 1 × 10 array, the number of nails required is (1 + 1) × (10 + 1) = 22 .
For a 2 × 5 array, the number of nails required is ( 2 + 1) × (5 + 1) = 18 .
Hence the minimum number of nails required is 18.

62
【Solution 2】
Clearly, the 10 posters should be arranged in a rectangular array, and there are four
ways to do so. We wish to maximize the number of four-way common corners.
For a 1 × 10 array, there are (1 − 1) × (10 − 1) = 0 such corners.
For a 2 × 5 array, there are (2 − 1) × (5 − 1) = 4 such corners.
Hence the minimum number of nails required is 4 + 2 × (2 + 5) = 18 .
Answer:18 nails
10. The diagram shows seven marked points, six on a
semicircular arc, including both endpoints of the
diameter, along with the centre of the arc. How many
triangles are there whose vertices are all chosen from
these points?
【Solution1】
If we do not take any of the three points on the diameter as vertices, there are 4 such
triangles.
If we take only one point on the diameter as a vertex, this can be chosen in 3 ways.
The other two vertices can be chosen in 6 ways. The number of triangles in this case
is 3 × 6 = 18.
If we take two points on the diameter as vertices, they can be chosen in 3 ways. The
other vertex can be chosen in 4 ways. The number of triangles in this case is 3×4=12.
The total number is 4+18+12=34.
【Solution2】
The first vertex can be chosen in 7 ways, the second one in 6 ways and the third 5
ways, for a total of 210. However, the same triangle can arise from choosing the
vertices in different orders. Each triangle arises 3 × 2 × 1 = 6 times, so that the total
number is reduced to 210 ÷ 6 = 35. From this, we must still subtract the one in which
all three chosen vertices are on the diameter. Hence the final count is 35 − 1=34.
Answer:34 triangles
11. The diagram shows an addition of a three-digit number, a X Y Z
two-digit number and a one-digit number, with a three-digit sum.
The same letter stands for the same digit and different letters Y Z
stand for different digits. A question mark can stand for any digit, + Z
including those represented by a letter. What is the maximum ? ? ?
value of the sum?
【Solution】
Since the sum is a three-digit number, it is at most 999. If it is 999, then Z = 3, but we
cannot have 2Y = 9. The next largest possible sum is 998. Here we have Z = 6, Y = 4,
X = 9, with 946+46+6=998.
Answer:998
12. Leon uses a code to convert a letter string consisting only of As, Bs and Cs, into a
number string consisting only of 0s and 1s, by replacing A with 101, B with 11
and C with 0. If the number string obtained is 110101101110101, what is the
number of letters in the original letter string?

63
【Solution】
If the first number of a string is 0, it must stand for C. If it is a 1, and the next letter is
another 1, the two 1s must stand for B, If the next letter is 0, then the third letter must
be 1 and these three letters must stand for A. Thus the number string can be read in a
unique way. In particular, 11-0-101-101-11-0-101 must stand for BCAABCA , and
the number of letters is 7.
Answer:7 letters
13. The total number of players on three badminton teams is 29. No two players on
the same team play against each other, while every two players on different teams
play each other exactly once. What is the maximum number of games played?
【Solution】
Suppose team A has at least two more players than team B. Transfer one player X
from team A to team B. Before the transfer, X plays every player on team B. After the
transfer, X plays every player left on team A, which is still more than the number of
players originally in team B. So the total number of games played has increased. It
follows that to maximize the total number of games played, the size of the teams
should not differ by more than 1. Hence we should have 10 players in two of the
teams and 9 players in the third team.
The total number of games is then 10 × 10 + 10 × 9 + 10 × 9=280.
Answer:280 games
14. Some of the squares in the 6 × 6 table are a b c d e f
shaded. The numbers of shaded squares in the
respective rows and columns are indicated on 1 6
the edge of the table, and there are no gaps 5 5
between the shaded squares in any row or 5 4
column. Show where the shaded squares are.
【Solution】 4 3
We label the rows 1 to 6 from bottom to top and the 2 2
columns a to f from left to right. Obviously, all of
2 1
c1, c2, c3, c4, c5 and c6 must be shaded, and this
means that a6, b6, d6, e6 and f 6 are not shaded. 2 3 6 5 2 1
(5 points)

a b c d e f
1 6
5 5
5 4
4 3
2 2
2 1
2 3 6 5 2 1

64
Hence all of d1, d2, d3, d4 and d5 must be shaded. This means that a1, b1, e1, f 1, a2,
b2, e2 and f 2 are not shaded. (5 points)

a b c d e f
1 6
5 5
5 4
4 3
2 2
2 1
2 3 6 5 2 1

This forces the shading of b3, b4 and b5.

a b c d e f
1 6
5 5
5 4
4 3
2 2
2 1
2 3 6 5 2 1

If the last shaded square in row 3 is f 3, it will be isolated. If it is e3, then a4 and a5
must be shaded. However, the lone square in column f must have a shaded square in
the same row in column e, and that row will have too many shaded squares. (5 points)

a b c d e f
1 6
5 5
5 4
4 3
2 2
2 1
2 3 6 5 2 1

65
It follows that a3 must be shaded, as well as e4 and e5.

a b c d e f
1 6
5 5
5 4
4 3
2 2
2 1
2 3 6 5 2 1

The last two shaded squares are a4 and f 5. (5 points)


a b c d e f
1 6
5 5
5 4
4 3
2 2
2 1
2 3 6 5 2 1

15. A three-digit number is 13 times the product of its digits. The hundreds digit is
larger than either of the other two digits. What is this number?
【Solution】
Since the three-digit number is equal to 13 times the product of its digits, it is
divisible by its hundreds digit, which is the largest of the three. If we increase both
the tens digit and the units digit to the hundreds digit, the quotient will be 111. If
instead we decrease both of them to 0, the quotient will be 100.
The actual quotient is between 100 and 111(5 points), and since it is divisible by 13, it
must be 104(5 points). The product of the tens digit and the units digit is 104 ÷ 13=8.
Since 8=2 × 4=1× 8, the three-digit number is one of 624, 642, 918 and 981(5 points).
The only one divisible by 13 is 13× 6× 2× 4= 624. The number is 624. (5 points)
Answer:624

66
注意:

允許學生個人、非營利性的圖書館或公立學校合理使用
本基金會網站所提供之各項試題及其解答。可直接下載
而不須申請。

重版、系統地複製或大量重製這些資料的任何部分,必
須獲得財團法人臺北市九章數學教育基金會的授權許
可。

申請此項授權請電郵 ccmp@seed.net.tw

Notice:

Individual students, nonprofit libraries, or schools are


permitted to make fair use of the papers and its
solutions. Republication, systematic copying, or
multiple reproduction of any part of this material is
permitted only under license from the Chiuchang
Mathematics Foundation.
Requests for such permission should be made by
e-mailing Mr. Wen-Hsien SUN ccmp@seed.net.tw

67
International Mathematics Assessments for Schools

2014 MIDDLE PRIMARY DIVISION FIRST ROUND PAPER


Time allowed:75 minutes

INSTRUCTION AND INFORMATION


GENERAL
1. Do not open the booklet until told to do so by your teacher.
2. No calculators, slide rules, log tables, math stencils, mobile phones or other
calculating aids are permitted. Scribbling paper, graph paper, ruler and compasses
are permitted, but are not essential.
3. Diagrams are NOT drawn to scale. They are intended only as aids.
4. There are 20 multiple-choice questions, each with 5 choices. Choose the most
reasonable answer. The last 5 questions require whole number answers between
000 and 999 inclusive. The questions generally get harder as you work through the
paper. There is no penalty for an incorrect response.
5. This is a mathematics assessment, not a test; do not expect to answer all questions.
6. Read the instructions on the answer sheet carefully. Ensure your name, school
name and school year are filled in. It is your responsibility that the Answer Sheet
is correctly coded.
7. When your teacher gives the signal, begin working on the problems.
THE ANSWER SHEET
1. Use only lead pencils.
2. Record your answers on the reverse side of the Answer Sheet (not on the question
paper) by FULLY filling in the circles which correspond to your choices.
3. Your Answer Sheet will be read by a machine. The machine will see all markings
even if they are in the wrong places. So please be careful not to doodle or write
anything extra on the Answer Sheet. If you want to change an answer or remove
any marks, use a plastic eraser and be sure to remove all marks and smudges.
INTEGRITY OF THE COMPETITION
The IMAS reserves the right to re-examine students before deciding whether to
grant official status to their scores.

68
2014 MIDDLE PRIMARY DIVISION FIRST ROUND PAPER

Questions 1-10, 3 marks each


1. What is the value of 2 + 0 + 1 + 4 + 2 × 0 × 1 × 4?
(A)0 (B)5 (C)7 (D)9 (E)15

2. Which of the following numbers is the smallest?


(A)298 (B)312 (C)231 (D)357 (E)101

3. Which of the following polygons has the greatest number of sides?


(A) (B) (C) (D) (E)

4. The word “2014IMAS” appears on the screen. After each minute, the leftmost
character moves over to become the rightmost character. How many minutes will
elapse before the word “2014IMAS” appears on the screen once again?
(A)5 (B)6 (C)7 (D)8 (E)9

5. The side length of each hexagon in the diagram is 1 cm. What is the perimeter, in
cm, of the figure formed from these hexagons?

(A)18 (B)20 (C)22 (D)24 (E)26

6. Thirty students numbered from 1 to 30 stand in a row. The teacher announces,


“Will those numbered from 1 to 10 inclusive take one step forward, and those
numbers 20 to 30 inclusive take one step backward.” How many students remain
in place?
(A)9 (B)10 (C)11 (D)20 (E)21

69
MP 2
7. Max throws four darts at the target shown in the diagram. All four darts hit the
target, each scoring a different number of points. What is the minimum number
of points Max has scored?
1
3
5
7
9
7
5
3
1

(A)4 (B)10 (C)16 (D)20 (E)24

8. Which of the polygonal board below cannot be obtained from a rectangular board
after one straight cut?
(A) (B) (C) (D) (E)

9. Benches are provided for children watching a movie. Each bench can seat 3
children. What is the minimum number of benches required to seat 25 children?
(A)7 (B)8 (C)9 (D)10 (E)11

10. A large bottle of apple juice costs 6.5 dollars while a small bottle of apple juice
costs 2.8 dollars. How many dollars less is the cost of a large bottle compared to
the total cost of three small bottles?

Item: Apple juice Item: Apple juice


Volume: 1500 mL Volume: 500 mL
Price: $ 6.50 Price: $ 2.80

(A)1.9 (B)2.1 (C)2.3 (D)2.8 (E)3.7

70
MP 3

Questions 11-20, 4 marks each


11. A shell may be traded in for two baskets of fruit or three baskets of vegetable.
Which of the following may not be obtained by trading in at most two shells?

(A)two baskets of fruit and three baskets of vegetable


(B)six baskets of vegetable
(C)two baskets of fruit
(D)three baskets of fruit and two baskets of vegetable
(E)three baskets of vegetable

12. Lana is a student in Grade 4. Which of the following is the closest approximation
to her age?
(A)120 hours (B)120 days (C)120 weeks
(D)120 months (E)120 years

13. There are 2 red balls, 6 yellow balls and 10 blue balls in a box. One ball is drawn
at random. Which of the following statement is correct?

2 red balls
6 yellow balls
10 blue balls

(A)The probability of drawing a blue ball is the lowest.


(B)It is equally likely to draw a ball of any colour.
(C)The probability of drawing a yellow ball is the highest.
(D)It is less likely to draw a yellow ball than a blue ball.
(E)The probability of drawing a yellow ball is the lowest.

71
MP 4
14. Whenever the hour hand and the minute hand of a clock coincide, the number of
germs in a dish increases by 10. Between 1:30 pm and 6:30 pm on the same day,
by how many has the number of germs increased?

(A)20 (B)30 (C)40 (D)50 (E)60

15. The diagram shows a cubical die moving on a 1 by 8 board by tilting over an
edge. The number on the face touching the board is imprinted on that square of
the board. The numbers in the first four squares are 4, 1, 2 and 5. What is the
total of all eight numbers?
1
4
3
4 1 2 5

(A)21 (B)22 (C)23 (D)24 (E)25

16. The two stars in the diagram represent the same number. The sum of the three
numbers in the second row is equal to twice the sum of the three numbers in the
first row. What number does each star represent?

5 6 ☆

☆ 19 20

(A)7 (B)8 (C)13 (D)17 (E)18

17. A workman is moving 40 panes of glass. He gets 2 dollars for each pane.
However, if he breaks one pane, he will have to pay 8 dollars instead of getting 2
dollars. If his total pay is 60 dollars, how many panes has he broken?
(A)1 (B)2 (C)3 (D)4 (E)5

18. Some children stand in a line and call out the numbers 1, 2 and 3 in cyclic order,
starting with 1. If the last child calls out 2, which of the following number can be
the number of children in the line?
(A)24 (B)25 (C)26 (D)27 (E)28

72
MP 5
19. Max has a red box which contains 6 blue boxes. Each blue box contains 4 green
boxes. How many boxes does Max have in total?
(A)10 (B)11 (C)24 (D)25 (E)31

20. The diagram shows a figure obtained by putting together 11 squares of the same
size. If the perimeter of the figure is 48 cm, what is the area, in cm2, of the
figure?

(A)11 (B)22 (C)33 (D)44 (E)48

Questions 21-25, 6 marks each


21. Some students are lined up in a rectangular array. Max is the 8th student in his
column from the front, and the 13th from the back. There are 15 students to his
left and 15 students to his right in the same row. How many students are in the
line-up?

22. The area of a triangle is 50 cm2. Each side is divided into five equal parts, and
some pairs of division points are joined as shown in the diagram. What is the
total area, in cm2 of the shaded regions?

23. Oliver arranges his toy ducks and toy turtles in a row as shown in the diagram.
He wishes to have all the toy ducks on the left and all the toy turtles on the right.
He may switch the position of any two adjacent toys. What is the minimum
number of switches he will require?

73
MP 6
24. Every digit of a seven-digit multiple of 3 is 0, 2 or 3, and there are more 3’s than
0’s. If each of 0, 2 and 3 appears at least once, what is the sum of all seven
digits?

25. When different buttons are pressed, a robot may move forward 1 cm, 3 cm or 5
cm. If buttons are pressed six times, how many different distances may the robot
have moved?

***

74
75
注意:

允許學生個人、非營利性的圖書館或公立學校合理使用
本基金會網站所提供之各項試題及其解答。可直接下載
而不須申請。

重版、系統地複製或大量重製這些資料的任何部分,必
須獲得財團法人臺北市九章數學教育基金會的授權許
可。

申請此項授權請電郵 ccmp@seed.net.tw

Notice:

Individual students, nonprofit libraries, or schools are


permitted to make fair use of the papers and its
solutions. Republication, systematic copying, or
multiple reproduction of any part of this material is
permitted only under license from the Chiuchang
Mathematics Foundation.
Requests for such permission should be made by
e-mailing Mr. Wen-Hsien SUN ccmp@seed.net.tw

76
Solution to
Fourth International Mathematics Assessment for Schools
Round 1 of Middle Division

1. What is the value of 2 + 0 + 1 + 4 + 2 × 0 × 1 × 4?


(A)0 (B)5 (C)7 (D)9 (E)15
【Suggested Solution】
2 + 0 + 1 + 4 + 2 × 0 × 1 × 4 = 7. Hence, we select (C).
Answer: (C)
2. Which of the following numbers is the smallest?
(A)298 (B)312 (C)231 (D)357 (E)101
【Suggested Solution】
Since 101 < 231 < 298 < 312 < 357, then the smallest number is 101. So, we select
(E).
Answer: (E)
3. Which of the following polygons has the greatest number of sides?
(A) (B) (C) (D) (E)

【Suggested Solution】
The polygon in option (A) has 4 sides, the polygon in option (B) has 12 sides, the
polygon in option (C) has 9 sides, the polygon in option (D) has 10 sides while the
polygon in option (E) has 10 sides. Hence, since the polygon in option (B) has the
greatest number of sides, so we select (B).
Answer: (B)
4. The word “2014IMAS” appears on the screen. After each minute, the leftmost
character moves over to become the rightmost character. How many minutes will
elapse before the word “2014IMAS” appears on the screen once again?
(A)5 (B)6 (C)7 (D)8 (E)9
【Suggested Solution】
There are eight characters in this word. It takes 8 minutes for the word "2014IMAS"
to reappear. So, we select (D).
Answer:(D)
5. The side length of each hexagon in the diagram is 1 cm.
What is the perimeter, in cm, of the figure formed from
these hexagons?
(A)18 (B)20 (C)22
(D)24 (E)26

77
【Suggested Solution】
The given diagram has 26 line segment of 1 cm long, so its perimeter is 26 cm. Hence,
we select (E).
Answer: (E)
【Note】By symmetry property of a diagram, We just need to count the number of
sides of the upper half and then be multiplied by 2.
6. Thirty students numbered from 1 to 30 stand in a row. The teacher announces,
“Will those numbered from 1 to 10 inclusive take one step forward, and those
numbers 20 to 30 inclusive take one step backward.” How many students remain
in place?
(A)9 (B)10 (C)11 (D)20 (E)21
【Suggested Solution】
We know that students with numbered 11 to 19 will not move one step forward or
take one step backward, so there are a total of 19-11 + 1 = 9 students, not moving,
remain in their original position. We select (A).
Answer: (A)
7. Max throws four darts at the target shown in the 1
diagram. All four darts hit the target, each scoring a 3
different number of points. What is the minimum 5
number of points Max has scored? 7
(A)4 (B)10 (C)16 9
(D)20 (E)24
7
【Suggested Solution】 5
From the given information, there is one target that Max 3
did not hit, so the minimum number of points Max has 1
scored is 1 + 3 + 5 + 7 = 16 points. Hence, the answer is (C).
Answer:(C)
8. Which of the polygonal board below cannot be obtained from a rectangular board
after one straight cut?
(A) (B) (C) (D) (E)

【Suggested Solution】
If a saw was able to cut a rectangular board thru the two non-consecutive vertices,
then one piece that was cut will be in the shape same as option (A). If a saw was
able to cut a rectangular board thru anyone of its vertices, then the shape in one piece
may either be same as that figure of option (A), option (B) or option (D).
If a saw will not cut a rectangular board thru anyone of its vertices, then one piece
that was cut will either be in the shape the same as that of option (A), option (B),
option (C) or option (D).
Hence, only the shape same as option (E) will never occur. So we select option (E).
Answer: (E)

78
9. Benches are provided for children watching a movie. Each bench can seat 3
children. What is the minimum number of benches required to seat 25 children?
(A)7 (B)8 (C)9 (D)10 (E)11
【Suggested Solution】
We know that 25 = 8 × 3 + 1, hence if there are 8 available benches, then at most 24
children will sit and 1 child cannot sit on any bench. Thus, we need at least 9 benches.
So, we select option (C).
Answer: (C)
10. A large bottle of apple juice costs 6.5 dollars while a small bottle of apple juice
costs 2.8 dollars. How many dollars less is the cost of a large bottle compared to
the total cost of three small bottles?

Item: Apple juice Item: Apple juice


Volume: 1500 mL Volume: 500 mL
Price: $ 6.50 Price: $ 2.80

(A)1.9 (B)2.1 (C)2.3 (D)2.8 (E)3.7


【Suggested Solution】
The cost of 3 small bottles of apple juice is $2.8 × 3 = $8.4. Hence buying 1 large
bottle of apple juice will be less than buying 3 small bottle of apple juices by $8.4-
$6.5 = $1.9. Therefore, we select option (A).
Answer: (A)

11. A shell may be traded in for two baskets of fruit or three baskets of vegetable.
Which of the following may not be obtained by trading in at most two shells?

(A)two baskets of fruit and three baskets of vegetable


(B)six baskets of vegetable
(C)two baskets of fruit
(D)three baskets of fruit and two baskets of vegetable
(E)three baskets of vegetable

79
【Suggested Solution】
We can use 2 shells to trade in the foods of option (A) or option (B), we can also use
1 shell to trade in the foods of option (C) or option (E), because it is not possible to
trade in 3 baskets of fruits, hence option (D) is unobtainable.
Answer: (D)
12. Lana is a student in Grade 4. Which of the following is the closest approximation
to her age?
(A)120 hours (B)120 days (C)120 weeks
(D)120 months (E)120 years
【Suggested Solution】
Since 120 hours < 120 days < 120 weeks < 3 years, the time in the first three
expression each less than 3 years, which is not practical to be the age of a grade 4
student. Option (E) is obviously cannot be the age for a grade 4 pupil. But 120
months = 10 years, which is more realistic. So, we select option (D).
Answer: (D)
13. There are 2 red balls, 6 yellow balls and 10 blue balls in a box. One ball is drawn
at random. Which of the following statement is correct?

2 red balls
6 yellow balls
10 blue balls

(A)The probability of drawing a blue ball is the lowest.


(B)It is equally likely to draw a ball of any colour.
(C)The probability of drawing a yellow ball is the highest.
(D)It is less likely to draw a yellow ball than a blue ball.
(E)The probability of drawing a yellow ball is the lowest.
【Suggested Solution】
Since the number of red balls inside the box is the least, then the probability of
drawing red ball is the smallest. We also note that the number of blue balls inside is
more than the number of red or number yellow balls, so the probability of drawing
blue ball is higher than any other ball. Because the number of yellow balls inside the
box is less than the number of blue balls, then the probability of drawing yellow ball
is lower than blue ball. Hence, we select option (D).
Answer: (D)
14. Whenever the hour hand and the minute hand of a
clock coincide, the number of germs in a dish
increases by 10. Between 1:30 pm and 6:30 pm on
the same day, by how many has the number of
germs increased?
(A)20 (B)30 (C)40 (D)50 (E)60

80
【Suggested Solution】
We know that the minute hand and the hour hand coincide once in every hour from 2
pm to 6 pm. The next time the two hands will be coincide must be after 6:30 pm.
Thus, from 1:30 pm to 6:30 pm, the two hands coincide 4 times. Hence the number of
germs increase by 40. So, we select option (C).
Answer:(C)
15. The diagram shows a cubical die moving on a 1 by 8 board by tilting over an
edge. The number on the face touching the board is imprinted on that square of
the board. The numbers in the first four squares are 4, 1, 2 and 5. What is the
total of all eight numbers?

1
4
3
4 1 2 5

(A)21 (B)22 (C)23 (D)24 (E)25


【Suggested Solution】
When rolling the cubical die on the tilted position, the following four numbers have
been seen keep repeating: 4, 1, 2 and 5, then total of all eight numbers is 4 + 1 + 2 + 5
+ 4 + 1 + 2 + 5 = 24. Hence, we select option (D).
Answer: (D)
16. The two stars in the diagram represent the same number. The sum of the three
numbers in the second row is equal to twice the sum of the three numbers in the
first row. What number does each star represent?

5 6 ☆

☆ 19 20

(A)7 (B)8 (C)13 (D)17 (E)18


【Suggested Solution】
From the given information, it shows the sum of the three numbers in the second row
is equal to twice the sum of the three numbers in the first row, it follows that the
difference of the sum of three numbers in the second row and the sum of three
numbers in the first row equal the sum of three numbers in the first row, that is; the
difference of these two rows is 19 + 20-5-6 = 28, then we have ☆ = 28-5-6 =
17. Thus, we select option (D).
Answer: (D)
17. A workman is moving 40 panes of glass. He gets 2 dollars for each pane.
However, if he breaks one pane, he will have to pay 8 dollars instead of getting 2
dollars. If his total pay is 60 dollars, how many panes has he broken?
(A)1 (B)2 (C)3 (D)4 (E)5

81
【Suggested Solution】
Assume the workman has not broken any glasses, then he gets $2 × 40 = $80, but he
receive only $60, so it means he broke (80-60) ÷ (2 + 8) = 2 pieces of glasses. Thus,
we select (B).
Answer: (B)
18. Some children stand in a line and call out the numbers 1, 2 and 3 in cyclic order,
starting with 1. If the last child calls out 2, which of the following number can be
the number of children in the line?
(A)24 (B)25 (C)26 (D)27 (E)28
【Suggested Solution】
From the given information, we are searching the number from the 5 options so that
when the total number of children divided by 3 will give a remainder of 2. It is only
option (C) from all options meets the requirement of the problem. Hence, we select
option (C).
Answer: (C)
19. Max has a red box which contains 6 blue boxes. Each blue box contains 4 green
boxes. How many boxes does Max have in total?
(A)10 (B)11 (C)24 (D)25 (E)31
【Suggested Solution】
From the given information, there are of 4 × 6 = 24 green boxes, 6 blue boxes and 1
red box. Thus, Max has 24 + 6 + 1 = 31 boxes in all. We must select (E).
Answer: (E)
20. The diagram shows a figure obtained by putting together 11
squares of the same size. If the perimeter of the figure is 48 cm,
what is the area, in cm2, of the figure?
(A)11 (B)22 (C)33
(D)44 (E)48
【Suggested Solution】
From the given diagram, we know the perimeter of given polygon
composed from 24 sides of the small square, using the given information, the side
length of each small square must be 2 cm. It follows that the area of each small
square is 4 cm2, there are 11 squares of the same size, hence the total area is 44 cm2.
Hence, we select option (D).
Answer: (D)
th
21. Some students are lined up in a rectangular array. Max is the 8 student in his
column from the front, and the 13th from the back. There are 15 students to his
left and 15 students to his right in the same row. How many students are in the
line-up?
【Suggested Solution】
From the given information, we know the formation has 8 + 13-1 = 20 rows, 15 +
15 + 1 = 31 columns. Thus, there are 20 × 31 = 620 students in the line-up.
Answer: 620

82
22. The area of a triangle is 50 cm2. Each side is divided into five equal parts, and
some pairs of division points are joined as shown in the diagram. What is the
total area, in cm2 of the shaded regions?

【Suggested Solution】
After connecting the division points by pairs, there will be 25 congruent equilateral
triangles as shown in the diagram at the right. So the area of each small equilateral
triangle is 50 ÷ 25 = 2 cm2. There are 10 small equilateral triangles in the shaded
region, then the total area of the shaded region is 2 × 10 = 20 cm2.
Answer: 020
23. Oliver arranges his toy ducks and toy turtles in a row as shown in the diagram.
He wishes to have all the toy ducks on the left and all the toy turtles on the right.
He may switch the position of any two adjacent toys. What is the minimum
number of switches he will require?

【Suggested Solution】
We know that for each switch of two adjacent toys may make one toy duck move at
most one position to the left, let us name the position of each toy from left to right as
1 to 11. Our main target is to arrange the toys in the manner that all 6 toy ducks will
be on the left side. Since the initial position of 6 toy ducks are 1, 3, 5, 8, 10, 11, then
we need to operate at least (1-1) + (3-2) + (5-3) + (8-4) + (10-5) + (11-6) =
17 times of switches in order to reach our goal. Hence, we must perform switches at
least 17 times. So, we select (C).
Answer: 017
24. Every digit of a seven-digit multiple of 3 is 0, 2 or 3, and there are more 3’s than
0’s. If each of 0, 2 and 3 appears at least once, what is the sum of all seven
digits?
【Suggested Solution】
We know that from the divisibility property of 3 of a certain number is: The sum of
all the digits in the number must be multiple of 3. Since 0 and 3 are both multiple of 3
but digit 2 is not a multiple of 3, if these seven-digit number must be a multiple of 3,
then digit 2 must appear either 3 times or 6 times in the number. If 2 will appear 6
times in the number, then only digit left must be fill in by either 0 or 3, but both digits
must appear at least once. Hence, 2’s must appear 3 times in the number with the
remaining 4 digits as digit 3 appear 3 times and digit 0 appear 1 time. Hence, the sum
of all these seven digits is 2 + 2 + 2 + 3 + 3 + 3 + 0 = 15.
Answer: 015

83
25. When different buttons are pressed, a robot may move
forward 1 cm, 3 cm or 5 cm. If buttons are pressed six
times, how many different distances may the robot have
moved?
【Suggested Solution 1】
When button 1 is pressed, the robot moves forward by 1 cm;
when button 2 is pressed, the robot moves forward by 3 cm and
when button 3 is pressed, the robot moves forward by 5 cm. This implies that the
maximum distance a robot can move forward is 5 cm once one of the buttons is
pressed, let us consider the following cases.
Case 1: When button 3 is not pressed, according to the number of button 2 pressed,
there are 7 different distances the robot can move and they are: 6 cm (pressing the
first button 6 times), 8 cm (pressing the first button 5 times and second button 1 time),
10 cm (pressing the first button 4 times and second button 2 times), 12 cm (pressing
the first button 3 times and second button 3 times), 14 cm (pressing the first button 2
times and second button 4 times), 16 cm (when pressing first button 1 time and
second button 5 times) and 18 cm (when pressing the second button 6 times and not
pressing the first button at all).
Case 2: When button 3 is pressed just once, the remaining 5 presses may either be
button 1 or button 2. according to the number of button 2 pressed, there are 6
different possible distances: 10 cm, 12 cm, 14 cm, 16 cm, 18 cm, 20 cm.
Case 3: When button 3 is pressed two times, the remaining 4 times may be to press
button 1 or button 2. according to the number of button 2 pressed, there are 5
different possible distances: 14 cm, 16 cm, 18 cm, 20 cm, 22 cm.
Case 4: When pressing button 3 three times, similarly, the robot can move forward to
any of these 4 different possible distances: 18 cm, 20 cm, 22 cm, 24 cm.
Case 5: When pressing button 3 four times, similarly, the robot can move forward to
any of these 3 different possible distances: 22 cm, 24 cm, 26 cm.
Case 6: When pressing button 3 five times, similarly, the robot can move forward to
any of these 2 different possible distances: 26 cm, 28 cm.
Case 7: When pressing button 3 six times. The robot can reach the distance of 30 cm.
In summary, the robot can move a distance of 6 cm, 8 cm, 10 cm, 12 cm, 14 cm, 16
cm, 18 cm, 20 cm, 22 cm, 24cm, 26 cm, 28 cm and 30 cm which is a total of 13
different distances.
In summary, the robot can move a distance of 6 cm, 8 cm, 10 cm, 12 cm, 14 cm, 16
cm, 18 cm, 20 cm, 22 cm, 24cm, 26 cm, 28 cm and 30 cm which is a total of 13
different distances.
【Suggested Solution 2】
The robot may move at least 1 × 6 = 6 cm and can reach as far as 5 × 6 = 30 cm.
Since it was given that the robot will always move forward in odd number of cm, so
when the robot move forward 6 steps, then the distance will be an even number in cm.
The possible distance that will be an even number cm between 6 cm to 30 cm are
possible as follow: 6 = 1 + 1 + 1 + 1 + 1 + 1; 8 = 1 + 1 + 1 + 1 + 1 + 3;
10 = 1+ 1 + 1 + 1 + 1 + 5; 12 = 1 + 1 + 1 + 1 + 3 + 5; 14 = 1 + 1 + 1 + 1 + 5 + 5;

84
16 = 1 + 1 + 1 + 3 + 5 + 5; 18 = 1 + 1 + 1 + 5 + 5 + 5; 20 = 1 + 1 + 3 + 5 + 5 + 5;
22 = 1 + 1 + 5 + 5 + 5 + 5; 24 = 1 + 3 + 5 + 5 + 5 + 5; 26 = 1 + 5 + 5 + 5 + 5 + 5;
28 = 3 + 5 + 5 + 5 + 5 + 5; 30 = 5 + 5 + 5 + 5+ 5 + 5.
Answer: 013

85
注意:

允許學生個人、非營利性的圖書館或公立學校合理使用
本基金會網站所提供之各項試題及其解答。可直接下載
而不須申請。

重版、系統地複製或大量重製這些資料的任何部分,必
須獲得財團法人臺北市九章數學教育基金會的授權許
可。

申請此項授權請電郵 ccmp@seed.net.tw

Notice:

Individual students, nonprofit libraries, or schools are


permitted to make fair use of the papers and its
solutions. Republication, systematic copying, or
multiple reproduction of any part of this material is
permitted only under license from the Chiuchang
Mathematics Foundation.
Requests for such permission should be made by
e-mailing Mr. Wen-Hsien SUN ccmp@seed.net.tw

86
4th International Mathematics Assessments for Schools
(2014-2015 )

Middle Primary Division Round 2


Time: 120 minutes

Printed Name: Code: Score:

Instructions:
 Do not open the contest booklet until you are told to do so.
 Be sure that your name and code are written on the space provided above.
 Round 2 of IMAS is composed of three parts; the total score is 100 marks.
 Questions 1 to 5 are given as a multiple-choice test. Each question has five
possible options marked as A, B, C, D and E. Only one of these options is correct.
After making your choice, fill in the appropriate letter in the space provided. Each
correct answer is worth 4 marks. There is no penalty for an incorrect answer.
 Questions 6 to 13 are a short answer test. Only Arabic numerals are accepted;
using other written text will not be honored or credited. Some questions have
more than one answer, as such all answers are required to be written down in the
space provided to obtain full marks. Each correct answer is worth 5 marks. There
is no penalty for incorrect answers.
 Questions 14 and 15 require a detailed solution or process in which 20 marks are
to be awarded to a completely written solution. Partial marks may be given to an
incomplete presentation. There is no penalty for an incorrect answer.
 Use of electronic computing devices is not allowed.
 Only pencil, blue or black ball-pens may be used to write your solution or answer.
 Diagrams are not drawn to scale. They are intended as aids only.
 After the contest the invigilator will collect the contest paper.
The following area is to be filled in by the judges;
the contestants are not supposed to mark anything here.
Total
Question 1 2 3 4 5 6 7 8 9 10 11 12 13 14 15 Score Signature

Score

Score

87
Middle Primary Division Round 2
Questions 1 to 5, 4 marks each
1. What is the sum of the digits of the value of 100 100  2015 ?
(A)27 (B)29 (C)30 (D)34 (E)39

Answer:

2. If 6  2  6  66  72 and 2  3  2  22  222  246 , what is the value of


53?
(A)3735 (B)605 (C)615 (D)625 (E)37035

Answer:

3. In a gymnastic competition, an athlete receives a score from each of seven judges.


After the highest score and the lowest score have been removed, the average of
the remaining five scores is the actual score for that athlete. If the seven judges
give scores of 9.2, 9.5, 9.3, 9.6, 9.1, 9.6 and 9.4 to an athlete, what is the actual
score for this athlete?
9.2 9.5 9.3 9.6 9.1 9.6 9.4

(A)9.3 (B)9.38 (C)9.4 (D)9.42 (E)9.5

Answer:

4. In a shopping mall, a ball is drawn from one of the five boxes shown below. A
door prize is won if the number of the ball is 1. From which box should a ball be
drawn so that the chance of winning a door prize is as large as possible?
(A) (B) (C)

2 3 1 2 1 1 2 1

(D) (E)

2 3 1 1 2 3 4 2

Answer:

88
MP 2
5. Which of the following five figures is not possible to trace without lifting the
pencil from the paper or retracing any part of it?
(A) (B) (C)

(D) (E)

Answer:

Questions 6 to 13, 5 marks each


6. At the bookstore, Lily spends half of her money buying mathematics books and
two-thirds of the remaining amount on Chinese literature books. She has just
enough money left to buy an English literature book which costs $18. How much
money does Lily have initially?

Answer: $

7. There are three kinds of objects, spheres, cylinders and cubes. Three spheres
have the same total weight as two cylinders, and five spheres have the same total
weight as six cubes. How many cubes will have the same total weight as five
cylinders?

Answer: cubes

89
MP 3
8. In a video game, 1 point is awarded for eating the first apple, 2 points for eating
the second apple, and so on, with 1 additional point awarded for the next apple
eaten. What is the total number of points awarded for eating ten apples?

Answer: points

9. In the month of May of a certain year, there are five Sundays and four Mondays.
On which day of the week does May 1 fall in that year?
(Using 0 to represent Sunday, 1 to represent Monday, 2 to represent Tuesday, 3 to represent
Wednesday, 4 to represent Thursday, 5 to represent Friday, 6 to represent Saturday.)

Answer:

10. In the following diagram, how many different triangles are there, including
overlapping triangles?

Answer: triangles

90
MP 4
11. There are 20 children at a party. The first girl shakes hands with 7 boys. The
second girl shakes hands with 8 boys. The third girl shakes hands with 9 boys,
and so on. The last girl shakes hands with all the boys. How many boys are at the
party?

Answer:

12. The school has six enrichment clubs. Mickey wants to join three of them.
However, there are two clubs running at the same time, he only can at most
choose one of them. How many different choices does he have?

Answer: choices

13. A square garden has a square pavilion at its centre. The distance on each outer
side of the pavilion to its corresponding side of the garden is 8 m. If the total area
of the part of the garden outside the pavilion is 448 m2, what is the area, in m2, of
the pavilion?

Garden

Pavilion

Answer: m2

91
MP 5
Questions 14 to 15, 20 marks each
(Detailed solutions are needed for these two problems)
14. One digit is chosen from each of the three groups {1, 4, 7}, {2, 5, 8} and {3, 6,
9}. The chosen digits are arranged in any order to form a three-digit number.
How many such three-digit numbers are divisible by 6?

Answer: numbers

92
MP 6
15. Each side of an equilateral triangle is divided into 4 equal parts by 3 points, and
these points are joined by lines parallel to the sides of the triangle, dividing into
16 small equilateral triangles. A tetriamond is a shape formed of 4 small
equilateral triangles joined edge to edge.

(a) Show that if 4 of the small triangles are painted, then it may be impossible to fit
any tetriamond inside the large triangle without covering up any part of the
painted small triangles. (4 marks)

(b) Prove that if 3 of the small triangles are painted, then it is always possible to fit
any tetriamond inside the large triangle without covering up any part of the
painted small triangles. (16 marks)

93
94
注意:

允許學生個人、非營利性的圖書館或公立學校合理使用
本基金會網站所提供之各項試題及其解答。可直接下載
而不須申請。

重版、系統地複製或大量重製這些資料的任何部分,必
須獲得財團法人臺北市九章數學教育基金會的授權許
可。

申請此項授權請電郵 ccmp@seed.net.tw

Notice:

Individual students, nonprofit libraries, or schools are


permitted to make fair use of the papers and its
solutions. Republication, systematic copying, or
multiple reproduction of any part of this material is
permitted only under license from the Chiuchang
Mathematics Foundation.
Requests for such permission should be made by
e-mailing Mr. Wen-Hsien SUN ccmp@seed.net.tw

95
Solution Key to Second Round of 4th IMAS
Middle Primary Division
1. What is the sum of the digits of the value of 100 100  2015 ?
(A)27 (B)29 (C)30 (D)34 (E)39
【Suggested Solution #1】
Since 100 100  2015  10000  2015  7985, then the sum of all the digits in the
difference of the given expression is 7  9  8  5  29 .
【Suggested Solution #2】
We know that 100 100  10000  (9999  1) while the sum of all its digits is
9  9  9  9  1  37, and the sum of the digits in 2015 is 2  0  1  5  8, then the
sum of all the digits in the difference of the given expression is 37  8  29.
Answer: (B)
2. If 6  2  6  66  72 and 2  3  2  22  222  246, what is the value of
53?
(A)3735 (B)605 (C)615 (D)625 (E)37035
【Suggested Solution】
5  3  5  55  555  615.
Answer: (C)
3. In a gymnastic competition, an athlete receives a score from each of seven judges.
After the highest score and the lowest score have been removed, the average of
the remaining five scores is the actual score for that athlete. If the seven judges
give scores of 9.2, 9.5, 9.3, 9.6, 9.1, 9.6 and 9.4 to an athlete, what is the actual
score for this athlete?
(A)9.3 (B)9.38 (C)9.4 (D)9.42 (E)9.5
【Suggested Solution】
After the highest score 9.6 and the lowest score 9.1 were removed, the average of the
remaining scores of the 5 judges is (9.2  9.3  9.4  9.5  9.6)  5  9.4 .
Answer: (C)
4. In a shopping mall, a ball is drawn from one of the five boxes shown below. A
door prize is won if the number of the ball is 1. From which box should a ball be
drawn so that the chance of winning a door prize is as big as possible?
(A) (B) (C)

2 3 1 2 1 1 2 1

(D) (E)

2 3 1 1 2 3 4 2

96
【Suggested Solution】
The chance (or probability) that the ball with label number 1 will be drawn from box
1 2 1 2 1
(A), (B), (C) and (D)is , , and  , respectively. Since box (E )does not
3 3 2 4 2
have the ball with a label of number 1, so the chance is 0. Hence, the chance that
the ball with a label number 1 will be drawn from box (B) is the most possible.
Answer: (B)
5. Which of the following five figures is not possible to trace without lifting the
pencil from the paper or retracing any part of it?
(A) (B) (C)

(D) (E)

【Suggested Solution】
Solving a problem such as searching for a path in a figure that visits each edge (or arc)
exactly once, we must consider the vertices associated with the each edge (or arc),
that is; the figure is possible to produce a path travelling each edge exactly once
depends on the degree (number of connection of each edge or arc to a particular
vertex) of each vertex, then the degree of each vertex is either odd number or even
number. Thus, a necessary condition for a certain figure to be traced without lifting
the pencil from the paper and retracing any of the edges (arcs) more than once, then it
must have exactly 0 or 2 vertices of odd degree.
In diagram (A), there are 10 vertices of even degree and no vertices of odd degree,
diagram (B) contains 28 vertices of even degree and 0 vertices of odd degree,
diagram (C) has 8 vertices of even degree and no vertices of odd degree, diagram (D)
has 8 vertices of even degree and 0 vertices of odd degree, diagram (E) has 3 vertices
of even degree and 4 vertices of odd degree, so it is not possible to draw diagram (E)
without lifting a pencil from the paper and without tracing any of the edges (arcs)
more than once.
Answer: (E)

97
6. At the bookstore, Lily spends half of her money buying mathematics books and
two-thirds of the remaining amount on Chinese literature books. She has just
enough money left to buy an English literature book which costs $18. How much
money does Lily have initially?
【Suggested Solution】
From the given information, we know that Lily spent one-third of the remaining half
of the money to buy the English book , that is equivalent to one-sixth of the money or
$18 to buy the English book, hence she has $18  6 or $108 originally.
Answer: $108
7. There are three kinds of objects, spheres, cylinders and cubes. Three spheres
have the same total weight as two cylinders, and five spheres have the same total
weight as six cubes. How many cubes will have the same total weight as five
cylinders?
【Suggested Solution】
From the given information, the total weight of 3 spheres has the same total weight as
2 cylinders, and the total weight of 5 spheres has the same total weight as 6 cubes,
then we know the total weight of 15 spheres and the total weight of 10 cylinders are
the same, likewise the total weight of 15 spheres is the same as the total weight of 18
cubes, it follows the total weight of 10 cylinders is the same as the total weight of 18
cubes. Therefore, the total weight of 5 cylinders has the same total weight as the
weight of 9 cubes.
Answer: 9 cubes
8. In a video game, 1 point is awarded for eating the first apple, 2 points for eating
the second apple, and so on, with 1 additional point is awarded for the next apple
to be eaten. What is the total number of points awarded for eating ten apples?

【Suggested Solution】
From the given information, we know the total number of points when reaching
eating the 10th apple will be rewarded 1  2  3  4  5  6  7  8  9  10  11 5  55
points.
Answer: 55 points
9. In the month of May of a certain year, there are five Sundays and four Mondays.
On which day of the week does May 1 fall in that year?
【Suggested Solution】
Refer the calendar below:
Mon Tue Wed Thu Fri Sat Sun
※ ※ ×
× × × × × × ×
× × × × × × ×
× × × × × × ×
× × × × × × ×

98
From the above calendar, there are 5 Sundays and 4 Mondays, so from the first
Sunday until the fifth Sunday, there will be 29 days. Since there are 31 days in the
month of May, then the remaining two days must fall before the first Sunday and not
after the fifth Sunday, which we label it using ※ in the calendar above. Therefore,
May 1 will fall on a Friday of that year.
Answer: Friday or 5
10. In the following diagram, how many different triangles are there, including
overlapping triangles?
【Suggested Solution】
In order to ensure the counting of triangles will not be
repeated, we will do the counting of triangles by
classification.
Case 1 There are no other triangles inside the triangle that we
consider, so a total of 12 triangles.
Case 2 There are two smaller triangles inside the big triangle
that we consider, so there are 2 such kinds of triangles.
Case 3 There are three smaller triangles inside the big triangle that we consider,
there are also 2 such kinds of triangles.
There is no certain kind of big triangle containing 4 or more small triangles inside it.
Thus, there are 12  2  2  16 triangles in different position.
Answer: 16 triangles
11. There are 20 children at a party. The first girl shakes hands with 7 boys. The
second girl shakes hands with 8 boys. The third girl shakes hands with 9 boys,
and so on. The last girl shakes hands with all the boys. How many boys are at the
party?
【Suggested Solution】
The 1st girl shakes with 6  1 boys, the 2nd girl shakes with 6  2 boys, the 3rd girl
shakes with 6  3 boys, …. Now, assuming there are n girls, it follows that the nth
girl shakes hands with 6  n boys. From here, we know that the number of boys is 6
more than the number of girls. Hence, the total number of boys  (20  6)  2  13,
while the number of girls  (20  6)  2  7.
Answer: 13 boys
12. The school has six enrichment clubs. Mickey wants to join three of them.
However, there are two clubs running at the same time, he only can at most
choose one of them. How many different choices does he have?
【Suggested Solution #1】
Let the six enrichment clubs be represented as A, B, C, D, E, F such that the two
clubs E and F have the same running time, then let us list out the possible clubs
Mickey might want to join:
A, B, C; A, B, D; A, B, E; A, B, F; A, C, D; A, C, E; A, C, F; A, D, E;
A, D, F; B, C, D; B, C, E; B, C, F; B, D, E; B, D, F; C, D, E; C, D, F.
Therefore, there are 16 different ways which Mickey may choose to participate in
three different clubs.

99
【Suggested Solution #2】
If Mickey didn’t participate in two clubs scheduled at the same time, so he must
select three clubs from the remaining 4 clubs, hence there will be 4 different ways of
joining them. If Mickey participates in any one of the two clubs scheduled at the
same time, then there are 2 possible ways of making choices, and then he will
participate in the two clubs from the remaining four clubs, then there will be
43
 6 different ways of joining them. Thus, there are a total of 6  2  12
2
different ways of joining them. Therefore, a total of 4  12  16 different ways of
selecting the 6 clubs.
【Suggested Solution #3】
Suppose the activities of all the six clubs are scheduled at different time, then Mickey
65 4
must select three clubs from six enrichment clubs, then there will be  20
3  2 1
different ways of selecting the clubs. Since the problem notifies us that there are two
clubs conducting their activities at the same time, and we know that there are 4
different ways which Mickey will participate in both clubs having activities at same
time and one of the four activities from the other four clubs. Therefore, Mickey can
participate 20  4  16 different ways of selecting the six clubs.
Answer: 16 different ways
13. A square garden has a square pavilion at its centre. The
Garden
distance on each outer side of the pavilion to its
corresponding side of the garden is 8 m. If the total area of
the part of the garden outside the pavilion is 448 m2, what Pavilion

is the area, in m2, of the pavilion?

【Suggested Solution】
Refer to the given diagram. Extend each side of the pavilion to its corresponding
boundary side of the garden so that it will divide the garden into four smaller
rectangles of equal area. Since the distance on each outer side of the pavilion to its
corresponding side of the garden is 8 m, it follows that the width of the rectangle is 8
m, but the area of the part of the garden outside the pavilion is 448 m2, so we have the
area of each small rectangle as 448  4  112 m2, so that the side length of the small
square is 112  8  14 m. Hence, the area of the pavilion is (14  8)2  62  36 m2.
14 8
8 8
14
8
8
14
8 8
8 14
Answer: 36 m2

100
14. One digit is chosen from each of the three groups {1, 4, 7}, {2, 5, 8} and {3, 6,
9}. The chosen digits are arranged in any order to form a three-digit number.
How many such three-digit numbers are divisible by 6?
【Suggested Solution】
By observation, we discover that each of the number in the first set when divided by
3 will give a remainder of 1, each number in the second set when divided by 3 will
give a remainder of 2 and each number in the third set when divided by 3 or when
each of them is divided by 3 will give a remainder of 0. Hence, according to the
meaning of the problem, we must form a three-digit number where the digit in each
place is selected from each set and the sum of all the digits formed when divided by 3
will give a remainder of 0, that is; the arrangement of all the three-digit numbers that
formulate is always divisible by 3. Hence, we just need to find all those three-digit
numbers from the above that are divisible by 2; or the digit in the units place is an
even number.
【Method #1】
Using Listing Method, we have the following numbers that are divisible by 2:
132, 312, 126, 216, 162, 612, 192, 912, 156, 516, 138, 318, 186, 816, 168, 618, 198,
918, 432, 342, 234, 324, 426, 246, 462, 642, 264, 624, 924, 294, 942, 492, 354, 534,
456, 546, 654, 564, 954, 594, 384, 834, 348, 438, 468, 648, 486, 846, 684, 864, 498,
948, 894, 984, 732 372, 726, 276, 762, 672, 792, 972, 756, 576, 738, 378, 786, 876,
768, 678, 978, 798. All these can compose a total of 72 three-digit numbers
divisible by 6.
【Method #2】
To form a three-digit number that meets the requirements of the problem, we have the
following three cases:
Case 1: when the digit in the units place is selected from the first set (the only
possibility is the number 4) while when the digit in the tens place is selected from the
second set and the digit in the hundreds place is selected from the third set, then there
will be 3  3 possible ways to form three-digit numbers, but the digit in the tens and
hundreds place can be interchanged in order to form another three-digit that satisfies
the requirement of the problem, so we have a total of 1 (3  3)  2  18 ways to form
three-digit numbers.
Case 2: when a digit in the units place is selected from the second set (we can choose
2 or 8 as the units digit) while when the digit in the tens place is selected from first
set and the digit in the hundreds place is selected from the third set, then there are
3  3 ways to form such kind of three-digit number. Since the digit in the tens and
hundreds place can be interchanged to form new three-digit number, so we have a
total of 2  (3  3)  2  36 ways to form three-digit numbers.
Case 3: when the digit in the units place is selected from the third set (the only
possibility is only the number 6) while when the digit in the tens place is selected
from the second set and the digit in the hundreds place is selected from the third set,
then there are 3  3 possible ways to form three-digit number. Likewise, digits in the
tens and hundreds place can be interchanged to form a new three-digit number, then
we have a total of 1 (3  3)  2  18 ways to form three- digit numbers.

101
Thus, there are a total of 18  36  18  72 three-digit numbers divisible by 6.
Answer: 72 three-digit numbers
【Marking Scheme】
 Is able to show that the three-digit number is formed, no matter how the
arrangement may be, all these three digits will still be divisible by 3, thus will be
rewarded 5 marks.
 Is able to explain the digit in units place of a three-digit number formed which
must always be an even number, thus will be rewarded 5 marks.
 Is able to use method #1 to list down all the 72 three-digit numbers correctly, thus
will be rewarded 10 marks. If some of those enumerated three-digit numbers
are incorrect, no marks will be rewarded; is able to use method #2 to present the
digit in units place which must be selected from the first, second and third sets
correctly, each will be rewarded 3 marks; lastly, is able to write down the total
numbers of three-digit numbers correctly, 1 mark will be rewarded.
15. Each side of an equilateral triangle is divided into 4 equal parts by 3 points, and
these points are joined by lines parallel to the sides of the triangle, dividing into
16 small equilateral triangles. A tetriamond is a shape formed by 4 small
equilateral triangles joined edge to edge.

(a) Show that if 4 of the small triangles are painted, then it will be impossible to fit
any tetriamond inside the large triangle without covering up any part of the
painted small triangles.(4 marks)
(b) Prove that if 3 of the small triangles are painted, then it is always possible to fit
any tetriamond inside the large triangle without covering up any part of the
painted small triangles.(16 marks)
【Suggested Solution】
(1) Let us paint black the 4 of the small triangles as figure below, then we cannot put
any of the tetriamond in the big triangle by preventing covering any of black-shaded
small triangles.

102
(2) We can use anyone of the three figures below to partition the given big triangle
into four regions, because we just need to paint black the three small triangles, then
there will be a certain region whose small triangles that have not been painted black,
so the four small triangles in this region will surely become a tetriamond.

【Marking Scheme】
(1) In the first sub-problem: if the position of small triangles are correctly painted
black, then reward it 4 marks.
(2) For sub-problem:
 The big triangle is correctly partitioned into four regions as the key, reward 8
marks.
 Is able to explain that there is one region without having shaded black, reward 4
marks.
 Is able to explain that the four small triangles in a certain is tetriamond, then no
matter which three of the four small triangles are shaded, it is possible to put the
tetriamond completely inside the big triangles and none of them will be covered
by the black triangles. Reward 4 marks.

103
注意:

允許學生個人、非營利性的圖書館或公立學校合理使用
本基金會網站所提供之各項試題及其解答。可直接下載
而不須申請。

重版、系統地複製或大量重製這些資料的任何部分,必
須獲得財團法人臺北市九章數學教育基金會的授權許
可。

申請此項授權請電郵 ccmp@seed.net.tw

Notice:

Individual students, nonprofit libraries, or schools are


permitted to make fair use of the papers and its
solutions. Republication, systematic copying, or
multiple reproduction of any part of this material is
permitted only under license from the Chiuchang
Mathematics Foundation.
Requests for such permission should be made by
e-mailing Mr. Wen-Hsien SUN ccmp@seed.net.tw

104
International Mathematics Assessments for Schools

2015 MIDDLE PRIMARY DIVISION FIRST ROUND PAPER


Time allowed:75 minutes

INSTRUCTION AND INFORMATION


GENERAL
1. Do not open the booklet until told to do so by your teacher.
2. No calculators, slide rules, log tables, math stencils, mobile phones or other
calculating aids are permitted. Scribbling paper, graph paper, ruler and compasses
are permitted, but are not essential.
3. Diagrams are NOT drawn to scale. They are intended only as aids.
4. There are 20 multiple-choice questions, each with 5 choices. Choose the most
reasonable answer. The last 5 questions require whole number answers between
000 and 999 inclusive. The questions generally get harder as you work through the
paper. There is no penalty for an incorrect response.
5. This is a mathematics assessment, not a test; do not expect to answer all questions.
6. Read the instructions on the answer sheet carefully. Ensure your name, school
name and school year are filled in. It is your responsibility that the Answer Sheet
is correctly coded.
7. When your teacher gives the signal, begin working on the problems.
THE ANSWER SHEET
1. Use only pencils.
2. Record your answers on the reverse side of the Answer Sheet (not on the question
paper) by FULLY filling in the circles which correspond to your choices.
3. Your Answer Sheet will be read by a machine. The machine will see all markings
even if they are in the wrong places. So please be careful not to doodle or write
anything extra on the Answer Sheet. If you want to change an answer or remove
any marks, use a plastic eraser and be sure to remove all marks and smudges.
INTEGRITY OF THE COMPETITION
The IMAS reserves the right to re-examine students before deciding whether to
grant official status to their scores.

105
2015 MIDDLE PRIMARY DIVISION FIRST ROUND PAPER

Questions 1-10, 3 marks each


1. What is the value of 2015  116 ?
(A)1889 (B)1890 (C)1898 (D)1899 (E)1989

2. The diagram shows five children of equal height standing in a row under a street
lamp. Which of them casts the shortest shadow?

(A)A (B)B (C)C (D)D (E)E

3. Anne ate half of a box of biscuits and then half of the remaining biscuits. If only
1 biscuit is left, how many biscuits were there originally?
(A) 4 (B) 6 (C) 8 (D) 10 (E)12

4. The sum of the digits displaced on each of the five watches is the same. Which of
the following can be the time displayed on the last watch?

(A)6:29 (B)8:31 (C)9:37 (D)10:00 (E)11:57

5. Eight pop bottles can be traded in for a pencil. What is the maximum number of
pencils that can be obtained from 34 pop bottles?
(A) 1 (B) 2 (C) 3 (D) 4 (E)5

106
MP 2
6. What is the minimum number of straight cuts required to divide a cylindrical
cake into eight identical pieces?

(A) 1 (B) 2 (C) 3 (D) 4 (E)5

7. The product of two two-digit multiples of 10 is 2000. What is their sum?


(A)120 (B)105 (C)100 (D)90 (E)80

8. Which of the following figures can be combined with the


given one to form a square?

(A) (B) (C)

(D) (E)

9. The subway system of a certain city consists of 3 2 1 16 15


16 stations in a closed loop, with trains going
in both directions. The fare is 1 dollar for a ride
of 1 station, 2 dollars for a ride of 2 or 3 4 14
stations, 3 dollars for a ride of 4 or 5 stations, 4
dollars for a ride of 6 or 7 stations, and 5 5 13
dollars for a ride of 8 or more stations. What is
the minimum cost for going from Station #2 to
Station #14? 6 12
(A) 1 (B) 2 (C) 3
(D) 4 (E)5 7 8 9 10 11

107
MP 3
10. Climbing the stairs, a boy takes 60 seconds to go from the 1st floor to the 4th floor.
Assuming that the distance between consecutive floors is the same and the boy
continues to climb at the same uniform speed as before, how many more seconds
will he take to reach the 8th floor?
(A)60 (B)80 (C)100 (D)110 (E)120

Questions 11-20, 4 marks each


11. Which of the following figures is the mirror reflection of the given figure?

(A) (B) (C)

(D) (E)

1
12. Of the following figures, which one does not have exactly of its area
4
shaded?
(A) (B) (C) (D) (E)

13. A, B and C share 36 grapes. If A gives 10 to B and B then gives 8 to C, each has
the same number of grapes. How many more grapes than C did A have initially?

(A)14 (B)16 (C)18 (D)20 (E)22

108
MP 4
14. The total weights of three groups of four animals are recorded as shown. Each
animal of the same type has the same weight. What is the weight of each sheep?

20

17

15

(A) 2 (B) 4 (C) 5 (D) 7 (E)9

15. A number is larger than another number by 416. The larger number is 9 times as
large as the smaller number. What is the sum of these two numbers?
(A)468 (B)500 (C)520 (D)530 (E)572

16. Every student consumes water at a constant rate. If a jug containing 420 litres of
water is just sufficient to supply 20 students for 7 days, for how many days can
the same jug supply 28 students?
(A) 3 (B) 4 (C) 5 (D) 6 (E)7

17. In the number triangle shown, every number not on the bottom row is equal to
the sum of the two numbers below it. What is the number represented by the “?”
mark?

65

40

13

25 ? 9 4

(A)3 (B)4 (C)8 (D)16 (E)26

18. Three cups of equal value are the door prizes won by five people. Since the cups
form a set, it is decided that a husband and wife pair among the five friends
should get them. In return, the couple offers 1800 dollars to be shared equally by
the other three friends. If each person has received an equal share, what is the
value, in dollars, of each cup?
(A)900 (B)1000 (C)1200 (D)1300 (E)1500

109
MP 5
19. What is the maximum value of the expression ( )  ( )  ( )  ( ) if a
different number chosen from 1, 2, 3, 4, 5, 6, 7, 8, 9 and 10 is used to fill each
bracketed space?
(A)81 (B)88 (C)91 (D)97 (E)99

20. How many of the following statements are correct?


A: The sum of your age on your birthday in 2015 and your birth year must
be 2015.
B: If today is Tuesday, then 217 days later it will again be Tuesday.
C: If we multiply 2015 by 8888 and then add 2015, the sum must be odd.
D: If the sum of the ages of three people is odd, then the sum of their ages
a year later must be even.
(A)0 (B)1 (C)2 (D)3 (E)4

Questions 21-25, 6 marks each

21. What is the total number of digits used to write down the first 31 positive
integers?

22. There are tokens of weight 1 gram and tokens of weight 4 grams. At least how
many tokens must be used in order to balance an object of weight 103 grams, if
tokens may also be placed on the same pan as the object?

110
MP 6
23. There are two classrooms on each of four floors in the school building, x01 and
x02 where x is the floor number. Martial Arts and Drama are on the first floor.
Calligraphy and Fine Arts are on the same floor. Music is directly above Fine
Arts and Painting is directly above Calligraphy. Modeling is directly above
Painting. Neither Martial Arts nor Music is in an odd-numbered room. In which
room is Dancing?

Building

401 402

301 302

201 202

101 102

24. During a certain period, each morning and afternoon is either wet or dry. The
total number of wet mornings and wet afternoons is 9. There are 8 dry mornings
and 7 dry afternoons A wet afternoon always follows a dry morning. In how
many days during this period are both morning and afternoon dry?

25. A standard deck of cards consists of two Jokers and thirteen cards in each of the
four suits: Spades ♠, Hearts ♥, Diamonds ♦ and Clubs ♣. What is the minimum
number of cards one must draw at random to ensure that six cards of the same
suit are drawn?

***

111
112
注意:

允許學生個人、非營利性的圖書館或公立學校合理使用
本基金會網站所提供之各項試題及其解答。可直接下載
而不須申請。

重版、系統地複製或大量重製這些資料的任何部分,必
須獲得財團法人臺北市九章數學教育基金會的授權許
可。

申請此項授權請電郵 ccmp@seed.net.tw

Notice:

Individual students, nonprofit libraries, or schools are


permitted to make fair use of the papers and its
solutions. Republication, systematic copying, or
multiple reproduction of any part of this material is
permitted only under license from the Chiuchang
Mathematics Foundation.
Requests for such permission should be made by
e-mailing Mr. Wen-Hsien SUN ccmp@seed.net.tw

113
Solution to
Fifth International Mathematics Assessment for Schools
Round 1 of Middle Division

1. What is the value of 2015  116 ?


(A)1889 (B)1890 (C)1898 (D)1899 (E)1989
【Suggested Solution】
We have 2015  116  2015  15  1  100  2000  100  1  1900  1  1899 .
Answer:(D)
2. The diagram shows five children of equal height standing in a row under a street
lamp. Which of them casts the shortest shadow?

(A)A (B)B (C)C (D)D (E)E


【Suggested Solution】
Since D is nearest to the lamp, her shadow is the shortest.
Answer:(D)
3. Anne ate half of a box of biscuits and then half of the remaining biscuits. If only
1 biscuit is left, how many biscuits were there originally?
(A) 4 (B) 6 (C) 8 (D) 10 (E)12
【Suggested Solution】
1 1 1 1
The last biscuit is   of all the biscuits, there were 1   4 biscuits
2 2 4 4
originally.
Answer:(A)
4. The sum of the digits displaced on each of the five watches is the same. Which of
the following can be the time displayed on the last watch?

(A)6:29 (B)8:31 (C)9:37 (D)10:00 (E)11:57

114
【Suggested Solution】
The constant digit sum is 12, and only 8:31 has this property.
Answer:(B)
5. Eight pop bottles can be traded in for a pencil. What is the maximum number of
pencils that can be obtained from 34 pop bottles?
(A) 1 (B) 2 (C) 3 (D) 4 (E)5
【Suggested Solution】
Since 34 is more than 4 times 8 but less than 5 times 8, at most 4 pencils can be
obtained.
Answer:(D)
6. What is the minimum number of straight cuts required to divide
a cylindrical cake into eight identical pieces?
(A) 1 (B) 2 (C) 3 (D) 4 (E)5
【Suggested Solution】
Since each straight cut can at most double the number of pieces, 3 cuts are required.
These are sufficient as a third cut can be along the plane halfway between the top and
the bottom of the cylinder.
Answer:(C)
7. The product of two two-digit multiples of 10 is 2000. What is their sum?
(A)120 (B)105 (C)100 (D)90 (E)80
【Suggested Solution】
If we divide the two numbers by 10, they become single-digit numbers with product
20  4  5 . Hence their sum is 40 + 50 = 90.
Answer:(D)
8. Which of the following figures can be combined with the
given one to form a square?

(A) (B) (C)

(D) (E)

【Suggested Solution】
Of the five figures, the only one which can combine with the
given one to form even a rectangle is (A) or (E), but the former
yields a rectangle which is not a square.
Answer:(E)

115
9. The subway system of a certain city consists of 3 2 1 16 15
16 stations in a closed loop, with trains going
in both directions. The fare is 1 dollar for a ride
of 1 station, 2 dollars for a ride of 2 or 3 4 14
stations, 3 dollars for a ride of 4 or 5 stations, 4
dollars for a ride of 6 or 7 stations, and 5 5 13
dollars for a ride of 8 or more stations. What is
the minimum cost for going from Station #2 to
Station #14? 6 12
(A) 1 (B) 2 (C) 3
(D) 4 (E)5 7 8 9 10 11
【Suggested Solution】
We can go from Station #2 through Stations #1, #16, and #15 to get to Station #14.
This ride of 4 stations costs 3 dollars.
Answer:(C)
10. Climbing the stairs, a boy takes 60 seconds to go from the 1 floor to the 4th floor.
st

Assuming that the distance between consecutive floors is the same and the boy
continues to climb at the same uniform speed as before, how many more seconds
will he take to reach the 8th floor?
(A)60 (B)80 (C)100 (D)110 (E)120
【Suggested Solution】
Going from the 1st floor to the 4th floor requires the climbing of 3 floors. Thus the
climbing of each floor requires 60  3  20 seconds. Going from the 4th floor to the
8th floor requires the climbing o 4 floors. Hence the number of additional seconds
required is 20  4  80 seconds.
Answer:(B)
11. Which of the following figures is the mirror reflection
of the given figure?

(A) (B) (C) (D) (E)

【Suggested Solution】
Given figure reflection
In the given figure, the “head” leans to the right,
and the “right arm” have small black dots at the tip.
Figures (A), (C) and (E) are the only one leaning to
the left, and Figures (E) are the only one with the
“left arm” have small black dots at the tip.
mirror
Answer:(E)

116
1
12. Of the following figures, which one does not have exactly of its area
4
shaded?
(A) (B) (C) (D) (E)

【Suggested Solution】
In Figure (D), the two strips in the middle have equal area, and are bigger than the
1
strips on the outside. Hence the shaded area is greater than of the whole figure. It
4
1
is easy to verify that in each of the other figures, the shaded area is exactly of the
4
whole figure.

Answer:(D)
13. A, B and C share 36 grapes. If A gives 10 to B and B then gives 8 to C, each has
the same number of grape. How many more grapes than C did A have initially?

(A)14 (B)16 (C)18 (D)20 (E)22


【Suggested Solution】
At the end, each has 36  3  12 grapes. Initially, A had 12  10  22 grapes while
C had 12  8  4 . Hence A had 22  4  18 more grapes than C.
Answer:(C)
14. The total weights of three groups of four animals are recorded as shown. Each
animal of the same type has the same weight. What is the weight of each sheep?

20

17

15

117
(A) 2 (B) 4 (C) 5 (D) 7 (E)9
【Suggested Solution】
Combining all three records, we see that the total weight of four animals of each type
is 20 + 17 + 15 = 52. Hence the total weight of one animal of each type is 52  4  13 .
It follows that each sheep weighs 20  13  7 .
Answer:(D)
15. A number is larger than another number by 416. The larger number is 9 times as
large as the smaller number. What is the sum of these two numbers?
(A)468 (B)500 (C)520 (D)530 (E)572
【Suggested Solution】
Eight times the smaller number is 416, so that it is 52. The sum is ten times the
smaller number. Hence it is 520.
Answer:(C)
16. Every student consumes water at a constant rate. If a jug containing 420 litres of
water is just sufficient to supply 20 students for 7 days, for how many days can
the same jug supply 28 students?
(A) 3 (B) 4 (C) 5 (D) 6 (E)7
【Suggested Solution】
The number of days is inversely proportional to the number of students. Hence the
20
number of days is  7  5.
28
Answer:(C)
17. In the number triangle shown, every number not on the
bottom row is equal to the sum of the two numbers below 65
it. What is the number represented by the “?” mark?
(A)3 (B)4 (C)8 40 25
(D)16 (E)26
【Suggested Solution】 28 12 13
Since 65  40  25 , 25  13  12 , 40  12  28 and
25 3 9 4
28  25  3 , we have ?=3.
Answer:(A)
18. Three cups of equal value are the door prizes won by five people. Since the cups
form a set, it is decided that a husband and wife pair among the five friends
should get them. In return, the couple offers 1800 dollars to be shared equally by
the other three friends. If each person has received an equal share, what is the
value, in dollars, of each cup?
(A)900 (B)1000 (C)1200 (D)1300 (E)1500
【Suggested Solution】
Each equal share is worth 1800  3  600 dollars, so that the total worth of the three
cups is 3000 dollars. It follows that each cup is worth 3000  3  1000 dollars.
Answer:(B)

118
19. What is the maximum value of the expression ( )  ( )  ( )  ( ) if a
different number chosen from 1, 2, 3, 4, 5, 6, 7, 8, 9 and 10 is used to fill each
bracketed space?
(A)81 (B)88 (C)91 (D)97 (E)99
【Suggested Solution】
Clearly, the maximum value is achieved by maximizing first the product and then the
sum, and finally subtracting the smallest possible value. Hence this value is
9 10  8  1  97 .
Answer:(D)
20. How many of the following statements are correct?
A: The sum of your age on your birthday in 2015 and your birth year must
be 2015.
B: If today is Tuesday, then 217 days later it will again be Tuesday.
C: If we multiply 2015 by 8888 and then add 2015, the sum must be odd.
D: If the sum of the ages of three people is odd, then the sum of their ages
a year later must be even.
(A)0 (B)1 (C)2 (D)3 (E)4
【Suggested Solution】
All four statements are obviously correct.
Answer:(E)
21. What is the total number of digits used to write down the first 31 positive
integers?
【Suggested Solution】
Each of the 31 numbers has a units digit. Each of 22 of them also has a tens digit.
Hence the total number of digit is 31 + 22 = 53.
Answer:053
22. There are tokens of weight 1 gram and tokens of weight 4
grams. At least how many tokens must be used in order to
balance an object of weight 103 grams, if tokens may also be
placed on the same pan as the object?
【Suggested Solution】
We must use 25 tokens of weight 4 grams just to reach 100 grams. We may add 3
tokens of weight 1 gram, bringing the total to 28. However, we can save 1 token by
using 26 tokens of weight 4 grams while placing a token of weight 1 gram with the
object. Thus the minimum number is 27.
Answer:027
23. There are two classrooms on each of four floors in Building
the school building, x01 and x02 where x is the
floor number. Martial Arts and Drama are on the 401 402
first floor. Calligraphy and Fine Arts are on the 301 302
same floor. Music is directly above Fine Arts and
Painting is directly above Calligraphy. Modeling 201 202

is directly above Painting. Neither Martial Arts 101 102


nor Music is in an odd-numbered room. There is a

119
room for Dancing, what is its room number ?
【Suggested Solution】
Modeling, Painting and Calligraphy are in the same room on the fourth, third and
second floors respectively, all in odd-numbered classrooms since Music is not. Now
Music and Fine Arts are on the third and second floors respectively. It follows that
Dancing is in classroom 402.
Building

401 402
Modeling Dancing

301 302
Painting Music
201 202
Calligraphy Fine Arts

101 102
Martial Arts Drama
Answer:402
24. During a certain period, each morning and afternoon is either wet or dry. The
total number of wet mornings and wet afternoons is 9. There are 8 dry mornings
and 7 dry afternoons. A wet afternoon always follows a dry morning. In how
many days during this period are both morning and afternoon dry?
【Suggested Solution】
The total number of mornings and afternoons is 9  7  8  24 , so that there are 12 of
each. Since the 4 wet mornings and 5 wet afternoons must occur in 9 different days,
the number of days in which both the morning and the afternoon are dry is 3.
1 2 3 4 5 6 7 8 9 10 11 12
Morning dry dry dry dry dry dry dry dry dry wet wet wet
Afternoon wet wet wet wet wet dry dry dry dry dry dry dry
Answer:003
25. A standard deck of cards consists of two Jokers and
thirteen cards in each of the four suits: Spades ♠, Hearts
♥, Diamonds ♦ and Clubs ♣. What is the minimum
number of cards one must draw at random to ensure that
six cards of the same suit are drawn?
【Suggested Solution】
Without six cards of the same suit, the most we can draw are 22 cards, consisting of
both Jokers and five cards of each suit. When we draw 23 cards, we must have six
cards of the same suit.
Answer:023

120
注意:

允許學生個人、非營利性的圖書館或公立學校合理使用
本基金會網站所提供之各項試題及其解答。可直接下載
而不須申請。

重版、系統地複製或大量重製這些資料的任何部分,必
須獲得財團法人臺北市九章數學教育基金會的授權許
可。

申請此項授權請電郵 ccmp@seed.net.tw

Notice:

Individual students, nonprofit libraries, or schools are


permitted to make fair use of the papers and its
solutions. Republication, systematic copying, or
multiple reproduction of any part of this material is
permitted only under license from the Chiuchang
Mathematics Foundation.
Requests for such permission should be made by
e-mailing Mr. Wen-Hsien SUN ccmp@seed.net.tw

121
5th International Mathematics Assessments for Schools
(2015-2016 )

Middle Primary Division Round 2


Time: 120 minutes

Printed Name: Code: Score:

Instructions:
 Do not open the contest booklet until you are told to do so.
 Be sure that your name and code are written on the space provided above.
 Round 2 of IMAS is composed of three parts; the total score is 100 marks.
 Questions 1 to 5 are given as a multiple-choice test. Each question has five
possible options marked as A, B, C, D and E. Only one of these options is correct.
After making your choice, fill in the appropriate letter in the space provided. Each
correct answer is worth 4 marks. There is no penalty for an incorrect answer.
 Questions 6 to 13 are a short answer test. Only Arabic numerals are accepted;
using other written text will not be honored or credited. Some questions have
more than one answer, as such all answers are required to be written down in the
space provided to obtain full marks. Each correct answer is worth 5 marks. There
is no penalty for incorrect answers.
 Questions 14 and 15 require a detailed solution or process in which 20 marks are
to be awarded to a completely written solution. Partial marks may be given to an
incomplete presentation. There is no penalty for an incorrect answer.
 Use of electronic computing devices is not allowed.
 Only pencil, blue or black ball-pens may be used to write your solution or answer.
 Diagrams are not drawn to scale. They are intended as aids only.
 After the contest the invigilator will collect the contest paper.
The following area is to be filled in by the judges;
the contestants are not supposed to mark anything here.
Total
Question 1 2 3 4 5 6 7 8 9 10 11 12 13 14 15 Score Signature

Score

Score

122
Middle Primary Division Round 2
Questions 1 to 5, 4 marks each
1. The operation * is such that 2  6  26  2  6  18 and 7  3  73  7  3  63. If a
and b are non-zero digits such that a  b  36 , what is the value of a?
(A)3 (B)4 (C)6 (D)7 (E)9

Answer:

2. I In how many different ways can three identical balls be distributed among three
distinct boxes, if we allow some boxes to be empty?
(A)6 (B)10 (C)12 (D)18 (E)27

Answer:

3. The diagram shows the playing board of a game where we start on square 1. We
move forward 2 squares and backwards 1 square. Then we move forward 3
squares and backwards 2 squares, and so on. Each time, we move forward one
square more than the last time, and the backward move is one square shorter than
the forward move. Moving from one square to the next, in either direction, is
called a step. How many steps does it take to reach square 11 for the first time?
1 2 3 4 5 6 7 8 9 10 11
(A)30 (B)32 (C)34 (D)36 (E)38

Answer:

4. A beam balance has a 1 g token, a 4 g token and a 7 g token. Each token may be
placed on the same pan as the object being weighed or on the other pan, or not at
all. How many different positive integral weights can be measured exactly using
this balance and the three tokens?

(A)8 (B)9 (C)10 (D)11 (E)12

Answer:

123
MP 2
5. There are three kinds of bottles, holding 0.4 L, 0.6 L and 1 L respectively. The
total capacity of several bottles is 10 L. How many possible values of the number
of bottles holding 0.6 L are there if there is at least one bottle of each kind?

(A)9 (B)10 (C)11 (D)12 (E)13

Answer:

Questions 6 to 13, 5 marks each


6. A three-digit number has digit-sum 18. The tens digit is 2 less than the hundreds
digit and the units digit is 2 less than the tens digit. What is this three-digit
number?

Answer:

7. The diagram shows a paper strip with four possible dividing lines. We may cut
along none, any or all of these lines. How many different sizes can the piece
containing the circle have?

Answer:

8. The diagram shows a 15 by 6 board on which the number 2016 are shaded. Each
square is 1 cm by 1 cm. Each long diagonal joins opposite vertices of a square
while each short diagonal joins midpoints of adjacent sides of a square. What is
the total area, in cm2, of the shaded regions?

Answer: cm2

124
MP 3
9. In the diagram, E, F, G and H are the respective midpoints of the sides CD, DA,
AB and BC of a square ABCD. If the area of the square KLMN is 4 cm2, what is
the area, in cm2, of ABCD?
A F D
L K
G E
M N

B H C
Answer: cm2

10. A cube is placed on the table, and a number is written on each face. The sum of
the two numbers on opposite faces is always 16. Honey can see three faces, and
the numbers on them add up to 24, and Lily can see the top face and the other
two lateral faces, and the numbers on them add up to 26. What is the number on
the top face?

Answer:

11. The diagram shows three wheels of fortune, generating a three-digit number. The
hundreds digit comes from the wheel on the left, the tens digit from the wheel in
the middle, and the units digit from the wheel on the right. A prize is awarded if
this three-digit number is divisible by 6. How many different winning numbers
are there?
1 2 0
4 2 8 4 3 1
3 6 2

Answer:

12. Daniel finished reading a comic book in 16 days. Then Ray starts reading the
same book, and gets to the halfway point of it in 5 less days. If Daniel reads 6
pages per day more than Ray, how many pages does this book have?

Answer:

125
MP 4
13. There are five different red points in the plane. The midpoint of each segment
joining two red points is painted black. What is the minimum number of black
points?

Answer:

Questions 14 to 15, 20 marks each


(Detailed solutions are needed for these two problems)
14. In how many ways can the numbers 1, 2, 3, 4, 5, 6, 7, 8 and 9 be divided into
three groups of three such that the sum of the numbers in the second group is 1
less than the sum of the numbers in the first group but 1 more than the sum of the
numbers in the third group?

Answer: ways

126
MP 5
15. Consider all integers between 100 and 999 inclusive such that the hundreds digit
is equal to the units digit. If S is the sum of all such numbers, what is the
digit-sum of S?

Answer:

127
注意:

允許學生個人、非營利性的圖書館或公立學校合理使用
本基金會網站所提供之各項試題及其解答。可直接下載
而不須申請。

重版、系統地複製或大量重製這些資料的任何部分,必
須獲得財團法人臺北市九章數學教育基金會的授權許
可。

申請此項授權請電郵 ccmp@seed.net.tw

Notice:

Individual students, nonprofit libraries, or schools are


permitted to make fair use of the papers and its
solutions. Republication, systematic copying, or
multiple reproduction of any part of this material is
permitted only under license from the Chiuchang
Mathematics Foundation.
Requests for such permission should be made by
e-mailing Mr. Wen-Hsien SUN ccmp@seed.net.tw

128
Solution Key to Second Round of IMAS 2015/2016
Middle Primary Division
1. The operation * is such that 2 ∗ 6 = 26 − 2 − 6 = 18 and 7 ∗ 3 = 73 − 7 − 3 = 63 . If a
and b are non-zero digits such that a ∗ b = 36 , what is the value of a?
(A)3 (B)4 (C)6 (D)7 (E)9
【Suggested Solution】
From the examples, it is reasonable to conclude that a ∗ b = (10a + b) − a − b = 9a .
From 9a = 36 , we have a = 4 .
Answer: (B)
2. In how many different ways can three identical balls be distributed among three
distinct boxes, if we allow some boxes to be empty?
(A)6 (B)10 (C)12 (D)18 (E)27
【Suggested Solution】
Add three balls in, use six 0s to represent the balls and two 1s to represent the
dividing lines separating the balls going into different boxes. The first 1 has 5
positions to choose from. Then the second 1 has 4 choices. However, since the two 1s
are not distinguished, the total number of ways is 5 × 4 ÷ 2 = 10 . After this, remove 1
ball from each box for allow some boxes to be empty.
Answer: (B)
3. The diagram shows the playing board of a game where we start on square 1. We
move forward 2 squares and backwards 1 square. Then we move forward 3
squares and backwards 2 squares, and so on. Each time, we move forward one
square more than the last time, and the backward move is one square shorter than
the forward move. Moving from one square to the next, in either direction, is
called a step. How many steps does it take to reach square 11 for the first time?
1 2 3 4 5 6 7 8 9 10 11
(A)30 (B)32 (C)34 (D)36 (E)38
【Suggested Solution】
After the first pair of moves, the farthest we reach is square 3. After each subsequent
couple, we reach another two squares farther. Hence it takes (11 − 3) ÷ 2 = 4 more
couples of moves, except that in the last couple, we do not have to move back. Hence
the total number of steps is (2 + 1) + (3 + 2) + (4 + 3) + (5 + 4) + 6 = 30 .
Answer: (A)
4. A beam balance has a 1 g token, a 4 g token and a 7 g
token. Each token may be placed on the same pan as the
object being weighed or on the other pan, or not at all.
How many different positive integral weights can be
measured exactly using this balance and the three
tokens?
(A)8 (B)9 (C)10 (D)11 (E)12

129
【Suggested Solution】
Using only one token, it can balance 1, 4 and 7. Using two tokens, they can balance
1 + 4 = 5 , 1 + 7 = 8 , 4 + 7 = 11 , 4 − 1 = 7 − 4 = 3 and 7 − 1 = 6 . Using all three
tokens, they can balance 1 + 4 + 7 = 12 , 7 − 1 − 4 = 2 , 1 + 7 − 4 = 4 and
7 + 4 − 1 = 10 . Note that 4 has been balanced before. Thus the total number of ways is
11.
Answer: (D)
5. There are three kinds of bottles, holding 0.4 L, 0.6 L and 1 L respectively. The
total capacity of several bottles is 10 L. How many possible values of the number
of bottles holding 0.6 L are there if there is at least one bottle of each kind?

(A)9 (B)10 (C)11 (D)12 (E)13


【Suggested Solution】
First take away one bottle of each kind, so that the relevant total capacity is now
reduced to 8. If we use an equal number of 0.4 L bottles and 0.6 L bottles, the number
of 0.6 L bottles can be any number from 0 to 8 inclusive. Since three 0.4 L bottles can
be replaced by two 0.6 L bottles and we have up to eight 0.4 L bottles, enough for
two replacements, the number of 0.6 L bottles can stretch to 8+2×2=12. Thus there
are 13 possible values.
Answer: (E)
6. A three-digit number has digit-sum 18. The tens digit is 2 less than the hundreds
digit and the units digit is 2 less than the tens digit. What is this three-digit
number?
【Suggested Solution】
The tens digit must be 18 ÷ 3 = 6 so that the three-digit number must be 864.
Answer: 864
7. The diagram shows a paper strip with four possible dividing lines. We may cut
along none, any or all of these lines. How many different sizes can the piece
containing the circle have?

【Suggested Solution】
There are 3 dividing line, including the end of the strip, which can serve as the left
edge of the piece containing the circle. The same can be said about the right edge.
Hence the total number of different sizes is 3 × 3 = 9 .
Answer: 9 sizes

130
8. The diagram shows a 15 by 6 board on which the number 2016 are shaded. Each
square is 1 cm by 1 cm. Each long diagonal joins opposite vertices of a square
while each short diagonal joins midpoints of adjacent sides of a square. What is
the total area, in cm2, of the shaded regions?

【Suggested Solution】

As shown in the diagram, both 2 and 0 can be transformed so that they consist of
complete squares like 6. The respective numbers of squares are 13, 14 and 14, while 1
consists of 6 complete squares and 3 half-squares. The total area is 48.5 cm2.
Answer: 48.5 cm2
9. In the diagram, E, F, G and H are the respective midpoints of the sides CD, DA,
AB and BC of a square ABCD. If the area of the square KLMN is 4 cm2, what is
the area, in cm2, of ABCD?
A F D
L K
G E
M N

【Suggested Solution】 B H C
If we rotate triangle BMG about G, it can combine with ALMG to form a square.
Similar transformations can be performed on the other sides of ABCD. Hence the area
of ABCD is 4 × 5 = 20 cm2.
A F D
L K
E
G
M N
Answer: 20 cm2
B H C

131
10. A cube is placed on the table, and a number is written on each
face. The sum of the two numbers on opposite faces is always ?
16. Honey can see three faces, and the numbers on them add up
to 24, and Lily can see the top face and the other two lateral
faces, and the numbers on them add up to 26. What is the
number on the top face?
【Suggested Solution】
Between them, Honey and Lily see all four lateral numbers once and the top number
twice. Since the sum of the four lateral numbers is 16 × 2 = 32 , the top number is
50 − 32
= 9.
2
Answer: 9
11. The diagram shows three wheels of fortune, generating a three-digit number. The
hundreds digit comes from the wheel on the left, the tens digit from the wheel in
the middle, and the units digit from the wheel on the right. A prize is awarded if
this three-digit number is divisible by 6. How many different winning numbers
are there?
1 2 0
4 2 8 4 3 1
3 6 2
【Suggested Solution】
The units digit must be even. If it is 0, we may have 120, 180, 240, 360, 420 or 480.
If it is 2, we may have 162, 222, 282, 342 or 462. The total is 11.
Answer: 11
12. Daniel finished reading a comic book in 16 days. Then Ray starts reading the
same book, and gets to the halfway point of it in 5 less days. If Daniel reads 6
pages per day more than Ray, how many pages does this book have?
【Suggested Solution】
Ray can finish the book in 2 × (16 − 5) = 22 days. So in 176 days, Daniel can read
the book 11 times while Ray can read it 8 times. Daniel has read 176 × 6 = 1056
pages more than Ray. Hence the book has 1056 ÷ 3 = 352 pages.
Answer: 352 pages
13. There are five different red points in the plane. The midpoint of each segment
joining two red points is painted black. What is the minimum number of black
points?
【Suggested Solution】
With three red points, we have 3 black points. When we add a fourth red point,
consider the two red points closest to it. The two black points generated by them with
the new red point must be new, bringing the total to 5. The same argument shows that
when a fifth red point is added, we must have at least 7 black points. Now if the red
points are at 1, 3, 5, 7 and 9 on the number lines, the black points are at 2, 3, 4, 5, 6, 7

132
and 8, and there are exactly 7 of them.
A B C D E
Black 2 Black 4 Black 6
Black 1 Black 3 Black 5 Black 7

Answer: 7
14. In how many ways can the numbers 1, 2, 3, 4, 5, 6, 7, 8 and 9 be divided into
three groups of three such that the sum of the numbers in the second group is 1
less than the sum of the numbers in the first group but 1 more than the sum of the
numbers in the third group?
【Suggested Solution】
The sum of all 9 numbers is 45. Hence the sums of the groups are 16, 15 and 14.
The second group must be one of (6, 5, 4), (7, 5, 3), (7, 6, 2), (8, 4, 3), (8, 5, 2),
(8, 6, 1), (9, 4, 2) or (9, 5, 1). (2 points)
If it is (6, 5, 4), the first group may be (9, 6, 2) or (8, 7, 1). (2 points)
If it is (7, 5, 3), the first group must be (9, 6, 1). (2 points)
If it is (7, 6, 2), the first group may be (9, 4, 3) or (8, 5, 3). (2 points)
If it is (8, 4, 3), the first group may be (9, 6, 1) or (9, 5, 2). (2 points)
If it is (8, 5, 2), the first group may be (9, 6, 1), (9, 4, 3) or (7, 6, 3). (2 points)
If it is (8, 6, 1), the first group (9, 5, 2), (9, 4, 3) or (7, 5, 4). (2 points)
If it is (9, 4, 2), the first group may be (8, 7, 1), (8, 5, 3) or (7, 6, 3). (2 points)
If it is (9, 5, 1), the first group may be (8, 6, 2) or (7, 6, 3). (2 points)
The total number of ways is 1 + 2 + 2 + 2 + 3 + 3 + 3 + 2 = 18 . (2 points)
Answer: 18
15. Consider all integers between 100 and 999 inclusive such that the hundreds digit
is equal to the units digit. If S is the sum of all such numbers, what is the
digit-sum of S?
【Suggested Solution】
The total number of integers of this form is 9 × 10 = 90 . (5 points) Form 45 pairs as
follows. Consider 101. Its partner 999 is chosen where the tens digit is obtained by
subtraction from 9 (9 − 0 = 9 ) and the other digits are obtained by subtraction from 10
(10 − 1 = 9 ). In this way,111 is paired with 989, 222 with 878, and so on. The sum of
the two numbers in each pair is always 1100. (10 points)
Hence S = 45 × 1100 = 49500 and the digit-sum of S is 18. (5 points)
Answer: 18

133
注意:

允許學生個人、非營利性的圖書館或公立學校合理使用
本基金會網站所提供之各項試題及其解答。可直接下載
而不須申請。

重版、系統地複製或大量重製這些資料的任何部分,必
須獲得財團法人臺北市九章數學教育基金會的授權許
可。

申請此項授權請電郵 ccmp@seed.net.tw

Notice:

Individual students, nonprofit libraries, or schools are


permitted to make fair use of the papers and its
solutions. Republication, systematic copying, or
multiple reproduction of any part of this material is
permitted only under license from the Chiuchang
Mathematics Foundation.
Requests for such permission should be made by
e-mailing Mr. Wen-Hsien SUN ccmp@seed.net.tw

134
International Mathematics Assessments for Schools

2016 MIDDLE PRIMARY DIVISION FIRST ROUND PAPER


Time allowed:75 minutes

When your teacher gives the signal, begin working on the problems.

INSTRUCTION AND INFORMATION


GENERAL
1. Do not open the booklet until told to do so by your teacher.
2. No calculators, slide rules, log tables, math stencils, mobile phones or other
calculating aids are permitted. Scribbling paper, graph paper, ruler and compasses
are permitted, but are not essential.
3. Diagrams are NOT drawn to scale. They are intended only as aids.
4. There are 20 multiple-choice questions, each with 5 choices. Choose the most
reasonable answer. The last 5 questions require whole number answers between
000 and 999 inclusive. The questions generally get harder as you work through the
paper. There is no penalty for an incorrect response.
5. This is a mathematics assessment, not a test; do not expect to answer all questions.
6. Read the instructions on the answer sheet carefully. Ensure your name, school
name and school year are filled in. It is your responsibility that the Answer Sheet
is correctly coded.

THE ANSWER SHEET


1. Use only pencils.
2. Record your answers on the reverse side of the Answer Sheet (not on the question
paper) by FULLY filling in the circles which correspond to your choices.
3. Your Answer Sheet will be read by a machine. The machine will see all markings
even if they are in the wrong places. So please be careful not to doodle or write
anything extra on the Answer Sheet. If you want to change an answer or remove
any marks, use a plastic eraser and be sure to remove all marks and smudges.

INTEGRITY OF THE COMPETITION


The IMAS reserves the right to re-examine students before deciding whether to
grant official status to their scores.

135
2016 MIDDLE PRIMARY DIVISION FIRST ROUND PAPER

Questions 1-10, 3 marks each


1. What is the value of 2016 − 20 × 16 ?
(A)1696 (B)1698 (C)1706 (D)1716 (E)1726

2. Cubes of side length 1 cm are available in two colors. They are used to form a
large cube of side length 3 cm, and two small cubes sharing a common face have
different colors. What is the positive difference between the numbers of cubes in
the two colors?
(A)0 (B)1 (C)2 (D)3 (E)4

3. How many different rectangles (including squares) in different positions are there
in the diagram below?

(A)5 (B)6 (C)7 (D)8 (E)9

4. In a 6 by 6 table, each row and each column contains the letters A, B, C, D, E and
F exactly once. All but four of the squares have been filled in. Which of the
following should be chosen to fill in the blank squares?
A B C D E F
B C D E F A
C D E F A B
D A B C
E B C D
F A B C D E
E F E F E F
(A) (B) (C)
C D F A A B
A B B C
(D) (E)
C D C D

136
MP 2
5. The letters IMAS are spelt out with the same size of black and white circles as
shown in the diagram below. If the total area of the black circles is 50 cm2 , what
is the total area, in cm2 , of the white circles?

(A)26 (B)28 (C)30 (D)32 (E)34

6. Barry is flying from A to B on a 13:00 flight. The distance between A and B is


1800 km, and the speed of the plane is 600 km per hour. If the flight is delayed
for 2 hours and no time difference between A and B, when will Barry arrive at B?
(A)15:00 (B)16:00 (C)17:00 (D)18:00 (E)19:00

7. Jane feeds her cat at regular intervals. The first feeding is at 7:00 and the sixth
feeding is at 22:00. Which of the following is not a feeding time for the cat?

(A)10:00 (B)12:00 (C)13:00 (D)16:00 (E)19:00

8. If Mary buys 3 dolls, she will have 150 dollars left. If she buys 4 dolls, she will
be 50 dollars short. All the dolls are at the same price. How much money does
Mary have on her?

(A)450 (B)550 (C)650 (D)750 (E)850

137
MP 3
9. How many four-digit numbers using each of 0, 2, 4 and 5 once are divisible by 5?
(A)4 (B)6 (C)8 (D)10 (E)12

10. Four teachers predict the order of finish among five classes, as shown in the table
below. Every prediction is wrong! In which place does Class 2 finish?
1st 2nd 3rd 4th 5th
Teacher A Class 4 Class 3 Class 2 Class 5 Class 1
Teacher B Class 4 Class 2 Class 5 Class 3 Class 1
Teacher C Class 3 Class 1 Class 4 Class 2 Class 5
Teacher D Class 1 Class 5 Class 2 Class 4 Class 3

(A)1st (B)2nd (C)3rd (D)4th (E)5th

Questions 11-20, 4 marks each

11. A new arithmetic operation “ ∇ ” is such that 6∇3 = 218 、 8∇ 4 = 232 and
9∇3 = 327 . What is the value of 10∇ 2 ?
(A)125 (B)205 (C)250 (D)520 (E)525

12. A bamboo stick is lowered vertically into water to a depth of 50 cm, and a mark
is made on the stick at the point of immersion. The stick is then taken out and
turned upside down. Then it is lowered vertically into water to a depth of 50 cm,
and a mark is made on the stick at the point of immersion. If the distance
between the two marks is 25 cm, what are the possible lengths, in cm, of the
bamboo stick?
(A)75 (B)100 (C)125 (D)75 or 125 (E)150 or 250

13. If a boy is chosen as captain, then the number of boys among the other members
is equal to the number of girls. If a girl is chosen as captain, then the number of
girls among the other members is one half the number of boys. How many boys
are there?
(A)2 (B)3 (C)4 (D)6 (E)8

138
MP 4
14. Use all of the numbers 1, 2, 3, 4, 5, 6, 7 and 8 to fill in the circles in the diagram
below so that the sum of the numbers in the four circles of each of the two
squares and each of the four trapezoids is constant. What is this constant sum?

(A)8 (B)10 (C)12 (D)16 (E)18

15. Four identical dice are stacked on a desk, as shown in the diagram below. How
many spots are on the face of the bottom die touching the top of the desk?

(A)1 (B)3 (C)4 (D)5 (E)6

16. C and D are two points on the segment AB, as shown in the diagram below. If AD
is 30 cm longer than BD and AC is 14 cm shorter than BC, what is the length, in
cm, of CD?

A C D B

(A)22 (B)24 (C)26 (D)28 (E)30

17. Simultaneously, X and Y start going from A to B while Z goes from B to A, each
at a different constant speed. Z's speed is 3 times that of X. Ten minutes after
their start, Y meets Z, and 10 minutes later, X meets Z. How many minutes will it
take for Y to complete the whole trip?
(A)10 (B)12 (C)14 (D)16 (E)18

139
MP 5
18. Hanna makes four straight cuts on a round cake. Into at most how many pieces
can she cut it?

(A)8 (B)10 (C)11 (D)12 (E)14

19. Eight children are sharing 61 balloons. Each gets at least one, and everyone
receives a different number. What is the smallest number of balloons that can go
to the child receiving more balloons than the others?
(A)10 (B)12 (C)14 (D)16 (E)18

20. In how many ways can we choose six different numbers from 1, 2, 3, 4, 5, 6, 7
and 8 so that their sum is a multiple of 4?
(A)6 (B)8 (C)12 (D)16 (E)24

Questions 21-25, 6 marks each


21. In reading a story book, Lance reads one page more each day than the preceding
day. On the fourth day, he reads 39 pages. After 9 days, he still has 48 pages to go.
How many pages are there in this book?

22. The diagram below shows two overlapping rectangles consisting of region I,
region II and the shaded region. The area of the rectangle ABCD is 6 times the
area of the shaded region. The total area of the regions I and II is 9 times the area
of the shaded region. The region II has area 12 cm2 . What is the area, in cm2 , of
the shaded region?
A B

D C
II

140
MP 6
23. Fanny has 20 coins each worth 5 pence. Trading some of them for coins each
worth 2 pence, she ends up with 32 coins. Then she trades some more 5-pence
coins for coins each worth 1 penny, and now she has 56 coins. How many
5-pence coin does Fanny still have?

24. Each of 1, 2, 3, 4, 5, 6, 7, 8 and 9 is written on one card. A takes away two cards
whose numbers have sum 14. B takes two cards whose numbers differ by 6. C
takes away two cards with numbers whose numbers have product 12. D takes
away two cards such that the number on one of them is twice the number on the
other. What number is on the card not taken away?

25. The sum of a three-digit number and a two-digit number is 199. The five digits
among the two numbers are all different. How many such three-digit numbers are
there?

***

141
142
注意:

允許學生個人、非營利性的圖書館或公立學校合理使用
本基金會網站所提供之各項試題及其解答。可直接下載
而不須申請。

重版、系統地複製或大量重製這些資料的任何部分,必
須獲得財團法人臺北市九章數學教育基金會的授權許
可。

申請此項授權請電郵 ccmp@seed.net.tw

Notice:

Individual students, nonprofit libraries, or schools are


permitted to make fair use of the papers and its
solutions. Republication, systematic copying, or
multiple reproduction of any part of this material is
permitted only under license from the Chiuchang
Mathematics Foundation.
Requests for such permission should be made by
e-mailing Mr. Wen-Hsien SUN ccmp@seed.net.tw

143
Solution to
Sixth International Mathematics Assessment for Schools
Round 1 of Middle Division
1. What is the value of 2016 − 20 × 16 ?
(A)1696 (B)1698 (C)1706 (D)1716 (E)1726
【Solution】
2016 − 20 × 16 = 2016 − 320 = 1696 .
Answer:(A)
2. Cubes of side length 1 cm are available in two colors. They are used to form a
large cube of side length 3 cm, and two small cubes sharing a common face have
different colors. What is the positive difference between the numbers of cubes in
the two colors?
(A)0 (B)1 (C)2 (D)3 (E)4

【Solution】
Since any two small cubes sharing a common face have different colors, the
arrangement of the cubes at the top, middle and bottom levels should be of the
following forms respectively:

The positive difference between the numbers of cubes in the two colors is 1.
Answer:(B)

144
3. How many different rectangles (including squares) in different positions are there
in the diagram below?

(A)5 (B)6 (C)7 (D)8 (E)9


【Solution】
In the diagram, there are two kinds of rectangles:
(i) those formed by a single rectangle (including square):
there are 5 such rectangles in different positions;
(ii) those formed by two rectangles:
in , there is one such rectangle; in , there is another such rectangle;
so there are 2 such rectangles in different positions.
Thus there are totally 5 + 2 = 7 rectangles in different positions.
Answer:(C)

4. In a 6 by 6 table, each row and each column contains the letters A, B, C, D, E and
F exactly once. All but four of the squares have been filled in. Which of the
following should be chosen to fill in the blank squares?
A B C D E F
B C D E F A
C D E F A B
D ○
1 ○
2 A B C
E ○
3 ○
4 B C D
F A B C D E

E F E F E F
(A) (B) (C)
C D F A A B

A B B C
(D) (E)
C D C D
【Solution】
Since there is no A in the fifth row and in the third column, ○
4 should be filled with A.
Similarly, ○
1 should be filled with E. Finally ○
2 and ○
3 should be filled with F.
Answer:(B)

145
5. The letters IMAS are spelt out with the same size of black and white circles as
shown in the diagram below. If the total area of the black circles is 50 cm2 , what
is the total area, in cm2 , of the white circles?

(A)26 (B)28 (C)30 (D)32 (E)34


【Solution】
There are 25 black circles and 15 white circles. Since the total area of the black
circles is 50 cm2 , the area of each circle is 50 ÷ 25 = 2 cm2. So the total area of the
white circles is 2 × 15 = 30 cm2.
Answer:(C)

6. Barry is flying from A to B on a 13:00 flight. The distance between A and B is


1800 km, and the speed of the plane is 600 km per hour. If the flight is delayed
for 2 hours and no time difference between A and B, when will Barry arrive at B?
(A)15:00 (B)16:00 (C)17:00 (D)18:00 (E)19:00
【Solution】
The flying time is 1800 ÷ 600 = 3 hours from A to B and the flight started at 15:00.
So Barry will arrive B at 18:00.
Answer:(D)

7. Jane feeds her cat at regular intervals. The first feeding is at 7:00 and the sixth
feeding is at 22:00. Which of the following is not a feeding time for the cat?

(A)10:00 (B)12:00 (C)13:00 (D)16:00 (E)19:00


【Solution】
Jane feeds her cat six times from 7:00 to 22:00. So the interval between successive
feedings is (22 − 7) ÷ 5 = 3 hours. Thus the feeding times are 7:00, 10:00, 13:00,
16:00, 19:00 and 22:00, except 12:00.
Answer:(B)

146
8. If Mary buys 3 dolls, she will have 150 dollars left. If she buys
4 dolls, she will be 50 dollars short. All the dolls are at the
same price. How much money does Mary have on her?
(A)450 (B)550 (C)650
(D)750 (E)850
【Solution】
The price of each doll is 150 + 50 = 200 dollars. So Mary has 3 × 200 + 150 = 750
dollars.
Answer:(D)
9. How many four-digit numbers using each of 0, 2, 4 and 5 once are divisible by 5?
(A)4 (B)6 (C)8 (D)10 (E)12
【Solution】
The units digit of a four-digit number is 0 or 5 if it is divisible by 5.
If the units digit of the four-digit number is 0, then there are 6 four-digit numbers,
namely 2450, 2540, 4250, 4520, 5240 and 5420, satisfying the conditions.
If the units digit of the four-digit number is 5, then there are only 4 four-digit
numbers, namely 2045, 2405, 4025 and 4205, satisfying the conditions since the
leading digit of a four-digit number cannot be 0.
So there are totally 6 + 4 = 10 four-digit numbers satisfying the conditions.
Answer:(D)
10. Four teachers predict the order of finish among five classes, as shown in the table
below. Every prediction is wrong! In which place does Class 2 finish?
1st 2nd 3rd 4th 5th
Teacher A Class 4 Class 3 Class 2 Class 5 Class 1
Teacher B Class 4 Class 2 Class 5 Class 3 Class 1
Teacher C Class 3 Class 1 Class 4 Class 2 Class 5
Teacher D Class 1 Class 5 Class 2 Class 4 Class 3
(A)1st (B)2nd (C)3rd (D)4th (E)5th
【Solution】
Since teacher A, B, C and D predict Class 2 to be 3rd, 2nd, 4th and 3rd, respectively, and
all of their predictions are wrong, Class 2 finishes in 1st or 5th place.
Since teacher A, B, C and D predict Class 5 to be 4th, 3rd, 5th and 2nd, respectively, and
all of their predictions are wrong, Class 5 finishes in 1st place.
Now we can conclude that Class 2 finishes in 5th place.
Answer:(E)
11. A new arithmetic operation “ ∇ ” is such that 6∇3 = 218 、 8∇ 4 = 232 and
9∇3 = 327 . What is the value of 10∇ 2 ?
(A)125 (B)205 (C)250 (D)520 (E)525
【Solution】
The product of the two given numbers forms the last two digits of the result and the
leading digit of the result is the quotient when the first number is divided by the
second. So 10∇ 2 = 520 .
Answer:(D)

147
12. A bamboo stick is lowered vertically into water to a depth of 50 cm, and a mark
is made on the stick at the point of immersion. The stick is then taken out and
turned upside down. Then it is lowered vertically into water to a depth of 50 cm,
and a mark is made on the stick at the point of immersion. If the distance
between the two marks is 25 cm, what are the possible lengths, in cm, of the
bamboo stick?
(A)75 (B)100 (C)125 (D)75 or 125 (E)150 or 250
【Solution】
If the bamboo stick was immersed twice in water without overlapping portions, the
length of the bamboo is 50 + 50 + 25 = 125 cm. If the bamboo stick was immersed
twice in water with overlapping portions, the length of the bamboo is
50 + 50 − 25 = 75 cm. So the possible lengths of the bamboo stick is 75 or 125 cm.
Answer:(D)

13. If a boy is chosen as captain, then the number of boys among the other members
is equal to the number of girls. If a girl is chosen as captain, then the number of
girls among the other members is one half the number of boys. How many boys
are there?
(A)2 (B)3 (C)4 (D)6 (E)8
【Solution1】
When boy is chosen as captain, the number of boys among the other members is
equal to the number of girls. So the number of boys is 1 more than the number of
girls and hence when a girl is chosen as captain, the number of boys is 2 more than
the number of girls among the other members. As we already know that in this
situation, the number of girls among the other members is one half the number of
boys, so the number of girls among the other members is 2. Thus the number of boys
is 2 × 2 = 4 .
【Solution2】
Assume there are x boys. Then there are x − 1 girls. From the given conditions,
x = 2( x − 1 − 1) . Thus x = 4 .
Answer:(C)

14. Use all of the numbers 1, 2, 3, 4, 5, 6, 7 and 8 to fill in the circles in the diagram
below so that the sum of the numbers in the four circles of each of the two
squares and each of the four trapezoids is constant. What is this constant sum?

(A)8 (B)10 (C)12 (D)16 (E)18

148
【Solution】
If we add the sums of the numbers in the four circles of the two squares and the four
trapezoids, then the number in each circle is counted three times. So the sum is
3 × (1 + 2 + 3 + 4 + 5 + 6 + 7 + 8) = 108 . Since the sum of the numbers in the four circles
of each of the two squares and each of the four trapezoids is constant, this constant is
108 ÷ 6 = 18 . The following figure shows a way to fill those numbers.

7 2
1 8

4 5
6 3
Answer:(E)
15. Four identical dice are stacked on a desk, as shown in the diagram below. How
many spots are on the face of the bottom die touching the top of the desk?

(A)1 (B)3 (C)4 (D)5 (E)6


【Solution】
For the top die, the face containing 1 spot is adjacent to the faces containing 2 and 5
spots. For the second die, the face containing 1 spot is adjacent to the faces
containing 4 spots. For the third die, the face containing 1 spot is adjacent to the faces
containing 3 spots. Thus we know the face containing 6 spots is the opposite face of
the face containing 1 spot. For the bottom die, the front face is the face containing 2
spots and the right face is the face containing 4 spots, so the face of the bottom die
touching the top of the desk contains 1 or 6 spots. If it contains 1 spot, then the right
face contains 5 spots, which is a contradiction. So the face of the bottom die touching
the top of the desk contains 6 spots.
Answer:(E)
16. C and D are two points on the segment AB, as shown in the diagram below. If AD
is 30 cm longer than BD and AC is 14 cm shorter than BC, what is the length, in
cm, of CD?
A C D B

(A)22 (B)24 (C)26 (D)28 (E)30

149
【Solution】
From the given conditions, AD-BD = AC + CD-BD = 30 cm and BC-AC = BD +
CD-AC = 14 cm. Adding the two equations, we get 2CD = 44 cm. Thus CD = 22
cm.
Answer:(A)
17. Simultaneously, X and Y start going from A to B while Z goes from B to A, each
at a different constant speed. Z's speed is 3 times that of X. Ten minutes after
their start, Y meets Z, and 10 minutes later, X meets Z. How many minutes will it
take for Y to complete the whole trip?
(A)10 (B)12 (C)14 (D)16 (E)18
【Solution 1】
Clearly, twenty minutes after their start, X meets Z. Since the speed of Z is three
1 3
times that of X, X completes the of the trip and Z completes the of the trip.
4 4
3 3
Thus Z completes the ÷ 2 = of the trip in ten minutes and hence Y completes the
4 8
3 5 5
1 − = of the trip in ten minutes. So Y needs 10 ÷ = 16 minutes to complete the
8 8 8
whole trip.
【Solution 2】
Assume the speed of X is x, then the speed of Z is 3x. Twenty minutes after their start,
X meets Z. So the length of the trip is (3x + x) × 20 = 80 x . And ten minutes after their
start, Y meets Z. So the sum of the speed of Y and Z is 80 x ÷ 10 = 8 x . Thus the speed
of Y is 8 x − 3 x = 5 x and hence Y needs 80 x ÷ 5 x = 16 minutes to complete the
whole trip.
Answer:(D)
18. Hanna makes four straight cuts on a round cake. Into at
most how many pieces can she cut it?
(A)8 (B)10 (C)11
(D)12 (E)14
【Solution】
We need to find how many parts a circle can be divided into at
most by 4 lines. A circle can be divided into at most 2 parts by a
line. If we draw another line, then each side of the original line
can be divided into at most 2 parts. So a circle can be divided
into 4 parts at most by two lines. If we draw the third line, then
each side of the original two lines can be divided into at most 2
parts. But one part is counted twice, hence a circle can be
divided into at most 4 + 3 = 7 parts by three lines. If we draw the
fourth line, then each side of the original three lines can be
divided into at most 2 parts. But two parts are both counted
twice, hence a circle can be divided into at most 7 + 4 = 11 parts by four lines.
Answer:(C)

150
19. Eight children are sharing 61 balloons. Each gets at least one, and everyone
receives a different number. What is the smallest number of balloons that can go
to the child receiving more balloons than the others?
(A)10 (B)12 (C)14 (D)16 (E)18
【Solution 1】
In order to get the smallest number of balloons that can go to the child receiving more
balloons than the others, the other seven children should get as many balloons as
possible. Suppose the eight children receive 1, 2, 3, 4, 5, 6, 7 and 8 balloons first and
hence they get totally 1 + 2 + 3 + 4 + 5 + 6 + 7 + 8 = 36 balloons. There are still
61 − 36 = 25 balloons left. Next, let each child receives 3 more balloons. Thus the
eight children receive 4, 5, 6, 7, 8, 9, 10 and 11 balloons. Finally, let the child who
receives 11 balloons receives the last balloon. So the smallest number of balloons that
can go to the child receiving more balloons than the others is 12.
【Solution 2】
If the number of balloons that can go to the child receiving more balloons than the
others is 11, than there are at most 4 + 5 + 6 + 7 + 8 + 9 + 10 + 11 = 60 balloons since
everyone receives a different number. This contradicts with that they are sharing 61
balloons. So the number of balloons that can go to the child receiving more balloons
than the others is 12 while the eight children receive 4, 5, 6, 7, 8, 9, 10 and 12
balloons.
Answer:(B)

20. In how many ways can we choose six different numbers from 1, 2, 3, 4, 5, 6, 7
and 8 so that their sum is a multiple of 4?
(A)6 (B)8 (C)12 (D)16 (E)24
【Solution】
Since 1 + 2 + 3 + 4 + 5 + 6 + 7 + 8 = 36 is a multiple of 4, as we choose six different
numbers so that their sum is a multiple of 4, the sum of the remainder two numbers
must be a multiple of 4, and vice versa. The ways to choose two different numbers so
that their sum is a multiple of 4 are (1, 3), (1, 7), (2, 6), (4, 8), (5, 3) and (5, 7). There
are 6 ways.
Answer:(A)

21. In reading a story book, Lance reads one page more each day than the preceding
day. On the fourth day, he reads 39 pages. After 9 days, he still has 48 pages to go.
How many pages are there in this book?
【Solution】
Since Lance reads 39 pages on the fourth day, he reads 39 − 3 = 36 pages on the first
day. So he has read 36, 37, 38, 39, 40, 41, 42, 43 and 44 pages in the first nine days.
Thus there are 36 + 37 + 38 + 39 + 40 + 41 + 42 + 43 + 44 + 48 = 408 pages in this
book.
Answer:408

151
22. The diagram below shows two overlapping rectangles consisting of region I,
region II and the shaded region. The area of the rectangle ABCD is 6 times the
area of the shaded region. The total area of the regions I and II is 9 times the area
of the shaded region. The region II has area 12 cm2 . What is the area, in cm2 , of
the shaded region?
A B

D C
II

【Solution】
The area of the region I is 6 − 1 = 5 times of the area of the shaded region, so the
area of the region II is 9 − 5 = 4 times of the area of the shaded region. Since the
region II has area 12 cm2 , the area of the shaded region is 12 ÷ 4 = 3 cm2.
Answer:003

23. Fanny has 20 coins each worth 5 pence. Trading some of them for coins each
worth 2 pence, she ends up with 32 coins. Then she trades some more 5-pence
coins for coins each worth 1 penny, and now she has 56 coins. How many
5-pence coin does Fanny still have?

【Solution】
Two 5-pence coins can be exchanged with five 2-pence coins and hence there are 3
more coins after such an exchange. Since Fanny has 32 − 20 = 12 more coins after
the first trading, she has taken 12 ÷ 3 × 2 = 8 5-pence coins for the first trading.
Similarly, one 5-pence coin can be exchanged with five 1-penny coins and hence
there are 4 more coins after such an exchange. After second trading, Fanny has
56 − 32 = 24 more coins, so she has taken 24 ÷ 4 = 6 5-pence coins for the second
trading. Thus she still have 20 − 14 = 6 5-pence coins.
Answer:006

24. Each of 1, 2, 3, 4, 5, 6, 7, 8 and 9 is written on one card. A takes away two cards
whose numbers have sum 14. B takes two cards whose numbers differ by 6. C
takes away two cards with numbers which have product 12. D takes away two
cards such that the number on one of them is twice the number on the other.
What number is on the card not taken away?

152
【Solution】
C takes away two cards with numbers whose numbers have product 12 and from 1, 2,
3, 4, 5, 6, 7, 8 and 9, we know 12 = 2 × 6 = 3 × 4 . So C takes away 2 and 6 or 3 and 4.
D takes away two cards such that the number on one of them is twice the number on
the other and from 1, 2, 3, 4, 5, 6, 7, 8 and 9, we know 2 = 2 ÷ 1 = 6 ÷ 3 = 8 ÷ 4 .
If C takes away 3 and 4, then D takes away 1 and 2. Now the remainder cards are 5, 6,
7, 8 and 9. There are no two cards whose numbers differ by 6. This is impossible.
If C takes away 2 and 6, then D takes away 4 and 8. Now the only two possible cards
taken away by A are 5 and 9 and the remainder cards are 1, 3 and 7. So B takes away
1 and 7 and hence 3 is not taken away on the card.
Answer:003

25. The sum of a three-digit number and a two-digit number is 199. The five digits
among the two numbers are all different. How many such three-digit numbers are
there?
【Solution】
The hundreds digit of the three-digit number must be 1.
Since all digits of these numbers are different, if the tens digits of the three-digit
number is 0, then since the maximum sum of the units digits of these two numbers is
8 + 9 = 17 , which is not 19, there is no carrying forward in the whole calculation and
the tens digit of the two-digit number is 9. In this case the sum of the unit digits of
these two number is 9 = 2 + 7 = 3 + 6 = 4 + 5 , so we have 3 × 2 = 6 possible
three-digit numbers.
If the tens digit of the three-digit number is not 0, then the pair of the tens digits of
these two numbers is one of (2, 7), (3, 6) and (4, 5). So we have 3 × 2 = 6 possible
tens digits of the three-digit number. And the pair of the unit digits of these two
numbers is one of (2, 7), (3, 6), (4, 5) and (0, 9) but different from the pair of the
tenth digits. So we also have 3 × 2 = 6 possible unit digits of the three-digit number.
Hence we have 3× 2 × 3× 2 = 36 possible three-digit numbers.
Thus there are 6 + 36 = 42 such three-digit numbers.
Answer:042

153
注意:

允許學生個人、非營利性的圖書館或公立學校合理使用
本基金會網站所提供之各項試題及其解答。可直接下載
而不須申請。

重版、系統地複製或大量重製這些資料的任何部分,必
須獲得財團法人臺北市九章數學教育基金會的授權許
可。

申請此項授權請電郵 ccmp@seed.net.tw

Notice:

Individual students, nonprofit libraries, or schools are


permitted to make fair use of the papers and its
solutions. Republication, systematic copying, or
multiple reproduction of any part of this material is
permitted only under license from the Chiuchang
Mathematics Foundation.
Requests for such permission should be made by
e-mailing Mr. Wen-Hsien SUN ccmp@seed.net.tw

154
6th International Mathematics Assessments for Schools
(2016-2017 )

Middle Primary Division Round 2


Time: 120 minutes

Printed Name: Code: Score:

Instructions:
Do not open the contest booklet until you are told to do so.
Be sure that your name and code are written on the space provided above.
Round 2 of IMAS is composed of three parts; the total score is 100 marks.
Questions 1 to 5 are given as a multiple-choice test. Each question has five
possible options marked as A, B, C, D and E. Only one of these options is correct.
After making your choice, fill in the appropriate letter in the space provided. Each
correct answer is worth 4 marks. There is no penalty for an incorrect answer.
Questions 6 to 13 are a short answer test. Only Arabic numerals are accepted;
using other written text will not be honored or credited. Some questions have more
than one answer, as such all answers are required to be written down in the space
provided to obtain full marks. Each correct answer is worth 5 marks. There is no
penalty for incorrect answers.
Questions 14 and 15 require a detailed solution or process in which 20 marks are
to be awarded to a completely written solution. Partial marks may be given to an
incomplete presentation. There is no penalty for an incorrect answer.
Use of electronic computing devices is not allowed.
Only pencil, blue or black ball-pens may be used to write your solution or answer.
Diagrams are not drawn to scale. They are intended as aids only.
After the contest the invigilator will collect the contest paper.
The following area is to be filled in by the judges;
the contestants are not supposed to mark anything here.
Total
Question 1 2 3 4 5 6 7 8 9 10 11 12 13 14 15 Score Signature

Score

Score

155
156
Middle Primary Division Round 2

Questions 1 to 5, 4 marks each

1. Among the numbers 5, 7, 11, 15 and 19, which number cannot be expressed as a
sum of two prime numbers?
(A)5 (B)7 (C)11 (D)15 (E)19

Answer:

2. There is a dart game in an amusement park where the goal is to hit the black
portions of the regular hexagon in order to win a prize. From the choices below,
which dart board will give the highest chance of winning?
(A) (B) (C)

(D) (E)

Answer:

3. What is the units digit of the value of 2 × 2 × ⋯ × 2 + 3 × 3 × ⋯ × 3 ?


2016 terms 2017 terms

(A)1 (B)3 (C)6 (D)7 (E)9

答:

4. Alex bought a total of nine pencils and ball pens. One ball pen costs $3 each,
while one pencil costs $2 each. If Alex spent a total of $22, how many ball pens
did he buy?
(A)2 (B)3 (C)4 (D)5 (E)6

Answer:

157
MP 2
5. In each of the following options, the dimensions of the rectangle are 10 cm by
6 cm. There are some shaded triangles inside each rectangle. The vertices of the
shaded triangles must be either at the endpoints of a line segment or points that
divide it into equal parts. From the options below, which figure has the largest
shaded region?
(A) (B) (C)

(D) (E)

Answer:

Questions 6 to 13, 5 marks each

6. In the diagram below, a square of area 81 cm2 is made up of three small identical
rectangles. What is the perimeter of one small rectangle?

Answer: cm

7. In adding up two numbers, Max has mistakenly read the tens digits ‘3’ of the first
number as ‘5’ and the hundreds digit ‘9’ of the second number as ‘6’. If Max got
2017 as the result, what should be the correct sum of the two numbers?
Answer:

8. A hotel staff distributes four room keys to four travelers randomly. It is known
that exactly two of the travelers get their correct room keys. How many different
ways are there of distributing the keys to the four travelers?
Answer: ways

158
MP 3
9. Refer to the diagram below, where 9 black bricks and 4 white bricks are arranged
alternately such that it will form a figure in which there are 5 bricks on its
diagonals, and all the outer bricks are black.

Using the similar pattern stated above, if we want to form figures in which there
are 7 bricks on its diagonals, how many black bricks are required?
Answer: black bricks

10. The average weight of five cars A, B, C, D and E is 200 kg. If the average weight
of cars A, B and C is 198 kg, while the average weight of cars C, D and E is
203 kg, what is the weight of car C, in kg?

Answer: kg

11. Fill in each ○ in the expression below with digits 1, 2, 3, 4, 5 and 6. Each digit
can be used once and two 3-digit numbers are formed. What is the minimum
difference of the expression below?
○○○-○○○=

Answer:

12. Six pairs of integers are formed from the list of positive integers 1, 2, 3, 4, 5, 6, 7,
8, 9, 10, 11 and 12. It is known that the sums of the five pairs are: 4, 6, 14, 20 and
21. What is the product of the remaining pair of numbers?

Answer:

13. The sum of two three-digit numbers is 999. If the digits in the two three-digit
numbers are all different without repetition, how many possible combinations are
there?

Answer: combinations

159
MP 4

Questions 14 to 15, 20 marks each


(Detailed solutions are needed for these two problems)
14. Fill in the circles with the numbers 1, 2, 3, 4, 5, 6 and 7. Each number can be
used once without repetitions. The sum of the digits inside the circles at the four
vertices of the square on the left is 15 and the sum of the digits inside the circles
at the vertices of the regular pentagon on the right is 24. How many possible
arrangements are there?

Answer: arrangements

160
MP 5
15. A bag contains 2017 balls that are numbered from 1 to 2017. At least how many
balls must be taken out so that among those balls, there will always be 3 balls, in
which the sum of the numbers on the first two balls is the number on the third
ball?

Answer: balls

161
162
注意:

允許學生個人、非營利性的圖書館或公立學校合理使用
本基金會網站所提供之各項試題及其解答。可直接下載
而不須申請。

重版、系統地複製或大量重製這些資料的任何部分,必
須獲得財團法人臺北市九章數學教育基金會的授權許
可。

申請此項授權請電郵 ccmp@seed.net.tw

Notice:

Individual students, nonprofit libraries, or schools are


permitted to make fair use of the papers and its
solutions. Republication, systematic copying, or
multiple reproduction of any part of this material is
permitted only under license from the Chiuchang
Mathematics Foundation.
Requests for such permission should be made by
e-mailing Mr. Wen-Hsien SUN ccmp@seed.net.tw

163
─────────────────────────────────────────────────
Solution Key to Second Round of IMAS 2016/2017
Middle Primary Division
─────────────────────────────────────────────────
1. Among the numbers 5, 7, 11, 15 and 19, which number cannot be expressed as a
sum of two prime numbers?
(A)5 (B)7 (C)11 (D)15 (E)19
【Suggested Solution】
Note that 5 = 2 + 3 , 7 = 2 + 5 , 15 = 2 + 13 and 19 = 2 + 17 .
Suppose that 11 can be expressed as a sum of two prime numbers, then one must be
an odd and the other must be an even. Since the only even prime number is 2, the
other number is 9, which is not a prime number, thus, a contradiction. Therefore the
answer is(C)
Answer:(C)
2. There is a dart game in an amusement park where the goal is to hit the black
portions of the regular hexagon in order to win a prize. From the choices below,
which dart board will give the highest chance of winning?
(A) (B) (C)

(D) (E)

【Suggested Solution】
By drawing line segments on figures (A), (B), (C), (D) and (E): (refer to the figures
below.).

(A) (B) (C) (D) (E)

2 4 3 6
Observe that the shaded area of figure (A) is = , shaded area of (B) is = ,
6 12 6 12
4 5 5
shaded area of (C) is , shaded area of (D) is , and shaded area of (E) is .
12 12 12
6 5 4
Since > > , therefore, (B) has the best chance to win a prize.
12 12 12
Answer:(B)

164
3. What is the units digit of the value of 2 × 2 × ⋯ × 2 + 3 × 3 × ⋯ × 3 ?
2016 terms 2017 terms

(A)1 (B)3 (C)6 (D)7 (E)9


【Suggested Solution】
The units digit of 2 × 2 × ⋯ × 2 cycles through the numbers 2, 4, 8 and 6. While the
n terms

units digit of 3 × 3 × ⋯ × 3 cycles through 3, 9, 7 and 1. Since 2016 = 4 × 504 , then


n terms

the units digit of 2 × 2 × ⋯ × 2 is 6. On the other hand, 2017 = 4 × 504 + 1 , then the
2016 terms

units digit of 3 × 3 × ⋯ × 3 is 3. Therefore the units digit of the value of the given
2017 terms

expression is 6 + 3 = 9 .
Answer:(E)
4. Alex bought a total of nine pencils and ball pens. One ball pen costs $3 each,
while one pencil costs $2 each. If Alex spent a total of $22, how many ball pens
did he buy?
(A)2 (B)3 (C)4 (D)5 (E)6
【Suggested Solution 1】
If Alex bought 9 ball pens, he should have spent a total of 9 × $3 = $27 , which is $5
more than his actual spent. Since the difference of the price of a ball pen and the
pencil is $3 − $2 = $1 , then the number of pencils bought is 5 ÷ 1 = 5 . Therefore the
number of ball pens bought is 9 − 5 = 4 .
【Suggested Solution 2】
If Alex bought 9 pencils, he should have spent a total of 9 × $2 = $18 , which is $4
less than his actual spent. Since the difference of the price of a ball pen and the pencil
is $3 − $2 = $1 , then the number of ball pens bought is 4 ÷ 1 = 4 .
Answer:(C)
5. In each of the following options, the dimensions of the rectangle are 10 cm by
6 cm. There are some shaded triangles inside each rectangle. The vertices of the
shaded triangles must be either at the endpoints of a line segment or points that
divide it into equal parts. From the options below, which figure has the largest
shaded region?
(A) (B) (C)

(D) (E)

165
【Suggested Solution 1】
By finding the areas of the shaded triangles.
1
Shaded area of figure (A) is 2 × × (10 ÷ 2) × 6 = 30 cm2;
2
Shaded area of figure (B) is
1 1
2 × × (10 ÷ 2) × (6 ÷ 2) + [10 × (6 ÷ 2) − 2 × × (10 ÷ 2) × (6 ÷ 2)] = 30 cm2;
2 2
1
Shaded area of figure (C) is 10 × × (10 ÷ 5) × (6 ÷ 2) = 30 cm2;
2
1
Shaded area of figure (E) is 10 × × (6 ÷ 2) × (10 ÷ 5) = 30 cm2.
2
Note that areas of figures (A), (B), (C) and (E) are all equal. Now we examine figure
(D).
Note that the shaded parts of figure (D) are equally divided into two equal parts by a
diagonal of the rectangle, with the sum of the areas equal.
Now, observe the diagonals of the 5 small triangles on the bottom half of the
rectangle. We can see that the bases of these five triangles are all equal to 2 cm, and
their heights are 1.2 cm, 2.4 cm, 3.6 cm, 4.8 cm and 6 cm, respectively, from left to
right which is in the ratio 1: 2: 3: 4: 5. Therefore, the sum of the area of the five
1
shaded triangles is × (1.2 + 2.4 + 3.6 + 4.8 + 6) × 2 = 18 cm2; therefore the total
2
shaded area of the figure is 36 cm2
Therefore, the figure that has the largest shaded region is figure (D).

【Suggested Solution 2】
By drawing line segments in the figures (A), (B), (C), and (E) below, observe that the
shaded areas of each figure is equal to the half of the total area of the rectangle.

(A) (B) (C) (D)


While in figure (D), observe that the sum of the area is greater than half of the entire
rectangle.

d e
a b c a
b
d c
e

Answer:(D)

166
6. In the diagram below, a square of area 81 cm2 is made up of three small identical
rectangles . What is the perimeter of one small rectangle?

【Suggested Solution】
Since the area of the square is 81 cm2, the side length of the square is 9 cm. Then the
width of each small rectangle is 3 cm. Therefore the perimeter of each small rectangle
is 2 × (3 + 9) = 24 cm.
Answer:24 cm
7. In adding up two numbers, Max has mistakenly read the tens digits ‘3’ of the first
number as ‘5’ and the hundreds digit ‘9’ of the second number as ‘6’. If Max got
2017 as the result, what should be the correct sum of the two numbers?
【Suggested Solution】
Mistakenly read the tens digit ‘3’ as ‘5’ causes an increase of 20 in the result.
Mistakenly read the hundreds digit ‘9’ as ‘6’ causes a decrease of 300 in the result.
Therefore the correct sum should be 2017 − 20 + 300 = 2297 .
Answer:2297
8. A hotel staff distributes four room keys to four travelers randomly. It is known
that exactly two of the travelers get their correct room keys. How many different
ways are there of distributing the keys to the four travelers?
【Suggested Solution】
4×3
Since 2 of the 4 travelers get their correct keys, then there is a total of = 6 ways
2
in doing this. Moreover, the 2 remaining travelers must be getting their correct keys,
so there is only 1 way in doing such. So the total number of ways is 6 × 1 = 6 ways.
Answer:6 ways
9. Refer to the diagram below, where 9 black bricks and 4 white bricks are arranged
alternately such that it will form a figure in which there are 5 bricks on its
diagonals, and all the outer bricks are black.

Using the similar pattern stated above, if we want to form figures in which there
are 7 bricks on its diagonals, how many black bricks are required?

167
【Suggested Solution 1】
Refer to the diagram on the right, you can see that the number
of black bricks used is 1 + 2 + 3 + 4 + 3 + 2 + 1 = 42 = 16 .
【Suggested Solution 2】
When there are 7 bricks on its diagonals, each line has 4 black
bricks, and will use 4 lines, therefore there is a total of
4 × 4 = 16 black bricks used.
【Suggested Solution 3】
Enclose the figure with a big square as shown. The total area of the white bricks is
same as that of the black bricks. Suppose each brick of side length equals 1, the area
1
of the big square is × 8 × 8 = 32 . Therefore, the total number of black bricks is
2
1
× 32 = 16 .
2
Answer:16 black bricks
10. The average weight of five cars A, B, C, D and E is 200 kg. If the average weight
of cars A, B and C is 198 kg, while the average weight of cars C, D and E is 203
kg, what is the weight of car C, in kg?
【Suggested Solution】
The total weight of cars A, B, C, D and E is 200 × 5 = 1000 kg. The total weight of cars
A, B and C is 198 × 3 = 594 kg, and the total weight of cars C, D and E is
203 × 3 = 609 kg. Therefore, the weight of C is 594 + 609 − 1000 = 203 kg.
Answer:203 kg
11. Fill in each ○ in the expression below with digits 1, 2, 3, 4, 5 and 6. Each digit
can be used once and two 3-digit numbers are formed. What is the minimum
difference of the expression below?
○○○ − ○○○=
【Suggested Solution】
All the digits are different. In order to make the difference minimum: the hundreds
digits of the two numbers should be of difference equals 1. Next, consider the
two-digit numbers formed by the tens and units digits of the minuend and the
subtrahend respectively. The minuend should be made smallest while the subtrahend
should be made the greatest.
So the minimum value of the expression is 412 − 365 = 47 .
Answer:47
12. Six pairs of integers are formed from the list of positive integers 1, 2, 3, 4, 5, 6, 7,
8, 9, 10, 11and 12. It is known that the sums of the five pairs are: 4, 6, 14, 20 and
21. What is the product of the remaining pair of numbers?
【Suggested Solution】
There is only one possible way to obtain the sum equals 4: 4 = 1 + 3
It follows that there is only way to obtain the sum equals 6 : 6 = 2 + 4
There are two possible ways to obtain the sum equals 21 and 20 respectively:
21 = 9 + 12 = 10 + 11 ; 20 = 8 + 12 = 9 + 11

168
Since each number can be used once, we should have 21 = 10 + 11 and 20 = 8 + 12 .
From this, we can deduce that 9 + 5 = 14 , and the remaining pair is 6 and 7, and their
product is 42.
Answer:42
13. The sum of two three-digit numbers is 999. If the digits in the two three-digit
numbers are all different without repetition, how many possible combinations are
there?
【Suggested Solution】
Since all the digits are different, in adding the ones, tens and hundreds digit to get a
sum of 9, there will not be any carry-overs. Therefore the only five possible pairs of
digits to get a sum of 9 : (9, 0), (8, 1), (7, 2), (6, 3), (5, 4). Note that there are 4 ways
to choose the hundreds digit ( (9, 0) cannot be used because the hundreds digit cannot
be 0), there are 4 × 2 = 8 ways to choose the tens digit, and 3 × 2 = 6 ways to
choose the units digit. Therefore, there are a total number of 4 × (4 × 2) × (3 × 2) = 192
ways that satisfies the condition.
Answer:192 combinations
14. Fill in the circles with the numbers 1, 2, 3, 4, 5, 6 and 7. Each number can be
used once without repetitions. The sum of the digits inside the circles at the
four vertices of the square on the left is 15 and the sum of the digits inside the
circles at the vertices of the regular pentagon on the right is 24. How many
possible arrangements are there?

【Suggested Solution】
Since the sum of all the digits is 1 + 2 + 3 + 4 + 5 + 6 + 7 = 28 , we know that the sum
of the 2 circles in the middle is 15 + 24 − 28 = 11 . (Refer to the figure below).

Right
Left Middle

Sum: 4 Sum: 11
Sum: 13

169
From this, we know that the only possible combination to get the sum of 4 for the
left-most column is 1 and 3(which can be interchanged). Total of 2 × 1 = 2 ways.
(5 points)
For the middle row, the only possible values to gett 11 = 5 + 6 = 4 + 7 , 2 ways and
can be interchanged. Total of 2 × 2 = 4 ways. (5 points)
For the right most column, possible value are 2, 4 and 7 (if 5 and 6 are used in the
middle column) and 2, 5 and 6 (if 4 and 7 are used in the middle column), and we can
arrange the numbers to be placed in the leftmost column in 3 × 2 ×1 = 6 ways.
(5 points)
Therefore, the total number of possible arrangements is 2 × 4 × 6 = 48 ways.
(5 points)
Answer:48 ways
15. A bag contains 2017 balls that are numbered from 1 to 2017. At least how many
balls must be taken out so that among those balls, there will always be 3 balls, in
which the sum of the numbers on the first two balls is the number on the third
ball.
【Suggested Solution】
Since 2017 = 1008 + 1009 < 1009 + 1010 , suppose we take out 1009~2017, which is a
total of 1009 numbers, we cannot ensure that there are three balls that will satisfy the
given conditions.(10 points)
So suppose we take 1010 balls out. Suppose the largest number out of the 1010 balls
is M, then the difference between M and the number of other balls taken out has 1009
different values, and both are less than 2017.(5 points) Since there are only 1007 balls
that have not been taken out, at least one difference M − x is the number y of the
balls taken out, where x is the number of the removed ball. So x, y, x + y = M are
taken out of the ball number which satisfies the condition above. (5 points)
Answer:1010 balls

170
注意:

允許學生個人、非營利性的圖書館或公立學校合理使用
本基金會網站所提供之各項試題及其解答。可直接下載
而不須申請。

重版、系統地複製或大量重製這些資料的任何部分,必
須獲得財團法人臺北市九章數學教育基金會的授權許
可。

申請此項授權請電郵 ccmp@seed.net.tw

Notice:

Individual students, nonprofit libraries, or schools are


permitted to make fair use of the papers and its
solutions. Republication, systematic copying, or
multiple reproduction of any part of this material is
permitted only under license from the Chiuchang
Mathematics Foundation.
Requests for such permission should be made by
e-mailing Mr. Wen-Hsien SUN ccmp@seed.net.tw

171
International Mathematics Assessments for Schools

2017 MIDDLE PRIMARY DIVISION FIRST ROUND PAPER


Time allowed:75 minutes

When your teacher gives the signal, begin working on the problems.

INSTRUCTION AND INFORMATION


GENERAL
1. Do not open the booklet until told to do so by your teacher.
2. No calculators, slide rules, log tables, math stencils, mobile phones or other
calculating aids are permitted. Scribbling paper, graph paper, ruler and compasses
are permitted, but are not essential.
3. Diagrams are NOT drawn to scale. They are intended only as aids.
4. There are 20 multiple-choice questions, each with 5 choices. Choose the most
reasonable answer. The last 5 questions require whole number answers between
000 and 999 inclusive. The questions generally get harder as you work through the
paper. There is no penalty for an incorrect response.
5. This is a mathematics assessment, not a test; do not expect to answer all questions.
6. Read the instructions on the answer sheet carefully. Ensure your name, school
name and school year are filled in. It is your responsibility that the Answer Sheet
is correctly coded.

THE ANSWER SHEET


1. Use only pencils.
2. Record your answers on the reverse side of the Answer Sheet (not on the question
paper) by FULLY filling in the circles which correspond to your choices.
3. Your Answer Sheet will be read by a machine. The machine will see all markings
even if they are in the wrong places. So please be careful not to doodle or write
anything extra on the Answer Sheet. If you want to change an answer or remove
any marks, use a plastic eraser and be sure to remove all marks and smudges.

INTEGRITY OF THE COMPETITION


The IMAS reserves the right to re-examine students before deciding whether to
grant official status to their scores.

172
173
2017 MIDDLE PRIMARY DIVISION FIRST ROUND PAPER

Questions 1-10, 3 marks each


1. What is the simplified value of 25 × 99 + 55 × 5 ?
(A)2750 (B)2850 (C)2900 (D)2950 (E)3000

2. In the arithmetic expression 1A + B1 = 100, what is the value of the digit B?


(A)5 (B)6 (C)7 (D)8 (E)9

3. As shown in the figure below, there is a 7 × 7 grid paper where each small
square has an area of 1 cm2, what is the area, in cm2, of the shaded part?

(A)18 (B)19 (C)20 (D)21 (E)22

4. Students are arranged in a square formation with 8 rows and 8 columns. Now, 3
rows and 3 columns are removed, how many students were reduced?
(A)9 (B)24 (C)27 (D)39 (E)45

5. A train travels between City A to City B. There are 4 other stops in-between these
two cities. How many different kinds of tickets are there? (Note: The ticket from
City A to City B and City B to City A must not be the same).
(A)8 (B)12 (C)20 (D)24 (E)30

6. In the figure shown below, a square is embedded in a rectangle. If AB = 9 cm


and CD = 15 cm, then what is the perimeter, in cm, of the rectangle?
C A B D

(A)48 (B)50 (C)52 (D)54 (E)56

174
MP 2
7. In a class, 22 students take painting course, 28 students take piano course, 10
students take both courses and 5 students take none of the courses. Find the
number of students in the class.
(A)40 (B)45 (C)50 (D)55 (E)65

8. As shown in the figure below, points C and D are on segment AB. The length of
AC is 6 cm longer than the length of CD. The length of BD is twice as the length
of AC. How much, in cm, is BD longer than AD?
A C D B

(A)3 (B)6 (C)9 (D)12 (E)15

9. The sum of four consecutive odd integers is 72. What is the largest integer among
the four integers?
(A)15 (B)17 (C)19 (D)21 (E)23

10. Each of the digits 1, 2, 3, 4, 5 and 6 are printed on one of the 6 cards. Choose two
different cards to form a 2-digit number. List down all possible 2-digit numbers
in increasing order on a sheet of paper, what is the 21st number in the list?
(A)43 (B)45 (C)46 (D)51 (E)61

Questions 11-20, 4 marks each

11. If (∆ ÷ 2 − 2) × 2 + 2=222, then what is the value of the symbol ∆ ?


(A)56 (B)224 (C)228 (D)876 (E)884

12. Positive integers are arranged in the array as shown below, what is the sum of all
the integers located on the fourth row?
1
2 3 4
5 6 7 8 9

(A)75 (B)81 (C)89 (D)91 (E)189

13. Define a new operation“ ∗ ”such that 4 ∗ 2 = 86 , 6 ∗ 3 = 189 , 8 ∗ 4 = 3212


and 9 ∗ 3 = 2712 . Find 10 ∗ 2 .
(A)128 (B)208 (C)2008 (D)2012 (E)2020

175
MP 3
14. The average score of Peter in several math exams is 88. On the last exam, he
must get 98 in order to raise his average score to 90. How many exams has he
taken in total?
(A)3 (B)4 (C)5 (D)6 (E)7

15. How many different ways are there to place 2 identical black balls and 2 identical
white balls in three different boxes A, B and C such that each box should contain
at least one ball?
(A)6 (B)9 (C)12 (D)15 (E)18

16. As shown in the figure below, there is a 3 × 3 grid paper where each small
square has an area of 1 cm2. Find the area, in cm2, of the shaded part.

(A)3 (B)4 (C)5 (D)6 (E)7

17. The sum of the digits of a 2-digit number is equal to 9. If the 2-digit number is
multiplied by 5, the sum of all digits is still 9. How many such 2-digit numbers
are there?
(A)4 (B)5 (C)7 (D)8 (E)9

18. As shown in the figure below, an ant starting from point A crawl along the path to
point B. It is required that both point C and point D can only pass once, how
many different paths can the ant crawl?
A

C
D

B
(A)6 (B)8 (C)10 (D)12 (E)18

19. Arrange black and white beads according to the pattern shown in the figure
below. If the total number of beads is 97, how many black beads are there?

(A)38 (B)39 (C)40 (D)41 (E)42

176
MP 4
20. There are 6 persons A, B, C, D, E and F attend a meeting. Some of them shake
hands with one another. In the final count up, A shakes 5 hands, B shakes 4, C
shakes 3, D shakes 2 and E shakes 1. How many does F shake?
(A)1 (B)2 (C)3 (D)4 (E)5

Questions 21-25, 6 marks each


21. The shape enclosed by solid lines in the figure below is composed of unit squares.
What is the maximum area of a rectangle that can be cut from the shape along
grid lines?

22. A box is filled with blue and green beads. The number of the blue beads is twice
as many as the green beads. Now take 6 blue beads and 4 green beads each time.
When the green beads are emptied, 6 blue beads are left. Please find the total
number of beads.

23. The teacher gave three students the same amount of money to buy stationery. A
bought 3 pens and has $3 left. B bought 5 crayons and has $5 left. C bought 7
pencils and has $3 left. If the price for each item is an integer, what is the least
total amount of money the teacher gave them?

24. Two students are computing A × B . The first student mistakenly took the last
digit of A as 2 instead of 7 and got a product of 418.The second student
mistakenly the tens digit of A as 3 instead of 2 and got a product of 703. What is
the value of B?

25. The students in a research class are clustered into two groups: the morning and
afternoon sessions. A student takes part in exactly one group in each session (the
two groups in each session can be different and the number of students in each
group can be different). Each group has at least one student and at most 4
students. Each student reports the number of students in the group he or she
belongs to in two sessions. One finds that no two students report the same pair of
numbers (with order, for example, (1, 4) and (4, 1) are different). What is
maximum number of students in the class?
***

177
178
179
注意:

允許學生個人、非營利性的圖書館或公立學校合理使用
本基金會網站所提供之各項試題及其解答。可直接下載
而不須申請。

重版、系統地複製或大量重製這些資料的任何部分,必
須獲得財團法人臺北市九章數學教育基金會的授權許
可。

申請此項授權請電郵 ccmp@seed.net.tw

Notice:

Individual students, nonprofit libraries, or schools are


permitted to make fair use of the papers and its
solutions. Republication, systematic copying, or
multiple reproduction of any part of this material is
permitted only under license from the Chiuchang
Mathematics Foundation.
Requests for such permission should be made by
e-mailing Mr. Wen-Hsien SUN ccmp@seed.net.tw

180
Solution to
7th International Mathematics Assessment for Schools
Round 1 of Middle Division

1. What is the simplified value of 25 × 99 + 55 × 5 ?


(A) 2750 (B) 2850 (C) 2900 (D) 2950 (E) 3000
【Solution】
25 × 99 + 55 × 5 = 25 × (99 + 11) = 25 × 110 = 2750 .
Answer: (A)
2. In the arithmetic expression 1A + B1 = 100, what is the value of the digit B?
(A) 5 (B) 6 (C) 7 (D) 8 (E) 9
【Solution】
We know the ones digit of 100 is 0, so the value of the digit A must be 9. Since
100 − 19 = 81 , then B = 8 .
Answer: (D)
3. As shown in the figure below, there is a 7 × 7 grid paper where each small
square has an area of 1 cm2, what is the area, in cm2, of the shaded figure?

(A) 18 (B) 19 (C) 20 (D) 21 (E) 22


【Solution】
The above shaded part can be assumed to assemble by exactly 20 small squares, so
the area of the shaded figure is 20 cm2.

Answer: (C)

181
4. Students are arranged in a square formation with 8 rows and 8 columns. Now, 3
rows and 3 columns are removed, how many students were reduced?
(A) 9 (B) 24 (C) 27 (D) 39 (E) 45
【Solution】
After removing 3 rows and 3 columns, the formation becomes a 5 × 5 square, then
the number of students reduced is 8 × 8 − 5 × 5 = 39 .

Answer: (D)
5. A train travels between City A to City B. There are 4 other stops in-between these
two cities. How many different kinds of tickets are there? (Note: The ticket from
City A to City B and City B to City A must not be the same).
(A) 8 (B) 12 (C) 20 (D) 24 (E) 30
【Solution】
From the given information, there are 6 train stations; each station must issue
different tickets to any one of the other 5 train stations. Hence, there are 6 × 5 = 30
different kinds of train tickets.
A B A B A B

A B A B A B

Answer: (E)
6. In the figure shown below, a square is embedded in a rectangle. If AB = 9 cm
and CD = 15 cm, then what is the perimeter, in cm, of the rectangle?
C A B D

F E
(A) 48 (B) 50 (C) 52 (D) 54 (E) 56

182
【Solution】
From the given figure, we know the width of the given rectangle is also the side
length of the given square. Hence, the sum of length and width of the given rectangle
is CD + AB = 15 + 9 = 24 cm. Hence, the perimeter of the rectangle is 24 × 2 = 48 cm.

Answer: (A)
7. In a class, 22 students take painting course, 28 students take piano course, 10
students take both courses and 5 students take none of the courses. Find the
number of students in the class.
(A) 40 (B) 45 (C) 50 (D) 55 (E) 65
【Solution】
From the given information, there are 22 + 28 − 10 = 40 students participated in
extra-curricular activities. Thus, there are 40 + 5 = 45 students in the class.

Both
Painting Painting Piano
and Piano

Answer: (B)
8. As shown in the figure below, points C and D are on segment AB. The length of
AC is 6 cm longer than the length of CD. The length of BD is twice as the length
of AC. How much, in cm, is BD longer than AD?
A C D B

(A) 3 (B) 6 (C) 9 (D) 12 (E) 15


【Solution 1】
A C D B

6
6 6
From the above figure, we know that the length of BD is 6 cm longer than the length
of AD.
【Solution 2】
Let CD = a, then AC = a + 6 , BD = 2 AC = 2(a + 6) = 2a + 12 .
Therefore, BD − AD = (2a + 12) − (a + 6 + a ) = 6 cm.
Answer: (B)
9. The sum of four consecutive odd integers is 72. What is the largest integer among
the four integers?
(A) 15 (B) 17 (C) 19 (D) 21 (E) 23
【Solution】
From the given information, we know the average of these four consecutive odd
integers is 72 ÷ 4 = 18, so the four consecutive odd integers are 15, 17, 19, 21.
Therefore, the largest odd integer is 21.
Answer: (D)

183
10. Each of the digits 1, 2, 3, 4, 5 and 6 are printed on one of the 6 cards. Choose two
different cards to form a 2-digit number. List down all possible 2-digit numbers
in increasing order on a sheet of paper, what is the 21st number in the list?
(A) 43 (B) 45 (C) 46 (D) 51 (E) 61
【Solution】
From the given information, among all 2-digit numbers there are 5 of them whose
tens digit is 1 and they are: 12, 13, 14, 15, 16. Similarly, there are also 5 possible
numbers whose tens digit is either 2, 3 and 4. Hence, the 21st number of the list is 51.
Answer: (D)
11. If (∆ ÷ 2 − 2) × 2 + 2=222, then what is the value of the symbol ∆ ?
(A) 56 (B) 224 (C) 228 (D) 876 (E) 884
【Solution】
Since ∆ ÷ 2 − 2 = (222 − 2) ÷ 2 = 220 ÷ 2 = 110,
then ∆ = (110 + 2) × 2 = 112 × 2 = 224.
Answer: (B)
12. Positive integers are arranged in the array as shown below, what is the sum of all
the integers located on the fourth row?
1
2 3 4
5 6 7 8 9

(A) 75 (B) 81 (C) 89 (D) 91 (E) 189
【Solution】
According to the pattern, the sum of the fourth row is
10 + 11 + 12 + 13 + 14 + 15 + 16 = 91 .
Answer: (D)
13. Define a new operation“ ∗ ”such that 4 ∗ 2 = 86 , 6 ∗ 3 = 189 , 8 ∗ 4 = 3212
and 9 ∗ 3 = 2712 . Find 10 ∗ 2 .
(A) 128 (B) 208 (C) 2008 (D) 2012 (E) 2020
【Solution】
From the given information, we deduce that the result of the operation is the product
followed by the sum. Hence, 10 × 2 = 20 , 10 + 2 = 12 , and we have 10 ∗ 2 = 2012 .
Answer: (D)
14. The average score of Peter in several math exams is 88. On the last exam, he
must get 98 in order to raise his average score to 90. How many exams has he
taken in total?
(A) 3 (B) 4 (C) 5 (D) 6 (E) 7
【Solution 1】
In order to increase the average score, 98 − 90 = 8 is distributed to previous exam
scores. The previous average exam increases by 2, thus, there are 8 ÷ 2 = 4 previous
exams, so he took a total of 4 + 1 = 5 exams.

184
【Solution 2】
The last exam is higher 98 − 88 = 10 than average score, this can raise his average
score from 88 to 90, that is increases the total average by 2, thus, he took a total of
10 ÷ 2 = 5 exams.
Answer: (C)
15. How many different ways are there to place 2 identical black balls and 2 identical
white balls in three different boxes A, B and C such that each box should contain
at least one ball?
(A) 6 (B) 9 (C) 12 (D) 15 (E) 18
【Solution】
The ball distribution in three boxes falls into three cases:
(i) BB, W, W; (ii) WW, B, B; (iii) BW, B, W;
For cases (i) or (ii) one needs to choose between three boxes to get the box with two
balls. For case (iii) one needs to arrange a sequence of the three boxes corresponding
to two different ball contents. So there is a total of 3 + 3 + 3 × 2 = 12 ways.
Answer: (C)
16. As shown in the figure below, there is a 3 × 3 grid paper where each small
square has an area of 1 cm2. Find the area, in cm2, of the shaded part.

(A) 3 (B) 4 (C) 5 (D) 6 (E) 7


【Solution】
(i)

Shaded part is 2 cm2

(ii)

Shaded part is
5 − 3 = 2 cm2

One computes by cut and join to get 2 + 2 = 4 cm2.


Using the Cutting and Pasting Method, we can easily find the area of shaded part is
2 + 2 = 4 cm2.
Answer: (B)

185
17. The sum of the digits of a 2-digit number is equal to 9. If the 2-digit number is
multiplied by 5, the sum of all digits is still 9. How many such 2-digit numbers
are there?
(A) 4 (B) 5 (C) 7 (D) 8 (E) 9
【Solution】
Let us list down all 2-digit numbers whose sum of all digits equals 9, there are 9 of
them: 18, 27, 36, 45, 54, 63, 72, 81 and 90. After multiplying each of them by 5, we
have 90, 135, 180, 225, 270, 315, 360, 405 and 450. We discovered that all of them
still have the property. Hence, there are a total of 9 such numbers.
Answer: (E)
18. As shown in the figure below, an ant starting from point A
A crawl along the path to point B. It is required that both
point C and point D can only pass once, how many C
different paths can the ant crawl?
(A)6 (B)8 (C)10 D
(D)12 (E)18
【Solution】 B
From the given figure, we know there are 2 paths from A to C; 3 ways from C to D; 2
ways from D to B. Thus, the total number of paths from A to B is 2 × 3 × 2 = 12 .
Answer: (D)

19. Arrange black and white beads according to the pattern shown in the figure
below. If the total number of beads is 97, how many black beads are there?


(A) 38 (B) 39 (C) 40 (D) 41 (E) 42
【Solution】
From the given figure, we discovered the beads are arranged as: W, B, W, W, B;
which is in a period of 5. Since 97 = 5 × 19 + 2 , there are 19 complete periods with a
remaining of first two beads in a period. There are two black beads in each period and
one black beads among the first two beads of a period. Thus, the total number of
black beads is 2 × 19 + 1 = 39 .
Answer: (B)
20. There are 6 persons A, B, C, D, E and F attend a meeting. Some of them shake
hands with one another. In the final count up, A shakes 5 hands, B shakes 4, C
shakes 3, D shakes 2 and E shakes 1. How many does F shake?
(A) 1 (B) 2 (C) 3 (D) 4 (E) 5
【Solution】
A, B, C, D and E totally shake 5 + 4 + 3 + 2 + 1 = 15 hands. Since each hand shaking
is counted twice, the total number of hands shaking is even. F should shake 1, 3 or 5
hands. If F shakes once, he shakes A; but B should shake A, C, D, E and now E
shakes twice, a contradiction. If F shakes 5 hands, both A and F shake hands with E,
also a contradiction. So F can only shake 3 hands and below is a possible situation:

186
A B C D E F
A ○ ○ ○ ○ ○
B ○ ○ ○ × ○
C ○ ○ × × ○
D ○ ○ × × ×
E ○ × × × ×
F ○ ○ ○ × ×
Answer: (C)
21. The shape enclosed by solid lines in the figure below is composed of unit squares.
What is the maximum area of a rectangle that can be cut from the shape along
grid lines?

【Solution】
Compute the area of rectangles of following shapes: 2 × 18 = 36 , 4 × 14 = 56 ,
6 × 12 = 72 , 8 × 8 = 64 , 10 × 6 = 60 , 12 × 2 = 24 . The maximal area is 72.

Answer: 072

22. A box is filled with blue and green beads. The number of the blue beads is twice
as many as the green beads. Now take 6 blue beads and 4 green beads each time.
When the green beads are emptied, 6 blue beads are left. Please find the total
number of beads.

187
【Solution 1】
Since the blue beads are twice as many as the green beads, if each time one takes 8
blue beads and 4 green beads instead, two kinds of beads are emptied at the same
time. Now 6 blue beads are left, one took the beads 6 ÷ (8 − 6) = 3 times. The
number of the blue beads is then 6 × 3 + 6 = 24 , the number of the green beads is
4 × 3 = 12 and there are 24 + 12 = 36 beads in total.
【Solution 2】
Stick 2 blue beads together as a big blue bead, then the number of the big blue beads
is same as the number of green beads. Now take 3 big blue beads and 4 green beads
each time. When the green beads are emptied, 3 big blue beads are left. That is one
took the beads 3 times. The number of the blue beads is then 3 × 2 × 3 + 3 × 2 = 24 , the
number of the green beads is 4 × 3 = 12 and there are 24 + 12 = 36 beads in total.
Answer: 036
23. The teacher gave three students the same amount of money to buy stationery. A
bought 3 pens and has $3 left. B bought 5 crayons and has $5 left. C bought 7
pencils and has $3 left. If the price for each item is an integer, what is the least
total amount of money the teacher gave them?
【Solution】
A bought 3 pens and left $3, the amount of money one student got is divisible by 3. B
bought 5 crayons and left $5, the amount of money is also divisible by 5, thus
divisible by 15. C bought 7 pencils and left $3, then one compute 15 = 7 × 2 + 1 ,
30 = 7 × 4 + 2 , 45 = 7 × 6 + 3 . The least amount the teacher gave to a single student is
45, totally for three students is 45 × 3 = 135 .
Answer: 135

24. Two students are computing A × B . The first student mistakenly took the last
digit of A as 2 instead of 7 and got a product of 418.The second student
mistakenly the tens digit of A as 3 instead of 2 and got a product of 703. What is
the value of B?
【Solution 1】
The first student got the last digit as 2 instead of 7, the product is 5 × B less than the
correct one; The second student got a result 10 × B more than the correct one. We
have B = (703 − 418) ÷ (5 + 10) = 19 .
【Solution 2】
From the given information, we know that A is 27. Hence B = 418 ÷ 22 = 19 .
Answer: 019
25. The students in a research class are clustered into two groups: the morning and
afternoon sessions. A student takes part in exactly one group in each session (the
two groups in each session can be different and the number of students in each
group can be different). Each group has at least one student and at most 4
students. Each student reports the number of students in the group he or she
belongs to in two sessions. One finds that no two students report the same pair of
numbers (with order, for example, (1, 4) and (4, 1) are different). What is
maximum number of students in the class?

188
【Solution】
Consider the two numbers reported by a student as an ordered pair. The first number
represents number of students in the morning session, the second number represents
the afternoon session. Since maximum number of students in a group is 4, there is a
total of 16 combinations as follows:
(1, 1) (1, 2) (1, 3) (1, 4)
(2, 1) (2, 2) (2, 3) (2, 4)
(3, 1) (3, 2) (3, 3) (3, 4)
(4, 1) (4, 2) (4, 3) (4, 4)
The pairs in row k (1 ≤ k ≤ 4 ) correspond to all students in a size k group in the
morning, the total number of which must be divisible by k, similarly for the kth
column. There are at most 3 students on the third row and the third column. If the pair
(3, 3) is removed the remaining 15 pairs satisfy the condition. So the maximum
number of students is 15.
(1, 1) (1, 2) (1, 3) (1, 4)
(2, 1) (2, 2) (2, 3) (2, 4)
(3, 1) (3, 2) (3, 3) (3, 4) Must remove 1
(4, 1) (4, 2) (4, 3) (4, 4)

Must remove 1

Answer: 015

189
注意:

允許學生個人、非營利性的圖書館或公立學校合理使用
本基金會網站所提供之各項試題及其解答。可直接下載
而不須申請。

重版、系統地複製或大量重製這些資料的任何部分,必
須獲得財團法人臺北市九章數學教育基金會的授權許
可。

申請此項授權請電郵 ccmp@seed.net.tw

Notice:

Individual students, nonprofit libraries, or schools are


permitted to make fair use of the papers and its
solutions. Republication, systematic copying, or
multiple reproduction of any part of this material is
permitted only under license from the Chiuchang
Mathematics Foundation.
Requests for such permission should be made by
e-mailing Mr. Wen-Hsien SUN ccmp@seed.net.tw

190
7th International Mathematics Assessments for Schools
(2017-2018)

Middle Primary Division Round 2


Time: 120 minutes

Printed Name: Code: Score:

Instructions:
Do not open the contest booklet until you are told to do so.
Be sure that your name and code are written on the space provided above.
Round 2 of IMAS is composed of three parts; the total score is 100 marks.
Questions 1 to 5 are given as a multiple-choice test. Each question has five
possible options marked as A, B, C, D and E. Only one of these options is correct.
After making your choice, fill in the appropriate letter in the space provided. Each
correct answer is worth 4 marks. There is no penalty for an incorrect answer.
Questions 6 to 13 are a short answer test. Only Arabic numerals are accepted;
using other written text will not be honored or credited. Some questions have more
than one answer, as such all answers are required to be written down in the space
provided to obtain full marks. Each correct answer is worth 5 marks. There is no
penalty for incorrect answers.
Questions 14 and 15 require a detailed solution or process in which 20 marks are
to be awarded to a completely written solution. Partial marks may be given to an
incomplete presentation. There is no penalty for an incorrect answer.
Use of electronic computing devices is not allowed.
Only pencil, blue or black ball-pens may be used to write your solution or answer.
Diagrams are not drawn to scale. They are intended as aids only.
After the contest the invigilator will collect the contest paper.
The following area is to be filled in by the judges;
the contestants are not supposed to mark anything here.
Total
Question 1 2 3 4 5 6 7 8 9 10 11 12 13 14 15 Score Signature

Score

Score

191
192
Middle Primary Division Round 2

Questions 1 to 5, 4 marks each

1. Arranged 30 triangles in a row and color them black and white in a pattern as
shown below. How many more black triangles than white triangles are there?
▲▲△▲▲△▲▲△……
(A)8 (B)10 (C)12 (D)14 (E)15

Answer:

2. It is known that A × B × C = 30 , B × C × D = 90 and C × D × E = 120 , what is


the value of A × C × E ?
(A)20 (B)30 (C)40 (D)50 (E)60

Answer:

3. Replace each ∆ by "+" or "-" in the expression 1∆ 2∆3∆8∆15 and compute


the value of the expression. How many different positive integers can we have
for the expression?
(A)8 (B)10 (C)12 (D)14 (E)16

Answer:

4. A rectangle is divided into 6 identical squares as shown in the figure below.


Color four squares black such that each row has at least one black square. Two
color methods are considered the same if they are identical after rotation. How
many different color methods are there?

(A)4 (B)5 (C)6 (D)7 (E)10

Answer:

193
MP 2
5. Two cars start moving from opposite ends of a road towards each other at a
constant speed. The faster car travels at 40 km per hour. After two hours, the
faster car is 20 km past the midpoint and 6 km away before meeting the slower
car. What is the speed of the slower car in km per hour?
(A)17 (B)19 (C)21 (D)23 (E)25

Answer:

Questions 6 to 13, 5 marks each

6. Select three different numbers from 1, 2, 3, 4, 5, 6, 7, 8, 9, 10, 11 and 12 such


that their average is 5. How many different possible combinations are there?

Answer: combinations

7. The pages of a book begin from No. 1, 2, 3, … such that two consecutive
numbers appear on both sides of a single page. When one page is torn, the sum of
the remaining page numbers is 1133. What is the sum of those two numbers on
the torn page?
Answer:

8. The number of boys in a class is twice as the number of girls. In a math test, the
average score of the class is 86, but average score of the girls is 90. What is the
average score of the boys?
Answer:

9. The road map of a certain city consists of 3 × 3 squares with each of the smaller
squares having a side length of 20 meters as shown in the figure. A street sweeper
starts cleaning from point A and every road thereafter until finally returning to A.
What is the shortest distance, in meters, in which he can do the task?
A

Answer: meters

194
MP 3
10. The perimeter of rectangle ABCD is 34 cm. Draw four squares extending
outward from its four sides, as shown in the figure below. If the sum of the areas
of the four new squares is 338 cm2, what is the area, in cm2, of rectangle ABCD?

A D

B C

Answer: cm2

11. Millie has 60 red beads, 50 black beads and a magic machine. Each time 4 red
beads are inserted into the machine, it gives out 1 black bead and for every 5
black beads is inserted, the machine gives out 2 red beads. After having operated
the machine 30 times, Millie has no more red beads. Find the number of black
beads she has now.

Answer: black beads

12. A 3-digit number abc is said to be a lucky number if a × b × c is also a


three-digit number. What is the least possible lucky number abc ?

Answer:

13. Rotate an equilateral triangle inscribed in a circle 40 degrees clockwise and


counter-clockwise, as in the figure below. How many triangles are there in the
figure?

Answer: triangles

195
MP 4

Questions 14 to 15, 20 marks each


(Detailed solutions are needed for these two problems)
14. A rectangle is divided into 12 unit squares such that 10 are white and 2 are black,
as shown in the figure below. To form a centrally symmetric picture by adding
some white squares but no black squares, what is the least number of white
squares needed? Please draw the centrally symmetric picture.

Answer: white squares,

196
MP 5
15. There are 12 different positive integers which satisfy the condition that the
product of every 5 numbers is even and that the sum of all 12 numbers is odd.
Find the least possible sum of these 12 positive integers.

Answer:

197
198
注意:

允許學生個人、非營利性的圖書館或公立學校合理使用
本基金會網站所提供之各項試題及其解答。可直接下載
而不須申請。

重版、系統地複製或大量重製這些資料的任何部分,必
須獲得財團法人臺北市九章數學教育基金會的授權許
可。

申請此項授權請電郵 ccmp@seed.net.tw

Notice:

Individual students, nonprofit libraries, or schools are


permitted to make fair use of the papers and its
solutions. Republication, systematic copying, or
multiple reproduction of any part of this material is
permitted only under license from the Chiuchang
Mathematics Foundation.
Requests for such permission should be made by
e-mailing Mr. Wen-Hsien SUN ccmp@seed.net.tw

199
Solution Key to Second Round of IMAS 2017/2018
Middle Primary Division
1. Arranged 30 triangles in a row and color them black and white in a pattern as
shown below. How many more black triangles than white triangles are there?
▲▲△▲▲△▲▲△……
(A)8 (B)10 (C)12 (D)14 (E)15
【Solution】
From the picture, it shows that the coloring is periodic with period 3, consisting of 2
black triangles and 1 white triangle. In each period, the number of black triangles is
one more than the white triangle. Since there are 30 ÷ 3 = 10 periods, so there are 10
more black triangles.
Answer:(B)
2. It is known that A × B × C = 30 , B × C × D = 90 and C × D × E = 120 , what is
the value of A × C × E ?
(A)20 (B)30 (C)40 (D)50 (E)60
【Solution】
A × C × E = ( A × B × C ) × (C × D × E ) ÷ ( B × C × D) , so A × C × E = 30 × 120 ÷ 90 = 40 .
Answer:(C)
3. Replace each ∆ by "+" or "-" in the expression 1∆ 2∆3∆8∆15 and compute
the value of the expression. How many different positive integers can we have
for the expression?
(A)8 (B)10 (C)12 (D)14 (E)16
【Solution 1】
Since 1 + 2 + 3 + 8 = 14 < 15 , so if the sign placed before 15 is negative, the result is
not positive. When the sign before 15 is a plus sign and since 1 is always positive, the
result is always positive. The difference between 2,3 and 8 are all distinct, so the
result for different sign choices are distinct. Thus, there are 2 × 2 × 2 = 8 different
positive integers.
【Solution 2】
There are 2 × 2 × 2 × 2 = 16 ways in filling the signs.
(1) 1 + 2 + 3 + 8 + 15 = 29 ;
(2) 1 + 2 + 3 + 8 − 15 = 14 − 15 < 0 , it is non-positive;
(3) 1 + 2 + 3 − 8 + 15 = 13 ;
(4) 1 + 2 − 3 + 8 + 15 = 23 ;
(5) 1 − 2 + 3 + 8 + 15 = 25 ;
(6) 1 + 2 + 3 − 8 − 15 = 6 − 23 < 0 , it is non-positive;
(7) 1 + 2 − 3 + 8 − 15 = 11 − 18 < 0 , it is non-positive;
(8) 1 − 2 + 3 + 8 − 15 = 10 − 15 < 0 , it is non-positive;
(9) 1 + 2 − 3 − 8 + 15 = 7 ;
(10) 1 − 2 + 3 − 8 + 15 = 9 ;

200
(11) 1 − 2 − 3 + 8 + 15 = 19 ;
(12) 1 + 2 − 3 − 8 − 15 = 3 − 26 < 0 , it is non-positive;
(13) 1 − 2 + 3 − 8 − 15 = 4 − 25 < 0 , it is non-positive;
(14) 1 − 2 − 3 + 8 − 15 = 9 − 20 < 0 , it is non-positive;
(15) 1 − 2 − 3 − 8 + 15 = 3 ;
(16) 1 − 2 − 3 − 8 − 15 = 1 − 28 < 0 , it is non-positive;
Therefore, we have 29, 13, 23, 25, 7, 9, 19 and 3, a total of 8 different positive
integers.
Answer:(A)
4. A rectangle is divided into 6 identical squares as shown in the figure below.
Color four squares black such that each row has at least one black square. Two
color methods are considered the same if they are identical after rotation. How
many different color methods are there?

(A)4 (B)5 (C)6 (D)7 (E)10


【Solution】
There are three rows in the figure and in every scenario, two of the rows will have
one black square and the third row will have two black squares.
(i) If the number of black squares are 1, 1 and 2 respectively in rows from top to
bottom, the colorings are as follows below:

(ii) If the number of black squares are 1, 2, and 1 respectively from top to bottom,
the colorings are as follows below:

(iii) If the number of black squares are 2, 1 and 1 respectively, the coloring is
rotationally equivalent to cases in (i).
In total, there are 7 coloring methods.
Answer:(D)

201
5. Two cars start moving from opposite ends of a road towards each other at a
constant speed. The faster car travels at 40 km per hour. After two hours, the
faster car is 20 km past the midpoint and 6 km away before meeting the slower
car. What is the speed of the slower car in km per hour?
(A)17 (B)19 (C)21 (D)23 (E)25
【Solution 1】
The faster car covers half the length of the road plus 20 km in two hours, while the
slower car covers 26 km less than the half length of the road. Thus, the faster car
covers 46 km more than the slower car in two hours. The speed difference is
46 ÷ 2 = 23 km and the speed of the slower car is 40 − 23 = 17 km.
【Solution 2】
The faster car travels 40 × 2 = 80 km in two hours, which is 20 km more than half
length of the road. Then, the road has a length of 2 × (80 − 20) = 120 km. The distance
of the two cars is 6 km, so the slower car travels 120 − 80 − 6 = 34 km for two hours.
Its speed is then 34 ÷ 2 = 17 km.
Answer:(A)
6. Select three different numbers from 1, 2, 3, 4, 5, 6, 7, 8, 9, 10, 11 and 12 such
that their average is 5. How many different possible combinations are there?
【Solution】
Write the sum of three numbers, which is 5 × 3 = 15 , as the sum of three distinct
positive integers. Enumerating all possible ways, we get,
15 = 12 + 2 + 1 = 11 + 3 + 1 = 10 + 4 + 1 = 10 + 3 + 2 = 9 + 5 + 1 = 9 + 4 + 2
= 8 + 6 +1 = 8 + 5 + 2 = 8 + 4 + 3 = 7 + 6 + 2 = 7 + 5 + 3= 6 + 5 + 4
In total, there are 12 different ways.
Answer: 12
7. The pages of a book begin from No. 1, 2, 3, … such that two consecutive
numbers appear on both sides of a single page. When one page is torn, the sum of
the remaining page numbers is 1133. What is the sum of those two numbers on
the torn page?
【Solution 1】
If the book is until page 50, the sum of all page numbers is
1 + 2 + 3 + ⋯ + 49 + 50 = 1275 , but 1275 − 1133 = 142 and there is no page before 50
has page numbers that has a sum of 142. This will result that the torn page is higher
than 50, so there is no possible solution;
If the book is until page 49, the sum is 1 + 2 + 3 + ⋯ + 48 + 49 = 1225 , and
1225 − 1133 = 92 is even, this is not possible for a sum of page numbers of a single
page;
If the book is until page 48, the sum is 1 + 2 + 3 + ⋯ + 47 + 48 = 1176 , and
1176 − 1133 = 43 , 43 = 21 + 22 , the torn page has numbers 21 and 22;
If the book in until less than or equal to page 47, sum of all page number is at most
1 + 2 + 3 + ⋯ + 46 + 47 = 1128 < 1133 , which is not possible.
Thus, the sum of page numbers of the torn page is 43 = 21 + 22 .

202
【Solution 2】
The two numbers on one page is consecutive, whose sum is odd. The total sum of all
page numbers is the sum of 1133 and an odd number, which is even. The sum of all
page numbers is sum of consecutive integers starting from 1, the total page numbers
is a multiple of 4 or a multiple of 4 plus 3. 1 + 2 + 3 + ⋯ + 46 + 47 = 1128 < 1133 , then,
the book has at least 48 pages. Possible pages numbers are 48, 51, 52, 55, 56, ….
When the book is at least 51 pages, tearing up one page leaves at least 49 different
page numbers, whose sum is at least 1 + 2 + 3 + ⋯ + 48 + 49 = 1225 > 1133 , which is
not possible for a solution.
For a book with 48 pages, 1 + 2 + 3 + ⋯ + 47 + 48 = 1176 , but
1176 − 1133 = 43 = 21 + 22 , this satisfies the condition of the problem.
Thus, the sum of page numbers of the torn page is 43 = 21 + 22 .
Answer: 43
8. The number of boys in a class is twice as the number of girls. In a math test, the
average score of the class is 86, but average score of the girls is 90. What is the
average score of the boys?
【Solution】
Divide all students into several groups so that each group has one girl and two boys.
Averagely, the sum of the scores of the three students in a group is 86 × 3 = 258 and
the score of the girl is 90. Then the total score of the two boys is 258 − 90 = 168 ,
whose average is 168 ÷ 2 = 84 .
Answer: 84
9. The road map of a certain city consists of 3 × 3 A
squares with each of the smaller squares having a side
length of 20 meters as shown in the figure. A street
sweeper starts cleaning from point A and every road
thereafter until finally returning to A. What is the
shortest distance, in meters, in which he can do the
task?
【Solution】
Eight intersections of the road map are the intersection points of odd number of roads.
To walk through each road at least once, 4 paths connecting pairs of the odd
intersections must be walked through twice. So, the sweeper must walk at least
(24 + 4) × 20 = 560 m. One cleaning path is shown below.
A

Answer: 560

203
10. The perimeter of rectangle ABCD is 34 cm. Draw four squares extending
outward from its four sides, as shown in the figure below. If the sum of the areas
of the four new squares is 338 cm2, what is the area, in cm2, of rectangle ABCD?

A D

B C

【Solution】
Mark each point as in the figure below and draw the rectangle BFGH. The rectangles
BFGH and ABCD are congruent. Quadrilateral EGID has four inner right angles and
both side lengths equal to sum of two sides of ABCD, so it is a square with side
length 34 ÷ 2 = 17 cm and area 17 × 17 = 289 cm2. The sum of areas of EFBA and
BHIC is 338 ÷ 2 = 169 cm2, so the area of ABCD is (289 − 169) ÷ 2 = 60 cm2.

E A D

F
B C
G H I
Answer: 60 cm2
11. Millie has 60 red beads, 50 black beads and a magic machine. Each time 4 red
beads are inserted into the machine, it gives out 1 black bead and for every 5
black beads is inserted, the machine gives out 2 red beads. After having operated
the machine 30 times, Millie has no more red beads. Find the number of black
beads she has now.
【Solution】
Operation (i): 4 red beads are inserted into the machine, it gives out 1 black bead.
Operation (ii): 5 black beads are inserted into the machine, it gives out 2 red beads.
Each operation reduces the number of total beads by 3. Since Millie has50 + 60 = 110
beads at the start, after 30 operations he has 110 − 30 × 3 = 20 beads left, all of which
are black. One operation sequence could be to operate step (ii) for 10 times to get 80
red beads and 0 black beads; then to operate step (i) for 20 times to get 20 black
beads and 0 red beads.
Answer: 20
12. A 3-digit number abc is said to be a lucky number if a × b × c is also a
three-digit number. What is the least possible lucky number abc ?

204
【Solution】
First, we have that both b and c are no more than 9, thus b × c ≤ 81 . In order that
a × b × c ≥ 100 , one has a ≥ 2 . When a = 2 , in order that a × b × c ≥ 100 , 9 × b is at
least 50, then b ≥ 6 . When a = 2 , b = 6 , a × b = 12 , in order that a × b × c ≥ 100 ,
c ≥ 9 . The least possible abc is 269, while a × b × c = 108 .
Answer: 269
13. Rotate an equilateral triangle inscribed in a circle 40
degrees clockwise and counter-clockwise, as in the figure
below. How many triangles are there in the figure?

【Solution】
(i) There are 9 triangles of same size but in different positions as the shaded triangle
in the figure below.

(ii) There are 9 triangles of same size but in different positions as the shaded triangle
in the figure below.

(iii) There are 9 triangles of same size but in different positions as the shaded triangle
in the figure below.

205
(iv) There are 3 triangles of same size but in different positions as the shaded triangle
in the figure below.

Totally there are 9 + 9 + 9 + 3 = 30 triangles.


Answer: 30.
14. A rectangle is divided into 12 unit squares such that
10 are white and 2 are black, as shown in the figure
below. To form a centrally symmetric picture by
adding some white squares but no black squares, what
is the least number of white squares needed? Please
draw the centrally symmetric picture.

【Solution】
Since no more black unit squares are added, the
symmetric center of the whole picture is the
symmetric center of the two black unit squares. (5
points)
4 white unit squares are already symmetric to one Symmetric
another with respect to this center. 6 more white unit center

squares are needed to be symmetric to those 6 alone


white unit squares, (5 points) as in the figure to the
right. (10 points)
Answer: 6
15. There are 12 different positive integers which satisfy the condition that the
product of every 5 numbers is even and that the sum of all 12 numbers is odd.
Find the least possible sum of these 12 positive integers.
【Solution】
Since every 5 numbers, the product should be even, at most 4 of the 12 numbers is
odd. (5 points)But since the sum of all the numbers is odd, there is an odd number of
odd numbers among them. So, the number of odd numbers is 1 or 3. (5 points)
(i) If there is only 1 odd number, taking sum of the smallest odd number and 11
smallest even numbers, one gets
1 + 2 + 4 + 6 + 8 + 10 + 12 + 14 + 16 + 18 + 20 + 22 = 133 .(5 points)
(ii) If there are 3 odd numbers, taking sum of the smallest 3 odd number and 9
smallest even numbers, one gets
1 + 3 + 5 + 2 + 4 + 6 + 8 + 10 + 12 + 14 + 16 + 18 = 99 .
So, the least possible sum of these 12 positive integers is 99. (5 points)
Answer: 99

206
注意:

允許學生個人、非營利性的圖書館或公立學校合理使用
本基金會網站所提供之各項試題及其解答。可直接下載
而不須申請。

重版、系統地複製或大量重製這些資料的任何部分,必
須獲得財團法人臺北市九章數學教育基金會的授權許
可。

申請此項授權請電郵 ccmp@seed.net.tw

Notice:

Individual students, nonprofit libraries, or schools are


permitted to make fair use of the papers and its
solutions. Republication, systematic copying, or
multiple reproduction of any part of this material is
permitted only under license from the Chiuchang
Mathematics Foundation.
Requests for such permission should be made by
e-mailing Mr. Wen-Hsien SUN ccmp@seed.net.tw

207
International Mathematics Assessments for Schools

2018 MIDDLE PRIMARY DIVISION FIRST ROUND PAPER


Time allowed:75 minutes

When your teacher gives the signal, begin working on the problems.

INSTRUCTION AND INFORMATION


GENERAL
1. Do not open the booklet until told to do so by your teacher.
2. No calculators, slide rules, log tables, math stencils, mobile phones or other
calculating aids are permitted. Scribbling paper, graph paper, ruler and compasses
are permitted, but are not essential.
3. Diagrams are NOT drawn to scale. They are intended only as aids.
4. There are 20 multiple-choice questions, each with 5 choices. Choose the most
reasonable answer. The last 5 questions require whole number answers between
000 and 999 inclusive. The questions generally get harder as you work through the
paper. There is no penalty for an incorrect response.
5. This is a mathematics assessment, not a test; do not expect to answer all questions.
6. Read the instructions on the answer sheet carefully. Ensure your name, school
name and school year are filled in. It is your responsibility that the Answer Sheet
is correctly coded.

THE ANSWER SHEET


1. Use only pencils.
2. Record your answers on the reverse side of the Answer Sheet (not on the question
paper) by FULLY filling in the circles which correspond to your choices.
3. Your Answer Sheet will be read by a machine. The machine will see all markings
even if they are in the wrong places. So please be careful not to doodle or write
anything extra on the Answer Sheet. If you want to change an answer or remove
any marks, use a plastic eraser and be sure to remove all marks and smudges.

INTEGRITY OF THE COMPETITION


The IMAS reserves the right to re-examine students before deciding whether to
grant official status to their scores.

208
209
2018 MIDDLE PRIMARY DIVISION FIRST ROUND PAPER

Questions 1-10, 3 marks each


1. 1. What is the value of 19 1  19  3  19  5  19  7   19 19 ?
(A)1900 (B)1919 (C)2900 (D)2919 (E)3800

2. If (  2  1)  2  2018 , then what is the value of  ?


(A)502 (B)503 (C)504 (D)505 (E)506

3. Five students sit along a circle and starts to call out some numbers one-by-one.
Student A calls out “1”, B calls out “2”, C calls out “3”, D calls out “4”, E calls
out “5” and then it returns back to Student A who calls out “6” and so on, where
each student increases the previously called number by one and calls it out.
Which student calls out the number “99”?
(A)A (B)B (C)C (D)D (E)E

4. In the figure shown below, four rectangles of the same size, denoted by I, II, III
and IV, are placed together, where ABCD and EFGH are both squares. If
rectangle I has a perimeter of 20 cm, then what is the perimeter, in cm, of
ABCD ?
A B
I
E F II
IV
H G
III
D C
(A)40 (B)60 (C)80 (D)100 (E)120

5. How many triangles in total are there in the figure below?

(A)4 (B)8 (C)12 (D)14 (E)16

210
MP 2
6. The six faces of a cube are colored using six different colors namely red, blue,
yellow, green, black and white in some order. Turn the cube arbitrarily and it
shows the two possible scenarios below. What is the color opposite to the face of
green?
White Red

Yellow Blue
Red Blac
k

(A)Red (B)Yellow (C)Blue (D)Black (E)White

7. Consider every positive integers whose digits do not include 2 and that sum of its
digits is equal to 3 and arrange all such integers in increasing order. What is the
sum of the three smallest integers that satisfy the conditions?
(A)36 (B)63 (C)144 (D)206 (E)414

8. In the 4  4 square table shown below, a △ is placed on the second row and
second column. What is the total number of squares with sides falling on the grid
lines and containing △?

(A)8 (B)10 (C)11 (D)12 (E)14

9. In the figure below, a frog jumps between the three circles. In each jump, it goes
from one circle into another circle. It is known that the frog starts from A and
ends at A after 4 jumps. How many different paths can the frog have?

A B C

(A)4 (B)5 (C)6 (D)7 (E)8

10. In 1202 A.D., Italian mathematician Fibonacci (1170~1250) wrote in his book
《Liber Abaci》 the following interesting problem: Exactly two months after
their birth, a pair of rabbits will give birth to a new pair (one male and one
female) and then give birth to a pair each month after that. There is only one pair
of new born rabbits at the beginning. If the rabbits never die, how many pair of
rabbits are there after exactly 12 months?
(A)144 (B)233 (C)234 (D)235 (E)377

211
MP 3

Questions 11-20, 4 marks each

11. Define “*” as an operation such that 4  2  82 , 6  3  183 , 8  4  324 ,


9  3  276 and 9  5  454 . What is the value of 10  2 ?
(A)55 (B)125 (C)202 (D)208 (E)2002

12. Bob got a score of 94 on foreign language test, and his average score on the
native language and math tests is 97. What is his average score on these three
tests?
(A)94 (B)94.5 (C)95 (D)95.5 (E)96

13. Color the surfaces of a 5 5 5 wooden cube red, then cut it into smaller 111
cube pieces. How many of these smaller cubes have exactly two red faces?
(A)36 (B)48 (C)61 (D)90 (E)98

14. A palindrome number is a positive integer that is the same when read forwards or
backwards. The numbers 909 and 1221 are examples of palindromes. How many
palindrome numbers are there between 10 and 1000?
(A)90 (B)99 (C)100 (D)106 (E)108

15. In the month of February of some year, there are more Saturdays than any other
days in a week. What day is the last day of this month?
(A)Wednesday (B)Thursday (C)Friday
(D)Saturday (E)Sunday

16. Four students namely Annie, Benny, Charlie and Deany all paid the same amount
of money to buy some number of notebooks together. After distributing the
notebooks, Annie, Benny and Charlie got 6, 7 and 11 notebooks more than Deany,
respectively. As such, to be fair, Annie, Benny and Charlie gave back a total of
$48 to Deany. What is the price of each notebook?
(A)2 (B)6 (C)8 (D)12 (E)16

17. A train left town A at 8:30 AM some day and arrived at town B at 1:50 AM of the
next day. There is no time difference between the two places. How long did the
train travelled for the trip?
(A)5 hours 20 minutes (B)10 hours 20 minutes (C)15 hours 20 minutes
(D)16 hours 20 minutes (E)17 hours 20 minutes

212
MP 4
18. Mike placed 4 identical squares, each with side length 5 cm and are
non-overlapping, to form a new figure as shown below. Find the perimeter, in cm,
of this new figure.

(A)15 (B)20 (C)30 (D)45 (E)50

19. Adam owns an old watch, which is slower than a normal watch by the same
amount of time for each hour. At 8 o'clock in the morning, the old watch reads 8
o'clock. At 9 o'clock in the morning, it reads 8:58. What time does it read when
the real time is 4 o'clock in the afternoon?
(A)3:42 (B)3:44 (C)3:46
(D)4:08 (E)4:16

20. Identical equilateral triangles are placed together into two figures as shown below.
One can cover Figure B using 5 pieces of Figure A. How many different ways
can we do the covering?

Figure A

Figure B
(A)3 (B)4 (C)5 (D)6 (E)7

Questions 21-25, 6 marks each


21. Put some unit cubes into a 3D model such that in the model, each cube touches
some other cubes in at least one point. It is known that the model looks like the
figure below from three directions of upright front, left and top. What is the least
number of unit cubes needed to make such a model?

213
MP 5
22. How many ways can we divide 6 students into 3 groups so that each group has
exactly 2 students?

23. In the figure below, ABCD, EFGH and AJKL are squares. The area of AJKL is
2018 cm2. If rectangles EFCI and JBFK both have an area of 1360 cm2, then
what is the area, in cm2, of CGHI?

A L D

I H
E

J K

B F C G

24. Three pairs of red, four pairs of yellow and five pairs of white socks are placed in
a bag. Now, blindly take a sock out each time. How many socked are needed to
be taken out to guarantee having six pairs of socks? (Note: Two socks of the
same color are considered a pair)

25. Cut the 6  6 square table below into rectangles along grid lines such that no
two rectangles are identical. What is the maximum number of rectangles one can
get? (Note: A square is considered a rectangle.)

***

214
215
注意:

允許學生個人、非營利性的圖書館或公立學校合理使用
本基金會網站所提供之各項試題及其解答。可直接下載
而不須申請。

重版、系統地複製或大量重製這些資料的任何部分,必
須獲得財團法人臺北市九章數學教育基金會的授權許
可。

申請此項授權請電郵 ccmp@seed.net.tw

Notice:

Individual students, nonprofit libraries, or schools are


permitted to make fair use of the papers and its
solutions. Republication, systematic copying, or
multiple reproduction of any part of this material is
permitted only under license from the Chiuchang
Mathematics Foundation.
Requests for such permission should be made by
e-mailing Mr. Wen-Hsien SUN ccmp@seed.net.tw

216
Solution to
2018 International Mathematics Assessment for Schools
Round 1 of Middle Primary Division
1. What is the value of 19 × 1 + 19 × 3 + 19 × 5 + 19 × 7 + ⋯ + 19 × 19 ?
(A)1900 (B)1919 (C)2900 (D)2919 (E)3800
【Solution】
19 × 1 + 19 × 3 + 19 × 5 + 19 × 7 + ⋯ + 19 × 19 = 19 × (1 + 3 + 5 + 7 + ⋯ + 19)
(1 + 19) × 10
= 19 ×
2
= 19 × 100
= 1900
Hence (A).
Answer:(A)
2. If (∆ × 2 − 1) × 2 = 2018 , then what is the value of ∆ ?
(A)502 (B)503 (C)504 (D)505 (E)506
【Solution】
(∆ × 2 − 1) × 2 = 2018
∆ × 2 − 1 = 2018 ÷ 2 = 1009
∆ × 2 = 1009 + 1 = 1010
∆ = 1010 ÷ 2 = 505
Hence (D).
Answer:(D)
3. Five students sit along a circle and starts to call out some numbers one-by-one.
Student A calls out “1”, B calls out “2”, C calls out “3”, D calls out “4”, E calls
out “5” and then it returns back to Student A who calls out “6” and so on, where
each student increases the previously called number by one and calls it out.
Which student calls out the number “99”?
(A)A (B)B (C)C (D)D (E)E
【Solution】
Taking division with remainders, we get 99 = 5 × 19 + 4 . It is student D who calls 99.
Hence (D).
Answer:(D)
4. In the figure shown below, four rectangles of the same A B
size, denoted by I, II, III and IV, are placed together, I
where ABCD and EFGH are both squares. If rectangle
I has a perimeter of 20 cm, then what is the perimeter, E F II
in cm, of ABCD ?
(A)40 (B)60 (C)80 IV
H G
(D)100 (E)120 III
D C

217
【Solution】
The side length of square ABCD is exactly the same as the sum of the length and
width of rectangle I. So, the perimeter of ABCD is (20 ÷ 2) × 4 = 40 cm. Hence (A).
Answer:(A)
5. How many triangles in total are there in the figure below?

(A)4 (B)8 (C)12 (D)14 (E)16


【Solution】
Refer to the figure below, there are four identical triangles for each case. Thus, the
total number of triangles is 4 × 4 = 16 . Hence (E).

Answer:(E)
6. The six faces of a cube are colored using six different colors namely red, blue,
yellow, green, black and white in some order. Turn the cube arbitrarily and it
shows the two possible scenarios below. What is the color opposite to the face of
green?
White Red

Yellow Blue
Red Blac

(A)Red (B)Yellow (C)Blue (D)Black (E)White


【Solution】
The red face is adjacent to the yellow, blue, black and white faces. Therefore, its
opposite face is colored green. Hence (A).
Answer:(A)
7. Consider every positive integers whose digits do not include 2 and that sum of its
digits is equal to 3 and arrange all such integers in increasing order. What is the
sum of the three smallest integers that satisfy the conditions?
(A)36 (B)63 (C)144 (D)206 (E)414

218
【Solution】
Since 3 = 1 + 1 + 1 , we can arrange such numbers in increasing order as 3, 30, 111,
300, …, etc. Therefore, the sum of the three smallest numbers is 3 + 30 + 111 = 144 .
Hence (C).
Answer:(C)
8. In the 4 × 4 square table shown below, a △ is placed on the second row and
second column. What is the total number of squares with sides falling on the grid
lines and containing △?

(A)8 (B)10 (C)11 (D)12 (E)14


【Solution】
Enumerating the squares according to the different sizes containing △. There are 1
such 1 × 1 square; 4 such 2 × 2 squares; 4 such 3 × 3 squares and 1 such 4 × 4
square. In total, there are 10 such squares containing △. Hence (B).

△ △ △ △ △

△ △ △ △ △

Answer:(B)
9. In the figure below, a frog jumps between the three circles. In each jump, it goes
from one circle into another circle. It is known that the frog starts from A and
ends at A after 4 jumps. How many different paths can the frog have?

A B C

(A)4 (B)5 (C)6 (D)7 (E)8

219
【Solution 1】
One can enumerate the following 6 ways: ABABA, ABACA, ABCBA, ACABA,
ACACA, ACBCA. Hence (C).
A B A
C A
B C A ×
A B A
A B A
C
C A
B A ×
C A
【Solution 2】
Denote three circles between the start circle and end circle as X, Y, Z:
A →X → Y → Z → A
Since X and Z are neither A, one distinguishes two cases as below:
(i) if Y is A, then X or Z each has two choices B or C, so there are 2 × 1 × 2 = 4
ways of jump.
(ii) if Y is not A, then X is determined by Y, as well as Z is also determined. There
are two choices of Y so there are 2 × 1 × 1 = 2 ways of jump.
Totally there are 4 + 2 = 6 ways of jump. Hence (C).
Answer:(C)
10. In 1202 A.D., Italian mathematician Fibonacci (1170~1250) wrote in his book
《Liber Abaci》 the following interesting problem: Exactly two months after
their birth, a pair of rabbits will give birth to a new pair (one male and one
female) and then give birth to a pair each month after that. There is only one pair
of new born rabbits at the beginning. If the rabbits never die, how many pair of
rabbits are there after exactly 12 months?
(A)144 (B)233 (C)234 (D)235 (E)377
【Solution】
By iteration, we get the following sequence:
1, 1, 2, 3, 5, 8, 13, 21, 34, 55, 89, 144, 233, …
where the 13th number is 233. The sequence is the so-called Fibonacci sequence,
where each term (starting from the third) is the sum of the two terms before it.
Hence (B).
Answer:(B)
11. Define “ *” as an operation such that 4 ∗ 2 = 82 , 6 ∗ 3 = 183 , 8 ∗ 4 = 324 ,
9 ∗ 3 = 276 and 9 ∗ 5 = 454 . What is the value of 10 ∗ 2 ?
(A)55 (B)125 (C)202 (D)208 (E)2002
【Solution】
From the examples it shows that the result is the products followed by the difference.
As 10 × 2 = 20 , 10 − 2 = 8 , we have 10 ∗ 2 = 208 . Hence (D).
Answer:(D)

220
12. Bob got a score of 94 on foreign language test, and his average score on the
native language and math tests is 97. What is his average score on these three
tests?
(A)94 (B)94.5 (C)95 (D)95.5 (E)96
【Solution 1】
His total score of native language and math is 97 × 2 = 194 , and total score of three
subjects is 194 + 94 = 288 , so the average of three is 288 ÷ 3 = 96 . Hence (E).
【Solution 2】
Since the average score on native language and math is 97 and the score for foreign
language is 94, the scores on native language and math are totally (97 − 94) × 2 = 6
more than the score for foreign language. Thus the average score of the three tests is
6 ÷ 3 = 2 more than the score for foreign language, i.e. the average score of the three
tests is 94 + 2 = 96 . Hence (E).
Answer:(E)
13. Color the surfaces of a 5 × 5 × 5 wooden cube red, then cut it into smaller
1 ×1 ×1 cube pieces. How many of these smaller cubes have exactly two red
faces?
(A)36 (B)48 (C)61 (D)90 (E)98
【Solution】
Unit cubes with two red faces are exactly those adjacent to edges of the large cube
but not at the corner. There are 5 − 2 = 3 such unit cubes on each edge of the large
cube, who has 12 edges. The total number of unit cubes with two red faces is then
12 × 3 = 36 . Hence (A).
Answer:(A)
14. A palindrome number is a positive integer that is the same when read forwards or
backwards. The numbers 909 and 1221 are examples of palindromes. How many
palindrome numbers are there between 10 and 1000?
(A)90 (B)99 (C)100 (D)106 (E)108
【Solution】
A two-digit number is a palindrome number if the two digits are the same but
non-zero, so there are 9 of them. A three-digit number is a palindrome number if the
first digit is equal to the third and non-zero, there are 90 of them. 1000 is not a
palindrome number. Totally there are 9 + 90 = 99 of palindrome numbers between
10 and 1000. Hence (B).
Answer:(B)
15. In the month of February of some year, there are more Saturdays than any other
days in a week. What day is the last day of this month?
(A)Wednesday (B)Thursday (C)Friday
(D)Saturday (E)Sunday
【Solution】
February in a flat year has 28 days, which is a multiple of 7. Each day in a week
appears four times in a flat year February. For a leap year February, the first day or
the last day is the same day in a week, which appear 5 times. Hence (D).
Answer:(D)

221
16. Four students namely Annie, Benny, Charlie and Deany all paid the same amount
of money to buy some number of notebooks together. After distributing the
notebooks, Annie, Benny and Charlie got 6, 7 and 11 notebooks more than Deany,
respectively. As such, to be fair, Annie, Benny and Charlie gave back a total of
$48 to Deany. What is the price of each notebook?
(A)2 (B)6 (C)8 (D)12 (E)16
【Solution】
Totally A, B, C takes 6 + 7 + 11 = 24 notebooks more than D. D takes 24 ÷ 4 = 6
notebooks less than average. He is paid back $48. Each notebook counts for
48 ÷ 6 = 8 dollars. Hence (C).
Answer:(C)
17. A train left town A at 8:30 AM some day and arrived at town B at 1:50 AM of the
next day. There is no time difference between the two places. How long did the
train travelled for the trip?
(A)5 hours 20 minutes (B)10 hours 20 minutes (C)15 hours 20 minutes
(D)16 hours 20 minutes (E)17 hours 20 minutes
【Solution】
The train used 15 hours 30 minutes on the first day; and 1 hours 50 minutes on the
next day. The total time is 17 hours 20 minutes. Hence (E).
Answer:(E)
18. Mike placed 4 identical squares, each with side length 5 cm and are
non-overlapping, to form a new figure as shown below. Find the perimeter, in cm,
of this new figure.

(A)15 (B)20 (C)30 (D)45 (E)50


【Solution】
The figure can be cut into three regions as below.

The left and right part must be covered by one square each and the middle region is
covered by two. The region has the same length of circumference as the minimum

222
rectangles covering it, which is 5 × 3 = 15 cm wide and 5 × 2 = 10 cm high. The
circumference is (15 + 10) × 2 = 50 cm. Hence (E).
A B

D C Answer:(E)
19. Adam owns an old watch, which is slower than a normal watch by the same
amount of time for each hour. At 8 o'clock in the morning, the old watch reads 8
o'clock. At 9 o'clock in the morning, it reads 8:58. What time does it read when
the real time is 4 o'clock in the afternoon?
(A)3:42 (B)3:44 (C)3:46
(D)4:08 (E)4:16
【Solution】
The watch is 2 minutes slower every hour. There are 8 hours from 8 am to 4 pm. The
watch is then 2 × 8 = 16 minutes behind. It reads 3:44. Hence (B).
Answer:(B)
20. Identical equilateral triangles are placed together into two figures as shown below.
One can cover Figure B using 5 pieces of Figure A. How many different ways
can we do the covering?

Figure A

Figure B
(A)3 (B)4 (C)5 (D)6 (E)7
【Solution】
The triangle X can be covered in three ways:
Case (i)
X

Then another piece of A is determined as below


X

223
The blank triangle area can be covered in two ways:
X X

This case contributes two cases in total

Case (ii)
X

This case is symmetric to case (i), so it contributes two cases.


Case (iii)
X

The blank area has only one covering method.


X

This case contributes 1.


In total, there are 2 + 2 + 1 = 5 ways of covering. Hence (C).
Answer:(C)
21. Put some unit cubes into a 3D model such that in the model, each cube touches
some other cubes in at least one point. It is known that the model looks like the
figure below from three directions of upright front, left and top. What is the least
number of unit cubes needed to make such a model?

224
【Solution】
It is obvious the model is contained in a cube of edge length 3 and contains some of
the 27 unit cubes of the large cube. The unit cube in the center has to appear in the
model. The cubes at the center of faces of the large cube must not appear. The cubes
at the middle of edges of the large cube must not appear. Moreover, one of the two
unit cubes at any adjacent corners of the large cube must appear. It is then obvious by
pigeonhole principle, one needs at least 5 unit cubes. One such construction is in the
figure below.

Answer:005
22. How many ways can we divide 6 students into 3 groups so that each group has
exactly 2 students?
【Solution】
Denote the six persons as A, B, C, D, E, F. There are five possibilities for the person
in the same group as A. Without loss of generality, assume A and B are in the same
group. Then there are 3 possibilities for the person in the same group as C. The last
group is also determined. Totally there are 5 × 3 = 15 ways of division.
Answer:015
23. In the figure below, ABCD, EFGH and AJKL are squares. The area of AJKL is
2018 cm2. If rectangles EFCI and JBFK both have an area of 1360 cm2, then
what is the area, in cm2, of CGHI?
A L D

I H
E

J K

B F C G

【Solution】
Since DL = AD − AL = AB − AJ = BJ and EFCI has the same area as JBFK, we
have EF = JK . Square AJKL and EFGH are of the same area. Then CGHI has area
2018 − 1360 = 658 cm2.
Answer:658

225
24. Three pairs of red, four pairs of yellow and five pairs of white socks are placed in
a bag. Now, blindly take a sock out each time. How many socked are needed to
be taken out to guarantee having six pairs of socks? (Note: Two socks of the
same color are considered a pair)
【Solution】
Since two socks of the same color forms a pair, one needs 12 socks. Now there are
three colors, there might be two single socks, so at least one needs to take out
12 + 1 + 1 = 14 socks.
Answer:014
25. Cut the 6 × 6 square table below into rectangles along grid lines such that no
two rectangles are identical. What is the maximum number of rectangles one can
get? (Note: A square is considered a rectangle.)

【Solution】
For more rectangles, the rectangles we get need to be as small in area as possible.
There one rectangle of area 1, 2 or 3; two rectangles of area 4(1 × 4 and 2 × 2 ); one of
area 5; two of area 6(1 × 6 and 2 × 3 ). These are 7 rectangles of smallest area. Since
1 + 2 + 3 + 4 + 4 + 5 + 6 + 6 = 31 < 6 × 6 = 36 < 1 + 2 + 3 + 4 + 4 + 5 + 6 + 6 + 7 = 38 ,
There are at most 8 different rectangles, while the figure below shows one such
example.

Answer:008

226
注意:

允許學生個人、非營利性的圖書館或公立學校合理使用
本基金會網站所提供之各項試題及其解答。可直接下載
而不須申請。

重版、系統地複製或大量重製這些資料的任何部分,必
須獲得財團法人臺北市九章數學教育基金會的授權許
可。

申請此項授權請電郵 ccmp@seed.net.tw

Notice:

Individual students, nonprofit libraries, or schools are


permitted to make fair use of the papers and its
solutions. Republication, systematic copying, or
multiple reproduction of any part of this material is
permitted only under license from the Chiuchang
Mathematics Foundation.
Requests for such permission should be made by
e-mailing Mr. Wen-Hsien SUN ccmp@seed.net.tw

227
8th International Mathematics Assessments for Schools
(2018-2019)

Middle Primary Division Round 2


Time: 120 minutes

Printed Name: Code: Score:

Instructions:
 Do not open the contest booklet until you are told to do so.
 Be sure that your name and code are written on the space provided above.
 Round 2 of IMAS is composed of three parts; the total score is 100 marks.
 Questions 1 to 5 are given as a multiple-choice test. Each question has five
possible options marked as A, B, C, D and E. Only one of these options is correct.
After making your choice, fill in the appropriate letter in the space provided. Each
correct answer is worth 4 marks. There is no penalty for an incorrect answer.
 Questions 6 to 13 are a short answer test. Only Arabic numerals are accepted;
using other written text will not be honored or credited. Some questions have more
than one answer, as such all answers are required to be written down in the space
provided to obtain full marks. Each correct answer is worth 5 marks. There is no
penalty for incorrect answers.
 Questions 14 and 15 require a detailed solution or process in which 20 marks are
to be awarded to a completely written solution. Partial marks may be given to an
incomplete presentation. There is no penalty for an incorrect answer.
 Use of electronic computing devices is not allowed.
 Only pencil, blue or black ball-pens may be used to write your solution or answer.
 Diagrams are not drawn to scale. They are intended as aids only.
 After the contest the invigilator will collect the contest paper.
The following area is to be filled in by the judges;
the contestants are not supposed to mark anything here.
Total
Question 1 2 3 4 5 6 7 8 9 10 11 12 13 14 15 Score Signature

Score

Score

228
229
Middle Primary Division Round 2
Questions 1 to 5, 4 marks each
1. What is the value of 100 − 97 + 94 − 91 + 88 − 85 + L + 4 − 1 ?
(A)45 (B)48 (C)51 (D)54 (E)57

Answer:

2. The figure below is formed by using 12 identical equilateral triangles. How many
equilateral triangles of different sizes (and which are located in different places)
are there?

(A)12 (B)14 (C)16 (D)18 (E)20

Answer:

3. Place the numbers 1, 2, 3, 4, 5 and 6, without repetition, into the six circles in the
figure below, where each circle should only have one number, such that the sum
of the three numbers on each side of the triangle are all equal. What is the
maximum possible value of this sum?

(A)9 (B)10 (C)11 (D)12 (E)13

Answer:

4. Multiply a two-digit number by 3 then add 10 to it. Now, we swap the order of
the two digits of the result. The resulting number is an integer among 95, 96, 97,
98 and 99. What is the original number?
(A)21 (B)22 (C)23 (D)24 (E)25

Answer:

230
MP 2
5. If the month of January of a certain year has four Saturdays and five Sundays,
then what day is January 17th of that particular year?
(A)Monday (B)Tuesday (C)Wednesday
(D)Thursday (E)Friday

Answer:

Questions 6 to 13, 5 marks each

6. A square has a perimeter 48 cm. Cut it into four identical small squares along the
dashed lines as shown below. What is the sum of the perimeters of the four
smaller squares?

Answer: cm

7. Lily went shopping and bought three items from three different stores. She then
noticed that whenever she was paying for an item, the money in her pocket was
exactly five times the amount to be paid. After shopping, she noticed that she has
$64 left in her pocket. How much money did she have before she went shopping?
Answer: dollars

8. In the figure below, ABCD is a square with side length of 10 cm and AFE is an
isosceles right triangle with hypotenuse of length 14 cm, where E is on the
extension of line AB. What is the area, in cm2, of the shaded region?
D C
F

A B E
Answer: cm2

231
MP 3
9. There are a total of 40 students in a class. 23 of them are able to ride bikes, 33 of
them are able to swim and 5 of them are unable to do either. How many students
in this class are able to ride bikes but are not able to swim?

Answer: students

10. A bridge is 1500 m long. A train passes through the bridge at a speed of 30 m per
second. The train is 300 m long. How long, in seconds, does it take for the train
to pass the bridge completely, starting from the time it entered the bridge?

Answer: seconds

11. In the figure below, color each of the six circles into 4 colors: Red, Yellow, Blue
and Black. Each circle should contain only one color, and any two circles
connected by a line segment should have different colors. In how many different
ways can we color the figure below? (Note: Coloring methods that are identical
by a reflection of the figure are NOT considered the same)

Answer: ways

12. In quadrilateral ABCD, ∠BAD = ∠BCD = 90° . Points E and F are on sides AD
and BC respectively and AB = 5 cm, CD = 10 cm, DE = 8 cm, BF = 6 cm, as
shown in the figure below. If the area of triangle BEF is 4 cm2 less than the area
of triangle DEF. What is the area, in cm2, of triangle DEF?
A E D

B
F
C
Answer: cm2

232
MP 4
13. The numbers 1, 2, 3, 4, 5 and 6 are written on the six faces of a unit cube without
repetition. Each face contains one number and the sum of the numbers in every
two opposite faces is 7. Put four such cubes side by side as shown in the figure
below, such that sum of every two numbers of every two touched faces is 8. Find
the number marked with “?” in the figure.
5

1

Answer:

Questions 14 to 15, 20 marks each


(Detailed solutions are needed for these two problems)
14. A mouse starts from the top left-most unit square marked with “I”, follows a route
to form the word “IMAS2019” by moving from one square to another square that
share a common side. How many different routes of eight squares are there?
I M A S
M A S 2 0
A S 2 0 1
S 2 0 1 9
0 1 9

Answer: routes

233
MP 5
15. An infinite sequence of numbers 1, 2, 3, 5, 8, 3, 1, 4, 5, 9, 4, … follows the
pattern such that starting from the third number, each number is equal to the units
digit of the sum of the two numbers in the sequence immediately preceding it.
What is the 2019th number of the sequence?

Answer:

234
235
注意:

允許學生個人、非營利性的圖書館或公立學校合理使用
本基金會網站所提供之各項試題及其解答。可直接下載
而不須申請。

重版、系統地複製或大量重製這些資料的任何部分,必
須獲得財團法人臺北市九章數學教育基金會的授權許
可。

申請此項授權請電郵 ccmp@seed.net.tw

Notice:

Individual students, nonprofit libraries, or schools are


permitted to make fair use of the papers and its
solutions. Republication, systematic copying, or
multiple reproduction of any part of this material is
permitted only under license from the Chiuchang
Mathematics Foundation.
Requests for such permission should be made by
e-mailing Mr. Wen-Hsien SUN ccmp@seed.net.tw

236
Solution Key to Second Round of IMAS 2018/2019
Middle Primary Division
1. What is the value of 100 − 97 + 94 − 91 + 88 − 85 + L + 4 − 1 ?
(A)45 (B)48 (C)51 (D)54 (E)57
【Solution 1】
100 − 97 + 94 − 91 + 88 − 85 + L + 4 − 1 = (100 − 97) + (94 − 91) + (88 − 85) + L + (4 − 1)
= 3144
+ 3 + 244
3 + L +33
17 terms

= 3 × 17 = 51
Hence (C).
【Solution 2】
100 − 97 + 94 − 91 + 88 − 85 + L + 4 − 1 = (100 + 94 + 88 + L + 4) − (97 + 91 + 85 + L + 1)
(100 + 4) × 17 (97 + 1) × 17
= −
2 2
17
= (104 − 98) ×
2
17
= 6 × = 51
2
Hence (C).
Answer:(C)
2. The figure below is formed by using 12 identical equilateral
triangles. How many equilateral triangles of different sizes
(and which are located in different places) are there?

(A)12 (B)14 (C)16 (D)18 (E)20


【Solution】
Let the area of a smallest equilateral triangle be 1 unit. There are 12 equilateral
triangles of area 1. There are 6 equilateral triangles of area 4 and there are 2
equilateral triangles of area 9. Hence there are 12 + 6 + 2 = 20 equilateral triangles at
different positions. Hence (E).

Answer:(E)

237
3. Place the numbers 1, 2, 3, 4, 5 and 6, without repetition, into the six circles in the
figure below, where each circle should only have one number, such that the sum
of the three numbers on each side of the triangle are all equal. What is the
maximum possible value of this sum?

(A)9 (B)10 (C)11 (D)12 (E)13


【Solution】
To make required sum the largest, the sum of the numbers of the three sides must
reach the maximum. Since the numbers at the vertices of the triangle are counted
twice, so the largest numbers should be placed at the vertices, that is, 4, 5 and 6. Then
sum of numbers on three sides is 1 + 2 + 3 + 4 + 5 + 6 + 4 + 5 + 6 = 36 , hence the sum
of number on each side is then 36 ÷ 3 = 12 . Therefore, the numbers can be placed as
follows. Hence (D).

3 2

5 1 6
Answer:(D)
4. Multiply a two-digit number by 3 then add 10 to it. Now, we swap the order of
the two digits of the result. The resulting number is an integer among 95, 96, 97,
98 and 99. What is the original number?
(A)21 (B)22 (C)23 (D)24 (E)25
【Solution 1】
Working backwards, swap the digits of the following numbers: 95, 96, 97, 98, 99 and
subtract by 10. The resulting numbers are 49, 59, 69, 79, 89, among which only 69 is
a multiple of 3, so the original number is 69 ÷ 3 = 23 . Hence (C).
【Solution 2】
Multiply an integer by 3 and add 10, the resulting number has a remainder of 1 when
it is divided by 3. Swapping the digits, the number also has remainder 1 when divided
by 3. Among 95, 96, 97, 98, 99, only 97 has remainder 1 divided by 3. Thus, the
original number is (79 − 10) ÷ 3 = 23 . Hence (C).
Answer:(C)

238
5. If the month of January of some year has four Saturdays and five Sundays, then
what day is January 17th of that particular year?
(A)Monday (B)Tuesday (C)Wednesday
(D)Thursday (E)Friday
【Solution】
There are 31 days in January, which is three days more than 4 weeks. Since there are
4 Saturdays and 5 Sundays, the first day of the month is a Sunday and January 17th is
a Tuesday. Hence (B)
Answer:(B)
6. A square has a perimeter 48 cm. Cut it into four identical small squares along the
dashed lines as shown below. What is the sum of the perimeters of the four
smaller squares?

【Solution 1】
The large square has perimeter 48 cm, hence its side length is 12 cm. Each small
square has side length 12 ÷ 2 = 6 cm and so perimeter 6 × 4 = 24 cm. Hence sum of
the perimeter is 24 × 4 = 96 cm.
【Solution 2】
After cutting into 4 small squares, the total perimeter will be increase by double of
the length of the dashed lines, which is equal to perimeter of the large square. Thus,
the total perimeter of 4 small squares is 48 × 2 = 96 cm.
Answer:96 cm
7. Lily went shopping and bought three items from three different stores. She then
noticed that whenever she was paying for an item, the money in her pocket was
exactly five times the amount to be paid. After shopping, she noticed that she has
$64 left in her pocket. How much money did she have before she went shopping?
【Solution 1】
Working backwards, she had 64 ÷ 4 × 5 = 80 dollars before the third purchase. She
had 80 ÷ 4 × 5 = 100 dollars before the second purchase. She had 100 ÷ 4 × 5 = 125
dollars before buying the first purchase.
【Solution 2】
4
Each time she bought, the money left is of the money before she bought. After
5
4 4 4 64
three purchases, the money left was × × = of the money before the first
5 5 5 125
64
purchase. So before the first purchase she had 64 ÷ = 125 dollars.
125
Answer:125 dollars

239
8. In the figure below, ABCD is a square with side length of 10 cm and AFE is an
isosceles right triangle with hypotenuse of length 14 cm, where E is on the
extension of line AB. What is the area, in cm2, of the shaded region?
D C
F

【Solution 1】 A B E
From the given information, BE = 14 − 10 = 4 cm. Since triangle EBG is also an
1
isosceles right triangle, its area is × 4 × 4 = 8 cm2. In isosceles right triangle AEF,
2
the length of the height on hypotenuse AE is equal to the half of the length of AE, so
1
the area of AEF is × 14 × 7 = 49 cm2. (or the area of isosceles right triangle AEF is
2
1 1
equal to × AE × AE = × 14 × 14 = 49 cm2.) Since the side length of square ABCD
4 4
is 10 cm, then the area of the shaded region is 10 × 10 − (49 − 8) = 59 cm2.
【Solution 2】
Since CG = CB − GB = CB − BE = CB − ( AE − AB ) = 10 − (14 − 10) = 6 cm. Since
triangle AEF is an isosceles right triangle, then ∠FAE = 45° and F is on the diagonal
of the square, CFG is also an isosceles right triangle. The shaded area is equal to sum
1 1
of area of triangle ACD and CFG, that is, × 10 × 10 + × 6 × 6 = 59 cm2.
2 4
Answer:59 cm2
9. There are a total of 40 students in a class. 23 of them are able to ride bikes, 33 of
them are able to swim and 5 of them are unable to do either. How many students
in this class are able to ride bikes but are not able to swim?
【Solution】
The number of students who are able to do at least one sport is 40 − 5 = 35 . Then the
number of students who can do both sports is 23 + 33 − 35 = 21 . The number of
students who can ride but not swim is then 23 − 21 = 2 students.
Answer:2 students
10. A bridge is 1500 m long. A train passes through the bridge at a speed of 30 m per
second. The train is 300 m long. How long, in seconds, does it take for the train
to pass the bridge completely, starting from the time it entered the bridge?
【Solution】
The train travels a distance of the bridge length and the train length, which is
1500 + 300 = 1800 m. Hence it takes 1800 ÷ 30 = 60 seconds for the train to leave
the bridge completely.
Answer:60 seconds

240
【Note】
In a problem about a train going through a bridge, the time depends on the length of
the bridge as well as on the length of train.
11. In the figure below, color each of the six circles into 4 colors: Red, Yellow, Blue
and Black. Each circle should contain only one color, and any two circles
connected by a line segment should have different colors. In how many different
ways can we color the figure below? (Note: Coloring methods that are identical
by a reflection of the figure are NOT considered the same)

【Solution】
Focusing on the four circles in the triangle area. Color the six circles in a specific
sequence. Firstly, there are 4 ways to color the top circle. Secondly, there are three
ways to color the circle right under the top circle, which is not the same color of the
first one. Thirdly, there are 2 ways to color the circle on the right vertex of the
triangle, which is not the same color as the first circle and as the second circle.
Fourthly, there are 2 ways to color the circle on the left vertex of the triangle. Fifthly
and Sixthly, there are 3 ways each to color the last two circles on the bottom line,
which has only one color excluded to use. Hence by multiplication principle, there
are 4 × 3 × 2 × 2 × 3 × 3 = 432 ways to color all six circles.

Answer:432 ways
12. In quadrilateral ABCD, ∠BAD = ∠BCD = 90° . Points E and F are on sides AD
and BC respectively and AB = 5 cm, CD = 10 cm, DE = 8 cm, BF = 6 cm, as
shown in the figure below. If the area of triangle BEF is 4 cm2 less than the area
of triangle DEF. What is the area, in cm2, of triangle DEF?

A E D

B
F
C

241
【Solution】
1 1
Connect BD. We know the area of triangle BDE is × DE × AB = × 8 × 5 = 20 cm2.
2 2
1 1
The area of BFD is × BF × CD = × 6 × 10 = 30 cm2. Then the sum of areas of
2 2
triangle BEF and DEF is 20 + 30 = 50 cm2. Since the difference of area of triangle
DEF and BEF is 4 cm2, hence the area of triangle DEF is (50 + 4) ÷ 2 = 27 cm2.
A E D

B
F
C Answer:27 cm2
13. The numbers 1, 2, 3, 4, 5 and 6 are written on the six faces of a unit cube without
repetition. Each face contains one number and the sum of the numbers in every
two opposite faces is 7. Put four such cubes side by side as shown in the figure
below, such that sum of every two numbers of every two touched faces is 8. Find
the number marked with “?” in the figure.
5

1
【Solution】
Since sum of numbers on opposite faces is 7, then 1 is opposite to 6, and 2 is opposite
to 5, and 3 is opposite to 4. The right face of the first cube has number 3 or 4.
If it is 3, the left face of the second cube has number 5, right face has number 2, left
face of the third cube has number 6, right face has 1, left face of the fourth cube has
number 7, which is a contradiction.
If it is 4, the left face of the second cube has number 4, right face has number 3, left
face of the third cube has number 5, right face has 2, left face of the fourth cube has
number 6, right face has number 1, which is the place with mark “?”.
Answer:1
14. A mouse starts from the top left-most unit square marked with “I”, follows a route
to form the word “IMAS2019” by moving from one square to another square that
share a common side. How many different routes of eight squares are there?
I M A S
M A S 2 0
A S 2 0 1
S 2 0 1 9
0 1 9

242
【Solution】
In the following table, each square is filled with the number of routes reaching it. The
number can be derived by recursion: each square is filled with sum of the numbers in
adjacent squares with previous marks. From the table, it shows that the number of
different routes with length eight is 34 + 34 = 68 .
1 1 1 1
1 2 3 4 4
1 3 6 10 14
1 4 10 20 34
4 14 34
Answer: 68 routes
【Marking Schemes】
Find correct number of routes to squares with 2: 4 + 6 + 4 = 14 , 5 points.
Find correct number of routes to squares with 0: 4 + 10 + 10 + 4 = 28 , 5 points.
Find correct number of routes to squares with 1: 14 + 20 + 14 = 48 , 5 points
Find correct number of routes to squares with 9: 34 + 34 = 68 , 5 points.

15. An infinite sequence of numbers 1, 2, 3, 5, 8, 3, 1, 4, 5, 9, 4, … follows the


pattern such that starting from the third number, each number is equal to the units
digit of the sum of the two numbers in the sequence immediately preceding it.
What is the 2019th number of the sequence?
【Solution 1】
This recurrence sequence follows a pattern when considering the remainders of
numbers in the sequence divided by a fixed number. Since 10 = 2 × 5 , we observe the
cases by dividing 2 and 5.
Considering remainders when each number of the sequence divided by 2, the pattern
is
1, 0, 1, 1, 0, 1, 1, 0, 1, ….
The period is 3.
Considering remainders when each number of the sequence divided by 5, the pattern
is
1, 2, 3, 0, 3, 3, 1, 4, 0, 4, 4, 3, 2, 0, 2, 2, 4, 1, 0, 1, 1, 2, 3, 0, ….
The period is 20.
Since 2019 is divisible by 3, the remainder of the 2019th number when divided by 2 is
the same as remainder of the third number, which is 1.
Since 2019 = 20 × 100 + 19 , the remainder of the 2019th number divided by 5 is the
same as remainder of the 19th number, which is 0.
The 2019th number is a digit with remainder 1 when divided by 2 and remainder 0
when divided by 5, hence this number is 5.

243
【Solution 2】
The first seventy terms of the sequence are
1, 2, 3, 5, 8, 3, 1, 4, 5, 9, 4, 3, 7, 0, 7, 7, 4, 1, 5, 6, 1, 7, 8, 5, 3, 8, 1, 9, 0, 9, 9, 8, 7, 5,
2, 7, 9, 6, 5, 1, 6, 7, 3, 0, 3, 3, 6, 9, 5, 4, 9, 3, 2, 5, 7, 2, 9, 1, 0, 1, 1, 2, 3, 5, 8, 3, 1, 4,
5, 9, ….
We can observe that it is a recurrence sequence with period 60. 2019 = 60 × 33 + 39 ,
so the 2019th term is same as the 39th term, which is 5.
Answer:5
【Note】
The sequence is formed by the unit digits of Fibonacci sequence with period 60.

【Marking Schemes】
Find periodic pattern of parity of the sequence is 3, 5 points.
Find periodic pattern of the remainders divided by 5 of the sequence is 20, 5 points.
(Or find the sequence has period 60, 10 points.)
Find the correct answer without a reasoning, 10 points only.

244
注意:

允許學生個人、非營利性的圖書館或公立學校合理使用
本基金會網站所提供之各項試題及其解答。可直接下載
而不須申請。

重版、系統地複製或大量重製這些資料的任何部分,必
須獲得財團法人臺北市九章數學教育基金會的授權許
可。

申請此項授權請電郵 ccmp@seed.net.tw

Notice:

Individual students, nonprofit libraries, or schools are


permitted to make fair use of the papers and its
solutions. Republication, systematic copying, or
multiple reproduction of any part of this material is
permitted only under license from the Chiuchang
Mathematics Foundation.
Requests for such permission should be made by
e-mailing Mr. Wen-Hsien SUN ccmp@seed.net.tw

245
International Mathematics Assessments for Schools

2019 ~ 2020 MIDDLE PRIMARY DIVISION FIRST ROUND PAPER


Time allowed:75 minutes

When your teacher gives the signal, begin working on the problems.

INSTRUCTION AND INFORMATION


GENERAL
1. Do not open the booklet until told to do so by your teacher.
2. No calculators, slide rules, log tables, math stencils, mobile phones or other
calculating aids are permitted. Scribbling paper, graph paper, ruler and compasses
are permitted, but are not essential.
3. Diagrams are NOT drawn to scale. They are intended only as aids.
4. There are 20 multiple-choice questions, each with 5 choices. Choose the most
reasonable answer. The last 5 questions require whole number answers between
000 and 999 inclusive. The questions generally get harder as you work through the
paper. There is no penalty for an incorrect response.
5. This is a mathematics assessment, not a test; do not expect to answer all questions.
6. Read the instructions on the answer sheet carefully. Ensure your name, school
name and school year are filled in. It is your responsibility that the Answer Sheet
is correctly coded.

THE ANSWER SHEET


1. Use only pencils.
2. Record your answers on the reverse side of the Answer Sheet (not on the question
paper) by FULLY filling in the circles which correspond to your choices.
3. Your Answer Sheet will be read by a machine. The machine will see all markings
even if they are in the wrong places. So please be careful not to doodle or write
anything extra on the Answer Sheet. If you want to change an answer or remove
any marks, use a plastic eraser and be sure to remove all marks and smudges.

INTEGRITY OF THE COMPETITION


The IMAS reserves the right to re-examine students before deciding whether to
grant official status to their scores.

246
247
2019 ~ 2020 MIDDLE PRIMARY DIVISION FIRST ROUND PAPER

Questions 1-10, 3 marks each


1. What is the simplified value of
13  12  11  ...  3  2  1  2  3  ...  11  12  13 ?
(A)181 (B)182 (C)183 (D)184 (E)185

2. If (  2  1)  3  2019 , then what is the value of  ?


(A)335 (B)336 (C)337 (D)3028 (E)3029

3. In the figure below, each unit square has side length of 1 cm. What is the area,
in cm2, of the shaded region?

(A)6 (B)8 (C)10 (D)12 (E)14

4. A square piece of paper, with a perimeter of 48 cm, is to be cut into three


rectangles of the same size as shown in the figure below. What is the perimeter,
in cm, of each rectangle that was cut out?

(A)12 (B)18 (C)24 (D)32 (E)40

5. In the equation below, fill in the five numbers 7, 50, 51, 55, and 63 into the five
circles to make the equality true. What is the sum of the numbers in the first three
circles?
   
(A)105 (B)106 (C)113 (D)114 (E)118

6. How many different 3-digit numbers are there such that the sum of its digits is
equal to 4?
(A)7 (B)8 (C)9 (D)10 (E)11

248
MP 2
7. There are a total 80 pandas in a zoo, all of which are housed in three different
halls. The number of pandas in Hall 1 is twice that in Hall 2, and the number of
pandas in Hall 2 is three times that in Hall 3. How many pandas are there in Hall
2?
(A)16 (B)24 (C)36 (D)48 (E)60

8. Every page of a book is either all text or all illustrations and that there are exactly
three pages of illustrations between every two pages of text, that is, if the last
page of the book is text only, then one page before and after the three-page
illustration are all-texts. If the last page is an illustration, in addition to the
illustrations on the last few pages, one page before and after the other three pages
of illustrations is text. If the first page of the book is text, and it has a total of 136
pages, how many pages are there in this book are illustrations only?
(A)104 (B)103 (C)102 (D)68 (E)34

9. For any positive integer, we compute by following the operating rules: If the
number is an even number, then we divide it by 2; and if it is odd, then we add 1
to it. We continue to do these operations until we arrive with the value of “1” for
the first time. How many numbers will leave a value of “1” after four operations?
(A)2 (B)3 (C)4 (D)5 (E)6

10. Among the numbers from 0 to 200, what is the average of all numbers that are
multiples of 3?
(A)95 (B)96 (C)99 (D)100.5 (E)101

Questions 11-20, 4 marks each

11. What is the tens digit of the simplified value of the following expression:
1  12  123  1234  12345  123456  1234567  12345678  123456789 ?
(A)0 (B)2 (C)4 (D)6 (E)8

249
MP 3
12. In the figure below, the largest rectangle has dimensions 9 cm long by 8 cm wide,
with a cross-section formed by two shaded rectangles in the middle. If the total
area of the shaded parts is exactly equal to the area of the un-shaded parts, then
what is the value, in cm, of x?
3 cm

x cm
8 cm

9 cm
(A)1 (B)2 (C)3 (D)4 (E)5

13. Use the digits 2, 0, 1, and 9 exactly once to form different 4-digit numbers.
How many of these 4-digits numbers will be greater than 1905?
(A)12 (B)13 (C)15 (D)17 (E)19

14. After deleting one of the six numbers 2, 3, 12, 26, 29 and 41, the five remaining
numbers can be divided into two groups with the same sum. What is the number
that was deleted?
(A)3 (B)12 (C)26 (D)29 (E)41

15. As shown in the diagram, three of the seven areas enclosed by the three circles
are each filled with a number, and it is known that the numbers 13, 15, 17, and 18
can be filled inside the four blank areas such that the sum of the four numbers in
each circle are all equal. What is the sum?

14 12

16

(A)58 (B)59 (C)60 (D)61 (E)62

250
MP 4
16. In the figure below, each unit square has a side length of 1 cm. How many
squares and rectangles are there?

(A)9 (B)12 (C)14 (D)15 (E)16

17. A theater has nine rows of seats. The first row has a total of 19 seats and starting
from the second row, each row has two more seats than the previous row. How
many seats does this theater have in total?
(A)173 (B)187 (C)243 (D)261 (E)280

18. Divide the six positive integers 2, 3, 7, 13, 18, and X into three groups so that the
first group contains one number, the second group contain two numbers, and the
third group contain three numbers, where X must belong in the third group. If the
number in the first group is greater than the sum of the numbers in the second
group, and that the sum of the numbers in the second group is greater than the
sum of the numbers in the third group, then what is the maximum possible value
of X?
(A)4 (B)5 (C)6 (D)7 (E)8

19. In the figure below, rectangle ABCD has a perimeter of 32 cm, where both
ABFE and EGHD are squares. If the perimeter of GFCH is 12 cm, then what is
the area, in cm2, of rectangle ABCD?
A E D

G H

B F C

(A)28 (B)39 (C)48 (D)55 (E)60

251
MP 5
20. As shown in the figure below, an ant starting from vertex A of the cuboid needs
to move along the edges to reach its destination, vertex B. If the ant can only pass
through three edges, how many possible paths can the ant crawl to reach the
destination?
A

(A)3 (B)6 (C)9 (D)12 (E)15

Questions 21-25, 6 marks each


21. A 5-digit number is formed by using digits that are all different. It is known that
four of the digits used are 3, 6, 8 and 9, and that the difference between any two
adjacent digits is greater than 3. How many 5-digit numbers are there that satisfy
the condition?

22. Joe has a total of 31 dollars that he will use to purchase four different items that
have unit prices of 2, 3, 5 and 7 dollars each respectively. If he needs to purchase
at least one piece of each item, and Joe has to use all his money without leftovers,
in how many different ways can he buy the goods?

23. It is known that the sum of the ages of A and B this year is 60 years old. 2 years
ago, the sum of the ages of B and C is also 60 years old and 3 years from now,
the sum of the ages of A and C is still 60 years old. How old is A this year?

24. A coin is placed in one of the unit squares in the grid as shown below. How many
rectangles (including squares) can be formed that contains the coin?

25. There are 25 boys in a class, 16 of whom are members of the football team, 12 in
the basketball team and 10 in the volleyball team. It is known that no one
participates in all three sports at the same time, and each boy participates in at
least one team. If five boys participate in both the football and basketball teams,
and three boys participate in both the basketball and volleyball teams. How many
boys are there that participate in both the football and volleyball teams?

***

252
253
注意:

允許學生個人、非營利性的圖書館或公立學校合理使用
本基金會網站所提供之各項試題及其解答。可直接下載
而不須申請。

重版、系統地複製或大量重製這些資料的任何部分,必
須獲得財團法人臺北市九章數學教育基金會的授權許
可。

申請此項授權請電郵 ccmp@seed.net.tw

Notice:

Individual students, nonprofit libraries, or schools are


permitted to make fair use of the papers and its
solutions. Republication, systematic copying, or
multiple reproduction of any part of this material is
permitted only under license from the Chiuchang
Mathematics Foundation.
Requests for such permission should be made by
e-mailing Mr. Wen-Hsien SUN ccmp@seed.net.tw

254
Solution to
2019~2020 International Mathematics Assessment for Schools
Round 1 of Middle Primary Division
1. What is the simplified value of
13 + 12 + 11 + ... + 3 + 2 + 1 + 2 + 3 + ... + 11 + 12 + 13 ?
(A)181 (B)182 (C)183 (D)184 (E)185
【Suggested Solution 1】
13 + 12 + 11 + ... + 3 + 2 + 1 + 2 + 3 + ... + 11 + 12 + 13 = 2  (1 + 2 + 3 + ... + 13) − 1
(1 + 13)  13
= 2 −1
2
= 14  13 − 1
= 182 − 1 = 181
Therefore, the answer is (A).
【Suggested Solution 2】
13 + 12 + 11 + ... + 3 + 2 + 1 + 2 + 3 + ... + 11 + 12 + 13 = (1 + 2 + 3 + ... + 13) + (2 + 3 + 4 + ... + 13)
(1 + 13)  13 (2 + 13)  12
= +
2 2
14  13 15  12
= +
2 2
= 91 + 90 = 181
Therefore, the answer is (A).
Answer: (A)
2. If ( 2 + 1)  3 = 2019 , then what is the value of  ?
(A)335 (B)336 (C)337 (D)3028 (E)3029
【Suggested Solution】
(  2 + 1)  3 = 2019
  2 + 1 = 2019  3 = 673
  2 = 673 − 1 = 672
 = 672  2 = 336
Therefore, the answer is (B).
(B)
Answer:
3. In the figure below, each unit square has side length of 1 cm.
What is the area, in cm2, of the shaded region?

(A)6 (B)8 (C)10 (D)12 (E)14

255
【Suggested Solution 1】
1
In the figure, the area of the 4 unshaded triangles are as follows:  6  1 = 3 cm2,
2
1 1 1
 4  2 = 4 cm2,  6  1 = 3 cm2 and  4  2 = 4 cm2. Therefore, the area of the
2 2 2
shaded area is 4  6 − 3 − 4 − 3 − 4 = 24 − 14 = 10 cm2. Therefore, the answer is (C).

【Suggested Solution 2】
Observe that the original figure is composed of four 2  3 rectangles as shown below,
which are flipped left, flipped right, flipped upside down and rotated 180 :

In each of these 2  3 rectangles, the areas of the two unshaded triangles are
1 3 1
 3  1 = cm2 and  2  2 = 2 cm2. Therefore, the area of the shaded region of the
2 2 2
3 5
original 4  6 rectangle is 4  (2  3 − − 2) = 4  = 10 cm2. Therefore, the answer
2 2
is (C).

【Suggested Solution 3】
Observe that the original figure is composed of four 2  3 rectangles as shown below,
which are flipped left, flipped right, flipped upside down and rotated 180 :
A
B
C
D E
In each of the 2  3 rectangle, the shaded area can be cut into two parts: triangle
ABC and trapezoid BCDE (as labeled in the figure above).
1 1 5
Now, notice that the total area of the shaded region is  2  1 +  (1 + 2)  1 =
2 2 2
cm , therefore, the area of the shaded region of the original 4  6 rectangle is
2

5
4  = 10 cm2. Therefore, the answer is (C).
2

【Suggested Solution 4】
Notice that there are 12 lattice points on the boundary placed on the shaded figure’s
perimeter, while there are 5 lattice points in the interior which are located inside the
figure. Thus, by applying Pick’s Theorem, the area of the shaded region is
12
5 + − 1 = 10 cm2. Therefore, the answer is (C).
2
Answer: (C)

256
4. A square piece of paper, with a perimeter of 48 cm, is to be cut into three
rectangles of the same size as shown in the figure below. What is the perimeter,
in cm, of each rectangle that was cut out?

(A)12 (B)18 (C)24 (D)32 (E)40


【Suggested Solution】
It is known that the side length of the square piece of paper is 48  4 = 12 cm,
therefore, each rectangle has length of 12 cm and width of 12  3 = 4 cm, Therefore,
the perimeter of each rectangle is 2  (12 + 4) = 32 . Therefore, the answer is (D).
Answer: (D)
5. In the equation below, fill in the five numbers 7, 50, 51, 55, and 63 into the five
circles to make the equality true. What is the sum of the numbers in the first three
circles?
+ + − =
(A)105 (B)106 (C)113 (D)114 (E)118
【Suggested Solution】
By moving the 4th term of the original equation to the other side, the equation can be
rewritten as follows:
+ + = +
Now, we have divided the five numbers into two groups, where on side, we have
three numbers and the other side, we have two numbers, and that the sum of the
numbers in each group are equal. Notice that 7 + 50 + 51 + 55 + 63 = 226 , therefore,
the sum of the numbers in each side of the equation should be 226  2 = 113 . Now,
notice that one possible permutation is 113 = 50 + 63 = 7 + 51 + 55 , thus, the sum of
the numbers in the first three circles is 113. Therefore, the answer is (C).
Answer: (C)
6. How many different 3-digit numbers are there such that the sum of its digits is
equal to 4?
(A)7 (B)8 (C)9 (D)10 (E)11
【Suggested Solution 1】
Observe that if 4 is written as a sum of up to three single digits, it can be one of the
following combinations: 4 = 1 + 3 = 2 + 2 = 1 + 1 + 2 , therefore, the number of 3-digit
numbers with sum of digits equals 4 are as follows: 103, 112, 121, 130, 202, 211, 220,
301, 310 and 400, for a total of 10. Therefore, the answer is (D).
【Suggested Solution 2】
Observe that if 4 is written as a sum of up to three digits, it can be one of the
following combinations: 4 = 1 + 3 = 2 + 2 = 1 + 1 + 2 ,
Therefore, the number of 3-digit numbers with sum of digits equal to 4 can be
counted in the following 4 cases:

257
Case 1: If the 3-digit number consists of the digits 0, 0 and 4: there are 3 such
numbers, but 004 and 040 are not 3-digit numbers, so only 1 number
satisfies the condition.
Case 2: If the 3-digit number consists of the digits 0, 1 and 3: there are a total of
3  2 1 = 6 such numbers, but 013 and 031 are not 3-digit numbers, so
there are 4 numbers that satisfies the condition.
Case 3: If the 3-digit number consists of the digits 0, 2 and 2: there are a total of 3
such numbers, but 022 is not a three-digit number, so there are 2 numbers
that satisfies the condition.
Case 4: If the 3-digit number consists of the digits 1, 1 and 2, then there are 3
numbers that satisfies the condition.
So, there are a total of 1 + 4 + 2 + 3 = 10 numbers that satisfy the condition of the
problem. Therefore, the answer is (D).
Answer: (D)
7. There are a total 80 pandas in a zoo, all of which are housed in three different halls.
The number of pandas in Hall 1 is twice that in Hall 2, and the number of pandas
in Hall 2 is three times that in Hall 3. How many pandas are there in Hall 2?
(A)12 (B)24 (C)36 (D)48 (E)60
【Suggested Solution】
The number of pandas in Hall 1 is twice that of Hall 2, and the number of pandas in
Hall 2 is three times that of Hall 3. Therefore, the number of pandas in Hall 1 is
80
2  3 = 6 times that of Hall 3. So the total number of pandas in Hall 3 is =8
1+ 3 + 6
pandas, therefore, the total number of pandas in Hall 2 is 8  3 = 24 . Therefore, the
answer is (B).
Answer: (B)
8. Every page of a book is either all text or all illustrations and that there are exactly
three pages of illustrations between every two pages of text, that is, if the last
page of the book is text only, then one page before and after the three-page
illustration are all-texts. If the last page is an illustration, in addition to the
illustrations on the last few pages, one page before and after the other three pages
of illustrations is text. If the first page of the book is text, and it has a total of 136
pages, how many pages are there in this book are illustrations only?
(A)34 (B)81 (C)102 (D)108 (E)120
【Suggested Solution】
It can be seen that except for the case where the last page is a text, every page of
text-only is followed by a three-page of illustration-only pages and the last text page
may be a 3, 2 or 1 illustration-only page. Now since 136 is a multiple of 4, it can be
judged that the last text page is followed by a 3-page illustration. Thus, the number of
3
illustration-only pages is 136  = 102 . Therefore, the answer is (C).
4
Answer: (C)

258
9. For any positive integer, we compute by following the operating rules: If the
number is an even number, then we divide it by 2; and if it is odd, then we add 1
to it. We continue to do these operations until we arrive with the value of “1” for
the first time. How many numbers will leave a value of “1” after four operations?
(A)2 (B)3 (C)4 (D)5 (E)6
【Suggested Solution】
The number obtained after the odd number is increased by one is always an even
number, and the number obtained by dividing the even number by 2 may be an odd
number or an even number, so it can be either of the two scenarios:
⚫ If the number obtained after the operation is an odd number, the number before
the operation is the odd number multiplied by 2;
⚫ If the number obtained after the operation is even, the number before the
operation is the even number multiplied by 2 or minus 1.
Since we obtain “1” after four operations which is an odd number, the number
obtained after three operations must be 1 2 = 2 .
Since we obtain “2” after three operations, which is an even number, the number
obtained after the second operation can be 2  2 = 4 or 2 − 1 = 1 , but the operation is
stopped if we get 1, so, the number obtained after the second operation must be 4.
Since we obtain “4” after two operations, which is an even number, the number
obtained after one operation can be 4  2 = 8 or 4 − 1 = 3 ,
(i) if an even number of 8 is obtained after an operation, the original number can
be 8  2 = 16 or 8 − 1 = 7 ,
(ii) if an odd number of 3 is obtained after one operation, the original number is
3 2 = 6 .
Therefore, it is known that a total of 3 numbers that can get a value of “1” after four
operations. Therefore, the answer is (B).
Answer: (B)
Note: There is one solution if we go one operation, which is 2; There is also one
solution if we do two operations, which is 4; There are two solutions if we do three
operations, which are 3 and 8; and there are three solutions if we do four operations,
which are 6, 7, and 16. The answer to this question is essentially a Fibonacci
sequence.

10. Among the numbers from 0 to 200, what is the average of all numbers that are
multiples of 3?
(A)95 (B)96 (C)99 (D)100.5 (E)101
【Suggested Solution 1】
From the numbers from 0 to 200, the numbers that are multiple of 3 are as follows:
0, 3, 6, ..., 198. Now, we do pairing of the following numbers: (0, 198), (3, 195), ...,
(96, 102), and we have an extra 99 which has no pair, from this, the average of every
pairwise is 99, so the average is also 99. So, the answer is (C).

259
【Suggested Solution 2】
It can be seen that among all the numbers from 0 to 200, the multiple of 3 is 0, 3, 6, ...,
198
198. So, in total we have + 1 = 67 terms. Therefore, the average of these numbers
3
(0 + 198)  67
2 99  67
is = = 99 . Therefore, the answer is (C).
67 67
Answer: (C)
11. What is the tens digit of the simplified value of the following expression:
1 + 12 + 123 + 1234 + 12345 + 123456 + 1234567 + 12345678 + 123456789 ?
(A)0 (B)2 (C)4 (D)6 (E)8
【Suggested Solution】
It can be observed that it is only necessary to consider adding the last two digits of
each number in the equation to get the last 2 digits.
Since 1 + 12 + 23 + 34 + 45 + 56 + 67 + 78 + 89 = 405 , then the tens digit is 0.
Therefore, the answer is (A).
Answer: (A)
3 cm
12. In the figure below, the largest
rectangle has dimensions 9 cm long
by 8 cm wide, with a cross-section
formed by two shaded rectangles in x cm
the middle. If the total area of the
shaded parts is exactly equal to the 8 cm
area of the un-shaded parts, then
what is the value, in cm, of x?

9 cm
(A)1 (B)2 (C)3 (D)4 (E)5
【Suggested Solution 1】
It can be seen that the area of the largest rectangle is 9  8 = 72 cm2, and because the
total area of the shaded portions is exactly equal to the area of the un-shaded portions,
then their area should be 72  2 = 36 cm2, thus, 3  8 + x  (9 − 3) = 36 , which yields
x = 2 . Therefore, the answer is (B).
【Suggested Solution 2】
It can be seen that the dimensions of the un-shaded portion have length 9 − 3 = 6 cm
and with 8 − x cm. and since the area of the largest rectangle is 9  8 = 72 cm2 and
because the total area of the shaded portions is exactly equal to the area of the
un-shaded portions, then their area should be 72  2 = 36 cm2, you have the equation
6(8 − x) = 36 . Solving for x, which will yield x = 2 . Therefore, the answer is (B).
Answer: (B)

260
13. Use the digits 2, 0, 1, and 9 exactly once to form different 4-digit numbers.
How many of these 4-digits numbers will be greater than 1905?
(A)12 (B)13 (C)15 (D)17 (E)19
【Suggested Solution】
Case 1: If the thousands digit is 1, there is one number (1920) that is greater than
1905.
Case 2: If the thousands digit is 2 or 9, then, the four-digit number is always greater
than 1905. If we have already selected the thousand digits, then there are 3
ways in choosing for the hundreds digit, then there are 2 ways in choosing
for the tens digit, and there is only 1 way in choosing for the units digit.
Therefore, there will be a total of 2  3  2 1 = 12 numbers that will be
greater than 1905 for this case.
So, there are a total of 1 + 12 = 13 numbers that will be greater than 1905. Therefore,
the answer is (B).
Answer: (B)

14. After deleting one of the six numbers 2, 3, 12, 26, 29 and 41, the five remaining
numbers can be divided into two groups with the same sum. What is the number
that was deleted?
(A)3 (B)12 (C)26 (D)29 (E)41
【Suggested Solution】
Notice that 2 + 3 + 12 + 26 + 29 + 41 = 113 , so the sum of the five numbers left must
be a multiple of 2, so we can deduce that the number deleted must be an odd number.
So we have 3 different cases:
Case 1: If the number deleted is 41, then the sum of the five numbers left is
2 + 3 + 12 + 26 + 29 = 72 . So, the sum of the numbers in each group must be
72  2 = 36 . So, the sum of the remaining numbers in the group that
contains 29 is 7, and on the remaining numbers, the sum of the numbers
less than 7 is 2 + 3 = 5  7 , so it’s impossible.
Case 2: If the number deleted is 29, then the sum of the five numbers left is
2 + 3 + 12 + 26 + 41 = 84 . So, the sum of the numbers in each group must be
84  2 = 42 . So, the sum of the remaining numbers in the group that
contains 41 must be 1, but we don’t have 1, so it’s impossible.
Case 3: If the number deleted is 3, then the sum of the five numbers left is
2 + 12 + 26 + 29 + 41 = 110 , So, the sum of the numbers in each group must
be 110  2 = 55 . So, the sum of the remaining numbers in the group that
contains 41 must be 14, out of the five numbers, we have 2 + 12 = 14 .
Therefore, the other group must be 26 + 29 = 55 , that is, one group
contains the numbers 41, 12 and 2 and the other group contains the
numbers 26 and 29.
Thus, the number deleted is 3. Therefore, the answer is (A).
Answer: (A)

261
15. As shown in the diagram, three of the seven areas enclosed
by the three circles are each filled with a number, and it is
known that the numbers 13, 15, 17, and 18 can be filled
inside the four blank areas such that the sum of the four 14 12
numbers in each circle are all equal. What is the sum?
(A)58 (B)59 (C)60
16
(D)61 (E)62
【Suggested Solution】
Notice that there are 3 odd numbers and 1 even number in the 4 numbers: 13, 15, 17,
and 18, and that the already filled-in numbers 12, 14, and 16 are all even numbers,
then, the only even number 18 must be filled-in to the center to make the sum of the
four numbers in each circle equal. Be sure to fill in the middle area so that the sum of
the four numbers in each circle is odd, otherwise the sum of the four numbers in the
circle where 18 is located is odd and the sum of the four numbers in the other two
circles it is even. (See Figure 1).

17

14 12 14 12
18 18
13 15
16 16

Figure 1 Figure 2

It can be seen that the sum of the numbers in the three circles at this time from largest
to smallest are as follows: 14 + 16 + 18 = 48 , 12 + 16 + 18 = 46 and 12 + 14 + 18 = 44 ,
thus, we fill in the numbers 13, 15 and 17, to the corresponding blank areas in the
circle sequentially as well to satisfy the conditions (See Figure 2). That is,
13 + 14 + 16 + 18 = 12 + 15 + 16 + 18 = 12 + 14 + 17 + 18 = 61 .
Therefore, the answer is (D).
Answer:(D)
16. In the figure below, each unit square has a side length of 1 cm. How many
squares and rectangles are there?

(A)9 (B)12 (C)14 (D)15 (E)16

262
【Suggested Solution】
It can be seen that the side length of each unit square in the figure is an integer, so we
can use the area to count for the rectangles. It can be seen that the area of this figure
is 6 cm2 and the pattern is not rectangular, so we consider the following 5 cases:
Case 1: The rectangle with an area of 1 square unit must be of dimension 1 1 , so
we have 6.
Case 2: The rectangle with an area of 2 square units must be of dimension 1 2 , so
we have 6.
Case 3: The rectangle with an area of 3 square units must be of dimension 1 3 , so
we have 2.
Case 4: The rectangle with an area of 4 square units must be of dimension 2  2 , so
we have 1.
Case 5: The rectangle with an area of 4 square units must be of dimension 1 5 ,
but we have no such rectangles.
So, in total, we have a total of 6 + 6 + 2 + 1 = 15 rectangles. Therefore, the answer is (D).
Answer: (D)
17. A theater has nine rows of seats. The first row has a total of 19 seats and starting
from the second row, each row has two more seats than the previous row. How
many seats does this theater have in total?
(A)173 (B)187 (C)243 (D)261 (E)280
【Suggested Solution】
It can be seen that the number of seats in the nine rows of seats are as follows: 19, 21,
23, 25, 27, 29, 31, 33 and 35 in order (from front to back), so the theater has a total of
(19 + 35)  9
19 + 21 + 23 + 25 + 27 + 29 + 31 + 33 + 35 = = 27  9 = 243 number of seats.
2
Therefore, the answer is (C).
Answer: (C)
18. Divide the six positive integers 2, 3, 7, 13, 18, and X into three groups so that the
first group contains one number, the second group contain two numbers, and the
third group contain three numbers, where X must belong in the third group. If the
number in the first group is greater than the sum of the numbers in the second
group, and that the sum of the numbers in the second group is greater than the
sum of the numbers in the third group, then what is the maximum possible value
of X?
(A)4 (B)5 (C)6 (D)7 (E)8
【Suggested Solution】
It can be seen that the number in the first group is greater than the sum of the
numbers in the second group, and is also greater than the sum of the number of the
third group, so therefore, the number in the first group must be the maximum of the
six numbers. Then because the unknown number X is in the third group, therefore, the
number in the first group must is 18. Now, the sum of the two numbers in the second
group must be less than 18. At this time, we check the different possible scenarios:
13 + 7 = 20  18  13 + 3 = 16 , thus, in order to maximize the value of X in the third

263
group, the two numbers of the second group must be taken as 13 and 3. Now, the sum
of the three numbers of the third group is 2 + 7 + X = 9 + X  16 , therefore, the largest
possible value of X is 6. Therefore, the answer is (C).
Answer: (C)
19. In the figure below, rectangle ABCD has a perimeter of 32 cm, where both ABFE
and EGHD are squares. If the perimeter of GFCH is 12 cm, then what is the area,
in cm2, of rectangle ABCD?
A E D

G H

B F C
(A)28 (B)39 (C)48 (D)55 (E)60
【Suggested Solution 1】
Since the perimeter of the rectangle ABCD is 32 cm, so BC + CD = 16 cm and since
the perimeter of rectangle GFCH is 12 cm, so FC + CH = 6 cm. Since it is also
known that ABFE and EGHD are squares, then AE = BF and ED = DH , thus,
AD = AE + ED = BF + DH = (BC − FC) + (CD − CH ) = 16 − 6 = 10 cm,
Thus, CD = 16 − BC = 16 − 10 = 6 cm, therefore, the area of rectangle ABCD is
6 10 = 60 cm2. Therefore, the answer is (E).
【Suggested Solution 2】
Let the side length of the square ABFE is a cm, and let the side length of the square
EGHD is b cm. It is also known that FC = GH = ED = b ,
GF = CH = CD − HD = AB − HD = a − b and the length of each line segment is
shown in the diagram below:
A a E b D

b b
a
b
G H
a −b a −b
B a F b C
Now, the perimeter of the rectangle GFCH is (a − b) + b + (a − b) + b = 2a = 12 cm, so
from this, a = 6 and the perimeter of rectangle ABCD is (a + b) + a + (a + b) + a =
4a + 2b = 24 + 2b = 32 cm, so b = 4 , so from this, the length of rectangle ABCD is
6 + 4 = 10 and width is 6 cm, therefore, the area of rectangle ABCD is 6 10 = 60 cm2.
Therefore, the answer is (E).
Answer: (E)

264
20. As shown in the figure below, an ant starting from vertex A of the cuboid needs
to move along the edges to reach its destination, vertex B. If the ant can only pass
through three edges, how many possible paths can the ant crawl to reach the
destination?
A

(A)3 (B)6 (C)9 (D)12 (E)15


【Suggested Solution】
The ant have a total of 3 edges starting from A, and after selecting the edge of the first
one, the second edge has two choices and the third edge is also determined. So, there
is a total of 3  2 = 6 different ways that the ant can travel from point A to B.
Therefore, the answer is (B).
Answer: (B)
21. A 5-digit number is formed by using digits that are all different. It is known that
four of the digits used are 3, 6, 8 and 9, and that the difference between any two
adjacent digits is greater than 3. How many 5-digit numbers are there that satisfy
the condition?
【Suggested Solution】
From the condition, it can be seen that the three digits 6, 8, and 9 cannot be adjacent.
That is, these three numbers must be on the ten thousands, hundreds and units digit
place. Now, notice that the difference between the digits 6 and 3 is exactly 3,
therefore, the digit 3 must be always between digits 8 and 9 and 6 cannot be a in the
hundreds place, otherwise, the digit 6 must be adjacent to digit 3. Because of this,
there are only two choices for hundreds place, and when hundreds place is already
chosen, there are two choices for the tens place and the units place are also
determined. Therefore, we have 2  2 = 4 ways, i.e., for the numbers in the form
6 X 839、6 X 938、839 X 6、938 X 6,X can only be located between 6 and 8, so it can
only take values 0, 1, 2, That is, there are a total of 4  3 = 12 five-digit numbers that
satisfies the condition.
Answer:012
22. Joe has a total of 31 dollars that he will use to purchase four different items that
have unit prices of 2, 3, 5 and 7 dollars each respectively. If he needs to purchase
at least one piece of each item, and Joe has to use all his money without leftovers,
in how many different ways can he buy the goods?
【Suggested Solution】
Since at least one item must be purchased for each item, when Joe buys one item for
each item, he will have a total of 31 − 7 − 5 − 3 − 2 = 14 dollars left. At this time, each
product does not necessarily need to be purchased again. Then by enumerating all
possible scenarios, we have the following:

265
14 = 7  2
=7+5+2
= 7 + 3+ 2 2
= 5 2 + 2 2
= 5 + 3 3
= 5 + 3+ 23
= 3 4 + 2
= 3 2 + 2  4
= 27
So, there are a total of 9 different ways.
Answer:009
23. It is known that the sum of the ages of A and B this year is 60 years old. 2 years
ago, the sum of the ages of B and C is also 60 years old and 3 years from now,
the sum of the ages of A and C is still 60 years old. How old is A this year?
【Suggested Solution】
Because the sum of the ages of B and C two years ago is 60 years old, then, the age
of B and C now is 60 + 2 + 2 = 64 years old. And 3 years from now, the sum of ages
of A and C is also 60 years old, so the sum of ages of A and C now is 60 − 3 − 3 = 54
years old. So, we know that the sum of the current ages of the three persons is
(60 + 64 + 54)  2 = 89 years old. So, this year, the age of A is 89 − 64 = 25 years old.
Answer:025
24. A coin is placed in one of the unit squares in the grid as shown below. How many
rectangles (including squares) can be formed that contains the coin?


【Suggested Solution 1】
It can be seen that the length of each side of each unit square in the figure is an
integer unit, so their area can be used to classify the rectangles. It can be seen that the
area of this square table is 12 square units, so the following can be obtained:
The rectangle with an area of 1 square unit must be of dimension 1 1, so we have 1.
The rectangle with an area of 2 square units must be of dimension 1 2 , so we have 3.
The rectangle with an area of 3 square units must be of dimension 1 3 , so we have 3.
The rectangle with an area of 4 square units, it can be as follows:
For dimensions 1 4 , so we have 3.
For dimensions 2  2 , so we have 2.
The rectangle with an area of 5 square units must be of dimension 1 5 , so we have 2.
The rectangle with an area of 6 square units, it can be as follows:
For dimensions 1 6 , so we have 1.
For dimensions 2  3 , so we have 3.
The rectangle with an area of 8 square units must be of dimension 2  4 , so we have 3.

266
The rectangle with an area of 10 square units must be of dimension 2  5 , so we have 2.
The rectangle with an area of 12 square units must be of dimension 2  6 , so we have 2.
So, we have a total of 1 + 3 + 3 + 3 + 2 + 2 + 1 + 3 + 3 + 2 + 1 = 24 different rectangles
(including squares) that can be formed that includes the coin.

【Suggested Solution 2】
Notice that there are 3 horizontal lines and 7 vertical lines in the grid, and each
rectangle formed must be bordered by two horizontal lines and two vertical lines. The
rectangle containing the coin must always have the bottom-most horizontal line,
therefore, there are two ways to choose the other horizontal line. Moreover, on the
three vertical lines on the left side of the coin, we must choose one, and on the four
vertical lines on the right side of the coin, we must also chose one, so there are 3  4
= 12 ways in choosing the vertical lines. So, there is a total of (1 2)  (3  4) = 24
different rectangles (including squares) that can be formed that includes the coin.
Answer:024
25. There are 25 boys in a class, 16 of whom are members of the football team, 12 in
the basketball team and 10 in the volleyball team. It is known that no one
participates in all three sports at the same time, and each boy participates in at
least one team. If five boys participate in both the football and basketball teams,
and three boys participate in both the basketball and volleyball teams. How many
boys are there that participate in both the football and volleyball teams?
【Suggested Solution 1】
Suppose there are x boys who participate in both the football and volleyball teams.
The following Venn Diagram represents the given problem:
Football Team
16 boys

x 5
0
Volleyball Team Basketball Team
10 boys 3 12 boys

Since16 + 12 + 10 − 5 − 3 − x = 25 , then x = 5 .
【Suggested Solution 2】
Since we know that the number of boys playing for only the basketball team is
12 − 5 − 3 = 4 boy, therefore, 25 − 4 = 21 boys participate in the football team or the
volleyball team. It is known that the total number of boys in the football and
volleyball teams is 16 + 10 = 26 boys, but we can see that 26 − 21 = 5 boys has
been counted twice, thus, a total of 5 boys participate in both the football team and
the volleyball team.
Answer:005

267
注意:

允許學生個人、非營利性的圖書館或公立學校合理使用
本基金會網站所提供之各項試題及其解答。可直接下載
而不須申請。

重版、系統地複製或大量重製這些資料的任何部分,必
須獲得財團法人臺北市九章數學教育基金會的授權許
可。

申請此項授權請電郵 ccmp@seed.net.tw

Notice:

Individual students, nonprofit libraries, or schools are


permitted to make fair use of the papers and its
solutions. Republication, systematic copying, or
multiple reproduction of any part of this material is
permitted only under license from the Chiuchang
Mathematics Foundation.
Requests for such permission should be made by
e-mailing Mr. Wen-Hsien SUN ccmp@seed.net.tw

268
9th International Mathematics Assessments for Schools
(2019-2020)

Middle Primary Division Round 2


Time: 120 minutes

Printed Name: Code: Score:

Instructions:
 Do not open the contest booklet until you are told to do so.
 Be sure that your name and code are written on the space provided above.
 Round 2 of IMAS is composed of three parts; the total score is 100 marks.
 Questions 1 to 5 are given as a multiple-choice test. Each question has five
possible options marked as A, B, C, D and E. Only one of these options is correct.
After making your choice, fill in the appropriate letter in the space provided. Each
correct answer is worth 4 marks. There is no penalty for an incorrect answer.
 Questions 6 to 13 are a short answer test. Only Arabic numerals are accepted;
using other written text will not be honored or credited. Some questions have more
than one answer, as such all answers are required to be written down in the space
provided to obtain full marks. Each correct answer is worth 5 marks. There is no
penalty for incorrect answers.
 Questions 14 and 15 require a detailed solution or process in which 20 marks are
to be awarded to a completely written solution. Partial marks may be given to an
incomplete presentation. There is no penalty for an incorrect answer.
 Use of electronic computing devices is not allowed.
 Only pencil, blue or black ball-pens may be used to write your solution or answer.
 Diagrams are not drawn to scale. They are intended as aids only.
 After the contest the invigilator will collect the contest paper.
The following area is to be filled in by the judges;
the contestants are not supposed to mark anything here.
Total
Question 1 2 3 4 5 6 7 8 9 10 11 12 13 14 15 Score Signature

Score

Score

269
270
Middle Primary Division Round 2
Questions 1 to 5, 4 marks each
1. Divide the number 10 into the sum of three distinct positive integers. How many
different ways are there altogether? (Note: if the arrangement of same three
positive integers are in different order, they are regarded as the same).
(A)1 (B)2 (C)3 (D)4 (E)5

Answer:

2. In a volleyball tournament, each game adopts the best three sets out of five sets
game, in which each team has 0, 1, 2, or 3 possible sets of winning. The
distribution chart shows a number of sets that a team can win as follows.

What is the median of the number of sets won by the team in the tournament?
(Note: The median of a group of data is the middle datum when the data are
arranged in ascending order of magnitude.)
(A)1 (B)1.5 (C)2 (D)2.5 (E)3

Answer:

3. In the following addition expression, the same symbol represents the same
number, and different symbols represent different numbers. It is known the
leftmost digit in both the addend and the sum is nonzero. What is the value of
△  □  ○?
○ 8 ○
+ △ □ ○
△ □ ○ 8
(A)10 (B)11 (C)12 (D)13 (E)14

Answer:

271
MP 2
4. Allan and Benny are computing the sum of a three-digit number and a two-digit
number. The computation of Allan is correct with an answer of 136, while Benny
is not doing it correctly because of a misalignment of digits in the vertical format
mistakenly adding the tens digit of three-digit number with the units digit of the
two-digit number and obtaining the tens digit in the sum. This is followed by
adding the hundreds digit of the three-digit number and the tens digit of the
two-digit number to get the hundreds digit in the sum with the final answer
becoming 244. What is the difference between the three-digit number and the
two-digit number?
(A)106 (B)108 (C)110 (D)112 (E)114

Answer:

5. As shown in the figure, both quadrilateral ABCD and quadrilateral BEFG are
squares, and AE  28 cm, AH  16 cm. What is the perimeter, in cm, of the
rectangle CDHG?

A B E

D C
H G F

(A)20 (B)24 (C)32 (D)48 (E)64

Answer:

Questions 6 to 13, 5 marks each

6. There are 45 students in the class sitting in a row. The position of the student A is
28th from left, and the position of student B is 35th from right. How many
students are there between student A and student B?
Answer: students

272
MP 3
7. There are 14 rows of seats in a semi-circular auditorium. In a certain performance,
18 spectators are seated at the first row, and, starting from the second row, each
row has three more spectators than the previous one. How many spectators are
there in the auditorium during the performance?

Answer: spectators

8. What is the value of (1 2  3 11)  (22  24  25  27) ? (if the answer is a
fraction, express the answer with simplest form)
Answer:

9. In a certain examination, Lily’s average score in the three subjects - Foreign


Language, Mathematics and Nature Science - was 90 points while the English
examination still needs to be scheduled to a further date. How many points must
Lily need to have in English to make the average score of the four subjects to be
92 points?
Answer: points

10. As shown in the figure, place five congruent small rectangles inside a large
square such that each side of all the rectangles is parallel to one of the sides of
the square. If the side length of the given large square is 48 cm, what is the area,
in cm2, of a small rectangle?

Answer: cm2

11. An electronic watch shows 7 o’clock thirty-one minutes on the surface of the
watch as 7:31. From 7:00 to 8:00 in the morning, how many times will three
distinct digits appear as a point of time on the surface of watch?
Answer: times

273
MP 4
12. Below is a tangram which was invented in China. It is a famous puzzle with
seven ingenious pieces: five isosceles right triangles of three sizes, one square
and one parallelogram. If the area of one of the smallest isosceles right triangle in
the 7-piece puzzle is 6 cm2, what is the area, in cm2 , of the largest isosceles right
triangle in this puzzle?

Answer: cm2
13. There are 24 regular unit cube blocks glued together as shown in the figure below.
How many faces of regular unit cubes blocks are glued together?

Answer: faces

Questions 14 to 15, 20 marks each


(Detailed solutions are needed for these two problems)
14. After arranging 1000 consecutive positive integers in a certain order, followed by
computing the sum of each successive three numbers from the beginning, a total
of 998 sums can be obtained. What is the largest possible number of odd integers
among these 998 sums?

Answer: odd integers

274
MP 5
15. Fill in the grid with the positive integers starting from 1 as shown in the figure,
and then use an “L-shaped block” (that is composed of three unit squares which
can be rotated or flipped) to frame three numbers and compute their sum.
For example, in the figure below, the sum of the three numbers framed by the
L-shaped block is 10  11  18  39. It is known that the sum of the three
numbers framed by an L-shaped block is 2019. What is the largest possible value
of the largest number among the three numbers framed by an L-shaped block?
1 2 3 4 5 6 7
8 9 10 11 12 13 14
15 16 17 18 19 20 21
22 23 24 25 26 27 28
… … … … … … …

Answer:

275
276
注意:

允許學生個人、非營利性的圖書館或公立學校合理使用
本基金會網站所提供之各項試題及其解答。可直接下載
而不須申請。

重版、系統地複製或大量重製這些資料的任何部分,必
須獲得財團法人臺北市九章數學教育基金會的授權許
可。

申請此項授權請電郵 ccmp@seed.net.tw

Notice:

Individual students, nonprofit libraries, or schools are


permitted to make fair use of the papers and its
solutions. Republication, systematic copying, or
multiple reproduction of any part of this material is
permitted only under license from the Chiuchang
Mathematics Foundation.
Requests for such permission should be made by
e-mailing Mr. Wen-Hsien SUN ccmp@seed.net.tw

277
Solution Key to Second Round of IMAS 2019/2020
Middle Primary Division

Questions 1 to 5, 4 marks each


1. Divide the number 10 into the sum of three distinct positive integers. How many
different ways are there altogether? (Note: if the arrangement of same three
positive integers are in different order, they are regarded as the same).
(A)1 (B)2 (C)3 (D)4 (E)5
【Suggested Solution】
Since 10  1  2  7  1  3  6  1  4  5  2  3  5 , there are four different ways of
express number 10. Therefore, choose D.
Answer:(D)
2. In a volleyball tournament, each game adopts the best three sets out of five sets
game, in which each team has 0, 1, 2, or 3 possible sets of winning. The
distribution chart shows a number of sets that a team can win as follows.

What is the median of the number of sets won by the team in the tournament?
(Note: The median of a group of data is the middle datum when the data are
arranged in ascending order of magnitude.)
(A)1 (B)1.5 (C)2 (D)2.5 (E)3
【Suggested Solution】
The distribution chart shows that the team played a total of 1  2  4  8  15 games,
when ranking the winning of each game based on the number of sets from lowest to
highest, the 8th position becomes the median. Since there are 0, 1, and 2 games of
winning, we conclude the median is 3. Therefore, the answer is option E.
Answer:(E)
3. In the following addition expression, the same symbol represents the same
number, and different symbols represent different numbers. It is known the
leftmost digit in both the addend and the sum is nonzero. What is the value of
△  □  ○?

278
○ 8 ○
+ △ □ ○
△ □ ○ 8
(A)10 (B)11 (C)12 (D)13 (E)14
【Suggested Solution】
We know that the sum of the addends in the ones place is 8, then ○ is equal to 4 or 9.
Since the sum of the addends in the hundreds place is at most 9  8  17 , so that the
number in the thousands place is 1, then  = 1. It follows ○ is equal to 8 or 9.
In summary, we have ○ equals 9 and the addend is 989, so the addition sentence
becomes:
9 8 9
+ 1 □ 9
1 □ 9 8
Now, it can be determined the addends in the tens place □ is 0. This implies that the
sum in the hundreds place is 0. Hence, the addition sentence is as follows
9 8 9
+ 1 0 9
1 0 9 8
Therefore, △  □  ○  1  0  9  10 . So we select option A as the answer.
Answer:(A)
4. Allan and Benny are computing the sum of a three-digit number and a two-digit
number. The computation of Allan is correct with an answer of 136, while Benny
is not doing it correctly because of a misalignment of digits in the vertical format
mistakenly adding the tens digit of three-digit number with the units digit of the
two-digit number and obtaining the tens digit in the sum. This is followed by
adding the hundreds digit of the three-digit number and the tens digit of the
two-digit number to get the hundreds digit in the sum with the final answer
becoming 244. What is the difference between the three-digit number and the
two-digit number?
(A)106 (B)108 (C)110 (D)112 (E)114
【Suggested Solution 1】
From the result computed by Benny, the units digit in the three-digit number 244
must be 4, then from the correct answer 136, it can be seen that the units digit of the
two-digit number is 6  4  2 . So that from the value in the tens digit computed by
Benny is 4, we know the tens digit of the three-digit number is 4  2  2 . And from
the correct sum, the tens digit of 136 gives the units digit of the two digits has to be
3  2  1, this implies the two-digit number is 12, so the three-digit number is
136  12  124 , then the difference between the three-digit number and the two-digit
number in the original additional sentence is 124  12  112 . Therefore, the answer is
option D.

279
【Suggested Solution 2】
When Benny was doing the addition sentence in the vertical form, he mistakenly
aligned the tens digit of that two-digit number in the position of the hundreds place,
so the two-digit number becomes 10 times that of the original. The sum of his
computations increased by 9 times as the original two-digit number. Then, the
two-digit number must be (244  136)  9  12 , and so the three-digit number is
136  12  124 . Therefore, the difference between the three-digit number and the
two-digit number in the original additional sentence is 124  12  112 . Hence, we
select option D.
Answer:(D)
5. As shown in the figure, both quadrilateral ABCD and quadrilateral BEFG are
squares, and AE  28 cm, AH  16 cm. What is the perimeter, in cm, of the
rectangle CDHG?
A B E

D C
H G F

(A)20 (B)24 (C)32 (D)48 (E)64


【Suggested Solution】
From the given figure, it can be seen that the quadrilateral AEFH is a rectangle.
But HG  HF  GF  AE  EF  AE  AH  28  16  12 cm,
DH  AH  AD  AH  AB  AH  HG  16  12  4 cm,
Then the perimeter of rectangle CDHG is 2  (12  4)  32 cm.
Hence, the answer is option C.
Answer:(C)

Questions 6 to 13, 5 marks each


6. There are 45 students in the class sitting in a row. The position of the student A is
28th from left, and the position of student B is 35th from right. How many
students are there between student A and student B?
【Suggested Solution #1】
Since there are 45 students in the class, and B's position is 35th from right to left, this,
means that there are 10 students are behind him. So, when counting from left to right,
B is in 10  1  11 (th) position. Therefore, there are a total of 28  11  1  16
students are in between A and B.
B A

… …
1 2 3 4 5 6 7 8 9 10 11 12 乙
… 26 27 28 乙
… 43 44 45
乙 乙

280
【Suggested Solution #2】
Because the class has a total of 45 students, and B's position is the 35th from right to
left, that is; there are 10 students from right to left that are behind him. So, if
counting from left to right, B's position is located in the 10  1  11th. On the other
hand, A's position is the 28th from the left to right, so there are 17 students from left to
right that are behind him. Hence, there are a total of 45  17  10  1  1  16 students
between A and B.
45 students
B A

… …
乙 乙
10 students 17 students
Answer: 16 students
7. There are 14 rows of seats in a semi-circular auditorium. In a certain performance,
18 spectators are seated at the first row, and, starting from the second row, each
row has three more spectators than the previous one. How many spectators are
there in the auditorium during the performance?

【Suggested Solution 1】
It can be seen from the first row of the auditorium, the number of spectators in each
row is 18, 21, 24, 27, 30, 33, 36, 39, 42, 45, 48, 51, 54, 57, then the total number of
spectators in the auditorium is
18  21  24  27  30  33  36  39  42  45  48  51  54  57  525 spectators.
【Suggested Solution 2】
Since there are 18 spectators sited in the first row at the auditorium and starting from
the second row there are 3 more spectators sitting in each row than in the previous,
then it is an arithmetic sequence with a common difference of 3, so the number of
spectators in the 14th row is 18  3  13  57 spectators. It follows the total number of
(18  57)  14
spectators in the auditorium is  75  7  525 spectators.
2
Answer: 525 spectators
8. What is the value of (1 2  3 11)  (22  24  25  27) ? (if the answer is a
fraction, express the answer with simplest form)

281
【Suggested Solution】
1 2  3  4  5  6  7  8  9  1 0 1 1
( 1 2 3    1 1 )  ( 2 2 2 4 2 5 2 7 )
2  1 1 3 8 5 5 3 9
 4  2  7  2  112.
Answer: 112
9. In a certain examination, Lily’s average score in the three subjects - Foreign
Language, Mathematics and Nature Science - was 90 points while the English
examination still needs to be scheduled to a further date. How many points must
Lily need to have in English to make the average score of the four subjects to be
92 points?
【Suggested Solution 1】
According to the given information, the total scores of the three subjects (Foreign
Language, Mathematics and Natural Science) is 90  3  270 points. Since the
average of four subjects is 92 points, then the total scores of the four subjects is
92  4  368 points. Hence, Lily must obtain a score of 368  270  98 points in
English.
【Suggested Solution 2】
If the average of four subjects is 92 points, then Lily must achieve more than 92
points in English, so that she can have extra points to make up for the shortcomings
in the other three subjects (Foreign Language, Mathematics and Natural Science),
because the average of three subject is only 90 points. Therefore, these three subjects
need a total of (92  90)  3  6 points to make up, that is, Lily’s English score must
have 92  6  98 points.
Answer: 98 points
10. As shown in the figure, place five congruent small rectangles inside a large
square such that each side of all the rectangles is parallel to one of the sides of
the square. If the side length of the given large square is 48 cm, what is the area,
in cm2, of a small rectangle?

【Suggested Solution 1】
From vertical view, it can be seen that the side length of the large square is the sum of
twice the length and twice the width of the small rectangle.
From horizontal view, it can be seen that the side length of the large square is the sum
of three times the width and a length of the small rectangle and then subtract the
width of the small rectangle, which is equivalent as the sum of three times the length
of the small rectangle.

282
This implies the length of each small rectangle is 48  3  16 cm and the length of
one small rectangle is just the sum of the width of two small rectangles. So that the
width of one small rectangle is 16  2  8 cm.
Hence, the area of one small rectangle is 16  8  128 cm2.
【Suggested Solution 2】
Let the length and width of each small rectangle represent as a cm and b cm
respectively, then from the given figure we obtain
 b  a  a  b  48 2a  2b  48
 
a  b  a  (a  b)  48  3a  48
So that a  16 , b  8 . Thus, the area of each small rectangle is 16  8  128 cm2.
Answer: 128 cm2
11. An electronic watch shows 7 o’clock thirty-one minutes on the surface of the
watch as 7:31. From 7:00 to 8:00 in the morning, how many times will three
distinct digits appear as a point of time on the surface of watch?
【Suggested Solution】
It can be seen that 8 or 7 will be displayed in the hour position. If it is 8 in the hour
hand, there is only 8:00 appearing; not suitable! If it is 7 in the hour hand, then the
possible numbers that can appear in the tens’ digit of the minute hand can only be 0, 1,
2, 3, 4 or 5. There are 6 possible digits for the tens’ place, while the ones’ digit of
the minute hand can be from 0~9. However, the digit 7 and then the already existing
tens’ digit in the minute hand must be excluded; then there are 8 possibilities. So,
there are 1 6  8  48 three distinct number point of time in total.
Answer: 48 different three-digit number point of time
12. Below is a tangram which was invented in China. It is a famous puzzle with
seven ingenious pieces: five isosceles right triangles of three sizes, one square
and one parallelogram. If the area of one of the smallest isosceles right triangle in
the 7-piece puzzle is 6 cm2, what is the area, in cm2 , of the largest isosceles right
triangle in this puzzle?

283
【Suggested Solution】
According to the relative position of the seven pieces in the given figure, it can be
seen that the square can be formed by assembling the hypotenuse of two smallest
isosceles right triangles side by side, while the parallelogram can be formed by
arranging each hypotenuse of each of the two smallest isosceles right triangles to be
the two longest sides of the parallelogram.
The largest isosceles right triangle is formed by one medium sized isosceles right
triangle and two smallest isosceles right triangles, so it follows the area of one largest
isosceles right triangle is 4 times the area of the smallest isosceles right triangles.
Therefore, the area of the largest isosceles right triangle is 6  4  24 cm2.
Answer:24 cm2
13. There are 24 regular unit cube blocks glued together as shown in the figure below.
How many faces of regular unit cubes blocks are glued together?

【Suggested Solution】
We know that 24 regular unit cubes will have a total of 6  24  144 faces. From the
given figure, we also know that there are (9  6  2)  3  4  64 faces that are not
sticking out. Therefore, there are a total of 144  64  80 faces of a regular unit
cube blocks glued together.
Answer: 80 faces
Questions 14 to 15, 20 marks each
(Detailed solutions are needed for these two problems)
14. After arranging 1000 consecutive positive integers in a certain order, followed by
computing the sum of each successive three numbers from the beginning, a total
of 998 sums can be obtained. What is the largest possible number of odd integers
among these 998 sums?
【Suggested Solution】
The sum of one odd number and two even numbers or the sum of three odd numbers
will always be an odd number, while the sum of one even number and two odd
numbers or the sum of three even numbers will always be an even number. Since
there are a total of 998 sums can be obtained, now assume the 998 sums are all odd
number, then
(i) there are no even numbers or exactly 2 even numbers between any two adjacent
odd numbers.
(ii) there shall be no odd number or exactly 1 odd number between any two adjacent
even numbers but not three consecutive even numbers.
(iii) there can be at most one odd number before the first even number and also after
the last even number.

284
It can be seen that there are 500 odd numbers and 500 even numbers in the 1000
consecutive positive integers, that is, there are as many odd numbers as even
numbers. So, it is impossible to arrange all of them in odd numbers, that is to say, it
is only possible to satisfy the conditions (i) and (ii) at the same time
 OOO  OOOEEOEEO  EEOOOO  OOO
a terms of O 250 term of EEO (250 a ) term of O

 OOO  OOOOEEOEE  OEEOOO  OOO


a terms of OOO 250 term of OEE (250 a ) term of O

However, no matter what is the value of a, the condition (iii) can not be satisfied.
Hence, it is impossible that 998 sums are all odd numbers, that is, at most 997 sums
are odd numbers. Here is one arrangement:
OEEOEE  OEEOOO  OOO
250 times of OEE group 250 times of OOO group

There is only 1 set of solution which contains an even number, then odd number and
followed by another odd number which will produce a sum of an even number, and
the rest of them will produce a sum of odd number.
Answer: 997 numbers
【Marking Scheme】
 Find that the sum of one odd number and two even numbers or the sum of three
odd numbers will always be an odd number, 5 marks.
 Show that it is impossible when 998 sums are all odd number, 10 marks.
 Give an example so that 997 sums are odd number and prove the example, 5
marks.
15. Fill in the grid with the positive integers starting from 1 as shown in the figure,
and then use an “L-shaped block” (that is composed of three unit squares which
can be rotated or flipped) to frame three numbers and compute their sum.
For example, in the figure below, the sum of the three numbers framed by the
L-shaped block is 10  11  18  39. It is known that the sum of the three
numbers framed by an L-shaped block is 2019. What is the largest possible value
of the largest number among the three numbers framed by an L-shaped block?
1 2 3 4 5 6 7
8 9 10 11 12 13 14
15 16 17 18 19 20 21
22 23 24 25 26 27 28
… … … … … … …
【Suggested Solution】
It can be found that this is an arrangement of 7 positive integers in a row as one
period, so the number in each row is increased by 7 from the previous row. Then, the
three numbers in the box may be:

285
 a, a  1 , a  7
a a 1
a7
2011
So that a  (a  1)  (a  7)  2019 , then a  , which is not a positive
3
integer, so it doesn’t meet the condition of the problem.

 a, a  1 , a  8
a a 1
a 8
2010
Here a  (a  1)  (a  8)  2019 , that is; a   670 , it follows that the
3
three numbers in the L-shaped block are 670, 671, 678.

 a, a  7 , a  8
a
a  7 a 8
2004
We have a  (a  7)  (a  8)  2019 , so a   668 , then the three numbers
3
in the L-shaped block are 668, 675, 676.

 a、 a  6 、 a  7
a
a6 a7
2006
Here, a  (a  6)  (a  7)  2019 , then a  , which is not a positive
3
integer, so it doesn’t meet the condition of the problem.

Therefore, the largest possible value of the largest number among the three numbers
framed by the L-shaped block is 678.
Answer: 678
【Marking Scheme】
 Find the situations so that the three numbers framed by the L-shaped block and
discuss whether the situation is held or not, and then find the three numbers
correctly as the situation holds. Each situation for 5 marks.

286
注意:

允許學生個人、非營利性的圖書館或公立學校合理使用
本基金會網站所提供之各項試題及其解答。可直接下載
而不須申請。

重版、系統地複製或大量重製這些資料的任何部分,必
須獲得財團法人臺北市九章數學教育基金會的授權許
可。

申請此項授權請電郵 ccmp@seed.net.tw

Notice:

Individual students, nonprofit libraries, or schools are


permitted to make fair use of the papers and its
solutions. Republication, systematic copying, or
multiple reproduction of any part of this material is
permitted only under license from the Chiuchang
Mathematics Foundation.
Requests for such permission should be made by
e-mailing Mr. Wen-Hsien SUN ccmp@seed.net.tw

287
International Mathematics Assessments for Schools

2020 ~ 2021 MIDDLE PRIMARY DIVISION FIRST ROUND PAPER


Time allowed:75 minutes

When your teacher gives the signal, begin working on the problems.

INSTRUCTION AND INFORMATION


GENERAL
1. Do not open the booklet until told to do so by your teacher.
2. No calculators, slide rules, log tables, math stencils, mobile phones or other
calculating aids are permitted. Scribbling paper, graph paper, ruler and compasses
are permitted, but are not essential.
3. Diagrams are NOT drawn to scale. They are intended only as aids.
4. There are 20 multiple-choice questions, each with 5 choices. Choose the most
reasonable answer. The last 5 questions require whole number answers between
000 and 999 inclusive. The questions generally get harder as you work through the
paper. There is no penalty for an incorrect response.
5. This is a mathematics assessment, not a test; do not expect to answer all questions.
6. Read the instructions on the answer sheet carefully. Ensure your name, school
name and school year are filled in. It is your responsibility that the Answer Sheet
is correctly coded.

THE ANSWER SHEET


1. Use only pencils.
2. Record your answers on the reverse side of the Answer Sheet (not on the question
paper) by FULLY filling in the circles which correspond to your choices.
3. Your Answer Sheet will be read by a machine. The machine will see all markings
even if they are in the wrong places. So please be careful not to doodle or write
anything extra on the Answer Sheet. If you want to change an answer or remove
any marks, use a plastic eraser and be sure to remove all marks and smudges.

INTEGRITY OF THE COMPETITION


The IMAS reserves the right to re-examine students before deciding whether to
grant official status to their scores.

288
2019 ~ 2020 MIDDLE PRIMARY DIVISION FIRST ROUND PAPER

Questions 1-10, 3 marks each


1. Which of the following numbers has the largest value?
(A)20201 (B)20210 (C)22100 (D)22010 (E)22001

2. If Mike has $45 and Scott has $54, how much more money does Scott have
compared to Mike?
(A)1 (B)4 (C)5 (D)9 (E)11

3. What is the numerical value of 20 − 19 + 2  0  2  0 − 20 + 21 ?


(A)0 (B)1 (C)2 (D)5 (E)6

4. Which of the following numbers is a multiple of 3?


(A)134 (B)178 (C)251 (D)577 (E)672

5. Harry arrived at school at 07:26 in the morning, which is 19 minutes before the
start of the first class. What time does the first class start in the morning?
(A)07:35 (B)07:45 (C)07:50 (D)07:55 (E)08:05

6. The diagram below shows a pattern that is made up using tangram puzzle pieces.
How many more triangles than quadrilaterals are there in the diagram?

(A)2 (B)3 (C)4 (D)5 (E)7

7. Jack miscalculated 25  48 as 25  4  8 . Which of the following operations


must he select in order to arrive at the correct answer?
(A)Divide by 4 (B)Divide by 8 (C)Multiply by 2
3
(D)Add 100 (E)Multiply by
2

289
MP 2
8. In the figure below, the area of each small square is 1 cm2. What is the total area,
in cm2, of the shaded region?

(A)24 (B)28 (C)32 (D)36 (E)40

9. Fold a square piece of paper diagonally twice and then cut a small corner as
shown in the diagram below. After performing the task, completely unfold the
paper. How many sides does our newly obtained figure have?

(A)6 (B)8 (C)10 (D)12 (E)16

10. What is the units digit of 1 + 11 + 111 + ... + 111...111 ?


2020 1's

(A)0 (B)1 (C)2 (D)5 (E)6

Questions 11-20, 4 marks each

11. Among the figures below, which has the most number of lines of symmetry?
(A) (B) (C)

(D) (E)

290
MP 3
12. Albert drove from Town A to Town B at a constant speed of 80 km per hour. If
the street sign shown below was seen during the trip, how many hours does it
take Albert to drive from Town A to Town B?
Town A Town B
90 km 70 km

(A)2 (B)2.5 (C)3 (D)1.125 (E)0.875

13. Among the shapes listed below, which cannot always be formed by using two
identical triangles?
(A)Rhombus (B)Trapezoid (C)Square
(D)Rectangle (E)Parallelogram

14. In the figure below, it is known that the side length of both squares is 2 cm. An
ant, whose initial position is at point O starts crawling continuously for 2020 cm
along the following path: “O-A-B-C-O-D-E-F-O-A-B-C-O-D-E-F-O-A…”. After
completing the task, where will the ant arrive?
A D

O
B E

C F

(A)Point B (B)Point E (C)Point D (D)Point C (E)Point F

15. The diagram below shows a clock displaying the time 10:10, where the hour
hand points between the numbers 10 and 11, the minute hand points exactly to
the number 2 and the second hand points to exactly to number 12. After exactly
50 minutes, what is the sum of the numbers pointed by the hour, minute, and
second hands?

(A)24 (B)28 (C)29


(D)35 (E)None of the above

291
MP 4
16. Given that the sum of the three smallest numbers among 11 consecutive positive
integers is 63, what is the largest number among these 11 numbers?
(A)11 (B)21 (C)22 (D)30 (E)31

17. The figure on the lower left shows a cube, where the numbers from 1 to 6 are
written on each face such that the sum of the numbers in any two opposite faces
are all equal. If the figure on the lower right shows the diagram of the flattened
view of the same cube, what is the number that is represented by ☆?

5 ☆

6 3
6

(A)1 (B)2 (C)3 (D)4 (E)5

18. In the expression below, letters A, B, C, D and E represent five different digits
from 0 to 9. Which of the following letters has the largest value?
A 8 B 4 C
- 4 D 6 E 2
1 3 5 7 9
(A)A (B)B (C)C (D)D (E)E

19. There are 23 identical beads in a drawer, of which 8 are white and 15 are black. If
you blindly take out the beads one at a time from the drawer, at least how many
must be taken out to ensure that there are more black beads than white beads?
(A)1 (B)8 (C)15 (D)17 (E)23

20. In the figure below, ABCD is a square with an area of 2020 cm2. Connect the
midpoints of each side to get square EFGH and then connect the midpoints of
each side of square EFGH to get square MNPQ. What is the area, in cm2, of the
shaded region?
A E B
Q M

H F

P N
D G C
(A)400 (B)404 (C)450 (D)500 (E)505

292
MP 5

Questions 21-25, 6 marks each


21. In a park, there is a viewing trail that was paved with identical regular pentagonal
stone bricks such that one side of a brick is connected to one side of another
brick, as shown in the diagram. The gardener plans to make fences to each side
of these stone bricks that are not in contact with the other stone bricks with a
wooden board. If a total of 2021 wooden boards are needed to pave this trail,
how many stone bricks are used?

22. A school conducted a survey to all of its 216 students in the fifth grade. The
results were as follows: 100 does not like to read comic books, 85 does not like
to read novels and 33 does not like to read both comic books and novels. How
many students like to read both comic books and novels?

23. Some overlapping circles divide the plane into several regions. Numbers are
filled in these areas such that for any pair of overlapping circles, say A and B, the
number in the intersection area of circle A and B is the average of the two
numbers in the areas solely in A or B. For example, the figure below shows
1+ 5 5 + 11 11 + 13
3= , 8= and 12 = .
2 2 2
11
12 13
8
5
3
1

In the figure below, there are five overlapping circles and ten numbers are filled
according to the conditions above. What is the sum of the numbers a and b?

c
17 15
e a
20 30
b 28 d

293
MP 6
24. The figure below shows one H-shaped paper, where each corner is a right angle.
What is the perimeter, in cm, of the shaded region?

4 cm 20 cm

16 cm

25. Grace wants to buy a total of 10 puppets, which have four kinds namely: Octopus,
Mouse, Rhino and Dog puppet using the $700 that she has. The unit prices of
these four puppets are $30, $50, $70 and $110, respectively. If she must buy at
least one type of each puppet, how many different purchase combinations are
there in total?

***

294
295
注意:

允許學生個人、非營利性的圖書館或公立學校合理使用
本基金會網站所提供之各項試題及其解答。可直接下載
而不須申請。

重版、系統地複製或大量重製這些資料的任何部分,必
須獲得財團法人臺北市九章數學教育基金會的授權許
可。

申請此項授權請電郵 ccmp@seed.net.tw

Notice:

Individual students, nonprofit libraries, or schools are


permitted to make fair use of the papers and its
solutions. Republication, systematic copying, or
multiple reproduction of any part of this material is
permitted only under license from the Chiuchang
Mathematics Foundation.
Requests for such permission should be made by
e-mailing Mr. Wen-Hsien SUN ccmp@seed.net.tw

296
Solution Key to the First Round of IMAS 20202021
Middle Primary Division
1. Which of the following numbers has the largest value?
(A)20201 (B)20210 (C)22100 (D)22010 (E)22001
【Suggested Solution】
Since 20201  20210  22001  22010  22100 , therefore, the answer is(C).
(C)
Answer:
2. If Mike has $45 and Scott has $54, how much more money does Scott have
compared to Mike?
(A)1 (B)4 (C)5 (D)9 (E)11
【Suggested Solution】
Scott has $54 − $45 = $9 more than Mike. Therefore, the answer is(D).
Answer: (D)
3. What is the numerical value of 20 − 19 + 2  0  2  0 − 20 + 21 ?
(A)0 (B)1 (C)2 (D)5 (E)6
【Suggested Solution】
Simplifying, 20 − 19 + 2  0  2  0 − 20 + 21 = 20 − 19 + 0 − 20 + 21 = 21 − 19 = 2 .
Therefore, the answer is(C).
Answer: (C)
4. Which of the following numbers is a multiple of 3?
(A)134 (B)178 (C)251 (D)577 (E)672
【Suggested Solution 1】
Observe that 134 = 3  44 + 2 , 178 = 3  59 + 1, 251 = 3  83 + 2 , 577 = 3 192 + 1
and 672 = 3  224 . Therefore, we can conclude that only 672 is a multiple of 3.
Therefore, the answer is(E).
【Suggested Solution 2】
We know from the divisibility of 3 rule that if the sum of the digits of a number is a
multiple of 3, then it is a multiple of 3. Now, we compute the digit sum for each
number: For 134, it is 1 + 3 + 4 = 8 . For 178, it is 1 + 7 + 8 = 16 . For 251, it is
2 + 5 + 1 = 8 . For 577, it is 5 + 7 + 7 = 19 . For 672, it is 6 + 7 + 2 = 15 . Since 8, 16
and 19 are not multiples of 3, but 15 is, therefore, we can conclude that only 672 is a
multiple of 3. Therefore, the answer is(E).
Answer: (E)
5. Harry arrived at school at 07:26 in the morning, which is 19 minutes before the
start of the first class. What time does the first class start in the morning?
(A)07:35 (B)07:45 (C)07:50 (D)07:55 (E)08:05
【Suggested Solution】
Since 26 + 19 = 45  60 , therefore the first class start at 07:45. Therefore, the answer
is(B).
Answer: (B)

297
6. The diagram on the right shows a pattern that is made up
using tangram puzzle pieces. How many more triangles
than quadrilaterals are there in the diagram?
(A)2 (B)3 (C)4
(D)5 (E)7
【Suggested Solution】
There are 5 triangles and 2 quadrilaterals used to form the pattern, so there are
5 − 2 = 3 more triangles than quadrilaterals. Therefore, the answer is(B).
Answer: (B)
7. Jack miscalculated 25  48 as 25  4  8 . Which of the following operations
must he select in order to arrive at the correct answer?
(A)Divide by 4 (B)Divide by 8 (C)Multiply by 2
3
(D)Add 100 (E)Multiply by
2
【Suggested Solution 1】
Since 25  48 = 1200 and 25  4  8 = 800 , by inspection, 800  4 = 200 ,
3
800  8 = 100 , 800  2 = 1600 , 800 + 100 = 900 and 800  = 1200 . Therefore, the
2
answer is(E).
【Suggested Solution 2】
By comparing the terms, we have 25  4  8  4 = 25  8  25  48 ,
25  4  8  8 = 25  4  25  48 , 25  4  8  2 = 25  64  25  48 ,
3
25  4  8 + 100 = 25  4  8 + 25  4 = 25  36  25  48 and 25  4  8  = 25  48 .
2
Therefore, the answer is(E).
Answer: (E)
8. In the figure below, the area of each small square is 1 cm2. What is the total area,
in cm2, of the shaded region?

(A)24 (B)28 (C)32 (D)36 (E)40


【Suggested Solution 1】
Since the area of each small square is 1 cm2 and by doing direct counting, it shows
that there are 32 small squares that are shaded in the figure, so the total area of the
shaded region is 32 cm2. Therefore, the answer is(C).

298
【Suggested Solution2】
Note that the area of each small square is 1 cm2. Observe that our original figure can
be formed by using four identical figures which is shown below.

Because there are 8 small squares that are shaded in the figure, so our original figure
will have a total of 4  8 = 32 shaded small squares, so the total area of the shaded
region is 32 cm2. Therefore, the answer is(C).
Answer: (C)
9. Fold a square piece of paper diagonally twice and then cut a small corner as
shown in the diagram below. After performing the task, completely unfold the
paper. How many sides does our newly obtained figure have?

(A)6 (B)8 (C)10 (D)12 (E)16


【Suggested Solution】
The task will basically remove portions of opposite corners of our original square,
which are its lower left and upper right corner and the newly generated sides will be
perpendicular or parallel to the sides of the original square.
The figure below shows our newly obtained figure, which has 8 sides.
Therefore, the answer is(B).

(B)
Answer:
10. What is the units digit of 1 + 11 + 111 + ... + 111...111 ?
2020 1's

(A)0 (B)1 (C)2 (D)5 (E)6


【Suggested Solution】
Notice that the units digit of each and every term in the expression is 1. So, we
calculate for the sum in the units digit place: 1 + 1 + 1 + ... + 1 + 1 + 1 = 2020 , which
2020 terms

yields that its units digit is 0. Therefore, the answer is(A).


(A)
Answer:

299
11. Among the figures below, which has the most number of lines of symmetry?
(A) (B) (C)

(D) (E)

【Suggested Solution】
Observe that the figure in option (A), which is a circle has an infinite number of lines
of symmetry, while figures in options (B), (C), (D), and (E) which is an equilateral
triangle, a square, a trapezoid and a regular hexagon, respectively have 3, 4, 1, and 6,
lines of symmetry, respectively. Thus, the circle has the most number of lines of
symmetry among all. Therefore, the answer is (A).
Answer: (A)
12. Albert drove from Town A to Town B at a constant speed of 80 km per hour. If
the street sign shown below was seen during the trip, how many hours does it
take Albert to drive from Town A to Town B?
Town A Town B
90 km 70 km

(A)2 (B)2.5 (C)3 (D)1.125 (E)0.875


【Suggested Solution】
We can easily deduce from the street sign that the distance between Town A and
Town B is 90 + 70 = 160 km. Thus, Albert needs a total of 160  80 = 2 hours to
travel from Town A to Town B. Therefore, the answer is (A).
Answer: (A)
13. Among the shapes listed below, which cannot always be formed by using two
identical triangles?
(A)Rhombus (B)Trapezoid (C)Square
(D)Rectangle (E)Parallelogram
【Suggested Solution】
From the figures that are drawn below, we can see that the rhombus, square, rectangle
and parallelogram can be formed by using two identical triangles. Only trapezoids
cannot be formed by using two identical triangles. Therefore, the answer is(B).

Answer:(B)

300
14. In the figure below, it is known that the side length of both squares is 2 cm. An
ant, whose initial position is at point O starts crawling continuously for 2020 cm
along the following path: “O-A-B-C-O-D-E-F-O-A-B-C-O-D-E-F-O-A…”. After
completing the task, where will the ant arrive?
A D

O
B E

C F
(A)Point B (B)Point E (C)Point D (D)Point C (E)Point F
【Suggested Solution】
Observe that the crawling path of the ant goes through the “O-A-B-C-O-D-E-F-O”
repeating cycle and the total length of each cycle is 2  8 = 16 cm. Since
2020 = 16 126 + 4 and 4 = 2  2 , the ant will eventually end up at the second point
after the start of the loop, which is point B. Therefore, our answer is(A).
(A)
Answer:
15. The diagram below shows a clock displaying the time 10:10, where the hour
hand points between the numbers 10 and 11, the minute hand points exactly to
the number 2 and the second hand points to exactly to number 12. After exactly
50 minutes, what is the sum of the numbers pointed by the hour, minute, and
second hands?

(A)24 (B)28 (C)29


(D)35 (E)None of the above
【Suggested Solution】
After exactly 50 minutes, the time will be 11:00. During this time, the hour hand
points exactly to the number 11, the minute hand points exactly to the number 12 and
the second hand points exactly to the number 12, so its sum is 11 + 12 + 12 = 35 .
Therefore, the answer is(D).
Answer: (D)
16. Given that the sum of the three smallest numbers among 11 consecutive positive
integers is 63, what is the largest number among these 11 numbers?
(A)11 (B)21 (C)22 (D)30 (E)31

301
【Suggested Solution 1】
Since the three smallest numbers each must differ by 1 and their sum is 63, so the
second smallest number must be 63  3 = 21 , so the smallest number must be
21 − 1 = 20 . Therefore, the largest number among these 11 consecutive positive
integers is 20 + 11 − 1 = 30 . Therefore, the answer is(D).
【Suggested Solution 2】
We can easily see that the largest number among these 11 consecutive positive
integers is 10 greater than the smallest number, 9 greater than the second smallest
number and 8 greater than the third smallest number. Since it is known that the sum
of the three smallest numbers is 63, so three times the largest number of these 11
consecutive positive integers is equal to 63 + 10 + 9 + 8 = 90 , therefore, the largest
number is 90  3 = 30 . Therefore, the answer is(D).
Answer: (D)
17. The figure on the lower left shows a cube, where the numbers from 1 to 6 are
written on each face such that the sum of the numbers in any two opposite faces
are all equal. If the figure on the lower right shows the diagram of the flattened
view of the same cube, what is the number that is represented by ☆?

5 ☆

6 3
6

(A)1 (B)2 (C)3 (D)4 (E)5


【Suggested Solution】
We use this convention for simplicity:
Let 6 be the front denoted as F, the opposite side of 6 be the back denoted as Ba, 3 is
located on the right denoted as R, the opposite side of 3 be the left denoted as L, 5 be
the top surface denoted as T and the opposite side of 5 be the bottom denoted as Bo.
According to the position of the opposite face of the cube, it can be seen that the net
have the following four scenarios:
L Ba R Ba T Ba Bo Ba
Bo R T T L Bo L Bo R R T L
F F F F
Notice that for all cases, the side marked “☆” will always be located at the back
position. Since the sum of the numbers of any two opposing faces is equal, it is
1+ 2 + 3 + 4 + 5 + 6
known that the value of this sum is = 7 . Therefore, the number
3
represented by “☆” is 7 − 6 = 1. Therefore, our answer is(A).
Answer: (A)

302
18. In the expression below, letters A, B, C, D and E represent five different digits
from 0 to 9. Which of the following letters has the largest value?
A 8 B 4 C
- 4 D 6 E 2
1 3 5 7 9
(A)A (B)B (C)C (D)D (E)E
【Suggested Solution】
We infer starting from the right to the left.
It can be seen that C is the units digit of 9 + 2 = 11, so C = 1.
Now, notice that the tens digit of the subtrahend is 4 and the tens digit of the result is
7. Since we know if we subtract this, there must be a borrow from the hundreds digit.
Therefore, E + 7 = 14 − 1 = 13 , so E = 6 .
Then we know that B − 1 is the units digit of 5 + 6 = 11 , thus B − 1 = 1, so B = 2 .
At this time, the thousands digit of the subtrahend is 8 and the thousands digit of the
result is 3, and since we know that there is no borrowing of 1 from the ten thousand
digits when subtracting thousands of digits, then 8 − 1 − D = 3 , so D = 4 .
Finally, it can be seen that A = 1 + 4 = 5 .
The completed expression is shown below:
5 8 2 4 1
- 4 4 6 6 2
1 3 5 7 9
So, the largest digit among all is represented by the letter E.
Therefore, our answer is(E).
Answer: (E)
19. There are 23 identical beads in a drawer, of which 8 are white and 15 are black. If
you blindly take out the beads one at a time from the drawer, at least how many
must be taken out to ensure that there are more black beads than white beads?
(A)1 (B)8 (C)15 (D)17 (E)23
【Suggested Solution 1】
If we arrange the removed beads in the pattern: "white, black, white, black, ...”, it can
be seen that at least 17 beads must be taken out to ensure that there are more black
beads than white beads. Note that if we removing less than 17 beads does not
guarantee that there are more black beads than white beads. So, the answer is(D).
【Suggested Solution 2】
When a total of 16 pieces are already taken out, the worst-case scenario is that 8
pieces of white beads and 8 pieces of black beads are taken out, wherein the number
of black beads taken out is not more than that of white beads. But when you take one
more bead, since we are sure that there are no more white beads inside the box,
assuring us of getting a black bead, which guarantees us that there are more black
beads than white beads, so at least 17 beads must be taken out. Therefore, the answer
is(D).
Answer: (D)

303
20. In the figure on the right, ABCD is a square with an area A E B
2
of 2020 cm . Connect the midpoints of each side to get
square EFGH and then connect the midpoints of each Q M
side of square EFGH to get square MNPQ. What is the
area, in cm2, of the shaded region? H F
(A)400 (B)404 (C)450
P N
(D)500 (E)505
【Suggested Solution 1】 D G C
Connect AC and BD, it can be seen that square ABCD is divided into 16 small
triangles of equal area, as shown in the figure below. Since the shaded part accounts
4
for four small triangles, then its area is 2020  = 505 cm2. Therefore, the answer
16
is(E). A E B
Q M

H F

P N
D G C
【Suggested Solution 2】
Because points E, F, G, and H are midpoints of the sides of square ABCD, therefore,
1
the each of the areas of triangles AEH, BFE, CGF and DHG are all of the area of
8
1 1
square ABCD. From this, the area of the square EFGH is 1 −  4 = the area of
8 2
1
square ABCD. Similarly, the area of square MNPQ is the area of square EFGH
2
1 1 1
and  = the area of square ABCD. So, the total sharded area is equal
2 2 4
1 1 1 1
to1 −  4 − = of the area of square ABCD, which is 2020  = 505 cm2.
8 4 4 4
Therefore, the answer is(E).
Answer: (E)
21. In a park, there is a viewing trail that was paved with
identical regular pentagonal stone bricks such that one …
side of a brick is connected to one side of another
brick, as shown in the diagram. The gardener plans to …
make fences to each side of these stone bricks that are
not in contact with the other stone bricks with a
wooden board. If a total of 2021 wooden boards are
needed to pave this trail, how many stone bricks are
used?

304
【Suggested Solution 1】
According to the conditions of the problem, we can easily see that for both ends,
which are the first and last stone bricks, we need to use 4 wooden boards each, while
for the rest of the bricks, we need to use 3 wooden boards each. So, there are a total
2021 − 4 − 4
of + 2 = 671 + 2 = 673 stone bricks used.
3
【Suggested Solution 2】
Observe that the first and last stone bricks will require one more wooden board,
compared to the other bricks, which requires 3 wooden boards per brick. So, there are
2021 − 2
a total of = 673 stone bricks used.
3
Answer:673
22. A school conducted a survey to all of its 216 students in the fifth grade. The
results were as follows: 100 does not like to read comic books, 85 does not like
to read novels and 33 does not like to read both comic books and novels. How
many students like to read both comic books and novels?
【Suggested Solution】
From the conditions, the number of students who does not like to read both comic
books and novels is 100 + 85 − 33 = 152 . Therefore, the number of students who like
to read both is 216 − 152 = 64 .
Survey to all Grade 5 Students

100
Doesn’t like to
85
read comic books 33 Doesn’t like to
read novels

Answer:064
23. Some overlapping circles divide the plane into several
regions. Numbers are filled in these areas such that for 11
any pair of overlapping circles, say A and B, the number 12 13
8
in the intersection area of circle A and B is the average 5
of the two numbers in the areas solely in A or B. For 3
1+ 5
example, the figure on the right shows 3 = , 1
2
5 + 11 11 + 13
8= and 12 = .
2 2
In the figure on the right, there are five overlapping c
circles and ten numbers are filled according to the 17 15
conditions above. What is the sum of the numbers a e a
and b? 20 30

b 28 d

305
【Suggested Solution】
Adding the five equations, we get
60 + 56 + 40 + 34 + 30
a+b+c+d +e= = 110 .
2
So,
a = ( a + b + c + d + e) − (e + c ) − (d + b) = 110 − 34 − 56 = 20
b = ( a + b + c + d + e) − (e + c ) − (a + d ) = 110 − 34 − 60 = 16
Therefore, the sum of a and b is 20 + 16 = 36 .
Answer:036
24. The figure on the right shows one H-shaped paper,
where each corner is a right angle. What is the
perimeter, in cm, of the shaded region?
4 cm 20 cm

16 cm
【Suggested Solution】
A E G B
Label the points as shown in the figure on the right. By
adding auxiliary lines EG and FH, it can be seen that the
perimeter of the shaded area is just equal to the sum of
the perimeter of rectangle ABCD, the two line segments Q M
EF and GH having side length of 20 cm and the two line
segments MN and PQ having side length of 4 cm. P N
Therefore, the perimeter of the shaded area is:
(16 + 20)  2 + 20 + 20 − 4 − 4 = 104 cm.
Answer:104 C F H D
25. Grace wants to buy a total of 10 puppets, which have four kinds namely: Octopus,
Mouse, Rhino and Dog puppet using the $700 that she has. The unit prices of
these four puppets are $30, $50, $70 and $110, respectively. If she must buy at
least one type of each puppet, how many different purchase combinations are
there in total?

306
【Suggested Solution 1】
Since Grace has to buy at least one of kind of each puppet and she must buy a total of
exactly 10 puppets, then according to the principle of parity, the number of purchases
of these four kinds of puppets has the following three cases namely:
⚫ Four odd numbers: (1, 1, 1, 7), (1, 1, 3, 5) and (1, 3, 3, 3)
⚫ Two odd and two even: (1, 1, 2, 6), (1, 1, 4, 4), (1, 2, 2, 5), (1, 2, 3, 4) and (2, 2, 3, 3)
⚫ Four even numbers:(2, 2, 2, 4)
For convenience of our calculations below, note that 700 − (30 + 50 + 70 + 110) = 440
and 700 − 2  (30 + 50 + 70 + 110) = 180 .
(i) If the number of units purchased of these four puppets is of the combination
(1, 1, 1, 7) then 6a = 440 , so a = 30 , 50, 70 or 110. But since 440 is not a
multiple of 6, then there are no solutions.
(ii) If the number of units purchased of these four puppets is of the combination
(1, 1, 3, 5) then 2a + 4b = 440 and simplifying, we get a + 2b = 220 . So, the
values of a and b are two different numbers among 30, 50, 70, and 110. Since
220 − a a a
b= = 110 − , therefore the units digit of must be 5, this implies
2 2 2
that the units digit of b must be 5 as well, thus, there are no solutions.
(iii) If the number of units purchased of these four puppets is of the combination
(1, 3, 3, 3) then 2a + 2b + 2c = 440 and simplifying, we get a + b + c = 220 .
So, the values of a, b and c are three different values among 30, 50, 70, and
110. At this point, it is known from 30 + 50 + 70 + 110 = 260 and that the
values of a, b and c is 260 − 220 = 40 , therefore, there are no solutions.
(iv) If the number of units purchased of these four puppets is of the combination
(1, 1, 2, 6) then a + 5b = 440 . So, the values of a and b are two different
440 − a a
numbers among 30, 50, 70, and 110. Since b = = 88 − , therefore it’s
5 5
a
units digit must be 0, but since the units digit of is 8, then it will take
5
values a = 40 , 90, 140, …, thus, there are no solutions.
(v) If the number of units purchased of these four puppets is of the combination
(1, 1, 4, 4) then 3a + 3b = 440 . So, the values of a and b are two different
numbers among 30, 50, 70, and 110. But since 440 is not divisible by 3, there
are no solutions.
(vi) If the number of units purchased of these four puppets is of the combination
(1, 2, 2, 5) then a + b + 4c = 440 , so, the values of a, b and c are three different
values among 30, 50, 70, and 110.
440 − (a + b) ( a + b)
Notice that c = = 110 − so we know that c  110 .
4 4
If c = 30 ,then a + b = 320 , there are no solutions.
If c = 50 , then a + b = 240 , there are no solutions.
If c = 70 , then a + b = 160 , one number takes 50 and another takes 110, thus, we
have our first purchase combination:

307
Octopus Mouse Rhino Dog
($30) ($50) ($70) ($110)
Solution 1 1 2 5 2
(vii) If the number of units purchased of these four puppets is of the combination
(1, 2, 3, 4) then the values of a, b and c are three different values among 30, 50,
70, and 110.
If c = 30 , then a + 2b = 350 , there are no solutions.
If c = 50 , then a + 2b = 290 , so b = 110 and a = 70 thus, we have our second
purchase combination:
Octopus Mouse Rhino Dog
($30) ($50) ($70) ($110)
Solution 2 1 4 2 3
If c = 70 , then a + 2b = 230 , there are no solutions.
If c = 110 , then a + 2b = 110 , so b = 30 and a = 50 , a = 70 thus, we have our
third purchase combination:
Octopus Mouse Rhino Dog
($30) ($50) ($70) ($110)
Solution 3 3 2 1 4
(viii) If the number of units purchased of these four puppets is of the combination
(2, 2, 3, 3) then a + b = 180 . So, the values of a and b are two different
numbers among 30, 50, 70, and 110. Let one number be 70 and another be 100,
thus, we have our fourth purchase combination:
Octopus Mouse Rhino Dog
($30) ($50) ($70) ($110)
Solution 4 2 2 3 3
(ix) If the number of units purchased of these four puppets is of the combination
(2, 2, 2, 4), then 3a = 180 and a = 60 . Since a can only take values among 30,
50, 70, and 110, thus, there are no solutions.
In summary, there are 4 purchase combinations:
Octopus Mouse Rhino Dog
($30) ($50) ($70) ($110)
Solution 1 1 2 5 2
Solution 2 1 4 2 3
Solution 3 3 2 1 4
Solution 4 2 2 3 3
【Suggested Solution 2】
Let a be the number of Octopus puppets bought, b be the number of Mouse puppets
bought, c be the number of Rhino puppets bought and d be the number of Dog
puppets bought, where 1  a  9 , 1  b  9 , 1  c  9 and 1  d  9 .
Using the conditions, we have the following equations:
 a + b + c + d = 10  a + b + c + d = 10 (1)
30a + 50b + 70c + 110d = 700  3a + 5b + 7c + 11d = 70
  (2)
And from (2) − (1)  3 , we get 2b + 4c + 8d = 40 , which yields b + 2c + 4d = 20 .

308
⚫ If d = 5 , then b = c = 0 , there are no solutions.
⚫ If d = 4 , then b + 2c = 4,Since b must be even, then b = 2,which implies c = 1
and a = 10 − 4 − 2 − 1 = 3 . So, we have our first solution which is (a, b, c, d) =
(3, 2, 1, 4).
⚫ If d = 3 , then b + 2c = 8 and since b must be even:
When b = 2 , then c = 3 and a = 10 − 3 − 2 − 3 = 2 . So, we have our second
solution which is (a, b, c, d) = (2, 2, 3, 3).
When b = 4 , then c = 2 and a = 10 − 3 − 4 − 2 = 1 . So, we have our third
solution which is (a, b, c, d) = (1, 4, 2, 3).
When b = 6 , then c = 1 and a = 10 − 3 − 6 − 1 = 0 , which is a contradiction.
⚫ If d = 2 , then b + 2c = 12 and since b must be even:
When b = 2 , then c = 5 and a = 10 − 2 − 2 − 5 = 1. So, we have our fourth
solution which is (a, b, c, d) = (1, 2, 5, 2).
When b = 4 , then c = 4 and a = 10 − 2 − 4 − 4 = 0 , which is a contradiction.
When b = 6 , then c = 3 and b + c + d = 2 + 6 + 3 = 11  10 , which is a
contradiction.
When b = 8 , then c = 2 and b + c = 10 , which is a contradiction.

⚫ If d = 1, then b + 2c = 16 and since b must be even:


When b = 2 , then c = 7 and a = 10 − 1 − 2 − 7 = 0 , which is a contradiction.
When b = 4 , then c = 6 and b + c = 10 , which is a contradiction.
When b = 6 , then c = 5 and b + c = 11  10 , which is a contradiction.
When b = 8 , then c = 4 and b + c = 12  10 , which is a contradiction.
In summary, there are 4 purchase combinations:
Octopus Mouse Rhino Dog
($30) ($50) ($70) ($110)
Solution 1 3 2 1 4
Solution 2 2 2 3 3
Solution 3 1 4 2 3
Solution 4 1 2 5 2
Answer:004

309
注意:

允許學生個人、非營利性的圖書館或公立學校合理使用
本基金會網站所提供之各項試題及其解答。可直接下載
而不須申請。

重版、系統地複製或大量重製這些資料的任何部分,必
須獲得財團法人臺北市九章數學教育基金會的授權許
可。

申請此項授權請電郵 ccmp@seed.net.tw

Notice:

Individual students, nonprofit libraries, or schools are


permitted to make fair use of the papers and its
solutions. Republication, systematic copying, or
multiple reproduction of any part of this material is
permitted only under license from the Chiuchang
Mathematics Foundation.
Requests for such permission should be made by
e-mailing Mr. Wen-Hsien SUN ccmp@seed.net.tw

310
10th International Mathematics Assessments for Schools
(2020-2021)

Middle Primary Division Round 2


Time: 120 minutes

Printed Name: Code: Score:

Instructions:
 Do not open the contest booklet until you are told to do so.
 Be sure that your name and code are written on the space provided above.
 Round 2 of IMAS is composed of three parts; the total score is 100 marks.
 Questions 1 to 5 are given as a multiple-choice test. Each question has five
possible options marked as A, B, C, D and E. Only one of these options is correct.
After making your choice, fill in the appropriate letter in the space provided. Each
correct answer is worth 4 marks. There is no penalty for an incorrect answer.
 Questions 6 to 13 are a short answer test. Only Arabic numerals are accepted;
using other written text will not be honored or credited. Some questions have more
than one answer, as such all answers are required to be written down in the space
provided to obtain full marks. Each correct answer is worth 5 marks. There is no
penalty for incorrect answers.
 Questions 14 and 15 require a detailed solution or process in which 20 marks are
to be awarded to a completely written solution. Partial marks may be given to an
incomplete presentation. There is no penalty for an incorrect answer.
 Use of electronic computing devices is not allowed.
 Only pencil, blue or black ball-pens may be used to write your solution or answer.
 Diagrams are not drawn to scale. They are intended as aids only.
 After the contest the invigilator will collect the contest paper.
The following area is to be filled in by the judges;
the contestants are not supposed to mark anything here.
Total
Question 1 2 3 4 5 6 7 8 9 10 11 12 13 14 15 Score Signature

Score

Score

311
Middle Primary Division Round 2

Questions 1 to 5, 4 marks each

1. Which of the following expressions has a value equal to 3?


(A) 2  0  2  1 (B) 2  0  2  1 (C) 2  0  2  1
(D) 2  0  2  1 (E) 2  0  2  1

Answer:

2. Which among the choices has the same pattern as the figure shown below?
(Note: figure can be rotated but not flipped)

(A) (B) (C) (D) (E)

Answer:

3. Nelson has some number of apples and one pear that he must place in a straight
line on the table. If we start counting from left to right, the pear is located on the
12th position, while counting from right to left, the pear is located on the 18 th
position. How many fruits are there on the table?
(A)30 (B)29 (C)28 (D)22 (E)20

Answer:

312
MP 2
4. Jack paved the sidewalk in the garden with tiles as shown in the figure below. If
the width of the sidewalk is 1 m, what is the total area, in m2, of the sidewalk?
23 m

17 m

13 m
(A)53 (B)104 (C)106 (D)221 (E)391
Answer:

5. What is the value of the expression 1234  2341  3412  4123 ?


(A)5555 (B)10000 (C)11110 (D)11111 (E)10101010
Answer:

Questions 6 to 13, 5 marks each


6. Using the dd/mm/yyyy (day/month/year) format, dates such as 01/02/2010 and
02/02/2020 are called "Palindromic Days" because the eight-digit numbers
formed (e.g. 01022010 and 02022020) are both palindromes. What is the
remainder when the “Palindromic Day” in the year 2021 is divided by 11?
(Note: A palindrome is a number which reads the same backward as forward.)
Answer:

7. In the figure below, we use a thread to string some black and white pearls into a
straight line. Starting from the black pearl marked with an arrow, we move the
pearls one by one from left to right. How many pearls must be moved so that the
number of black pearls on the left is equal to the number of white pearls on the
right?

Answer: pearls

313
MP 3
8. There are 3 air conditioning units in John's house and one new air conditioning
unit was installed over the summer. However, due to power limitations of his
electric meter, if all 4 air conditioners are turned on at the same time, the fuse
will overload, so only 3 of them can be turned on at the same time. So, in a 24
hour period over the summer, if 3 air-conditioning units are always turned on,
then at least how long is the most used air conditioning unit turned on?
Answer: hours

9. The following figure shows five circles that intersect each other in pairs. If each
circle has an area of 6 m2 and each overlapping area of two intersecting circles is
1 m2, then what is the total area, in m2, covered by the five circles?

Answer: m2

10. The following figure is made up of some number of identical unit squares. Now,
Alex wants to form a bigger square by attaching unit squares on the figure
(without changing the current figure). At least how many unit squares must he
use in order to do so?

Answer:

314
MP 4
11. Using some number of black and white balls of the same size, we arrange the
figures as shown in the diagram below. If Figure 1 uses 8 balls, Figure 2 uses 13
balls, Figure 3 uses 20 balls and so on. How many balls are there in Figure 10?

Figure 1 Figure 2 Figure 3 Figure 4


Answer: balls

12. The diagram below shows seven rooms that are numbered from 1 to 7 such that
each room has a door connected to any of its adjacent rooms. Starting from room
1, our goal is to reach room 7, by passing through other rooms. If you can only
move to a room with the larger number, then how many different routes are
there?
2 4 6

1 3 5 7

Answer: routes
13. Fill in the letters a, b, c, d and e in each of the cells of the 5  5 table shown
below such that each letter appears exactly once in each row and column. What
letter should be placed in the cell where the "☆" is located?

a b
a
☆ d
b e
e d

Answer:

315
MP 5

Questions 14 to 15, 20 marks each


(Detailed solutions are needed for these two problems)
14. If 2021 is expressed as a sum of 100 prime numbers, where the largest prime
number used must be as small as possible, then what is the smallest possible value
of the largest prime number among these 100 prime numbers? Give an example
that satisfies these conditions.

Answer: The smallest possible value of the largest prime number is .


We can choose the following primes to satisfy the conditions:

316
MP 6
15. The two figures below are composed of small equilateral triangles of the same
size. We want to put several pieces of the parallelogram cardboard (which can be
rotated or flipped that is shown on the left) into the equilateral triangle grid that is
shown on the right without overlapping each other. The sides of these
parallelogram cardboards are required to be placed along the grid lines of the
equilateral triangle grid and cannot exceed the sides of it. What is the maximum
number of pieces of parallelogram cardboard that can be placed into the
triangular grid?
Justify your answer and show one way to place the cardboards.

The maximum number of pieces is .


Right figure is one way to place those
cardboards.

Answer:

317
318
注意:

允許學生個人、非營利性的圖書館或公立學校合理使用
本基金會網站所提供之各項試題及其解答。可直接下載
而不須申請。

重版、系統地複製或大量重製這些資料的任何部分,必
須獲得財團法人臺北市九章數學教育基金會的授權許
可。

申請此項授權請電郵 ccmp@seed.net.tw

Notice:

Individual students, nonprofit libraries, or schools are


permitted to make fair use of the papers and its
solutions. Republication, systematic copying, or
multiple reproduction of any part of this material is
permitted only under license from the Chiuchang
Mathematics Foundation.
Requests for such permission should be made by
e-mailing Mr. Wen-Hsien SUN ccmp@seed.net.tw

319
Solution Key to the Second Round of 10th IMAS 2020/2021
Middle Primary Division

Questions 1 to 5, 4 marks each


1. Which of the following expressions has a value equal to 3?
(A) 2  0  2  1 (B) 2  0  2  1 (C) 2  0  2  1
(D) 2  0  2  1 (E) 2  0  2  1
【Suggested Solution】
Calculating them, we have 2  0  2  1  0  1  1, 2  0  2  1  2  2  4 ,
2  0  2  1  2  0  1  3 , 2  0  2  1  2  3  5 and 2  0  2  1  0  2  2 . Since
only the third option is equal to 3. So, the answer is (C).
Answer:(C)
2. Which among the choices has the same pattern as the figure shown below?
(Note: figure can be rotated but not flipped)

(A) (B) (C) (D) (E)

【Suggested Solution】
From the original figure, the four black circles located inside the large circle are
arranged in a clockwise direction from smallest to largest, while the four black circles
inside the large circle in options (A), (B), (C) and (D) are arranged from smallest to
largest, but in a counterclockwise direction. Thus, only the four black circles inside
the large circle in option (E) are arranged in a clockwise direction from smallest to
largest. So, the answer is (E).
Answer:(E)
3. Nelson has some number of apples and one pear that he must place in a straight
line on the table. If we start counting from left to right, the pear is located on the
12th position, while counting from right to left, the pear is located on the 18 th
position. How many fruits are there on the table?
(A)30 (B)29 (C)28 (D)22 (E)20

320
【Suggested Solution】
Counting from left to right, the pear is located on the 12th position, so we can
conclude that there are 11 apples on the left of the pear. Similarly, counting from right
to left, the pear is located on the 18th position, so we can conclude that there are 17
apples on the right of the pear. So, in total there are 11  17  1  29 fruits on the
table. So, the answer is (B).
Answer:(B)
4. Jack paved the sidewalk in the garden with tiles as shown in the figure below. If
the width of the sidewalk is 1 m, what is the total area, in m2, of the sidewalk?
23 m

17 m

13 m
(A)53 (B)104 (C)106 (D)221 (E)391
【Suggested Solution】
Divide the sidewalk into three sections of linear sidewalks as follows: upper
(horizontal), middle (vertical) and lower (horizontal) and the area of each section is
23  1  23 m2, 17  1  17 m2 and 13  1  13 m2, respectively. So, the total area of the
sidewalk is 23  17  13  53 m2. So, the answer is(A).
Answer:(A)
5. What is the value of the expression 1234  2341  3412  4123 ?
(A)5555 (B)10000 (C)11110 (D)11111 (E)10101010
【Suggested Solution 1】
All four terms of the expression contain all four digits 1, 2, 3 and 4 exactly once and
each digit appears in each of the place values exactly once, so the value of the
original expression is equal to (1  2  3  4) 1111  11110 . So, the answer is (C).
【Suggested Solution 2】
1234  2341  3412  4123
 (1  2  3  4)  (1  2  3  4)  10  (1  2  3  4)  100  (1  2  3  4)  1000
 10  100  1000  10000
 11110
So, the answer is (C).
Answer:(C)

321
Questions 6 to 13, 5 marks each
6. Using the dd/mm/yyyy (day/month/year) format, dates such as 01/02/2010 and
02/02/2020 are called "Palindromic Days" because the eight-digit numbers
formed (e.g. 01022010 and 02022020) are both palindromes. What is the
remainder when the “Palindromic Day” in the year 2021 is divided by 11?
(Note: A palindrome is a number which reads the same backward as forward.)
【Suggested Solution】
In the year 2021, the only “Palindromic Day” is 12/02/2021, so the eight-digit
number is 12022021. Using the divisibility of 11 rule, the difference between the sum
of all digits on odd placements and the sum of all digits on even placements of this
palindrome is (1  0  2  2)  (2  2  0  1)  0 . Hence, when it divided by 11, the
remainder is 0.
Answer:0
【Comments】
It can be seen that the digit form of a “Palindrome Day” is always in the form of
abcddcba , so the difference between the sum of all digits on odd placements and the
sum of all digits on even placements of this palindrome is
(a  c  d  b)  (b  d  c  a)  0 ,
therefore, the number of palindromes in this form is always a multiple of 11.
7. In the figure below, we use a thread to string some black and white pearls into a
straight line. Starting from the black pearl marked with an arrow, we move the
pearls one by one from left to right. How many pearls must be moved so that the
number of black pearls on the left is equal to the number of white pearls on the
right?

【Suggested Solution】
Judging from the number of black and white pearls in the diagram, there are 7 black
pearls on the left and 4 white pearls on the right. Now, starting from the black pearl
marked with an arrow, move the pearls one by one from left to right:
After the first move, there will be 6 black pearls on the left and 4 white pearls on the
right.
After the second move, there will be 5 black pearls on the left and 4 white pearls on
the right.
After the third move, there will be 5 black pearls on the left and 5 white pearls on the
right.
Continuing to do so will make the number of black pearls on the left less than the
number of white pearls on the right.
Therefore, after moving 3 pearls, the number of black pearls on the left is equal to the
number of white pearls on the right.
Answer:3 pearls

322
8. There are 3 air conditioning units in John's house and one new air conditioning
unit was installed over the summer. However, due to power limitations of his
electric meter, if all 4 air conditioners are turned on at the same time, the fuse
will overload, so only 3 of them can be turned on at the same time. So, in a 24
hour period over the summer, if 3 air-conditioning units are always turned on,
then at least how long is the most used air conditioning unit turned on?
【Suggested Solution 1】
To use the longest air conditioner for the least time, it is necessary to make the use
time of each air conditioner as evenly as possible. You might as well label these 4 air
conditioners as A, B, C, and D respectively, because it uses 3 units each time. It can
be seen that there are 4 combinations of 4 air conditioners, namely ABC, ABD, ACD,
and BCD. Therefore, each combination takes 24  4  6 hours on average in 24
hours. Since each air conditioner will appear in three combinations, each air
conditioner needs to be turned on for 18 hours. Even the one that has been used for
the longest time needs to be used for at least 18 hours.
【Suggested Solution 2】
To make the air conditioner that uses the longest time the least, it is necessary to
make the use time of each air conditioner as evenly as possible. If 3 air conditioners
are turned on at the same time, the total number of hours that all air conditioners are
turned on in 24 hours is 24  3  72 hours. Now that these hours are to be evenly
distributed to each air conditioner, each air conditioner must be turned on for
72  4  18 hours, that is, the air conditioner that has been used for the longest time
must be used for at least 18 hours.
The 24 hours are equally divided into four periods of A, B, C, and D, each of which is
6 hours. Turn the first air conditioner on for ABC for three times, turn on the second
air conditioner for three times on ABD, turn on the third air conditioner for three
times on ACD, and turn on the fourth air conditioner for three times on BCD. Each air
conditioner uses 18 hours.
Answer:18 hours
9. The figure on the right shows five circles that
intersect each other in pairs. If each circle has an area
of 6 m2 and each overlapping area of two intersecting
circles is 1 m2, then what is the total area, in m2,
covered by the five circles?

【Suggested Solution】
Since the area of each circle is 6 m2 and the overlapping area of each of two
intersecting circles is 1 m2, so the total area covered by the five circles in the figure is
equal to the area of the five circles minus the area of the five overlapping parts, so the
total area covered by the five circles is 5  6  5  1  30  5  25 m2.
Answer:25 m2

323
10. The figure on the right is made up of some
number of identical unit squares. Now, Alex
wants to form a bigger square by attaching unit
squares on the figure (without changing the
current figure). At least how many unit squares
must he use in order to do so?
【Suggested Solution】
Observe that in the figure, the longest length is 9 units, and the widest length is 6
units. Therefore, to form a square, its side length must be at least 9 units, that is, at
least 9  9  81 small square pieces are required. Since there are already 32 squares,
so at least 81  32  49 unit squares need to be added.

Answer:49 unit squares


11. Using some number of black and white balls of the same size, we arrange the
figures as shown in the diagram below. If Figure 1 uses 8 balls, Figure 2 uses 13
balls, Figure 3 uses 20 balls and so on. How many balls are there in Figure 10?

Figure 1 Figure 2 Figure 3 Figure 4


【Suggested Solution 1】
Observe that on first four figures, it can be deduced that there are 10  2  12 balls
on each side of the large equilateral triangle formed by the white balls in Figure 10.
Then there are two black balls outside the two ends of the third column of the big
triangle formed by white balls and above the big triangle formed by white balls is a
small equilateral triangle formed by black balls, with 10  1  9 balls on each side.
So, in Figure 10, there are a total of
(1  2  3  ...  11  12)  2  (1  2  3  ...  8  9)  78  2  45  125 balls.

324
【Suggested Solution 2】
Observe that on first four figures, we can see that the total number of balls in the
Figure 2 is 5 more than the total number of balls in Figure 1, the total number of balls
in the Figure 3 is 7 more than the total balls in Figure 2, the total number of balls in
the Figure 4 is 9 more than the total balls in Figure 3, … , the total number of balls in
the Figure n is 2n  1 more than the total balls in Figure n  1 . So, the total number
of balls in Figure 10 is 5  7  9  11  13  15  17  19  21  117 more than the total
number of balls in Figure 1, therefore, in Figure 10, there are a total of 8  117  125
balls.
Answer:125 balls
12. The diagram below shows seven rooms that are numbered from 1 to 7 such that
each room has a door connected to any of its adjacent rooms. Starting from room
1, our goal is to reach room 7, by passing through other rooms. If you can only
move to a room with the larger number, then how many different routes are
there?
2 4 6

1 3 5 7

【Suggested Solution】
To reach room 2, you can only walk in from room 1, so there is only 1 route.
To reach room 3, you can walk in from room 1 or 2, so there are 1  1  2 routes in
total.
To reach room 4, you can walk in from room 2 or 3, so there are 1  2  3 routes in
total.
To reach room 5, you can walk in from room 3 or 4, so there are 2  3  5 routes in
total.
To reach room 6, you can walk in from room 4 or 5, so there are 3  5  8 routes in
total.
To reach room 7, you can walk in from room 6 or 7, so there are 5  8  13 routes in
total.
The answer can be obtained by marking the number on the graph as follows.

2 4 6
1 3 8

1 3 5 7
1 2 5 13

Answer:13 routes

325
13. Fill in the letters a, b, c, d and e in each of the cells of the 5  5 table shown
below such that each letter appears exactly once in each row and column. What
letter should be placed in the cell where the "☆" is located?

a b
a
☆ d
b e
e d
【Suggested Solution】
For easier convention, label the columns into A, B, C, D and E and label the rows as 1,
2, 3, 4 and 5 as shown below:
1 2 3 4 5
A a b
B a
C ☆ d
D b e
E e d
From the conditions of the problem, it is known that a, b, d and e cannot be filled in
the cell A3, so the letter c must be filled in.
Because B3 cannot be filled with a, c, d and e , therefore it is b, so E3 can only be
filled with a.
Because E1 cannot be filled with a, b, d and e it must be filled with c, so E4 can only
be filled with b.
1 2 3 4 5
A a b c
B b a
C ☆ d
D b e
E c e a b d
Because C1 cannot be filled with a, b, c and d, it must be filled with e, so B1 can only
be filled with d.
Because B2 cannot be filled with a, b, d and e, so it must be filled with c, and we
know that B4 can only be filled in with e.
Because C2 cannot be filled with b, c, d, and e, so a must be filled in, so D2 can only
be filled in with d.

326
1 2 3 4 5
A a b c
B d c b e a
C e a d
D b d e
E c e a b d
Then because A4 cannot be filled with a, b, c and e, so it must be filled with d, so A5
can only be filled with e.
Finally, because C4 cannot be filled with a, b, d and e, it must be filled with c.
Therefore, C5 can only be filled with b, D4 can only be filled with a, D5 can only be
filled with c.
Completing the table, we can see that the cell with "☆" should be filled with the a.
1 2 3 4 5
A a b c d e
B d c b e a
C e a d c b
D b d e a c
E c e a b d
Answer:a

Questions 14 to 15, 20 marks each


(Detailed solutions are needed for these two problems)

14. If 2021 is expressed as a sum of 100 prime numbers, where the largest prime
number used must be as small as possible, then what is the smallest possible value
of the largest prime number among these 100 prime numbers? Give an example
that satisfies these conditions.
【Suggested Solution】
Observe that all prime numbers are odd expect for the number “2”. Therefore, from
the parity of these numbers, it is known that when the number of odd numbers is odd,
its sum is also odd.
Because 2021 is an odd number, it must be divided into an even number of prime 2
and an odd number of odd prime numbers. So, in order to make the largest prime
number in this set as small as possible, we have to make take the odd prime numbers
as small as possible.
Now, since 2021  100  20  21 , and the smallest prime number greater than 20 is 23,
so there must be a prime number not less than 23 in the 100 prime numbers,
otherwise the sum will not be greater than 19 100  1900 , so we have to use the
prime number 23 as much as possible.

327
Now, we consider the case where 2021  23  87  20 . In this scenario, we can use
up to eighty-seven 23s, but the minimum of the remaining 100  87  13 prime
numbers is 2  13  26  20 , doesn’t meet the conditions.
Now, consider the case where we will be using eight-six 23s, which leaves us with
2021  86  23  43 . The remaining 14 prime numbers whose sum is 43, we can use
the following scenario: one 2, twelve 3s and one 5.
Therefore, among these 100 prime numbers, the smallest value of the largest prime
number is 23.
Answer:23
【Marking Scheme】
 Explain that 2021 must be a split into an even number of prime 2 and an odd
number of odd prime numbers. .............................................................. 5 points
 Show that there must be a prime number not less than 23 among the 100 prime
numbers .................................................................................................... 5 points
 Conclude that the minimum value of the largest prime number is 23 ..... 5 points
 Give an example that meets the conditions .............................................. 5 points

15. The two figures below are composed of small equilateral triangles of the same
size. We want to put several pieces of the parallelogram cardboard (which can be
rotated or flipped that is shown on the left) into the equilateral triangle grid that is
shown on the right without overlapping each other. The sides of these
parallelogram cardboards are required to be placed along the grid lines of the
equilateral triangle grid and cannot exceed the sides of it. What is the maximum
number of pieces of parallelogram cardboard that can be placed into the
triangular grid?
Justify your answer and show one way to place the cardboards.

【Suggested Solution】
Colour the large equilateral triangle grid in black and white as shown in the left figure
below, and observe that there are 36 black triangles and 28 white triangles. From here,
we can see that Each piece will always cover two gray triangles and two
white triangles, Therefore, you can put up to 28  2  14 pieces of the parallelogram
cardboard as shown in the left picture below. The picture below on the right shows
one possible example on how to place the cardboards.

328
【Marking Scheme】
 To colour the large equilateral triangle grids into black and white
................................................................................................................ 5 points
 Observe that each piece of cardboard will cover two black triangles and two
white triangles......................................................................................... 5 points
 Observe that there are 36 black triangles and 28 white triangles
(or a difference of 8) ............................................................................... 5 points
 Claim that a maximum of 14 pieces can be put in and a coverage method that
meets this maximum value is given ....................................................... 5 points

329
注意:

允許學生個人、非營利性的圖書館或公立學校合理使用
本基金會網站所提供之各項試題及其解答。可直接下載
而不須申請。

重版、系統地複製或大量重製這些資料的任何部分,必
須獲得財團法人臺北市九章數學教育基金會的授權許
可。

申請此項授權請電郵 ccmp@seed.net.tw

Notice:

Individual students, nonprofit libraries, or schools are


permitted to make fair use of the papers and its
solutions. Republication, systematic copying, or
multiple reproduction of any part of this material is
permitted only under license from the Chiuchang
Mathematics Foundation.
Requests for such permission should be made by
e-mailing Mr. Wen-Hsien SUN ccmp@seed.net.tw

330
International Mathematics Assessments for Schools

2021 ~ 2022 MIDDLE PRIMARY DIVISION FIRST ROUND PAPER


Time allowed:75 minutes

When your teacher gives the signal, begin working on the problems.

INSTRUCTION AND INFORMATION


GENERAL
1. Do not open the booklet until told to do so by your teacher.
2. No calculators, slide rules, log tables, math stencils, mobile phones or other
calculating aids are permitted. Scribbling paper, graph paper, ruler and compasses
are permitted, but are not essential.
3. Diagrams are NOT drawn to scale. They are intended only as aids.
4. There are 20 multiple-choice questions, each with 5 choices. Choose the most
reasonable answer. The last 5 questions require whole number answers between
000 and 999 inclusive. The questions generally get harder as you work through the
paper. There is no penalty for an incorrect response.
5. This is a mathematics assessment, not a test; do not expect to answer all questions.
6. Read the instructions on the answer sheet carefully. Ensure your name, school
name and school year are filled in. It is your responsibility that the Answer Sheet
is correctly coded.

THE ANSWER SHEET


1. Use only pencils.
2. Record your answers on the reverse side of the Answer Sheet (not on the question
paper) by FULLY filling in the circles which correspond to your choices.
3. Your Answer Sheet will be read by a machine. The machine will see all markings
even if they are in the wrong places. So please be careful not to doodle or write
anything extra on the Answer Sheet. If you want to change an answer or remove
any marks, use a plastic eraser and be sure to remove all marks and smudges.

INTEGRITY OF THE COMPETITION


The IMAS reserves the right to re-examine students before deciding whether to
grant official status to their scores.

331
2021 ~ 2022 MIDDLE PRIMARY DIVISION FIRST ROUND PAPER

Questions 1-10, 3 marks each


1. Five identical pencils are positioned on the table as shown below. Which two
pencils are lying on the same plane?
a b
c

e
(A)b and c (B)b and d (C)b and e (D)c and d (E)c and e

2. Five children ride on a seesaw. The diagrams below are images showing the
weight comparisons between two different persons.
b a
c e
a e

i ii iii
d d

b e

iv v

Who has the heaviest weight?


(A)a (B)b (C)c (D)d (E)e

3. Tom and Tim play a badminton match. The first player to reach 21 points wins a
game and the first player to win two games wins the match. If Tom wins the first
and third games and gets a total of 60 points for the whole match, what score did
Tom get in the second game?
(A)8 (B)10 (C)15 (D)18 (E)20

332
MP 2
4. Which of the following is closest to 2021 kg?
(A)2 cars, and the weight of each car is 1013 kg.
(B)21 motorcycles, and the weight of each motorcycle is 101 kg.
(C)155 bicycles, and the weight of each bicycle is 13 kg.
(D)61 desks, and the weight of each desk is 33 kg.
(E)44 refrigerators, and the weight of each refrigerator is 46 kg.

5. Let ★ be a positive integer such that (★  2)  (★  2)  2021, what is the


value of ★?
(A)42 (B)43 (C)44 (D)45 (E)47

6. What is the value of 2021  2020  2019  2018  2017  ...  2  1 ?


(A)1 (B)1010 (C)1011 (D)1012 (E)2021

7. Josh has 3 dogs, namely Jack, Sparrow and Tom, where each of them have
different weights. If Jack and Sparrow weigh 12 kg together, Sparrow and Tom
weigh 16 kg together and Tom and Jack weigh 14 kg together, then what is the
total weight, in kg, of all 3 dogs?
(A)21 kg (B)24 kg (C)26 kg (D)28 kg (E)42 kg

8. A certain year in the 21st Century is a perfect square number. What year is it?
(Note: 1  12 , 4  22 , 9  32 , …, so we call 1, 4, 9, … perfect square numbers.)
(A)1936 (B)2021 (C)2025 (D)2116 (E)2209

9. Peter keeps all of his socks in a messy non-transparent drawer under his bed. He
has 8 black socks and 6 white socks. He takes the socks out from the box, one
sock at a time. How many attempts does he need to make sure that he gets 2
socks of the same colour? (Note: There is no difference between a left and a
right sock.)
(A)2 (B)3 (C)4 (D)7 (E)9

10. The puzzle shown in the diagram below has one rule: Always follow the
direction of the arrows. How many allowable routes from “in” to “out” adhere to
the rule?

In Out

(A)6 (B)7 (C)8 (D)9 (E)10

333
MP 3

Questions 11-20, 4 marks each


11. The diagram below contains five pairs of figures (square, triangle, circle, heart
and star) that are placed on a grid. We must connect any two same figures using a
single path such that all connecting paths must run only along the grey segments
of the grid and no connecting paths may intersect.

Jane draws five set of connecting paths, as shown in the options. Exactly four of
them are satisfying the conditions. Which option is NOT a correct set of
connecting paths?
(A) (B)

(C) (D) (E)

334
MP 4
12. There are four pairs of square patterns as shown below. Inside each square, there
is a wire element. In one of the squares (marked with a question mark), the wire
element is missing. Which of the following is its pattern?

?
(A) (B) (C) (D) (E)

13. Alan and Benjie use the staircase shown in the diagram below to play a game
where the goal is to reach a certain level on the staircase first. At the beginning,
both of them will start in step 1, and in each turn, they will be playing the “Rock,
Paper and Scissors” game to move around the staircase. The winner in each game
gets to move 4 consecutive steps upward (or downward or a combination of both)
for showing a rock; 5 consecutive steps for showing scissors and 6 consecutive
steps for showing a paper. So, for example, when someone reaches step 10, he
must go down to the step 9 and so on, finally returning to the step 1 and then
goes back up doing the same procedure again until somebody wins. To illustrate
further, when a person shows a rock on the first game and wins and then in the
second game shows a paper and wins, he will then move and land on step 9.

10
9
8
7
6
5
4
3
2
1
Since Alan is currently on step 1 of the stairs, what is the least number of times
he has to win in order to reach step 2? Note: If a game is a tie (where both
players show the same hand, i.e. paper and paper), then they don’t move.
(A)2 (B)3 (C)4 (D)5 (E)8

335
MP 5
14. A young-looking mother took her child to the park to play. People thought she
was the child's elder sister. Curiously, one person asked her age and her smart
child replied: "Four years ago, my mother was 7 times my age, but now she is 4
times my age." How old is the mother?
(A)21 (B)24 (C)28 (D)32 (E)36

15. The diagram below is a square that is divided into four parts. Which two of them
have the same area?

a c

b d

(A)a and b (B)a and c (C)b and c (D)b and d (E)c and d

16. Select two numbers from 2, 3, 4, 5, 6, 7 and 8 to form a simplified proper


1
fraction. If the product of any two such simplified proper fractions is , they are
2
considered as one pair. How many pairs are there in total?
(A)4 (B)5 (C)6 (D)7 (E)8

17. Let’s play a game where we start counting from the thumb with the number 1, the
index finger is 2, the middle finger is 3, the ring finger is 4 and the little finger is
5, then after which we count in backward order, where the ring finger is 6, the
middle finger is 7, the index finger is 8, and the thumb is 9; then we count in
forward order again, and we keep on counting in this manner, as shown on the
diagram. Which finger will it land into when I count to 2021?
8 7 6
2 3 4
5
9
1

(A)thumb (B)index (C)middle (D)ring (E)little

336
MP 6
18. The diagram below is made up of nine identical squares which form the letter “I”.
How many different rectangles located in different positions (including squares)
are there in the diagram?

(A)9 (B)12 (C)18 (D)22 (E)25

19. Four persons A, B, C and D will participate in a 100-meter race. The following
table shows the predicted rankings of each of these four participants before the
race. Line ① is participant B’s prediction. After the race, all four of them
checked the final ranking and after comparing it with their predictions, they
found out that nobody predicted their own personal ranking correctly. It is known
that everyone has correctly predicted the winner of at least one from the first to
third placers, none of them correctly predicted who finished last and exactly two
participants correctly predicted the third place winner. Who are the participants
that predicted line ②, line ③ and line ④? Note: write the answer in this order.
First Place Second Place Third Place Four Place
① D B C A
② D A B C
③ A B C D
④ C A B D
(A)C, D, A (B)C, A, D (C)D, C, A (D)D, A, C (E)A, C, D

20. The three circles below have some number of points on their circumference.
Connect all the points on the same circle using straight lines and count the
number of regions these segments have partitioned the circle into.

2 points, 3 points, 4 points,


2 regions. 4 regions. 8 regions.
If there are 5 points on the circumference of a circle, how many regions have
been partitioned at the most?
(A)10 (B)12 (C)15 (D)16 (E)20

337
MP 7

Questions 21-25, 6 marks each


21. In a race, each athlete must complete a 100-meter distance at their own fixed
speed, and the athlete who reaches the finish line first wins. If athlete A has
reached the finish line, athlete B still has 10 meters to complete the race; and
when athlete B reaches the finish line, athlete C is still 20 meters away. How
many meters was athlete A ahead of athlete C?

22. There are four cards with digits 0, 1, 2 and 3 written on them. Choose any three
cards and arrange them to form all possible three-digit numbers (where 0 is not
allowed to be the leading digit). What is the average of all possible three-digit
numbers that are odd?

23. Fill-in the numbers 1, 2, 3, 4, 6, 9, 12, 18, and 36 into each of the unit squares
exactly once on the grid below, so that the product of the three numbers on each
horizontal, vertical and diagonal lines are all equal. Which number must be filled
in the unit square marked with “☆”?

24. There are three kinds of tokens having different weights: 5 grams, 10 grams and
20 grams. The total weight of 19 tokens is 250 grams. If the number of 5 gram
tokens and the number of 20 gram tokens are interchanged, then the total weight
of tokens are reduced to 190 grams. What is the number of tokens having a
weight of 10 grams?

25. A strange math teacher gave his students this problem. He first wrote the
following eight digits: 1, 2, 3, 4, 5, 7, 8 and 9 on the board. He then asked his
students to split these digits into two groups, such that each group has four digits;
and then arrange and combine the four digits in each group to form two 2-digit
numbers and add them together. Finally, the result of adding the two 2-digit
numbers in each group must be the same 2-digit number. What the largest
possible value of this sum?

***

338
注意:

允許學生個人、非營利性的圖書館或公立學校合理使用
本基金會網站所提供之各項試題及其解答。可直接下載
而不須申請。

重版、系統地複製或大量重製這些資料的任何部分,必
須獲得財團法人臺北市九章數學教育基金會的授權許
可。

申請此項授權請電郵 ccmp@seed.net.tw

Notice:

Individual students, nonprofit libraries, or schools are


permitted to make fair use of the papers and its
solutions. Republication, systematic copying, or
multiple reproduction of any part of this material is
permitted only under license from the Chiuchang
Mathematics Foundation.
Requests for such permission should be made by
e-mailing Mr. Wen-Hsien SUN ccmp@seed.net.tw

339
Solution to
2021~2022 International Mathematics Assessment for Schools
Round 1 of Middle Primary Division
1. Five identical pencils are positioned on the table a b
as shown on the right. Which two pencils are c
lying on the same plane?
(A)b and c (B)b and d (C)b and e
(D)c and d (E)c and e

【Suggested Solution】
d
We denote the face of table as Level 0 and a pencil high
as Level 1. Then, the nib and top of pencil a are both on
Level 1. The nib and top of pencil e are both on Level 0. e
The nib of pencil b and d are both on Level 1 while the
top of pencil b and d are both on Level 0, hence they
lying in the same plane. Even the top of pencil c is on
Level 0 but its nib is little higher than Level 1, it is not
lying in the same plane with pencil b and d.
Answer: (B)
2. Five children ride on a seesaw. The diagrams below are images showing the
weight comparisons between two different persons.
b a
c e
a e

i ii iii
d d

b e

iv v

Who has the heaviest weight?


(A)a (B)b (C)c (D)d (E)e
【Suggested Solution】
From seesaw (i), we have a is heavier than b. From seesaw (ii), we have c is heavier
than e . From seesaw (iii), we have e is heavier than a . From seesaw (iv), we have b
is heavier than d . From seesaw (v), we have e is heavier than d. So the weights from
heaviest to lightest is c, e, a, b and d.
Answer: (C)

340
3. Tom and Tim play a badminton match. The first player to reach 21 points wins a
game and the first player to win two games wins the match. If Tom wins the first
and third games and gets a total of 60 points for the whole match, what score did
Tom get in the second game?
(A)8 (B)10 (C)15 (D)18 (E)20
【Suggested Solution】
Since Tom wins the first and third games, he gets 21 points in each of the first and
third game. So Tom gets 60 − 21 − 21 = 18 points in the second game.
Answer: (D)
4. Which of the following is closest to 2021 kg?
(A)2 cars, and the weight of each car is 1013 kg.
(B)21 motorcycles, and the weight of each motorcycle is 101 kg.
(C)155 bicycles, and the weight of each bicycle is 13 kg.
(D)61 desks, and the weight of each desk is 33 kg.
(E)44 refrigerators, and the weight of each refrigerator is 46 kg.
【Suggested Solution】
The total weight of 2 cars is 2 1013 = 2026 kg,
the total weight of 21 motorcycles is 21101 = 2121 kg,
the total weight of 155 bicycles is 155 13 = 2015 kg.
the total weight of 61 desks is 61 33 = 2013 kg,
the total weight of 44 refrigerators is 44  46 = 2024 kg,
Since 2026 − 2021 = 5 、 2121 − 2021 = 100 、 2021 − 2015 = 6 、 2021 − 2013 = 8 、
2024 − 2021 = 3 。So 44 refrigerators are closest to 2021 kg.
Answer: (E)
5. Let ★ be a positive integer such that (★ − 2)  (★ + 2) = 2021 , what is the
value of ★?
(A)42 (B)43 (C)44 (D)45 (E)47
【Suggested Solution 1】
Since we know that 2021 = 43  47 , then it easy to see that ★ = 45 .
【Suggested Solution 2】
Observe that the equation is in the form (★ − 2)  (★ + 2) = ★  ★ − 2  2 = 2021 ,
simplifying we get ★ ★ = 2021 + 4 , which yields ★  ★ = 2025 . Since ★ is
positive integer, we have ★ = 45 .
Answer:(D)
6. What is the value of 2021 − 2020 + 2019 − 2018 + 2017 + ... − 2 + 1 ?
(A)1 (B)1010 (C)1011 (D)1012 (E)2021
【Suggested Solution】
2021 − 2020 + 2019 − 2018 + 2017 + ... − 2 + 1
= (2021 − 2020) + (2019 − 2018) + (2017 − 2016) + ... + (3 − 2) + 1
= 1011
(C)
Answer:

341
7. Josh has 3 dogs, namely Jack, Sparrow and Tom, where each of them have
different weights. If Jack and Sparrow weigh 12 kg together, Sparrow and Tom
weigh 16 kg together and Tom and Jack weigh 14 kg together, then what is the
total weight, in kg, of all 3 dogs?
(A)21kg (B)24 kg (C)26 kg (D)28 kg (E)42 kg
【Suggested Solution】
If we add the weight of each pair then each dog participates 2 times in the sum. Thus
12 + 14 + 16
the total weight of all 3 dogs is = 21 kg.
2
Answer: (A)
8. A certain year in the 21st Century is a perfect square number. What year is it?
(Note: 1 = 12 , 4 = 22 , 9 = 32 , …, so we call 1, 4, 9, … perfect square numbers.)
(A)1936 (B)2021 (C)2025 (D)2116 (E)2209
【Suggested Solution】
All the years of the 21st century are referred as four-digit numbers such as 20□□ ,
that is, a total of 100 years from 2000 to 2099. Because 442 = 1936 and 462 = 2116
so that 442  20□□  462 , but we know that 452 = 2025 . It follows the required
year number is 2025. Hence, the year number is 2025.
Answer: (C)
9. Peter keeps all of his socks in a messy non-transparent drawer under his bed. He
has 8 black socks and 6 white socks. He takes the socks out from the box, one
sock at a time. How many attempts does he need to make sure that he gets 2
socks of the same colour? (Note: There is no difference between a left and a right
sock.)
(A)2 (B)3 (C)4 (D)7 (E)9
【Suggested Solution】
He needs 3 attempts, works case scenario is he takes a 2 socks of different colour in
the first 2 tries and the 3rd one will be the same colour as one of the first 2.
Answer:(B)
10. The puzzle shown in the diagram below has one rule: Always follow the
direction of the arrows. How many allowable routes from “in” to “out” adhere to
the rule?

In Out

(A)6 (B)7 (C)8 (D)9 (E)10

342
【Suggested Solution】
The are 10 allowable routes.
The number of routes to each intersection is shown in the diagram below.
B C
1 1+2=3

H I
1+1=2 2

A G 3+4+3=
in 1 1 2+2=4 10 out
J D
L K
1+1=2 2

1 1+2=3
F E
From A, there is only a single path to each of B, G and F. To reach H, one can either
pass through B or G, and so there are 1 + 1 = 2 ways. To reach I, one can only come
from H and hence, there are 2 ways to reach I. Meanwhile, to reach C, one can come
from B or I, hence, a total of 1 + 2 = 3 ways. Using the same reasoning, we obtain
the following conclusions:
To reach intersection L, there are 2 ways.
To reach the intersection K, there are 2 ways.
To reach the intersection E, either one comes from F or K, hence a total of 1 + 2 = 3
ways.
To reach the intersection J, either one comes from I or K, hence a total of 2 + 2 = 4
ways.
To reach the intersection D, one can come from C, J or E, and so we conclude that
there is a total of 3 + 4 + 3 = 10 different paths.
Answer: (E)
11. The diagram below contains five pairs of figures (square, triangle, circle, heart
and star) that are placed on a grid. We must connect any two same figures using a
single path such that all connecting paths must run only along the grey segments
of the grid and no connecting paths may intersect.

343
Jane draws five set of connecting paths, as shown in the options. Exactly four of
them are satisfying the conditions. Which option is NOT a correct set of
connecting paths?
(A) (B)

(C) (D) (E)

【Suggested Solution】
As we combine the five connecting paths, we can
find that the path in option (A) will intersect with
the paths in option (B) and (E).
Since exactly four paths are satisfying the
conditions, so the path in option (A) is not correct.
The diagram below is a solution.

(A)
Answer:

344
12. There are four pairs of square patterns as shown below. Inside each square, there
is a wire element. In one of the squares (marked with a question mark), the wire
element is missing. Which of the following is its pattern?

?
(A) (B) (C) (D) (E)

【Suggested Solution】
Each pair of patterns always gives the same pattern; see the diagrams below.
The missing wire element is highlighted.

+ =

+ =

+ =

+ =

(A)
Answer:
13. Alan and Benjie use the staircase shown in the diagram below to play a game
where the goal is to reach a certain level on the staircase first. At the beginning,
both of them will start in step 1, and in each turn, they will be playing the “Rock,
Paper and Scissors” game to move around the staircase. The winner in each game
gets to move 4 consecutive steps upward (or downward or a combination of both)
for showing a rock; 5 consecutive steps for showing scissors and 6 consecutive
steps for showing a paper. So, for example, when someone reaches step 10, he
must go down to the step 9 and so on, finally returning to the step 1 and then
goes back up doing the same procedure again until somebody wins. To illustrate
further, when a person shows a rock on the first game and wins and then in the
second game shows a paper and wins, he will then move and land on step 9.

345
10
9
8
7
6
5
4
3
2
1
Since Alan is currently on step 1 of the stairs, what is the least number of times
he has to win in order to reach step 2? Note: If a game is a tie (where both
players show the same hand, i.e. paper and paper), then they don’t move.
(A)2 (B)3 (C)4 (D)5 (E)8
【Suggested Solution】
Let us find the possible combination of the game, in order to answer the problem
correctly.
Since Alan stopped on the level 2. If the answer just depends on one game, then it is
impossible. Therefore, it must be from the level 10 moving down to the level 2. There
are 9 move-up steps and 8 move-down steps, with a total of 17 steps. Now, let us use
the sum of 4 steps, 5 steps and 6 steps to express 17. So, we have
4 + 4 + 4 + 5 = 17 or 6 + 6 + 5 = 17 , it is either one of the two, consider Alan win at
least 3 times, which will be Alan won 1 time with scissors and 2 times with paper.
Answer: (B)
14. A young-looking mother took her child to the park to play. People thought she
was the child's elder sister. Curiously, one person asked her age and her smart
child replied: "Four years ago, my mother was 7 times my age, but now she is 4
times my age." How old is the mother?
(A)21 (B)24 (C)28 (D)32 (E)36
【Suggested Solution 1】
Since the age difference of mother and her child are never change, hence four years
ago the difference was 7 − 1 = 6 times of the child’s age, and now the difference was
4 − 1 = 3 times of the child’s age. After four years, the difference from 6 times
change to 3 times of the child’s age, then we can get of the child’s age was double.
Hence, the child is 8 years old now. Therefore, the mother’s age is 32 years old now.
【Suggested Solution 2】
Suppose the ages of the mother and child at present are A and B respectively, and two
equations can be listed by the given conditions as follows: ( B − 4)  7 = A − 4 and
A = 4B . Therefore, the child’s age is B = 8 and the mother’s age is A = 8  4 = 32
years old.
Answer: (D)

346
15. The diagram below is a square that is divided into
four parts. Which two of them have the same area?

a c

b d

(A)a and b (B)a and c (C)b and c (D)b and d (E)c and d
【Suggested Solution】
Add the grid as the diagram shown on the right.
Thus the area of part a is 14 unit square, the area of part b
is 17 unit square, the area of part c is 17 unit square, the
a c
area of part d is 16 unit square. Hence part b and c have the
same area.
Answer: (C) b d

16. Select two numbers from 2, 3, 4, 5, 6, 7 and 8 to form a simplified proper


1
fraction. If the product of any two such simplified proper fractions is , they are
2
considered as one pair. How many pairs are there in total?
(A)4 (B)5 (C)6 (D)7 (E)8
【Suggested Solution】
Let us begin with the smallest number and be sure the fractions that you formed
cannot be reduced.
2
When the denominator is 3, we have .
3
3
When the denominator is 4, we have .
4
2 3 4
When the denominator is 5, there are 3 simplified fractions can be formed: , , .
5 5 5
5
When the denominator is 6, we have .
6
2 3
When the denominator is 7, there are 5 simplified fractions can be formed: , ,
7 7
4 5 6
, ,
7 7 7
3 5 7
When the denominator is 8, there are 3 simplified fractions can be formed: , , .
8 8 8

347
1
The product of any of the two simplified fractions as mentioned above will be ,
2
they are:
2 3 2 1 3 5 3 1
First Pair:  = = ; Second Pair:  = = ;
3 4 4 2 5 6 6 2
5 4 4 1 7 4 4 1
Third Pair:  = = ; Fourth Pair:  = =
8 5 8 2 8 7 8 2
Therefore, there are four possible pairs in total.
(A)
Answer:
17. Let’s play a game where we start counting from the thumb with 8 7 6
the number 1, the index finger is 2, the middle finger is 3, the 2 3 4
ring finger is 4 and the little finger is 5, then after which we 5
count in backward order, where the ring finger is 6, the middle 9
finger is 7, the index finger is 8, and the thumb is 9; then we 1
count in forward order again, and we keep on counting in this
manner, as shown on the diagram. Which finger will it land into
when I count to 2021?
(A)thumb (B)index (C)middle (D)ring (E)little
【Suggested Solution】
Obviously, solving this problem is not relying on counting from finger to finger, but
finding out the pattern. From the given diagram, we know that 8 numbers form a
cycle, so no matter how big the number is, we only need to divide by 8 first, and after
knowing the remainder and starting counting the remainder, then we will know the
number is located in which finger.
Divide 2021 by 8, and the final remainder is 5. Just simply count the final count to
the little finger.
Answer: (E)
18. The diagram on the right is made up of nine identical squares
which form the letter “I”. How many different rectangles located in
different positions (including squares) are there in the diagram?

(A)9 (B)12 (C)18 (D)22 (E)25


【Suggested Solution】
The only possible rectangles we can have in the diagram are as follows: 1 by 1 square,
1 by 2 rectangle, 1 by 3 rectangle, 1 by 4 rectangle and 1 by 5 rectangles.
Counting them, we have nine 1 by 1 squares, eight 1 by 2 rectangles, five 1 by 3
rectangles, two 1 by 4 rectangles and one 1 by 5 rectangle.
In total, there are 9 + 8 + 5 + 2 + 1 = 25 rectangles.
Answer:(E)

348
19. Four persons A, B, C and D will participate in a 100-meter race. The following
table shows the predicted rankings of each of these four participants before the
race. Line ① is participant B’s prediction. After the race, all four of them
checked the final ranking and after comparing it with their predictions, they
found out that nobody predicted their own personal ranking correctly. It is known
that everyone has correctly predicted the winner of at least one from the first to
third placers, none of them correctly predicted who finished last and exactly two
participants correctly predicted the third place winner. Who are the participants
that predicted line ②, line ③ and line ④? Note: write the answer in this order.
First Place Second Place Third Place Four Place
① D B C A
② D A B C
③ A B C D
④ C A B D
(A)C, D, A (B)C, A, D (C)D, C, A (D)D, A, C (E)A, C, D
【Suggested Solution】
This is like a guessing game. We must use reasoning skill to give the correct ranking.
First, since no one guessed the fourth placer correctly, therefore, the participant B was
the fourth placer. And we know that line ② and line ④, guess that participant B
won the third placer, which is wrong, so the third placer must be participant C, line
① and line ③, guess that participant B won the second placer, which is wrong, so
the second placer must be participant A and this implies D is the first placer.
Hence, the first placer is participant D, second placer is participant A, third placer is
participant C and fourth placer is participant B.
Then we will find out who are those people who do the prediction of line ②, line ③
and line ④? Let's look at the prediction of line ②, since the ranking of participant B
and participant C are wrong, because participant B is the who do the prediction of
line ①, so line ② should be the prediction of participant C. Carefully observe the
prediction of line ④, only the ranking of participant A is correct, so participant A
cannot be the one who predict line ④, because no one has predict his own ranking.
Thus, in summary, we have line ① is predicted by participant B, line ② is
predicted by participant C, line ③ must be predicted by participant A and line ④ is
predicted by participant D .
First Place Second Place Third Place Four Place
Final rankings D A C B
① B D B C A
② C D A B C
③ A A B C D
④ D C A B D
Answer: (B)

349
20. The three circles below have some number of points on their circumference.
Connect all the points on the same circle using straight lines and count the
number of regions these segments have partitioned the circle into.

2 points, 2 regions. 3 points, 4 regions. 4 points, 8 regions.


If there are 5 points on the circumference of a circle, how many regions have
been partitioned at the most?
(A)10 (B)12 (C)15 (D)16 (E)20
【Suggested Solution】
Let any three segments not meet at a same point then we can get at most 16 regions
as the diagram shown.

5 points,
16 regions.

(D)
Answer:
21. In a race, each athlete must complete a 100-meter distance at their own fixed
speed, and the athlete who reaches the finish line first wins. If athlete A has
reached the finish line, athlete B still has 10 meters to complete the race; and
when athlete B reaches the finish line, athlete C is still 20 meters away. How
many meters was athlete A ahead of athlete C?
【Suggested Solution】
Since each athlete finished the race at his own fixed speed. When athlete A reached
the finish line, athlete B was still 10 meters far, and the ratio of the distance of athlete
100
A to athlete B in the same time was ; that is, athlete B’s speed was 90% of
90
athlete A’s speed. Similarly, athlete C’s speed was 80% of athlete B’s speed. It
follows athlete C’s speed was 90%  80% = 72% of athlete A’s speed.
Hence, when athlete A reaches the finish line, athlete C should be at 72 meters. Thus,
athlete A is 100 − 72 = 28 meters ahead of athlete C.
Answer:028
22. There are four cards with digits 0, 1, 2 and 3 written on them. Choose any three
cards and arrange them to form all possible three-digit numbers (where 0 is not
allowed to be the leading digit). What is the average of all possible three-digit
numbers that are odd?

350
【Suggested Solution】
From the given information, there are only two even digits and two odd digits, we
have 2 way choose the unit digit, then we have 2 way choose the leading digit, we
have 2 way to arrange the tens digits. Hence we have 2  2  2 = 8 such 3-digit
numbers, we list down all of those 3-digit numbers:
103, 123, 201, 203, 213, 231, 301, 321.
their sum is 1696 and its average is 1696  8=212 .
Answer:212
23. Fill-in the numbers 1, 2, 3, 4, 6, 9, 12, 18, and 36 into each of the
unit squares exactly once on the grid below, so that the product of
the three numbers on each horizontal, vertical and diagonal lines ☆
are all equal. Which number must be filled in the unit square
marked with “☆”?
【Suggested Solution 1】
The product of those nine numbers is 1 2  3  4  6  9 12 18  36 =
1 2  3  2  2  2  3  3  3  2  2  3  2  3  3  2  2  3  3
Hence the product of each horizontal, vertical and diagonal line all equal to
2  2  2  3  3  3 = 216 . We have
216 = 36  6  1 = 36  3  2 = 18  12  1 = 18  6  2 12 1 18
= 18  4  3 = 12  9  2 = 12  6  3 = 9  6  4 9 6 4
Since 6 appear in four multiplication, it must placed at ☆.
2 36 3
We can complete the table as shown on the right.
【Suggested Solution 2】
Labelled the unit squares as diagram shown. Since the product of the a b c
three numbers on each horizontal, vertical and diagonal lines are all
d ☆ e
equal we have a  h = b  g = c  f = d  e 。Then we can get :
1 36 = 2 18 = 3 12 = 4  9 . f g h
Hence, 6 must placed at ☆. We can complete the table as shown below.
12 1 18
9 6 4
2 36 3
Answer:006
24. There are three kinds of tokens having different weights: 5 grams, 10 grams and
20 grams. The total weight of 19 tokens is 250 grams. If the number of 5 gram
tokens and the number of 20 gram tokens are interchanged, then the total weight
of tokens are reduced to 190 grams. What is the number of tokens having a
weight of 10 grams?
【Suggested Solution】
Suppose originally, the number of tokens of the 20 grams is 1 more than the number
of tokens of 5 grams, now, let us interchange the number of tokens of the 20 grams
and the number of tokens of 5 grams, then the total tokens is 20 − 5 = 15 grams less

351
than the original number.
From the given information, we know the original weight of those 19 tokens is 250
and after exchanging the number of tokens between the 5 grams and 20 grams, there
are 250 − 190 = 60 grams less. Because 60  15 = 4 , this indicates that there are 4
more tokens of 20 grams than 5 grams.
When reduced 4 pieces of tokens of 20 grams then the number of tokens of 20 grams
than 5 grams is the same and the total tokens is 250 − 20  4 = 170 grams.
At this time, the number of tokens becomes 19 − 4 = 15 pieces.
If 15 tokens are all of 10 grams, which has a total of 150 grams. But there must be a
total tokens of 170 grams, it is 20 grams less, so there are tokens of 5 grams and 20
grams. If we merged one piece of 5 grams token and one piece 20 grams token as a
big token, then 1 big token is (5 + 20) − 10  2 = 5 more than two tokens of 10 grams.
It follows the number of big tokens are 20  5 = 4 pieces and this implies there are
(170 − 25  4)  10 = 7 pieces of tokens of 10 grams. Therefore, there are 4 tokens of
5 grams, 8 tokens of 20 grams and 7 tokens of 10 grams.
Answer:007
25. A strange math teacher gave his students this problem. He first wrote the
following eight digits: 1, 2, 3, 4, 5, 7, 8 and 9 on the board. He then asked his
students to split these digits into two groups, such that each group has four digits;
and then arrange and combine the four digits in each group to form two 2-digit
numbers and add them together. Finally, the result of adding the two 2-digit
numbers in each group must be the same 2-digit number. What the largest
possible value of this sum?
【Suggested Solution 1】
As the student of this strange math teacher, he or she must split the 8 digits into two
groups then combine these 4 digits in each group into two 2-digit numbers and add
them together. Since the result of adding the two numbers in each group is a same
2-digit number, so the first digit of these two number have at most one of digits 5, 7,
8, 9 otherwise their sum will became 3-digit number. We can not choose 9 as one of
first digit or their sum will carry and became 3-digit number.
If we choose 7 and 8 for each one number’s first digit of these two group, the sum of
these two group must be 7w + 2x = 8y + 1z , where w, x, y, z be one of digits 3, 4, 5,9.
There must have a carry on, so it is impossible.
If we choose 5 and 8, the sum of these two group have two case 5w + 4x = 8y + 1z ,
where w, x, y, z be one of digits 2, 3, 7, 9, or the sum of these two group must be
5w + 3x = 8y + 1z , where w, x, y, z be one of digits 2, 4, 7, 9. The former must have a
curry on, so it is impossible. For the latter, we may let (w, x) = (7, 9) or (9, 7) and (y, z)
= (2, 4) or (4, 2) and obtained their same sum both are 96.
If we choose 5 and 7, the sum of these two group have two case 5w + 4x = 7y + 2z ,
where w, x, y, z be one of digits 1, 3, 8, 9, or the sum of these two group be
5w + 3x = 7y + 2z , where w, x, y, z be one of digits 1, 4, 8, 9, or the sum of these two
group must be 5w + 2x = 7y + 1z , where w, x, y, z be one of digits 3, 4, 8, 9. The

352
former two case must have a curry on, so it is impossible. For the last, we may let (w, x)
= (8, 9) or (9,8) and (y, z) = (3, 4) or (4,3) and obtained their same sum both are 87.
Hence, the largest value of this sum is 96.
【Suggested Solution 2】
Since total sum of all digits is 39 have remainder 3 when divided by 9. Hence, the
resulting number should have remainder 6 divided by 9. The largest such 2-digit
number 96. And 96 can be realized as 57 + 39 and 78 + 21, so the largest is 96.
Answer:096

353
注意:

允許學生個人、非營利性的圖書館或公立學校合理使用
本基金會網站所提供之各項試題及其解答。可直接下載
而不須申請。

重版、系統地複製或大量重製這些資料的任何部分,必
須獲得財團法人臺北市九章數學教育基金會的授權許
可。

申請此項授權請電郵 ccmp@seed.net.tw

Notice:

Individual students, nonprofit libraries, or schools are


permitted to make fair use of the papers and its
solutions. Republication, systematic copying, or
multiple reproduction of any part of this material is
permitted only under license from the Chiuchang
Mathematics Foundation.
Requests for such permission should be made by
e-mailing Mr. Wen-Hsien SUN ccmp@seed.net.tw

354
International Mathematics Assessments for Schools

2021 ~ 2022 Middle Primary Division 2nd Round Paper


Time: 120 minutes

Printed Name: Code: Score:

Instructions:
⚫ Do not open the contest booklet until you are told to do so.
⚫ Be sure that your name and code are written on the space provided above.
⚫ Round 2 of IMAS is composed of three parts; the total score is 100 marks.
⚫ Questions 1 to 5 are given as a multiple-choice test. Each question has five
possible options marked as A, B, C, D and E. Only one of these options is correct.
After making your choice, fill in the appropriate letter in the space provided. Each
correct answer is worth 4 marks. There is no penalty for an incorrect answer.
⚫ Questions 6 to 13 are a short answer test. Only Arabic numerals are accepted;
using other written text will not be honored or credited. Some questions have more
than one answer, as such all answers are required to be written down in the space
provided to obtain full marks. Each correct answer is worth 5 marks. There is no
penalty for incorrect answers.
⚫ Questions 14 and 15 require a detailed solution or process in which 20 marks are
to be awarded to a completely written solution. Partial marks may be given to an
incomplete presentation. There is no penalty for an incorrect answer.
⚫ Use of electronic computing devices is not allowed.
⚫ Only pencil, blue or black ball-pens may be used to write your solution or answer.
⚫ Diagrams are not drawn to scale. They are intended as aids only.
⚫ After the contest the invigilator will collect the contest paper.
The following area is to be filled in by the judges;
the contestants are not supposed to mark anything here.
Total
Question 1 2 3 4 5 6 7 8 9 10 11 12 13 14 15 Score Signature

Score

Score

355
2021 ~ 2022 Middle Primary Division 2nd Round Paper

Questions 1 to 5, 4 marks each

1. Determine the logic of the patterns on Figure A and use that knowledge to find
which unit square should Figure B be placed into.

(1)

(2)
(3)
Figure B
(4)

(5)
Figure A
(A)1 (B)2 (C)3 (D)4 (E)5

Answer:

2. Three farmers Alvin, Benny and Charlie went out to harvest chestnuts where
Alvin was able to get 116 chestnuts, Benny 112 chestnuts and Charlie 96
1
chestnuts. On their way home, one of them gave of his chestnuts to one of
4
1
the other two. Then, another one gave of his chestnuts to one of other two.
4
1
Finally, the third farmer gave of his chestnuts to another farmer. After all of
4
these, the total number of chestnuts of all three farmers are the same. At the start,
1
which farmer gave out of his chestnuts to which farmer?
4
(A)Alvin to Benny (B)Alvin to Charlie (C)Benny to Alvin
(D)Benny to Charlie (E)Charlie to Alvin

Answer:

356
MP 2
3. Peter will be attending a two-day summer camp, where he plans to wear different
colour of socks in both days. He keeps all of his socks in a messy non-transparent
drawer under his bed and it is known that he has 8 black socks and 6 white socks.
If he takes out 1 sock at a time, how many socks does he need to get to be sure
that he gets 2 pairs of socks of different colour?
(A)4 (B)6 (C)8 (D)10 (E)12

Answer:

4. In the sequence,
1, 2, 4, 8, 6, …
Starting from the second term, each term is formed by multiplying the previous term
by 2 and then taking the remainder when divided by 10. What is the 2022nd term?
(A)1 (B)2 (C)4 (D)6 (E)8
Answer:

5. A tetrahedron is a regular pyramid made up of four equilateral triangles. It is


possible to paint each face with a different colour --- red, green, yellow and blue.
Five different views of the tetrahedron are shown below. Which among the
choices is inconsistent with the other four?

red green

(A) (B) (C) (D) (E)

green green yellow yellow green


blue red red blue yellow
red yellow blue green red

Answer:

357
MP 3

Questions 6 to 13, 5 marks each


6. One day, Robert went to a library 1600 meters away from home to meet with his
classmates at 13 : 20. Robert leaves home at 13 : 00, walking at a constant speed
of 80 meters per minute. At exactly 13 : 05, Robert's mother noticed that he left
his wallet at home, so she chased him walking at a speed of 120 meters per
minute. Robert only realized he left his wallet at home at 13 : 10, so he walked
back casually at a speed of 80 meters per minute. Robert and his mother met
halfway through. After taking his wallet, he again walked back towards the
library at a speed of 80 meters per minute. If we ignore the time Robert took to
get the wallet from his mother, how much time, in minutes, did he waste?
Answer: minutes

7. Mr. Andrew's office is located in City T, while he lives in City S. Every day after
timing-off from work, he takes the same train to return to his home city. The train
station in city S is still some distance away from his home, and his wife travels
by car to the train station to pick him up. It is known that the train and his wife
arrive at the train station at the same time. One day, due to some reasons, his wife
left their house 30 minutes later than usual. Upon arriving at the train station in
City S, Andrew noticed that his wife hasn’t arrived yet so he decided to walk
home. On his way walking home, he met his wife along the way and rode the car
immediately. Upon reaching home, Andrew observed that they arrived home 22
minutes later than usual. How long, in minutes, did Andrew walk?
Station Mr. Andrew’s home

Answer: minutes

8. There are four cards with digits 0, 1, 2 and 3 written on each of them. Choose
three different cards and arrange them to form all possible three-digit numbers,
where 0 is not allowed be leading digit. What is the sum of all those three-digit
numbers that are odd?
Answer:

9. Three positive 9-digit numbers A, B and C each contains all the digits from 1 to 9
exactly once. If A − B = C , then what is the largest possible value of integer C?
Answer:

358
MP 4
10. In a line, there is at least one girl and at least one boy. Two children with exactly
4 or 7 other children between them are of the same gender. What is the maximum
number of children in this line?
Answer: children

11. In the diagram below, ABC is an isosceles triangle with AB AC and


BC 48 cm. If DE : DF = 5 : 7, what is the length, in cm, of DC?
A

F
E

B D C

Answer: cm

12. Numbers 1, 2, 3 and 4 are to be placed into the circles in the diagram below,
where each circle must be filled with exactly one number. If two circles
connected by a line should not contain the same number, find the total number of
ways in filling the diagram.

Answer: ways

359
MP 5
13. How many cubes which are shown in Figures 1 and 2 are found inside the
rectangular pattern below?

Figure 1

Figure 2

Answer: cubes

360
MP 6

Questions 14 to 15, 20 marks each


(Detailed solutions are needed for these two problems)
14. A continuous look-and-say sequence is formed this way: Start with three 1’s;
read the terms as “three 1’s” and write down what you read as “31” at the end of
the sequence; read the next group of unread identical numbers as “one 3” and
write “13” at the end; continue this process to get an infinite sequence 1, 1, 1, 3,
1, 1, 3, 2, 1, ….
(a) What is the largest possible digit you could write down? Why? (5 points)
(b) What is the 20th term of this sequence? Why? (5 points)
(c) Will the number “333” appear in the sequence? Why or why not? (10 points)

Answer: (a) The largest possible digit is . The reason is:

(b) The 20th term of this sequence is . The reason is:

(c) □ Yes. □ No. The reason is:

15. Divide the numbers 1, 2, …, 15 into five groups, such that each group contains
three numbers and sum of numbers in each group is 24. Please provide an
example.

Answer: Group 1: { , , } Group 2: { , , }


Group 3: { , , } Group 4: { , , }
Group 5: { , , }

361
362
注意:

允許學生個人、非營利性的圖書館或公立學校合理使用
本基金會網站所提供之各項試題及其解答。可直接下載
而不須申請。

重版、系統地複製或大量重製這些資料的任何部分,必
須獲得財團法人臺北市九章數學教育基金會的授權許
可。

申請此項授權請電郵 ccmp@seed.net.tw

Notice:

Individual students, nonprofit libraries, or schools are


permitted to make fair use of the papers and its
solutions. Republication, systematic copying, or
multiple reproduction of any part of this material is
permitted only under license from the Chiuchang
Mathematics Foundation.
Requests for such permission should be made by
e-mailing Mr. Wen-Hsien SUN ccmp@seed.net.tw

363
Solution to
2021~2022 International Mathematics Assessment for Schools
Round 2 of Middle Primary Division
1. Determine the logic of the patterns on Figure A and use that knowledge to find
which unit square should Figure B be placed into.

(1)

(2)

(3)
Figure B
(4)

(5)
Figure A
(A)1 (B)2 (C)3 (D)4 (E)5
【Suggested Solution】
In each row and each column, the shaded wedges add up to make a complete square.

So, square (1) should be filled by , square (2) should be filled by ,

square (3) should be filled by , square (4) should be filled by ,

and square (5) should be filled by . Hence the answer is (B).

(B)
Answer:

364
2. Three farmers Alvin, Benny and Charlie went out to harvest chestnuts where
Alvin was able to get 116 chestnuts, Benny 112 chestnuts and Charlie 96
1
chestnuts. On their way home, one of them gave of his chestnuts to one of
4
1
the other two. Then, another one gave of his chestnuts to one of other two.
4
1
Finally, the third farmer gave of his chestnuts to another farmer. After all of
4
these, the total number of chestnuts of all three farmers are the same. At the start,
1
which farmer gave out of his chestnuts to which farmer?
4
(A)Alvin to Benny (B)Alvin to Charlie (C)Benny to Alvin
(D)Benny to Charlie (E)Charlie to Alvin
【Suggested Solution 1】
From the given information, we know the total number of chestnuts harvested by the
three farmers is 116 + 112 + 96 = 324 chestnuts. So at the end, each of them has
324  3 = 108 chestnuts.
1
We solve by backwards, someone gave of his chestnuts to another farmer. Before
4
3
a certain number of chestnuts was given out, there must be 108  = 144 chestnuts.
4
Meaning, this certain farmer must have given out 144 − 108 = 36 chestnuts. So, one
of the three farmers had only 108 − 36 = 72 chestnuts before receiving chestnuts
from another farmer. Therefore, the number of chestnuts that these three farmers had
before third round, in descending order, is 144, 108 and 72 chestnuts.
1
Before the second round, if 108 is after gave of his chestnuts to another farmer,
4
3
then he must have 108  = 144 chestnuts before given out. Then at least one have
4
1
144 chestnuts original, this is impossible. Hence 72 is after gave of his chestnuts
4
3
to another farmer, then he must have 72  = 96 chestnuts before given out, and
4
another farmer must have 108 − 24 = 84 chestnuts before receiving chestnuts from
him. Hence before second round, these three farmers had at that particular situation,
in descending order, is 144, 96 and 84 chestnuts.
1
Before the second round, if 144 is after gave of his chestnuts to another farmer,
4
3
then he must have 144  = 192 chestnuts before given out, this is impossible.
4
1
Hence 84 is after gave of his chestnuts to another farmer, then he must have
4
3
84  = 112 chestnuts before given out, and another farmer must have 144 − 28 = 116
4

365
chestnuts before receiving chestnuts from him.
Hence, in first round farmer Benny gave to farmer Alvin, in second round farmer
Charlie gave to farmer Benny, in third round farmer Alvin gave to farmer Charlie.
【Suggested Solution 2】
Put per 4 pieces of chestnuts in a bag, then Alvin have 29 bags, Benny have 28 bags
and Charlie have 24 bags. So at the end, each of them has (29 + 28 + 24)  3 = 27
bags chestnuts.
Since 29 is an odd number, so farmer Alvin cannot be first or second farmer who
1 1
gave of his chestnuts to another farmer, he must be the third farmer who gave
4 4
of his chestnuts to another farmer, otherwise after that Benny or Charlie cannot have
1
integer bags of chestnuts. If Charlie is the first farmer who gave of his chestnuts
4
to another farmer, then Alvin have 35 bags or Benny have 34 bags, in any case after
that nor Alvin or Benny cannot be 27 bags, this is impossible.
1
If Benny is the first farmer who gave of his chestnuts to farmer Charlie, then
4
Charlie have 31 bags and Alvin have 29 bags, in any case after second round nor
Alvin or Charlie will be 27 bags, this is impossible. If Benny is the first farmer who
1
gave of his chestnuts to farmer Alvin, then Alvin have 36 bags and Charlie have
4
24 bags. For let at the end each of them has 27 bags, the second round must farmer
1 1
Charlie gave of his chestnuts to farmer Benny and then farmer Alvin gave of
4 4
his chestnuts to farmer Charlie.
Hence, in first round farmer Benny gave to farmer Alvin, in second round farmer
Charlie gave to farmer Benny, in third round farmer Alvin gave to farmer Charlie.
Answer: (C)
3. Peter will be attending a two-day summer camp, where he plans to wear different
colour of socks in both days. He keeps all of his socks in a messy non-transparent
drawer under his bed and it is known that he has 8 black socks and 6 white socks.
If he takes out 1 sock at a time, how many socks does he need to get to be sure
that he gets 2 pairs of socks of different colour?
(A)4 (B)6 (C)8 (D)10 (E)12
【Suggested Solution 1】
He needs 3 attempts to gets a pair of socks in same colour. If the works case scenario
is he takes a 2 black socks and 1 white sock in the first 3 tries, as worst case scenario
is to take 6 black socks more in the next 6 tries and the 10th one is guaranteed to be
sure gets 2 pairs of socks in different colour. Hence, he needs 10 attempts.
【Suggested Solution 2】
The worst case scenario is to take 8 black socks in the first 8 tries, and need 2 more
tries guaranteed to gets 1 pairs of white socks. Hence, he needs 10 attempts.
Answer: (D)

366
4. In the sequence,
1, 2, 4, 8, 6, …
Starting from the second term, each term is formed by multiplying the previous term
by 2 and then taking the remainder when divided by 10. What is the 2022nd term?
(A)1 (B)2 (C)4 (D)6 (E)8
【Suggested Solution】
The sequence is 1, 2, 4, 8, 6, 2, 4, 8, 6 …etc and the pattern (2,4,8,6) is repeated.
Thus when we remove the 1 we find that when 2021 divided by 4 the remainder is 1
and thus the 2022nd term is 2.
Answer: (B)
5. A tetrahedron is a regular pyramid made up of four
equilateral triangles. It is possible to paint each face with a
different colour --- red, green, yellow and blue. Five
different views of the tetrahedron are shown below. Which
among the choices is inconsistent with the other four?
red green

(A) (B) (C) (D) (E)

green green yellow yellow green


blue red red blue yellow
red yellow blue green red
【Suggested Solution】
(B) (E)

green yellow green


red green yellow
yellow red red
If we rotate option (B), then we will find it contradicts (E). So the other three options
are correct.
From (A) we know the bottom face is painted yellow colour, then from other three
vertices its view as following:
(I)

green yellow yellow green


blue blue red red
red green blue yellow

(II) (III) (I) = (D) (II) = (C) (III) = (B)


Hence, the view of option (E) is inconsistent with the other four.
(E)
Answer:

367
6. One day, Robert went to a library 1600 meters away from home to meet with his
classmates at 13 : 20. Robert leaves home at 13 : 00, walking at a constant speed
of 80 meters per minute. At exactly 13 : 05, Robert's mother noticed that he left
his wallet at home, so she chased him walking at a speed of 120 meters per
minute. Robert only realized he left his wallet at home at 13 : 10, so he walked
back casually at a speed of 80 meters per minute. Robert and his mother met
halfway through. After taking his wallet, he again walked back towards the
library at a speed of 80 meters per minute. If we ignore the time Robert took to
get the wallet from his mother, how much time, in minutes, did he waste?
【Suggested Solution】
We noticed that at 13:10, Robert had left home 80 10 = 800 meters and his mother
was 120  5 = 600 meters away, so the distance between them is 800 − 600 = 200
meters. The time taking Robert and his mother to meet and turn over the wallet from
this point is 200  (80 + 120) = 1 minute. And Robert went back to the library after
meeting his mother, that is, walking the same distance and time again, then the time
wasted round trip should be 1 2 = 2 minutes.
Answer:2 minutes.
7. Mr. Andrew's office is located in City T, while he lives in City S. Every day after
timing-off from work, he takes the same train to return to his home city. The train
station in city S is still some distance away from his home, and his wife travels
by car to the train station to pick him up. It is known that the train and his wife
arrive at the train station at the same time. One day, due to some reasons, his wife
left their house 30 minutes later than usual. Upon arriving at the train station in
City S, Andrew noticed that his wife hasn’t arrived yet so he decided to walk
home. On his way walking home, he met his wife along the way and rode the car
immediately. Upon reaching home, Andrew observed that they arrived home 22
minutes later than usual. How long, in minutes, did Andrew walk?
【Suggested Solution】
If Mr. Andrew had been waiting at the train station at the beginning, his wife would
have spent the time from where she met Mr. Andrew to the train station and then
back to the place she met Mr. Andrew. In other words, if his wife drove to the train
station from where she met Mr. Andrew, the one-way journey would be 4 minutes.
So if Mr. A insists on waiting at the train station, he will be waiting for 30 − 4 = 26
minutes, and because Mr. Andrew is not willing to wait at all. It means that Mr. A
walked for 26 minutes.

Station Mr. Andrew’s home

Meeting place
Answer:26 minutes

368
8. There are four cards with digits 0, 1, 2 and 3 written on each of them. Choose
three different cards and arrange them to form all possible three-digit numbers,
where 0 is not allowed be leading digit. What is the sum of all those three-digit
numbers that are odd?
【Suggested Solution】
From the given information, there are only two even digits and two odd digits, we
have 2 way choose the unit digit, then we have 2 way choose the leading digit, we
have 2 way to arrange the tenth-digit. Hence we have 2  2  2 = 8 such 3-digit
numbers, we list down all of those 3-digit numbers: 103, 123, 201, 203, 213, 231, 301,
321, and their sum is 1696.
Answer:1696
9. Three positive 9-digit numbers A, B and C each contains all the digits from 1 to 9
exactly once. If A − B = C , then what is the largest possible value of integer C?
【Suggested Solution】
This problem is actually an interesting combination of 9 digits from 1 to 9. When
these 9 digits are arranged in order from largest to smallest; in which we represent it
as the 9-digit number A, and then the reverse of these 9-digit number as B, the
requirements of the problem can be met. The subtraction sentence is as follows:
987654321 − 123456789 = 864197532 . Thus, the largest value of C is 864197532.
Answer:864197532
10. In a line, there is at least one girl and at least one boy. Two children with exactly
4 or 7 other children between them are of the same gender. What is the maximum
number of children in this line?
【Suggested Solution】
Number the children 1, 2, 3 and so on in order. We first show that the line cannot
have 12 or more children. By symmetry, we may assume that #1 is a girl. By
hypothesis, #6 and #9 are also girls. From #6 we see that #11, #3, #8 must be girls.
Starting from #9, we see that #4, #12, #7, #2, #10 and #5 must also be girls. Now the
first 12 children are all girls. It follows that the remaining children are all girls. This
contradicts the condition that there is at least one boy. If #12 does not exist, then #7,
#2, #10 and #5 can be boys while the others are girls. It follows that the maximum
number of children in the line is 11. Answer:11 children
11. In the diagram below, ABC is an isosceles triangle with A
AB AC and BC 48 cm. If DE : DF = 5 : 7, what is
the length, in cm, of DC?
【Suggested Solution 1】
Since AB AC , we have EBD = FCD , then triangles
DE DB
BDE and CDF are similar, so = , DC = 28 cm. E
F
DF DC
【Suggested Solution 2】
B D C
Connect AD and let [*] denote the area of region *.
Since AB AC and the ratio of DE to DF is 5 : 7, [ABD] : [ADC] = 5 : 7.
7 7
Hence BD : DC = 5 : 7. So DC =  BC =  48 = 28 cm. Answer:28 cm
5+7 12

369
12. Numbers 1, 2, 3 and 4 are to be placed into the circles in the diagram below,
where each circle must be filled with exactly one number. If two circles
connected by a line should not contain the same number, find the total number of
ways in filling the diagram.

【Suggested Solution】
Look at the four circles at vertices of the square. If two circles at upper left and lower
right are filled with the same number, each of other two circles has three options. We
get 4  3  3 = 36 ways. If these two circles are filled with different number, each of
other two circles has two options, we get 4  3  2  2 = 48 ways. Totally we get 84
ways to fill the four circles of the square. Once the circles at the square vertices are
filled, each remaining circle has a number different to two distinct numbers, thus two
options. So there are 84  2  2  2  2 = 1344 ways to fill all circles.
Answer:1344 ways
13. How many cubes which are shown in Figures 1 and 2 are found inside the
rectangular pattern below?

Figure 1

Figure 2

370
【Suggested Solution】

Answer:6 cubes
14. A continuous look-and-say sequence is formed this way: Start with three 1’s;
read the terms as “three 1’s” and write down what you read as “31” at the end of
the sequence; read the next group of unread identical numbers as “one 3” and
write “13” at the end; continue this process to get an infinite sequence 1, 1, 1, 3,
1, 1, 3, 2, 1, ….
(a) What is the largest possible digit you could write down? Why? (5 points)
(b) What is the 20th term of this sequence? Why? (5 points)
(c) Will the number “333” appear in the sequence? Why or why not? (10 points)
【Suggested Solution】
(a) The largest would be 3. Otherwise, if there is a number more than 3, there will be
four equal numbers before this number. Among these four numbers, two numbers
describe a digit appears before, two numbers describe frequency. If consecutive
descriptions of a same digit would be described together, a contradiction.
(b) Write down the sequence as
1, 1, 1, 3, 1, 1, 3, 2, 1, 1, 3, 1, 2, 2, 1, 1, 3, 1, 1, 2,
2, 2, 1, 1, 3, 2, 1, 3, 2, 2, 1, 1, 3, …
so the 20 term is “2”.
th

(c) If there is “333” in the sequence, take the first appearance of “333”. Then one of
the first two 3’s describes frequency, implying there are three 3’s before, a
contradiction.
Answer:(a) 3 (b) 2
【Marking Scheme】
(a) Find the largest number is 3 given 2 points, with correct reason given 3 points.
(b) Find the 20th term is 2 given 2 points, with correct reason given 3 points.
(c) Take the first appearance of “333” given 2 points, proof there is a contradiction
given 8 points.

371
15. Divide the numbers 1, 2, …, 15 into five groups, such that each group contains
three numbers and sum of numbers in each group is 24. Please provide an
example.
【Suggested Solution】
We can write 24 as following:
24 = 1 + 8 + 15 = 1 + 9 + 14 = 1 + 10 + 13 = 1 + 11 + 12 = 2 + 7 + 15 = 2 + 8 + 14
= 2 + 9 + 13 = 2 + 10 + 12 = 3 + 6 + 15 = 3 + 7 + 14 = 3 + 8 + 13 = 3 + 9 + 12
= 3 + 10 + 11 = 4 + 5 + 15 = 4 + 6 + 14 = 4 + 7 + 13 = 4 + 8 + 12 = 4 + 9 + 11
= 5 + 6 + 13 = 5 + 7 + 12 = 5 + 8 + 11 = 5 + 9 + 10 = 6 + 7 + 11 = 6 + 8 + 10
=7+8+9
When we put {1, 8, 15} as a group, the we cannot choose these three numbers again.
24 = 2 + 9 + 13 = 2 + 10 + 12 = 3 + 7 + 14 = 3 + 9 + 12 = 3 + 10 + 11 = 4 + 6 + 14
= 4 + 7 + 13 = 4 + 9 + 11 = 5 + 6 + 13 = 5 + 7 + 12 = 5 + 9 + 10 = 6 + 7 + 11
When we put {2, 9, 13} as a group, the we cannot choose these three numbers again.
24 = 3 + 7 + 14 = 3 + 10 + 11 = 4 + 6 + 14 = 5 + 7 + 12 = 6 + 7 + 11
There are only one 4, one 5 remained, so we must choose {4, 6, 14}, {5, 7, 12}.
Then the last one is 24 = 3 + 10 + 11.
Hence, one of example is: {1, 8, 15}, {2, 9, 13}, {3, 10, 11}, {4, 6, 14}, {5, 7, 12}.

【Marking Scheme】
⚫ Write all the three numbers with sum 24................................................ 5 points
⚫ Choose any one group and deleted the duplicate numbers .................... 5 points
⚫ Choose any one group remained and deleted the duplicate numbers .... 5 points
⚫ Find a correct example ........................................................................... 5 points

372
International Mathematics Assessments for Schools

2022 / 2023 MIDDLE PRIMARY DIVISION FIRST ROUND PAPER


Time allowed:75 minutes

When your teacher gives the signal, begin working on the problems.

INSTRUCTION AND INFORMATION


GENERAL
1. Do not open the booklet until told to do so by your teacher.
2. No calculators, slide rules, log tables, math stencils, mobile phones or other
calculating aids are permitted. Scribbling paper, graph paper, ruler and compasses
are permitted, but are not essential.
3. Diagrams are NOT drawn to scale. They are intended only as aids.
4. There are 20 multiple-choice questions, each with 5 choices. Choose the most
reasonable answer. The last 5 questions require whole number answers between
000 and 999 inclusive. The questions generally get harder as you work through the
paper. There is no penalty for an incorrect response.
5. This is a mathematics assessment, not a test; do not expect to answer all questions.
6. Read the instructions on the answer sheet carefully. Ensure your name, school
name and school year are filled in. It is your responsibility that the Answer Sheet
is correctly coded.

THE ANSWER SHEET


1. Use only pencils.
2. Record your answers on the reverse side of the Answer Sheet (not on the question
paper) by FULLY filling in the circles which correspond to your choices.
3. Your Answer Sheet will be read by a machine. The machine will see all markings
even if they are in the wrong places. So please be careful not to doodle or write
anything extra on the Answer Sheet. If you want to change an answer or remove
any marks, use a plastic eraser and be sure to remove all marks and smudges.

INTEGRITY OF THE COMPETITION


The IMAS reserves the right to re-examine students before deciding whether to
grant official status to their scores.

373
2022 / 2023 MIDDLE PRIMARY DIVISION FIRST ROUND PAPER
Questions 1-10, 3 marks each
1. Among the 16 hexagons in the diagram below, what is the maximum number of
hexagons that contain the same set of symbols, in the same number, not
necessarily arranged in the same order?

(A)2 (B)3 (C)4 (D)5 (E)6

2. There are three kinds of toys and each one has a fixed price. The sums of the
prices for each row and column are given, except for the last row and column, as
shown in the diagram below.

$180

$190

$240 $130 *
What is the value of the symbol *?
(A)$160 (B)$170 (C)$200 (D)$210 (E)$220

374
MP 2
3. What is the value of the expression (1234 + 2341 + 3412 + 4123)  5 ?
(A)1111 (B)1234 (C)2022 (D)2222 (E)2468

4. Compute all possible products of two one-digit positive integers to construct a 9


by 9 multiplication table. How many two-digit numbers can be obtained in the
table?
(A)14 (B)23 (C)58 (D)67 (E)81

5. How many different triangles can you find in the diagram below?

(A)50 (B)42 (C)30 (D)28 (E)24

6. Eight friends dined at a restaurant and decided to equally divide the amount of
money that had to be paid. Because Tom left his money at home, his other seven
friends paid an extra $25 each to cover his share. How much was the total bill of
the eight friends?
(A)$175 (B)$200 (C)$800 (D)$1225 (E)$1400

7. The diagram below shows a strange dartboard. If the dart hits one zone, the
player scores the number of points written in that zone. If the dart hits the central
point, it scores 90; if the dart hits the segment between two zones, it scores the
higher score of the two zones.

30 50

70 0

20 80
10 90

Using three darts at a time, how many different ways are there of obtaining a
total score of exactly 100? Note that all darts must be on the dartboard.
(A)4 (B)5 (C)6 (D)7 (E)8

375
MP 3
8. There are three segments OA, OB and OC such that AOB = 22 and
BOC = 56 , as shown in the diagram below.
C
D

O A

If AOC is split in two equal parts by the segment OD, what is the measure, in
degrees, of BOD ?
(A)17 (B)18 (C)19 (D)20 (E)21

9. Suppose that your height this year is 10% more than it was last year, and last
year your height was 20% more than it was the year before. During the past two
years, your height has increased by what percentage?
(A)30% (B)31% (C)32% (D)34% (E)40%

10. The number 638*977 is a multiple of 7. How many different possible values of
the middle digit represented by the symbol * are there?
(A)1 (B)2 (C)3 (D)4 (E)5

Questions 11-20, 4 marks each


11. The digits 1, 2, 3, 4, 5, 6, 7, 8 and 9 are to be placed in the nine circles in the
diagram exactly once such that the sums along each of the three lines are all
equal. If each circle can only be filled by exactly one digit, what is the smallest
value possible for this sum?

(A)15 (B)16 (C)17 (D)18 (E) 19

376
MP 4
12. Two squares of side 8 cm intersect such that the corner of one square coincides
with the centre of the other square, as shown in the diagram below.

B C

A
D

E
If it is known that BC = DE , what is the area, in cm2, of the shaded region?
(A)2 (B)8 (C)12 (D)16 (E)32

13. At a certain store, the cost of 3 avocados is the same as that of 5 oranges, the cost
of 4 oranges is the same as that of 11 apples, and the cost of 5 apples is the same
as that of 24 bananas.

=
=

=
What is the number of bananas that can be bought with the money for buying two
avocados?
(A)22 (B)44 (C)264 (D)42 (E)48

377
MP 5
14. The diagram below shows four diagrams, each consisting of eight unit squares,
and a V-shaped piece consisting of three unit squares.
V-shaped

A B C D
We use three copies of the V-shaped piece to try to cover each of the diagrams. The
pieces may be turned or flipped, and they are allowed to overlap.
Which diagram CANNOT be covered?
(A)A only (B)B only (C)C only
(D)D only (E)All of them can be covered

15. How many odd multiples of 3 are there between 1800 and 2022?
(A)36 (B)37 (C)38 (D)74 (E)75

16. The average of 50 numbers is 38. If two of these numbers, namely 37 and 63, are
discarded, what will be the average of the remaining numbers?
(A)36 (B)36.5 (C)37 (D)37.5 (E)38

17. A rectangle ABCD is divided into nine smaller rectangles as shown in the
diagram below. The number written inside a small rectangle denotes its perimeter,
in cm. Notice that the perimeter of one of the small rectangles has an unknown
value, *. If the perimeter of rectangle ABCD is 50 cm, what is the value of the
symbol *? Note that the diagram is not drawn to scale.
(A)20 (B)21 (C)23 (D)26 (E)29
A B
19

15 17 7

*
D C

18. The digits 1, 2, 3, 4, 5, 6, 7, 8 and 9 are arranged in a circle, where every chain of
three consecutive digits in the clockwise direction forms a three-digit number.
What is the maximum value of the sum of all nine such numbers?
(A)4551 (B)4662 (C)4773 (D)4884 (E)4995

378
MP 6
19. For her birthday, Alice brought a box of candies, which she wanted to share with
her classmates. If she gave each child 2 candies, there would be 6 candies left in
the box. If she gave each child 3 candies, then exactly 5 children would be left
without candies. How many candies were there in the box?
(A)22 (B)28 (C)33 (D)42 (E)48

20. The diagram below shows the net of an unfolded cube.

Which of the five cubes below it is not a made-up version of the flattened one?
(A) (B) (C) (D) (E)

Questions 21-25, 6 marks each


21. The diagram below shows a partially built 6  6  6 cube. How many unit cubes
are missing?

379
MP 7
22. A 9  7 grid of squares with one shaded square is shown in the diagram below.

How many different rectangles bounded by the gridlines can we construct so that
all rectangles contain the shaded square? (Note: a square is also a rectangle.)

23. When one of ten consecutive positive integers is removed, the sum of the
remaining nine is 2022. Which number is removed?

24. The ages of Tom and his two grandchildren add up to 80 years. The ages of the
two grandchildren are the digits of the grandpa's age. How old is grandpa Tom?

25. Each number in a 4  5 table is 0, 1 or 2. The sum of the numbers in each row
and in each column is a multiple of 3. What is the maximum number of 1’s in this
table?

***

380
Solution to
2022/2023 International Mathematics Assessment for Schools
Round 1 of Middle Primary Division
1. Among the 16 hexagons in the diagram below, what is the maximum number of
hexagons that contain the same set of symbols, in the same number, not
necessarily arranged in the same order?

(A)2 (B)3 (C)4 (D)5 (E)6


【Solution】

:1 :0 :2 :1
:1 :4 :2 :1
:1 :1 :1 :2
:1 :0 :0 :0
:2 :1 :1 :2
:1 :1 :1 :1
:1 :2 :3 :1
:2 :1 :1 :0
:0 :1 :1 :3
:2 :1 :0 :1
:2 :2 :1 :1
:1 :2 :1 :1
:1 :1 :2 :2
:1 :1 :0 :1
:1 :0 :2 :1
:1 :3 :0 :2
:1 :1 :2 :2
:2 :0 :2 :0
:0 :2 :0 :1
:2 :0 :2 :1

381
Count the number of each symbol in each hexagon, as shown on the diagram above.
So, there are 4 hexagons (marked yellow) contain the same symbols.
Answer:(C)
2. There are three kinds of toys and each one has a fixed price. The sums of the
prices for each row and column are given, except for the last row and column, as
shown in the diagram below.

$180

$190

$240 $130 *
What is the value of the symbol *?
(A)$160 (B)$170 (C)$200 (D)$210 (E)$220
【Solution 1】
The total price of the toys in the first column from left to right, we know each
robot is worth $240  3 = $80 .
Then, from the total price of the toys in the first row from top to bottom, we know
each car is worth ($180 − $80)  2 = $50 .
And, from the total price of the toys in the second row from top to bottom, we
know each bear is worth $190 − 2  $80 = $30 .
So, the value of * is $80 + $50 + $30 = $160 .
【Solution 2】
The total price of the toys in the second column from left to right, we know two
cars and one bear are totally worth $130.
Then, from the total price of the toys in the second row from top to bottom, we
know two robots and one bear are totally worth $190.
And the value of * is the total price of one car, one car, one robot and one bear, so
we know the value of * is ($130 + $190)  2 = $160 .
Answer:(A)
3. What is the value of the expression (1234 + 2341 + 3412 + 4123)  5 ?
(A)1111 (B)1234 (C)2022 (D)2222 (E)2468
【Solution】
Since each of the digits 1, 2, 3 and 4 appears exactly once in each place, the sum is

382
equal to (1 + 2 + 3 + 4) 1111 = 10 1111 and the quotient is equal to
10 1111  5 = 2  1111 = 2222 .
Answer:(D)
4. Compute all possible products of two one-digit positive integers to construct a 9
by 9 multiplication table. How many two-digit numbers can be obtained in the
table?
(A)14 (B)23 (C)58 (D)67 (E)81
【Solution 1】
The multiplication table has 9 rows and 9 columns, there are 81 numbers in the table.
× 1 2 3 4 5 6 7 8 9
1 1 2 3 4 5 6 7 8 9
2 2 4 6 8 10 12 14 16 18
3 3 6 9 12 15 18 21 24 27
4 4 8 12 16 20 24 28 32 36
5 5 10 15 20 25 30 35 40 45
6 6 12 18 24 30 36 42 48 54
7 7 14 21 28 35 42 49 56 63
8 8 16 24 32 40 48 56 64 72
9 9 18 27 36 45 54 63 72 81
All numbers appear in this table, along with some which have less than 2 digits. Now
there are 9 distinct numbers on the diagonal. But the products 11 = 1 , 2  2 = 4 and
3  3 = 9 are one-digit numbers. Because the table is symmetric with respect to the
diagonal, and 1 2 , 1  3 , …, 1  9 , 2  3 and 2  4 are also one-digit numbers.
There are 3 + 10  2 = 23 one-digit numbers on the multiplication table in total. Hence
there are 81 − 23 = 58 two-digit numbers which can be obtained in the table.
【Solution 2】
The multiplication table has 9 rows and 9 columns, there are 81 numbers in the table.
× 1 2 3 4 5 6 7 8 9
1 1 2 3 4 5 6 7 8 9
2 2 4 6 8 10 12 14 16 18
3 3 6 9 12 15 18 21 24 27
4 4 8 12 16 20 24 28 32 36
5 5 10 15 20 25 30 35 40 45
6 6 12 18 24 30 36 42 48 54
7 7 14 21 28 35 42 49 56 63
8 8 16 24 32 40 48 56 64 72
9 9 18 27 36 45 54 63 72 81
Count the number of two-digit numbers directly, because the table is symmetric with
respect to the diagonal, there are (5 + 6 + 5 + 4 + 3 + 2 + 1)  2 + 6 = 58 two-digit
numbers which can be obtained in the table.
Answer:(C)

383
5. How many different triangles can you find in the diagram below?

(A)50 (B)42 (C)30 (D)28 (E)24


【Solution 1】
All triangles in this diagram have a common vertex, which is the topmost point. Any
triangle is made of three points: one being the topmost point and the other two points
are chosen on one of the two parallel horizontal lines.
On each parallel line there are 7 points. If we choose the leftmost point on the line,
then we can pair it with any of the 6 remaining points on the line to form a triangle. If
we choose the next point, then we can pair it with other 5 remaining points to form a
triangle that was not considered before, and so on. If we proceed like this, then the
number of triangles in the diagram are 2  (6 + 5 + 4 + 3 + 2 + 1) = 42 .
【Solution 2】
Label the 12 smaller areas inside the triangles as A, B, C, D, E, F, G, H, I, J, K and L
respectively as shown below.

A C E G I K
B D F H J L
From here – using only one segment – we can form 6 triangles namely – A, C, E, G, I
and K.
Using two smaller areas we have 11 namely – AC, CE, EG, GI, IK, AB, CD, EF, GH,
IJ and KL.
Using three smaller areas, we have 4 namely – ACE, CEG, EGI and GIK.
Using four smaller areas, we have 8 namely – ACEG, CEGI, EGIK, ABCD, CEDF,
EGFH, GIHJ and KIJL.
Using five smaller areas, we 2 namely have ACEGI and CEGIK.
Using six smaller areas, we have 5 namely ACEGIK, ABCDEF, CDEFGH, EFGHIJ
and GHIJKL.
Using seven smaller areas, we can’t make any triangles.
Using eight smaller areas, we 3 namely have ABCDEFGH, CDEFGHIJ and
EFGHIJKL.
Using nine smaller areas, we can’t make any triangles.
Using ten smaller areas, we have 2 namely ABCDEFGHIJ and CDEFGHIJKL.
Using eleven smaller areas, we can’t make any triangles.
Using all twelve – we can have 1 namely ABCDEFGHIJ.
So, we have in total 6 + 11 + 4 + 8 + 2 + 5 + 3 + 2 + 1 = 42 triangles.
Answer: (B)

384
6. Eight friends dined at a restaurant and decided to equally divide the amount of
money that had to be paid. Because Tom left his money at home, his other seven
friends paid an extra $25 each to cover his share. How much was the total bill of
the eight friends?
(A)$175 (B)$200 (C)$800 (D)$1225 (E)$1400
【Solution 1】
Tom’s share is $25  7 = $175 , so the total bill was 8  $175 = $1400 .
【Solution 2】
The amount of money paid by each of seven people is $25 more than the amount of
money paid by each of eight people, so the amount of money paid by each of eight
people is $25  7 = $175 from the figure below.
$25

Thus the total bill was 8  $175 = $1400 .


【Solution 3】
Denote by $x the sum equally divided among the eight friends. The total bill was
S = 8 x . On the other side, the entire bill was initially split between only seven
friends. Thus S = 7( x + 25) = 7 x + 175 . From 8x = 7 x + 175 , we get that x = 175 .
Therefore, the total bill was 8  175 = 1400 .
Answer:(E)
7. The diagram below shows a strange dartboard. If the dart hits one zone, the
player scores the number of points written in that zone. If the dart hits the central
point, it scores 90; if the dart hits the segment between two zones, it scores the
higher score of the two zones.

30 50

70 0

20 80
10 90

Using three darts at a time, how many different ways are there of obtaining a
total score of exactly 100? Note that all darts must be on the dartboard.
(A)4 (B)5 (C)6 (D)7 (E)8

385
【Solution】
If the lowest score is 0, then there are 4 ways:
0 + 10 + 90 , 0 + 20 + 80 , 0 + 30 + 70 and 0 + 50 + 50 .
If the lowest score is 10, then there are 2 ways:
10 + 10 + 80 and 10 + 20 + 70 .
If the lowest score is 20, then there is 1 way:
20 + 30 + 50 .
If the lowest score is at least 30, then the score is could be 30 + 30 + 30 = 90 or at
least 30 + 30 + 50  100 .
So, there are 4 + 2 + 1 = 7 different ways of scoring a total of 100.
Answer:(D)
8. There are three segments OA, OB and OC C
such that AOB = 22 and BOC = 56 ,
as shown in the diagram below. D
B
If AOC is split in two equal parts by the
segment OD, what is the measure, in
degrees, of BOD ? O A
(A)17 (B)18 (C)19 (D)20 (E)21
【Solution】
We have AOC = AOB + BOC = 22 + 56 = 78 .
1 78
Hence DOC = AOC = = 39 and BOD = BOC − DOC = 56 − 39 = 17 .
2 2
Answer:(A)
9. Suppose that your height this year is 10% more than it was last year, and last year
your height was 20% more than it was the year before. During the past two years,
your height has increased by what percentage?
(A)30% (B)31% (C)32% (D)34% (E)40%
【Solution 1】
My height this year is 1.1 times my height last year, and my height last year is 1.2
times my height the year before last year. So my height this year is 1.1 1.2 = 1.32
times my height the year before last year, i.e., during the last two years, my height has
increased by 32%.
【Solution 2】
Suppose my height the year before last year was 100 cm. Since last year my height
was 20% more than it was the year before, my height last year was 120 cm. Since my
height this year is 10% more than it was last year, and my height this year is 132 cm.
So, during the last two years, my height has increased by 32 cm, i.e. increased by 32%.
Answer:(C)
10. The number 638*977 is a multiple of 7. How many different possible values of
the middle digit represented by the symbol * are there?
(A)1 (B)2 (C)3 (D)4 (E)5

386
【Solution 1】
Observe that
6380977 = 911568  7 + 1
6381977 = 911711 7
6382977 = 911853  7 + 6
6383977 = 911996  7 + 5
6384977 = 912139  7 + 4
6385977 = 912282  7 + 3
6386977 = 912425  7 + 2
6387977 = 912568  7 + 1
6388977 = 912711 7
6389977 = 912853  7 + 6
So there are 2 possible values of the middle digit represented by the symbol *.
【Solution 2】
Since 63 and 77 are multiples of 7, we can reduce the given multiple of 7 to 8*9.
Now we take * = 0 first. Since 809 = 7  115 + 4 , 809 is not a multiple of 7.
Next, we take * = 1. Since 819 = 7 117 , 819 is a multiple of 7 and we can judge the
next multiple of 7 is 819 + 70 = 889 = 7  127 . So there are 2 possible values of the
middle digit represented by the symbol *.
【Solution 3】
638*977 is a multiple of 7 if and only if 977 − 38* + 6 = 983 − 38* is a multiple of 7.
Since 983 − 389 = 594 and 983 − 380 = 603 , the value of 983 − 38* is between
594 and 603. Since only 595 and 602 are multiples of 7 in this interval, there are two
possible value of 983 − 38* , namely 595 and 602, so that the middle digit
represented by the symbol * is 8 and 1, respectively.
Answer:(B)
11. The digits 1, 2, 3, 4, 5, 6, 7, 8 and 9 are to be placed in the nine circles in the
diagram exactly once such that the sums along each of the three lines are all
equal. If each circle can only be filled by exactly one digit, what is the smallest
value possible for this sum?

(A)15 (B)16 (C)17 (D)18 (E)19

387
【Solution】
In order to get the smallest possible value for the sum on
all three lines, we must place the smallest digits (1, 2 and 3) 1
in the three corners, as those digits will be reused. Once we
placed all the digits in the provided circles, the three equal 6 9
sums on the lines will add up to 51 = 45 (the sum of all 9
digits) +(1 + 2 + 3) . Therefore, the sum of digits on one line 7 5
will be 51  3 = 17 . By knowing this fact, we can now fill
3 4 8 2
in the other empty circles as shown in the diagram.
Answer: (C)
12. Two squares of side 8 cm intersect such that the corner of one square coincides
with the centre of the other square, as shown in the diagram below.

B C

A
D

If it is known that BC = DE , what is the area, in cm2, of the shaded region?


(A)2 (B)8 (C)12 (D)16 (E)32
【Solution】
Connect AC. Then the area of the shaded region is equal to B C
the sum of the areas of triangle ABC and ACD.
In triangle ABC and ACD, the length of the altitudes from
A is half of the side length of the square. So the sum of the A
D
areas of triangle ABC and ACE is
1 8 1 8
 BC  +  CD  = 2( BC + CD) = 2( DE + CD) = 2CE = 16 E
2 2 2 2
Therefore, the area of the shaded region is16 cm2.
Answer:(D)
【Note】
In fact, we don’t need the condition “ BC = DE ”. As show
in the diagram on the right, connect AC and AE, then we B C
can observe that the area of triangle ABC and ADE are the
same, since AE = AC , ACB = 45 = AED and
EAD = EAC − DAC = 90 − DAC = DAB − DAC A
D
= CAB . Hence triangle ABC and ADE are congruent.
Therefore, the area of the shaded region is a quarter of the E
1
area of the square, that is  8  8 = 16 cm2.
4

388
13. At a certain store, the cost of 3 avocados is the same as that of 5 oranges, the cost
of 4 oranges is the same as that of 11 apples, and the cost of 5 apples is the same
as that of 24 bananas.

=
=

=
What is the number of bananas that can be bought with the money for buying two
avocados?
(A)22 (B)44 (C)264 (D)42 (E)48
【Solution 1】
Let v, o, a and b be the prices for an avocado, an orange, an apple and an banana,
respectively. From the given condition we have that: 3v = 5o , 4o = 11a and
5a = 24b . Therefore, 12v = 4  3v = 4  5o = 5  4o = 5  11a = 11 5a = 11 24b .
Divide by 6 to get that 2v = 11 24b  6 = 44b , so 44 bananas that can be bought
with the money from two avocados.
【Solution 2】
Since the largest common multiple of 4, 5 and 11 is 220, we may let the price of an
avocado be 220 dollars, then an orange costs 220  3  5 = 132 dollars, and then an
apple costs 132  4  11 = 48 dollars and a banana costs 48  5  24 = 10 dollars.
Since two avocados cost 220  2 = 440 dollars, 440  10 = 44 bananas that can be
bought with the money from two avocados.
Answer: (B)
14. The diagram below shows four diagrams, each consisting of eight unit squares,
and a V-shaped piece consisting of three unit squares.
V-shaped

A B C D

389
We use three copies of the V-shaped piece to try to cover each of the diagrams. The
pieces may be turned or flipped, and they are allowed to overlap.
Which diagram CANNOT be covered?
(A)A only (B)B only (C)C only
(D)D only (E)All of them can be covered
【Solution】
Following are the ways to cover diagram (A), (B) and (D) by using three copies of
the V-shaped piece.

For diagram (C), one V-shaped piece can only cover exactly one of the green squares.
There are four green squares, so it is impossible to cover diagram (C) by using three
copies of the V-shaped piece.

Answer: (C)
15. How many odd multiples of 3 are there between 1800 and 2022?
(A)36 (B)37 (C)38 (D)74 (E)75
【Solution 1】
Since 1800 = 3  600 and 2022 = 3  674 , there are (674 − 600)  2 = 37 odd
multiples of 3 between 1800 and 2022.
【Solution 2】
The odd multiples of 3 are there between 1800 and 2022, the smallest is 1803 and the
largest is 2019, there are (2019 − 1803)  6 + 1 = 37 numbers.
Answer: (B)
16. The average of 50 numbers is 38. If two of these numbers, namely 37 and 63, are
discarded, what will be the average of the remaining numbers?
(A)36 (B)36.5 (C)37 (D)37.5 (E)38
【Solution 1】
The sum of all original 50 numbers is 50  38 = 1900 , so the sum of all 48 numbers is
1900 − 37 − 63 = 1800 after the two number being discarded. Hence the average of
the remaining numbers is 1800  48 = 37.5 .

390
【Solution 2】
We can observe that the area of the shaded region is 25 − 1 = 24 from the figure
below, so the average of the 48 remaining numbers is decreased by 24  48 = 0.5 , i.e.
their average is 38 − 0.5 = 37.5 .

25

38 1

48 Answer:(D)
17. A rectangle ABCD is divided into nine smaller rectangles as shown in the
diagram below. The number written inside a small rectangle denotes its perimeter,
in cm. Notice that the perimeter of one of the small rectangles has an unknown
value, *. If the perimeter of rectangle ABCD is 50 cm, what is the value of the
symbol *? Note that the diagram is not drawn to scale.
(A)20 (B)21 (C)23 (D)26 (E)29
A B
19

15 17 7

*
D C
【Solution】
From the diagram, one can see that the perimeter of the large rectangle is equal to the
sum of the perimeters of the four outer small rectangles for which the perimeter is
given minus the perimeter of the middle small rectangle. Therefore, we have that
19 + 7 + * + 15 − 17 = 50
* = 26
Answer: (D)
18. The digits 1, 2, 3, 4, 5, 6, 7, 8 and 9 are arranged in a circle, where every chain of
three consecutive digits in the clockwise direction forms a three-digit number.
What is the maximum value of the sum of all nine such numbers?
(A)4551 (B)4662 (C)4773 (D)4884 (E)4995

391
【Solution】
Each digit appears once as the hundreds digit, once as the tens digit and once as the
units digit of a three-digit number. Hence the sum of all nine numbers is constant and
equal to (100 + 10 + 1)(1 + 2 + 3 + 4 + 5 + 6 + 7 + 8 + 9) = 111 45 = 4995 .
Answer: (E)
19. For her birthday, Alice brought a box of candies, which she wanted to share with
her classmates. If she gave each child 2 candies, there would be 6 candies left in
the box. If she gave each child 3 candies, then exactly 5 children would be left
without candies. How many candies were there in the box?
(A)22 (B)28 (C)33 (D)42 (E)48
【Solution 1】
Let n be the number of Alice’s colleagues. If she gave each child 2 candies, there
would be 6 candies left. Then, the number of candies is 6 plus twice the number of
colleagues. If she would give each child 3 candies each, then exactly 5 children
would be left without candies, that is Alice gave exactly 3 candies to n − 5 colleagues.
From the two relations, we get that 6 + 2n = 3n − 15 , whence n = 21.
Therefore, there are 2  21 + 6 = 3  21 − 15 = 48 candies in the box.
【Solution 2】
Gave each child 2 candies, there would be 6 candies left in the box.
Gave each child 3 candies each, exactly 5 children would without candies, that is 15
candies short.
From these two cases, the number of Alice’s colleagues is (6 +15)  (3 −1) = 21 .
Therefore, there are 2  21 + 6 = 3  21 − 15 = 48 candies in the box.
Answer:(E)
20. The diagram below shows the net of an unfolded cube.

Which of the five cubes below it is not a made-up version of the flattened one?
(A) (B) (C) (D) (E)

392
【Solution】
The cube is the diagram below:

Option (A) is not made-up version of the flattened one. It should be the following
diagram:

The faces shown in option (B) is the three squares with red edges:

The faces shown in option (C) is the three squares with red edges:

The faces shown in option (D) is the three squares with red edges:

393
The faces shown in option (E) is the three squares with red edges:

Answer: (A)
21. The diagram below shows a partially
built 6  6  6 cube. How many unit
cubes are missing?

【Solution 1】
Top
Left Back

Right

Front

Bottom
Observe that there are 6  6 = 36 unit cubes in each level of the 6  6  6 cube.
In the top level, there are only 21 unit cubes, so 36 − 21 = 15 unit cubes are missing
in this level.
In the second level from top to bottom, there are 7 unit cubes more than the top level.
Thus there are 21 + 7 = 28 unit cubes, so 36 − 28 = 8 unit cubes are missing in this
level.
In the third level from top to bottom, there are 2 unit cubes more than the second
level. Thus there are 28 + 2 = 30 unit cubes, so 36 − 30 = 6 unit cubes are missing
in this level.

394
In the fourth level from top to bottom, there are 2 unit cubes more than the third level.
Thus there are 30 + 2 = 32 unit cubes, so 36 − 32 = 4 unit cubes are missing in this
level.
In the fifth level from top to bottom, there is 1 unit cube more than the fourth level.
Thus there are 32 + 1 = 33 unit cubes, so 36 − 33 = 3 unit cubes are missing in this
level.
Hence the number of missing unit cubes in total is 15 + 8 + 6 + 4 + 3 = 36 .
【Solution 2】
Observe that there are 6  6 = 36 unit cubes in each level of the 6  6  6 cube.
In the first level from front to back, there are only 22 unit cubes, so 36 − 22 = 14
unit cubes are missing in this level.
In the second level from front to back, there is 1 unit cubes more than the first level.
Thus there are 22 + 1 = 23 unit cubes, so 36 − 23 = 13 unit cubes are missing in this
level.
In the third level from front to back, there are 7 unit cubes more than the second level.
Thus there are 23 + 7 = 30 unit cubes, so 36 − 30 = 6 unit cubes are missing in this
level.
In the fourth level from front to back, there are 3 unit cubes more than the third level.
Thus there are 30 + 3 = 33 unit cubes, so 36 − 33 = 3 unit cubes are missing in this
level.
Hence the number of missing unit cubes in total is 14 + 13 + 6 + 3 = 36 .
【Solution 3】
Observe that if we add 7 unit cubes to the top level, 2  2 = 4 unit cubes to the second
level, 2  3 = 6 unit cubes to the third level, 1 4 = 4 unit cubes to the fourth level
and 3  5 = 15 unit cubes to the fifth level, then we can get a 6  6  6 cube.
Hence the number of missing unit cubes in total is 7 + 4 + 6 + 4 + 15 = 36 .
Answer: 036
22. A 9  7 grid of squares with one shaded square is shown in the diagram below.

How many different rectangles bounded by the gridlines can we construct so that
all rectangles contain the shaded square? (Note: a square is also a rectangle.)

395
【Solution】
A rectangle is constructed by picking four sides. We must count the number of ways
we can choose each side so that the shaded square stays inside the rectangle, and then
the product of these four numbers will be the required number of different rectangles.
There are 2 ways to choose the upper boundary of the rectangle and 3 ways to choose
the right boundary. There are 8 ways to choose the lower boundary and 5 ways to
choose the left boundary. Their product is 2  3  8  5 = 240 ways.
Answer:240
23. When one of ten consecutive positive integers is removed, the sum of the
remaining nine is 2022. Which number is removed?
【Solution 1】
Let the first of the ten consecutive numbers be x and let the number removed be
x + y , 0  y  9 . The sum of the ten numbers is 10 x + 45 so that
10x + 45 − x − y = 2022 . This simplifies to 9x = 1977 + y . When 1977 divided by 9,
the remainder is 6. Hence we must have y = 3 so that x = (1977 + 3)  9 = 220 , so
that the number removed is 220 + 3 = 223 .
【Solution 2】
Let the first of the ten consecutive numbers be x. Then the sum of the ten numbers is
10 x + 45 . So the unit-digit of this sum is 5. Since the sum of the remaining nine
numbers is 2022, the unit-digit of the erased number is 3.
Observe that the sum of the remaining nine numbers is 9 x + 45 , 9 x + 44 ,
9 x + 43 , … or 9 x + 36 , so the smallest number of the remaining numbers is between
2022 − 45 2 2022 − 36 2
= 219 and = 220 . Hence the smallest number of the
9 3 9 3
remaining numbers is 220. Thus the erased number is 223.
Answer: 223
24. The ages of Tom and his two grandchildren add up to 80 years. The ages of the
two grandchildren are the digits of the grandpa's age. How old is grandpa Tom?
【Solution 1】
Let ab be the grandpa’s age, then a and b are the grandchildren’ ages. Observe that
a  8 and b  10 . We also know that ab + a + b = 80 , equivalent to 11a + 2b = 80 .
As 2b and 80 are even numbers, the number a must also be even. Since b  10 , we
have 11a  80 − 2  9 = 62 , that is a  6 . Hence we have a = 6 , then
b = (80 −11 6)  2 = 7 , so the grandpa’s age is 67.
【Solution 2】
If the grandpa’s age is 7b , then 7 and b are the grandchildren’ ages. Then we know
7b + 7 + b = 80 , equivalent to 77 + 2b = 80 . This is impossible since 2b is an even
number.
If the grandpa’s age is 6b , then 6 and b are the grandchildren’ ages. Then we know
6b + 6 + b = 80 , equivalent to 66 + 2b = 80 . So b = (80 − 66)  2 = 7 . Thus the
grandpa’s age is 67.
If the grandpa’s age is equal or less than 59, then the sum of two grandchildren’ ages

396
is equal or lager than 80 − 59 = 21 , that is at least one grandchild’s age is over 10.
This is impossible, since the ages of the two grandchildren are two digits of the
grandpa's age.
So the grandpa’s age is 67.
Answer: 067
25. Each number in a 4  5 table is 0, 1 or 2. The sum of the numbers in each row
and in each column is a multiple of 3. What is the maximum number of 1’s in this
table?
【Solution】
In each of the 4 rows, if none of the five numbers is 2, then there must be at least two
0s. Hence twice the number of 2s plus the number of 0s is at least 8. In each of the 5
columns, if none of the numbers is 0, then there must be at least two 2s. Hence twice
the numbers of 0s plus the number of 2s is at least 10. It follows that three times the
number of 0s plus three times the number of 2s is at least 18, so that the total number
of 0s and 2s is at least 6. Since there are 4  5 = 20 numbers in the table, the number
of 1s is at most 20 − 6 = 14 . The following table shows that the number of 1s may be
as high as 14.
1 1 1 1 2
1 1 1 1 2
0 0 1 1 1
1 1 0 0 1
Answer: 014

397

You might also like